You are on page 1of 165

Infectious Diseases

A Case Study Approach


NOTICE
Medicine is an ever-changing science. As new research and clinical experience broaden our knowl-
edge, changes in treatment and drug therapy are required. The author and the publisher of this
work have checked with sources believed to be reliable in their efforts to provide information that
is complete and generally in ac:c:ord with the standards accepted at the time of publication. How-
ever, in view of the possibility of human error or changes in medical sciences, neither the author
nor the publisher nor any other party who has been involved in the preparation or publication
of this work warrants that the information contained herein is in every respect accurate or com-
plete, and they disclaim the complete responsibility for any errors or omissions or for the results
obtained from use of the information contained in this work. Readers are encouraged to confirm
the information contained herein with other sources. For example and in particular, readers are
advised to check the product information sheet included in the package of each drug they plan to
administer to be certain that the information contained in this work is accurate and that changes
have not boon made in the recommended dose or in the contraindications for administration. This
recommendation is of particular importance in connection with new or infrequently used drugs.
Infectious Diseases
A Case Study Approach

Editor
Jonathan C. Cho, PharmD, MBA, BCIDP, BCPS
Clinical Associate Professor
Clinical Pharmacist, Infectious Diseases
Ben and Maytee Fisch College of Pharmacy
The University of Texas at Tyler
Tyler, Tens

New York Chicago San Francisco Athens London Madrid Mexico City
Milan New Delhi Singapore Sydney Toronto
Copyright© 2020 by McGraw-Hill Education. All rights reserved. Except as permitted under the United States
Copyright Act of 1976, no part of this publication may be reproduced or distributed in any form or by any means,
or stored in a database or retrieval system, without the prior written permission of the publisher.

ISBN: 978-1-26-045511-3
MHID: 1-26-045511-4

The material in this eBook also appears in the print version of this title: ISBN: 978-1-26-045510-6,
MHID: 1-26-045510-6.

eBook conversion by codeMantra


Version 1.0

All trademarks are trademarks of their respective owners. Rather than put a trademark symbol after every occur-
rence of a trademarked name, we use names in an editorial fashion only, and to the benefit of the trademark owner,
with no intention of infringement of the trademark. Where such designations appear in this book, they have been
printed with initial caps.

McGraw-Hill Education eBooks are available at special quantity discounts to use as premiums and sales promo-
tions or for use in corporate training programs. To contact a representative, please visit the Contact Us page at
www.mhprofessional.com.

TERMS OF USE

This is a copyrighted work and McGraw-Hill Education and its licensors reserve all rights in and to the work. Use
of this work is subject to these terms. Except as permitted under the Copyright Act of 1976 and the right to store
and retrieve one copy of the work, you may not decompile, disassemble, reverse engineer, reproduce, modify,
create derivative works based upon, transmit, distribute, disseminate, sell, publish or sublicense the work or any
part of it without McGraw-Hill Education's prior consent. You may use the work for your own noncommercial
and personal use; any other use of the work is strictly prohibited. Your right to use the work may be terminated if
you fail to comply with these terms.

THE WORK IS PROVIDED "AS IS." McGRAW-HILL EDUCATION AND ITS LICENSORS MAKE NO
GUARANTEES OR WARRANTIES AS TO THE ACCURACY, ADEQUACY OR COMPLETENESS OF
OR RESULTS TO BE OBTAINED FROM USING THE WORK, INCLUDING ANY INFORMATION THAT
CAN BE ACCESSED THROUGH TIIE WORK VIA HYPERLINK OR OTHERWISE, AND EXPRESSLY
DISCLAIM ANY WARRANTY, EXPRESS OR IMPLIED, INCLUDING BUT NOT LIMITED TO IMPLIED
WARRANTIES OF MERCHANTABILITY OR FITNESS FOR A PARTICULAR PURPOSE. McGraw-Hill
Education and its licensors do not warrant or guarantee that the functions contained in the work will meet your
requirements or that its operation will be uninterrupted or error free. Neither McGraw-Hill Education nor its
licensors shall be liable to you or anyone else for any inaccuracy, error or omission, regardless of cause, in the
work or for any damages resulting therefrom. McGraw-Hill Education has no responsibility for the content of any
information accessed through the work. Under no circumstances shall McGraw-Hill Education and/or its licen-
sors be liable for any indirect, incidental, special, punitive, consequential or similar damages that result from the
use of or inability to use the work, even if any of them has been advised of the possibility of such damages. This
limitation ofliability shall apply to any claim or cause whatsoever whether such claim or cause arises in contract,
tort or otherwise.
CONTENTS

Contributors vii 18. Diabetic Foot Infection 75


Preface ix
Ka11a R. Stover

19. Vertebral Osteomyelltls 79


1. lnfluenZll 1
Meghan N. Jeffres
Maria Heaney, Jason Gallagher
20. Prostlletfc Joint Infection 83
2. Acute Otitls Media 5
Amelia K. Sojjan
Aimee Dtwner, Jennifer E. Girotto
21. lntra-Abdomlnal lnfedlons 87
3. Acute Bronchitis 9
Jamie L. Wagner
Jenana Maker
22. Clostrldlo ldu dlftklle Infection 91
4. Pharyngitis 13
Rebecca L. Dunn, Jonathan C. Cho
Prank S. Yu. Jonathan C. Cho
2J. Trav•l•r's Dlarrhu 95
s. Communlt,._Acqulred Pneumon ia 17
Amber B. Giles
Sean N. Avedissian, Marc H. Scheetz
24. Hepatitis (
6. Hospital-Acquired and
Lindsey Childs-Kean
Ventilator-Associated Pneumon ia 21
Stephanie E. Giancola, Elizabeth B. Hirsch 25. Syphllls 103
Trent G. Towtu
7. Cystitis 25
Kristy M. Shaeer
26. Herpes 107
Elias B. Chahine
8. Pyelonephrltls 29
Maryke V. Worle)i 27. ChlamJc:lia and Gononh • 111
9. Bacterial Meningitis Takova D. Wallace-Ga)', Jonathan C. Cho
33
Jonathan C. Cho 28. Bacterial Vaginosis, Vulvovaginal
C.ndidias is, and Trichomoniasis 115
10. Viral Encephalitis 37
Elizabeth A. Cook, Jessica Wooster,
AnnllOJd
Jonathan C. Cho
11. Infective Endocardltls 41
29. Superfici. i Fung11l Infections 121
Rachel A. Foster, P. Brandon Bookstaver
Winter J. Smith, Jonathan C. Cho
12. Sepsis 47
30. Cryptococcus 125
Emily L. Heil
Paul 0. Gubbins
13. Human Immunodeficiency Virus
31. Aspargillosis 129
and Opportun istic Infections 51
Ashley H. Marx
Elizabdh Sherman
32. Protozoan s 133
14. Febrlle Neutropen la 55
Lisa Awry
Wesley D. Kufel
33. Nematode s 137
15. Skin and Soft Tissue Infection I 61
Jessica Robinson
Michael Kelsch
34.. Tuberculosl.s 141
16. Skin and Soft Tissue Infection 11 67
David Cluck
Madeline King
Index 145
17. Necrotlzlng Fasclltfs 71
Tianrui Yan& Jonathan C. Cho
v
This page intentionally left blank
Sean N. Avedlsslan, PhannO, MSc Rachel A. Foster, PhannD, MBA, BCIDP
Midwestern University Chicago College of Pharmacy, Intennountain Healthcare, Murray, Utah
Downers Grove, Illinois Chapter 11
Northwestern Memorial Hospital, Department of Pharmacy,
Chicago, Illinois Jason Gallagher, PharmD, BCPS
Chapters Temple University School of Pharmacy, Philadelphia,
Pennsylvania
Usa Avery, PharmD, BCPS, BCIDP Chapter l
Wegmans School of Pharmacy/St John Fisher College,
Rochester, New York Stephani• E. Giancola, PharmD, BCIDP, BCPS
Chapter 32 Brooke Army Medical Center, Fort Sam Houston, Texas
Chapter6
P. Brandon Bookstaver, PharmD, BCPS
University of South Carolina College of Pharmacy, Amber B. Giles, PharmD, MPH, BCPS, AAHIVP
Columbia, South Carolina Presbyterian College School of Pharmacy, Clinton, South
Chapter 11 Carolina
Chapter23
Ellas B. Chahlne, PharmD, BCPS, BCIDP
Palm Beac.b. Atlantic University Lloyd L. Gregory School of Jenniftlr E. Girotto, Ph1nnD, BCPPS, BCIDP
Pharmacy, West Palm Beach, Florida University of Connecticut School of Pharmacy, Connecticut
Chapter 26 Children's Medical Center, Hartford. Connecticut
Chapter2
Undsey Chllds-K..n, PharmD, MPH, BCPS
University of Florida College of Pharmacy, Gainesville, Paul O. Gubbins, PhannD
Florida University of Missouri - Kansas City, School of Pharmacy at
Chapter 24 Missouri State University, Spring6.eld. Missouri
Chapter30
Jonathan c. Cho, PharmD, MBA, BCIDP, BCPS
Department of Clinical Sciences Maria Heaney, PharmD
Ben and Maytee Fisch College of Phannacy, University of Temple University School of Pharmacy, Philadelphia,
Texas, Tyler, Texas Pennsylvania
Chapters 4, 9, 17, 22, 27, 28, d- 29 Chapter l

David Cluck, PhannD, BCPS, BCIDP, AAHIVP Emily L. Hail, PharmD, BCIDP
East Tennessee State University-Gatton College of University of Maryland School of Pharmacy, Baltimore,
Pharmacy, Department of Pharmacy Practice. Johnson Maryland
City, Tennessee Chapter 12
Chapter34
Elizabeth B. Hirsch, PharmD, BCPS
Elizabeth A. Cook, PharmD, AE-C, BCACP, CDE University of Minnesota College of Pharmacy, Minneapolis,
Ben and Maytee Fisch College of Pharmacy, University Minneapolis
of Texas, Tyler, Texas Chapter6
Chapter28
Meghan N. Jeffres, PharmD, BCIDP
Aimee DassnH, PharmD, BCIDP University of Colorado Skaggs School of Pharmacy and
Children's Health, Dallas, Texas Pharmaceutical Sciences, Aurora, Colorado
Chapter2 Chapter 19

Rebecca L. Dunn, PharmD, BCPS Michael Kelsch, PharmD, BCPS


Ben and Maytee Fisch College of Pharmacy, University North Dakota State University School of Pharmacy, Sanford
of Texas, Tyler, Texas Medical Center Fargo, Fargo, North Dakota
Chapter22 Chapter 15

vii
viii CONTRIBUTORS

Madeline King, PharmD, BCIDP Amelia K. Sofjan, PharmD, BCPS


Philadelphia College of Pharmacy - University of the University of Houston College of Pharmacy, Houston, Texas
Sciences, Philadelphia, Pennsylvania Chapter 20
Chapter 16
Kayla R. Stover, PharmD, BCIDP, BCPS
Wesley D. Kufel, PharmD, BCIDP, BCPS, AAHIVP University of Mississippi School of Pharmacy, Jackson,
Binghamton University School of Pharmacy and Mississippi
Pharmaceutical Sciences, State University of New York; Chapter 18
Upstate Medical University, State University of New York;
Upstate Medical University Hospital, Binghamton, New Trent G. Towne, PharmD, BCPS
York Manchester University College of Pharmacy, Natural
Chapter 14 & Health Sciences, Fort Wayne, Indiana
Chapter 25
Ann Lloyd, PharmD, BCPS-AQ ID, BCIDP
The University of Oklahoma College of Pharmacy, Tulsa, Jamie L. Wagner, PharmD, BCPS
Oklahoma University of Mississippi School of Pharmacy, Jackson,
Chapter 10 Mississippi
Chapter 21
Jenana Maker, PharmD, BCPS
University of the Pacific Thomas J Long School of Pharmacy Takova D. Wallace-Gay, PharmD, BCACP
and Health Sciences, Stockton, California Ben and Maytee Fisch College of Pharmacy, University
Chapter 3 of Texas, Tyler, Texas
Chapter 27
Ashley H. Marx, PharmD, BCPS, BCIDP
University of North Carolina Eshelman School of Pharmacy, Jessica Wooster, PharmD, BCACP
Chapel Hill, North Carolina Ben and Maytee Fisch College of Pharmacy, University
Chapter 31 ofTexas, Tyler, Texas
Chapter 28
Jessica Robinson, PharmD, BCPS, BCIDP
University of Charleston School of Pharmacy, Charleston, Marylee V. Worley, PharmD, BCPS
West Virginia Nova Southeastern University College of Pharmacy, Fort
Chapter 33 Lauderdale, Florida
Chapter 8
Marc H. Scheetz, PharmD, MSc
Midwestern University Chicago College of Pharmacy, Tianrui Yang, PharmD, BCPS
Downers Grove, Illinois Northwestern Memorial Hospital, Ben and Maytee Fisch College of Pharmacy, University
Department of Pharmacy, Chicago, Illinois of Texas, Tyler, Texas
Chapter 5 Chapter 17

Kristy M. Shaeer, PharmD, MPH, BCIDP, AAHIVP Frank S. Yu, PharmD


University of South Florida College of Pharmacy, Tampa, Ben and Maytee Fisch College of Pharmacy, University
Florida ofTexas, Tyler, Texas
Chapter 7 Chapter 4

Elizabeth Sherman, PharmD, AAHIVP


Nova Southeastern University College of Pharmacy,
Memorial Healthcare System, Division of Infectious
Diseases, Southeast AIDS Education and Training Center,
Fort Lauderdale, Florida
Chapter 13

Winter J. Smith, PharmD, BCPS


Ben and Maytee Fisch College of Pharmacy, University
ofTexas, Tyler, Texas
Chapter 29
PREFACE

T
he goal of Infectious Diseases: A Case Study Approach Pharmacy Education's Coding Systems for Colleges of Phar-
is to provide healthcare students with a valuable macy and the 2016 American College of Clinical Pharmacy's
infectious diseases pharmacotherapy resource. With Pharmacotherapy Didactic Curriculum Toolkit. Authors of
the growing need of antimicrobial stewardship programs, this casebook chapters are comprised of infectious diseases
healthcare professionals competent in infectious diseases pharmacist faculty from Colleges of pharmacy across the
pharmacotherapy are necessary. United States. All these individuals have vast experiences
This casebook is designed to teach infectious diseases and training in infectious diseases and are widely recognized
through patient cases that closely resemble situations as experts in their field.
healthcare professionals will likely face during their clini- I hope that you will find this casebook useful during your
cal practice. Infectious diseases-related topics covered in studies!
this book range from bacterial infections, to sexually trans- Jonathan C. Cho, PharmD, MBA, BCIDP, BCPS
mitted diseases, to antimicrobial dosing recommendations. The University of Texas at Tyler
Topics were selected based on the Accreditation Council for Tyler, Texas

ix
This page intentionally left blank
1 Influenza
Maria Heaney Jason Gallagher

PATIENT PRESENTATION Amlodipine 10 mg PO daily


Atorvastatin 20 mg PO daily
Chief Complaint
Fluticasone-salmeterol 100 mcg/50mcg1 puff BID
"I feel like I got hit by a truck. My body aches intensely, f m
weak, I have a fever, fm lying under 3 blankets and can't get Levothyroxine 112 mcg PO daily
warm, and I can't stop shivering:' Montelukast 10 mg PO QHS

History of Present Illness Physical Examination


AW is a 3~year-old female presenting to the emergency depart-
ment with a 2-day history of myalgia, c.hills. and fever to 102.F ~ V'ttal Signs
for which she has been using over-the-counter acetaminophen. Temp 101.7.F, P 84, RR 18, BP 136/84 mm Hg, Sa01 97%,
She reports a rapid onset of symptoms. including nasal conges- Ht 5'5", Wt 64.5 kg
tion and cough associated with chest pain. She denies a sore
throat. but complains of nausea and vomiting that began a few ~ Genel'tll
weeks ago and has worsened acutely. She reports 3 episodes of Lethargic female with headache
emesis this morning prior to coming to the emergency depart-
ment She states she has never received the influema vaccine.
but all of her family members are vaccinated annually. ~ HEENT
Normocephalic, atraumatic, PERRLA, EOMI, pale/dry
Past Medical History mucous membranes and conjunctiva
Asthma, HTN, HLD, hypothyroidism
~ Pulmonary
Surgical History Stridorous breath sounds, equal lung expansion, cough
Appendectomy 6 years ago present

Family History ~ Cardiovascular


Father has a history of MI; mother has HTN; sister has a RRR. no murmurs, rubs, or gallops
history of breast cancer.
~ Abdomen
Soclal History Soft, non-tender, non-distended, normoactive bowel sounds
Married with 3 young. healthy children (ages I. 3, and 6 years).
Works as a 7th grade teacher. Denies illicit drug or tobacco use. ~ Genitourinary
Drinks alcohol socially.
Deferred
Allergies
~ Neurology
Penicillin (rash), egg (anaphylaxis)
Lethargic. AAO X3
Vaccines
um. refuses infiuenza vaccine due to egg allergy ~ Extremities
Warm and well perfused, no edema
Home Medications
Albuterol metered-dose inhaler 2 puffs q4h PRN shortness ~ Skin
of breath No rashes, or lesions

1
2 INFECTIOUS DISEASES: A CASE STUDY APPROACH

Laboratory Findings C. Both A and B


Na= 142 mEq/mL Ca = 8.6 mg/dL AST= 27 IU/L D. None of her family members should be offered
chemoprophylaxis
K = 3.8 mEq/mL Mg= 1.9 mgldL ALT = 19 IU/L
7. Which of the following would be an appropriate chemo-
Cl = 98 mEq/mL Phos = 3.6 mg/dL T Bili = 0.5 mg/dL prophylaxis regimen for a patient with a CrCl of 80 to 85
C02 = 24 mEq/mL Hgb = 15.1 mg/dL AJk Phos = 36 IU/L mL/min?
BUN= 17 mg/dL Hct = 15.1 g/dL Lactate = 0.7 A. Oseltamivir 30 mg PO twice daily
mmoUL B. Peramivir 600 mg IV once
C. Zanamivir 10 mg inhaled once daily
SCr = 0.8 mg/dL Plt = 184 x 103/ hCG = 84,000
mm3 D. Oseltamivir 30 mg PO once daily
mIU/mL
Glu = 142 mg/dL WBC = 8.2 x 103/ 8. Which of the following influenza vaccines is indicated for
mm3 AW?
A. Fluzone Quadrivalent
B. Fluzone High-Dose
C. FluMist Quadrivalent
D. AW should not receive the influenza vaccine
QUESTIONS
9. Which of the following statements is true?
1. Which of the following laboratory tests is the best option to
A. Zanamivir is an acceptable alternative antiviral for
assist in diagnosing AW in the emergency department?
A. An influenza viral culture treatment of influenza in AW
B. AW should receive the influenza vaccine once treat-
B. A rapid influenza cell culture
ment with an antiviral is initiated
C. A rapid influenza molecular assay
D. A rapid influenza antigen detection test C. AW is at a relatively low risk for complications second-
ary to influenza infection
2. Which of AW's symptoms and/or physical exam findings D. Both A and B
support a diagnosis of influenza?
A. Rapid onset of fever and respiratory symptoms
B. History of nausea and vomiting ANSWERS
C. Body aches 1. Explanation: The correct answer is C. Based on AW's
D. Both A and C presentation, it is likely that she has influenza. Influenza
testing can have antimicrobial stewardship implications
3. The influenza test performed for AW is positive for influenza
and can influence infection prevention and control deci-
A. Which of the following antiviral regimens is most
sions. During influenza season, outpatients and patients
appropriate to start?
presenting to the emergency department should be tested
A. Baloxavir 20 mg PO once daily
for influenza if they present with acute-onset respira-
B. Amantadine 200 mg PO once daily
tory symptoms and are at high risk for complications
C. Oseltamivir 75 mg PO BID
secondary to influenza infection if the result may influ-
D. Zanamivir 10 mg inhaled daily
ence clinical management. Testing may be considered
4. What is the most appropriate duration oftherapy for uncom- for patients who are not at high risk for complications if
plicated influenza treated with oseltamivir? results may influence clinical management with regard
A. 3 days to antiviral therapy, avoidance of unnecessary antibiot-
B. 5 days ics or other diagnostic tests, and shortened time in the
C. 7 days emergency department. Testing is recommended for all
D. 10 days hospitalized patients during influenza season who have
acute respiratory illness, have exacerbation of a chronic
5. Which of the following adjunctive therapies should be added
pulmonary or cardiac comorbidity, or are immunosup-
to AW's antiviral regimen?
pressed with respiratory or other nonspecific symptoms.
A. Methylprednisolone 60 mg IV daily
All of the above methods are reliable for detecting influ-
B. IVIG 1 mg/kg IV once
enza. A rapid influenza cell culture has a high sensitivity
C. Piperacillin-tazobactam 3.375 g IV every 6 hours +
and specificity; however, it takes 1 to 3 days to produce
vancomycin 1 g IV every 12 hours
results which would not be ideal for rapid testing in the
D. None of the above
emergency department. While a viral culture has a high
6. Which of AW's family members should receive post-exposure sensitivity and specificity, it takes 3 to 10 days for results.
chemoprophylaxis? Since there are other testing methods available with a
A. Her 1-year-old child more rapid time to results, a viral culture is not the most
B. Her 3-year-old child efficient method for emergency department diagnostics.
CHAPTER 1 I INFLUENZA 3

The rapid molecular assay produces results in 15 to 20 situations, such as in immunocompromised patients with
minutes, and the rapid antigen detection test in 10 to severe influenza infection, a 10-day course may be consid-
15 minutes. These methods are the most efficient for ered. Critical illness is another scenario in which a longer
diagnosing influenza in the emergency department. Both duration of treatment may be considered, though there is
methods have a high specificity, or the ability to detect a no well-defined duration in this population. 1 AW is not
true negative result The difference between the two is the immunocompromised, nor is she critically ill, thus a 5-day
sensitivity, or the ability to detect a true positive result. course of therapy is adequate to treat her influenza.
The rapid molecular assay detects influenza A or B viral
RNA and has a high sensitivity, while the rapid antigen 5. Explanation: The correct answer is D. Corticosteroid
detection test has a low to moderate sensitivity and is administration in patients with influenza has been asso-
not ideal for use in hospitalized patients. It is guideline ciated with increased mortality in hospitalized patients
recommended to use a rapid molecular assay over a rapid in observational studies and is not recommended for
antigen detection test, when possible, to improve the adjunctive therapy. Immunotherapy, such as with IVIG,
detection of influenza infection.1 can have immune modulating effects and neutralize viral
activity; however, significant clinical benefit over antiviral
2. Explanation: The correct answer is D. Influenza is char- agents has not yet been determined in clinical study. IVIG
acterized by a rapid onset of respiratory symptoms, body should not routinely be used as adjunctive therapy. Lastly,
aches, and exhaustion. 1 Though nausea and vomiting can piperacillin-tazobactam is an antibacterial agent Bacte-
be associated with influenza infection, this is not a typical rial coinfection is possible in patients with influenza,
presentation for adults. Additionally, AW has a several- and may be present upon initial influenza diagnosis or
week history of nausea, which is not a characteristic of may manifest later and result in clinical deterioration.
influenza infection. Bacterial coinfection should be investigated and empiri-
3. Explanation: The correct answer is C. Influenza treatment cally treated in patients presenting with severe disease
should be initiated promptly in patients with suspected consisting of extensive pneumonia on imaging, respira-
or confirmed infection regardless of vaccination history tory failure, hypotension, and fever. Bacterial coinfection
if they are hospitalized, if they are outpatients with severe may also be considered in patients who deteriorate after
illness or at high risk of complications, including immuno- initial improvement on an antiviral agent or in those who
suppressed patients or those with chronic comorbidities, fail to improve within 3 to 5 days of antiviral therapy.
children <2 years old, adults 65 years or older, or women In patients with severe bacterial pneumonia complicat-
who are pregnant or within 2 weeks postpartum. Oselta- ing influenza, Staphylococcus aureus, including MRSA,
mivir, peramivir, and zanamivir are all guideline-recom- accounts for many cases, thus an anti-MRSA antibiotic
mended agents for the treatment of influenza. Baloxavir is should be included in empiric bacterial coverage in such
the most recently approved antiviral agent indicated for the cases. 1 For AW. antibiotics are not required for adjunctive
treatment of uncomplicated influenza. This is not the cor- therapy because her presenting symptoms did not include
rect answer choice because it is given as a single dose based evidence of severe pneumonia, respiratory failure, or
on weight. For patients 40 to 80 kg, the dose is 40 mg once, hypo tension.
and for patients ~80 kg, the dose is 80 mg once. An impor-
6. Explanation: The correct answer is D. Post-exposure
tant counseling point for baloxavir is that administration
chemoprophylaxis with antivirals is not recommended
must be separated from polyvalent cations such as calcium,
routinely, but may be considered in certain patient popu-
iron, and magnesium. Baloxavir is also not recommended
lations. Chemoprophylaxis may be administered to adults
for hospitalized patients with severe cases of influenza
and children ~3 months old at high risk of complications
due to a lack of data in this population.2 Zanamivir 10 mg
from influenza, such as patients who are severely immu-
inhaled daily is a chemoprophylaxis dose. The treatment
nocompromised and for whom vaccination is contraindi-
dose of zanamivir is 10 mg inhaled twice daily. Addition-
cated. If adults and children ~3 months old are exposed
ally, there is limited data for the use of inhaled zanamivir
to influenza and are household contacts of an immuno-
in hospitalized patients with severe cases of influenza, and
compromised patient, it is recommended that chemopro-
this is not a preferred agent in pregnancy due to concern
phylaxis is administered for 7 days after exposure along
for lower lung volumes which may result in reduced drug
with the inactivated influenza vaccine. Patients exposed
exposure and bronchospasm. Adamantane antivirals such
to influenza who do not receive chemoprophylaxis should
as amantadine are no longer recommended due to viral
be monitored closely and treated promptly if symptoms
resistance. Osdtamivir is the preferred antiviral in hospital-
develop, especially those at high risk for complications
ized patients, as well as in pregnancy.•
from infection, including children <5 years old (and
4. Explanation: The correct answer is B. According to the especially <2 years old), elderly patients ~65 years old,
guideline recommendations and FDA approval, immu- women who are pregnant or postpartum, morbidly obese
nocompetent adults with influenza should receive a 5-day patients, nursing home residents, and patients with chronic
course when being treated with oseltamivir. In certain pulmonary, cardiac, or metabolic diseases. 1 Pre-exposure
4 INFECTIOUS DISEASES: A CASE STUDY APPROACH

chemoprophylaxis with oseltamivir or zanam1vrr may for complications, should receive the influenza vaccine
also be considered in patients 2!3 months old at high risk even if they are infected with influenza in order to pro-
for complications, including aforementioned populations tect against other influenza strains and future infection.
as well as hematopoietic stem cell transplants recipients According to the CDC, it is acceptable to administer an
within 6 to 12 months posttransplant and lung transplant inactivated influenza vaccine during treatment with an
recipients. Pre-exposure chemoprophylaxis is given for antiviral. Live attenuated influenza vaccines should be
the duration of influenza season in most cases.1 AW avoided until 48 hours after completing therapy with
reported that all members of her family are healthy and an antiviral. 5 In pregnancy, inhaled zanamivir is not
have received the influenza vaccine, thus post-exposure preferred for treatment due to concern for reduced drug
chemoprophylaxis is not indicated in this situation. Her distribution and bronchospasm resulting from lower lug
I-year-old and 3-year-old children, however, should be volumes. Zanamivir is also not recommended for use
very closely monitored for symptoms as they are at high in patients with respiratory disease, including asthma.
risk for complications from influenza. Oseltamivir is the preferred agent for treatment in
pregnancy.1
7. Explanation: The correct answer is C. The chemoprophy-
laxis dose of oseltamivir is 75 mg PO once daily for patients
with normal renal function. Oseltamivir 30 mg PO twice
daily is a renally adjusted treatment dose, and 30 mg PO REFERENCES
once daily is a renally adjusted dose that can be used for 1. Uyeki TM, Bernstein HH, Bradley JS, et al. Clinical
treatment or prophylaxis. Peramivir is not indicated practice guidelines by the Infectious Diseases Society of
for chemoprophylaxis. Pre-exposure prophylaxis is given America: 2018 update on diagnosis, treatment, chemo-
for the duration of influenza activity in the community in prophylaxis, and institutional outbreak management of
most cases, whereas post-exposure chemoprophylaxis is seasonal influenza. Clin Infect Dis. 2018;68(6):el-e47.
given for 7 days after the initial influenza exposure.1 2. Baloxavir marboxil [package insert] . San Francisco,
CA: Genentech USA, Inc.; 2018. Available at https://
8. Explanation: The correct answer is A. Fluzone High-
www.gene.com/download/pdf/xofluza_prescribing.pdf.
Dose is an inactivated influenza vaccine indicated for
Accessed April 18, 2019.
patients ~65 years. FluMist Quadrivalent is a live attenu-
3. Centers for Disease Control and Prevention. Recommended
ated influenza vaccine (LAIV).3 An egg allergy of any
immunization schedule for adults aged 19 years or older,
severity is no longer a contraindication to LAIV admin-
United States, 2018. Available at https://www.cdc.gov/
istration and is listed as safe for administration by the
vaccines/schedules/hcp/imz/adult.html Accessed February
Advisory Committee on Immunization Practices (ACIP).
7, 2019.
For AW, however, we would avoid the use of the LAIV
4. Grohskopf LA, Sok.olow LZ, Broder KR, Walter EB, Fry
because it is not recommended for administration to
AM, Jernigan DB. Prevention and control of seasonal
pregnant women according to ACIP.4
influenza with vaccines: Recommendations of the Advi-
9. Explanation: The correct answer is B. Since AW is preg- sory Committee on Immunization Practices-United
nant as evidenced by her elevated hCG, she is at high States, 2018-19 influenza season. MMWR Recomm Rep.
risk for complications secondary to influenza, including 2018;67(3):1-20.
cardiopulmonary disease, premature labor, and fetal 5. Centers for Disease Control and Prevention. Influenza
loss. Chronic pulmonary disease, including asthma, also vaccination: A summary for clinicians. CDC Web site.
puts patients at a higher risk for complications. 1 Patients, Available at https://www.cdc.gov/flu/professionals/
especially pregnant women and others at higher risk vaccination/vax-summary.htm. Accessed April 18, 2019.
2 Acute Otitis Media
Aimee Dassner Jennifer E. Girotto

PATIENT PRESENTATION Physical Examination


Chief Complaint .... Vital Signs (while crying)
"Increased irritability and right ear pain." Temp 100.7°F, P 140 bpm, RR 35, BP 100/57 mm Hg. Ht 81 cm,
Wt23.7kg
History of Present Illness
.... General
JL is a 22-month-old female who presents to her primary care
provider (PCP) with a 2-day history of rhinorrhea and a 1-day
Fussy, but consolable by Mom; well-appearing
history of increased irritability, fever (to 101.5°F per Mom),
.,.. HEENT
and right-ear tugging. Mom denies that JL has had any nausea,
vomiting, or diarrhea. Normocephalic, atraumatic, moist mucous membranes,
normal conjunctiva, clear rhinorrhea, moderate bulging
and erythema of right tympanic membrane with middle-ear
Past Medical History
effusion
Full-term birth via spontaneous vaginal delivery. Hospitalized
at 9 months of age for respiratory syncytial virus-associated .,.. Pulmonary
bronchiolitis. Two episodes of acute otiti.s media (AOM), with
last episode about 6 months earlier. Good air movement throughout, clear breath sounds bilaterally

.... Cardiovascular
Surgical History
Normal rate and rhythm, no murmur, rub or gallop
None
.,.. Abdomen
Social History Soft, non-distended, non-tender, active bowel sounds
Ll~s with mother, father, and her 5-year-old brother who
attends kindergarten. JL attends daycare 2 d/wk, and stays at .... Genitourinary
home with maternal grandmother 3 d/wk.
Normal female genitalia, no dysuria or hernaturia

Allergies .... Neurology


No known drug allergies Alert and appropriate for age

Immunizations .,.. Extremities


lmmuniution Age Administered Normal

Hepatitis B Birth
D'Thp/Hep B/IPV 2 mo, 4 mo, 6 mo QUESTIONS
Hib 2 mo, 4 mo, 6 mo, 15 mo 1. Which of the following clinical criteria is not part of the
diagnostic evaluation or staging of acuteotitis media
PCV13 2 mo, 4 mo, 6 mo, 12 mo (AOM) fur this patient?
Influenza 6 mo, 8 mo, 18 mo A. Rhinorrhea
MMR 12mo B. Fever
C. Otalgia
Varicella 12mo
D. Contour of the tympanic membrane
2. Which of the following is a risk factor for AOM?
Home Medications
A. Vaginal delivery
Vitamin D drops 600 IU/d B. History of RSV at 9 months
5
6 INFECTIOUS DISEASES: A CASE STUDY APPROACH

C. Day care attendance C. Influenza (Flu)


D. Immunizations up to date D. Both A and C
E. All of the above
3. Which ofthe following best describes the clinical presentation
of AOM for this patient? 9. If JL continued to have multiple episodes of AOM, pro-
A. Non-severe, bilateral phylactic antibiotics should be considered to reduce the
B. Non-severe, unilateral frequency of AOM episodes in which of the following
C. Severe, bilateral situations:
D. Severe, unilateral A. 3 episodes of AOM in 6 months
B. 4 episodes of AOM in 1 year
4. What is the recommended management for AOM in this
C. 6 episodes of AOM in 2 years
patient?
D. Never
A. Culture the middle ear fluid, then treat with culture-
directed antibiotics
B. Acetaminophen 15 mg/kg PO q6h, with patient follow- ANSWERS
up in 2 to 3 days
1. Explanation: The correct answer is A. Per the 2013
C. Acetaminophen 15 mg/kg PO q6h PRN and amoxicillin
American Academy of Pediatrics (AAP) guidelines
45 mg/kg PO ql2h
for "The Diagnosis and Management of Acute Otitis
D. Acetaminophen 15 mg/kg PO q6h PRN and cefdinir
Media," 1 clinical criteria for the diagnosis and severity
14 mg/kg PO q24h
staging (non-severe versus severe) of AOM include otor-
E. Acetaminophen 15 mg/kg PO q6h PRN and amoxicillin/
rhea, otalgia, fever, visualization of tympanic membrane
clavulanate 30 mg/kg q8h
(TM) contour (normal, mild bulging, moderate bulging
5. Forty-eight hours after initial presentation, the patient's or severe bulging) and color, and presence of middle
mother calls the PCP to report persistent otalgia and fevers ear effusion (MEE). AOM should not be diagnosed in
(Tm... 102°F), with complaints of new-onset left ear pain. patients without MEE. Additionally, a moderate to severe
Which of the following would be the most appropriate bulging TM is the most important clinical sign in the
antimicrobial therapy to start for this patient? diagnosis of AOM, and has been highly associated with
A. Amoxicillin bacterial etiology of infection.
B. Amoxicillin/clavulanate
2. Explanation: The correct answer is C. Day care atten-
C. Cefdinir
dance is a well-known risk factor for AOM. Other risk
D. Azithromycin
factors for AOM is a family member with AOM, parental
6. You have recommended that JL be prescribed amo.xicillin at smoking, and pacifier usage. 2
a dose of 30 mg/kg/dose q8h. The community pharmacist
3. Explanation: The correct answer is B. This patient only
calls and asks to verify if this dose is correct or if 45 mg/kg/
has complaints and signs of otalgia in the right ear, which
dose q 12h would be better. Which ofthe following would be
would make this a unilateral presentation of AOM. Severe
a reason that you would prefer the 30 mg/kg/dose q8h over
AOM is defined as a toxic-appearing child, or persistent
a dose of 45 mg/kg/dose q 12h?
otalgia >48 hours, or a temperature ~39°C (102.2°F) in
A. Increased rates of H. influenzae resistance in your
the past 48 hours. This patient does not meet any of these
community
criteria. 1
B. Increased rates of oral penicillin non-susceptible pneu-
mococci in your community 4. Explanation: The correct answer is B. All children with
C. Increased rates of M. catarrhalis in your community AOM and otalgia should be offered pain management, so
D. There is no reason that 30 mg/kg/dose q8h would be acetaminophen (an analgesic and antipyretic) is appropri-
preferred ate to prescribe for this patient. Both viral and bacterial
pathogens can cause AOM, but identification of causative
7. What is the appropriate duration of antibiotic therapy for
pathogens is not routinely performed for non-refractory
treatment of AOM for JL?
AOM in clinical practice.u The majority of AOM epi-
A. 5 days
sodes are viral in origin and are self-limiting, as are many
B. 7 days
types of bacterial AOM. Pneumococcal AOM is the least
C. 10 days
likely bacterial AOM to resolve on its own. Severe AOM,
D. 14 days
AOM in patients <6 months and non-severe bilateral
8. Per the current AAP guidelines for the diagnosis and man- AOM in young patients (6 to 24 months) have been
agement of acute otitis media, which of the following rou- most often associated with increased rates of clinical
tinely administered pediatric vaccine(s) is/are recommended failure. Patients without these criteria are generally rec-
by the AAP to help prevent AOM in infants and children? ommended to have an initial period of observation prior
A. Pneumococcal conjugate vaccine (PCV13) to antibiotic prescribing. If AOM worsens or does not
B. H. influenzae type b (Hib) improve after 48 to 72 hours of observation, a bacterial
CHAPTER 2 I ACUTE OTITIS MEDIA 7

etiology that is unlikely to self-resolve can be presumed infection). if they have received antibiotics in the past
and antibiotics should be prescribed. AAP guideline 30 days, or if they fail initial therapy with amoxicillin.1
recommendations for approach to initial management of Cefdinir is listed in the AAP guidelines as an alternative
AOM in children are summarized in Table 2.1, adapted first-line treatment fur penicillin-allergic patients.1 However,
from AAP guidelines.1 it is important to note that cefdinir has deaeased empiric
in vitro S. pneumoniae susceptibilities compared to amox-
5. Explanation: The correct answer is A. The most com-
icillin (70% to 80% versus 84% to 92% susceptible).
mon bacterial. pathogens in AOM are Streptococcus
Lastly, azithromycin is not recommended for AOM,
pneumoniae, non-typeable Haemophilus influenzae, and
due to poor activity against both S. pneumoni~ and
Moraxella catarrhalis. The cawative etiology of bacterial
H. influenzae.1
AOM has shifted since the onset of routine pneurnococ-
cal. vaccination with PCV7 in 2000, and its subsequent 6. EJ;planation: The correct answer is B. Change of the
replacement with PCV13 in 2010. Specifically, the preva- dose of amox:icillin from q12h to q8h improves the time
lence of circulating penicillin-resistant S. pneumoniae above the MIC up until an MIC of approximately 2 11gl
strains has decreased. ml.' Since both H. influenzae and M. catarrhalis produce
When properly dosed, amox:icillin (90 mg/kg/d as q8h beta-lactamase, a switch to amoxicillin/clavulanate would
dosing) can provide adequate time above the minimum be most appropriate instead of a change in dose.
inhibitory concentl'ation (MIC) for S. pneumoniae isolates
7. E:l:plmation: The correct answer is C. While the optimal
with a penicillin MIC :S:2 p.glmL (current susceptibility
duration of therapy for AOM is unknown, 10 days of anti-
breakpoint). The addition of clavulanate to amoxicillin
biotics is the current standard duration of therapy. Several
(amoxicillin/davulanate) provides additional coverage
studies have evaluated shorter courses of antibiotics for
against beta-lactamase-producing organisms, but does
treatment of AOM, with results suggesting that 7 and 5
not provide any additional coverage against resistant
days of antibiotic therapy may be equally as effective as
S. pneumoniae over amoxicillin. Although 18% to 42% of
10 days in children aged 2 to 5 years and ~6 years with
H. influenzae and 100% ofM. catarrhalis isolates produce
mild to moderate AOM, respectl.vely.1 However, JL is not
beta-lactamase, 48% to 75% of AOM infections with
yet 2 years old, so these shorter treatment courses would
these organisms, respectively, are self-limiting. Therefore,
not be appropriate. Notably, a 2016 New England Jour-
high-dose amox:icillin (90 mglkg/d) is reoommended as
nal of Medicine article evaluating a 5-day versus 10-day
first-line treatment for antibiotic-naive (no antibiotics
course of amoxicillin/clavulanate for the treatment of
within 30 days) patients as a narrow-spec::trum, affordable
AOM in children 6 to 23 months of age found that chil-
antibiotic with minimal adverse effects.1 Amoxicillin/
dren treated with 5 days of antibiotics were more likely to
clavulanate is recommended for AOM treatment only in
experience clinical failure.'
the following situations: if a patient presents with concur-
rent bilateral conjunctivitis (suggestive of H. influenzae 8. Explanation: The correct answer is D. AAP guidelines rec-
ommend vaccination with the pneumococcal conjugate and
influenza vaccines according to the schedule set forth by the
Advisory Committee on Immunization Practices for
the prevention of AOM in all children.1 Note that although
TABLE2.1. Recommendations for Initial
the bacteria H. influenzae is associated with AOM, type b
Management of AOM
H. influenzae was associated more with systemic disease
Pain (eg, epiglottitis, pneumonia) as opposed to AOM.
Presentation Age Antibiotica Management
9. Explanation: The correct answer is D. Current AAP
Severe. guidelines recommend against the use of antimicrobial
Unilateral prophylaxis for the prevention of recurrent AOM in chil-
or Bilateral AIJy YES YES dren.1 The potential small benefit of antimicrobial pro-
Non-severe, <6months YES YES phylaxis for AOM does not outweigh the risks of adverse
Unilateral effects from antibiotics, effects of prolonged antibiotic
Additional
~6months use on increasing antimicrobial resistance, and additional
Observation•
cost to patient families. Patients who experience recurrent
Non-severe, <24months YES YES AOM (defined as 3 episodes in 6 months or 4 episodes in
Bilateral Additional 1 year) are candidates for tympanostomy tube placement.
~24months
Observation•
"The decision to manage with additional observation should be REFERENCES
made in conjunction with the patient's family and follow-up should
be ensured at 48 to 72 hours. Anb"bictic::s should be initiated if the 1. Lleberthal AS, Carroll AE, Chonmaitree T, et al. The diag-
c:hild worsens, or fails to improve within 48 to 72 boll1'$. nosis and management of acute otitis media. Pediatrics.
Source: Adapted from reference 1. 2013;131:e964-e999.
8 INFECTIOUS DISEASES: A CASE STUDY APPROACH

2. Uhari M, Mantysaari K, Niemela M. A meta-analytic treatment of acute otitis media in children. Pediatr Drugs.
review of the risk factors for acute otitis media. Clin Infect 2008;10(5):329-335.
Dis. 1996;22(6): 1079-1083. 4. Hoberman A, Paradise JL, Rockette HE, et al. Shortened
3. Fallon RM, Kuti JL, Doern GV, et al. Pharmacodynamic antimicrobial treatment for acute otitis media in young
target attainment of oral ~-lactams for the empiric children. N Engl] Med. 2016;375(25):2446-2456.
3 Acute Bronchitis
Jenana Maker

PATIENT PRESENTATION .,. General


Well-developed female in NAD
Chief Complaint
"I can't stop coughing." .- HEENT
PERRLA. EOMI, TMs intact, moist mucous membranes, mild
History of Present Illness pharyngeal erythem.a present
MT is a 35-year-old female who presents to her primary care
physician with cough for the last 10 days. She explains that her .,. Neck/Lymph Nodes
6-year-old son had fever. runny nose, and cough about 2 weeks
Supple, no lymphadenopathy
ago but got better after a few days. She explains that she got sick
shortly after him and developed nasal congestion, sore throat.
.- Pulmonary
and a productive cough. While her congestion has improved.
she continues to cough and is concerned that her symptoms CTA, no crackles/wheezing, cough present
still persist She denies any fever, clillls. dyspnea, or hemoptysis.
.- Cardiovascular
Past Medical History NSR. no mlr/g
Hypothyroidism. insomnia
.,. Abdomen
Surgical History Soft. non-tender, non-distended, bowel sounds present
None
.- Genitourinary
Deferred
Family History
Father has H'IN and hyperlipidemia. mother has history of .- Neurology
breast cancer and is in remission. One younger sister (age 30).
who is alive and well. A&O X3, CN intact

.- MS/Extremities
Social History
Deferred
Married with two children (ages 6 and 8). works as a dental
hygienist, denies smoking and illicit drug use
Laboratory Findings
Allergies Na =136 mEq/L Hgb= 14g/dL
NKDA K=3.9mEq/L Hct=40%
Cl= 100 mEq/L Pit= 201 x 10'/mm'
Home Medications co2 = 22 mEq/L WBC = 9 x 103/mm'
Levothyroxine 50 mcg orally once daily
BUN= 12 mgldL
Alpraz.olam. 0.25 mg orally nightly as needed for insomnia
Ser = 0.8 mg/dl.
Physical Examination Glu = 98 mgldL

.- Vital Signs
Temp 98.s•F, P 68, RR 16, BP 115173 mm Hg. 0 2 saturation
.,. Rapid lnlluenza Test
98%, Ht 5'9", Wt 63 kg Negative

9
10 INFECTIOUS DISEASES: A CASE STUDY APPROACH

QUESTIONS multiple days and up to 3 weeks. In addition to cough,


patients may experience sputum production with or with-
1. Which of the following is a hallmark symptom of acute
out purulence. Further, some patients may experience mild
bronchitis?
dyspnea, wheezing, and bronchial hyperresponsiveness.
A. Cough
As cough persists, some patients may complain of sub-
B. Fever
sternal or chest wall pain when coughing. Fever is rardy
C. Sputum production
present in patients with acute bronchitis and typically
D. Nasal congestion
indicates influenza or pneumonia (rules out Answer B).
2. Which of the following clinical or laboratory parameters is Sputum production and nasal congestion may or may not
needed to establish the diagnosis of acute bronchitis in this be present (rules out Answers C and D).1.2
patient?
2. Explanation: The correct answer is C. The diagnosis of
A. Sputum culture
acute bronchitis is typically established with a physical
B. Chest X-ray
exam and patient's clinical presentation. Specifically, her
C. Patient's clinical presentation and physical exam
findings of a persistent cough and sputum production
D. Spirometry testing
are indicative of acute bronchitis. A sputum culture is
3. Which of the following microorganisms is the most likely typically not indicated as bacteria are rardy implicated
pathogen responsible for MT's symptoms? in acute bronchitis (rules out Answer A). Chest X-ray is
A. Streptococcus pneumoniae typically reserved for patients with signs/symptoms sus-
B. Mycoplasma pneumoniae picious of pneumonia (rules out Answer B). Spirometry
C. Bordetella pertussis testing is not indicated and is largdy used in patients with
D. Respiratory viruses chronic respiratory conditions such as asthma, COPD, or
interstitial lung disease (rules out Answer D). 1.2
4. Which of the following antimicrobials would be most
appropriate to recommend for MT? 3. Explanation: The correct answer is D. Respiratory viruses
A. Amoxicillin/clavulanic acid 875/125 mg orally twice account for 85% to 95% of all acute bronchitis cases. The
daily most commonly isolated viruses include rhinovirus, influ-
B. Azithromycin 500 mg orally once, then 250 mg orally enza A and B, parainfluenza, respiratory syncytial virus,
once daily and coronavirus. Of note, patients who test positive for
C. Osdtamivir 75 mg orally twice daily influenza virus in the setting of fever and cough may need
D. Antimicrobial therapy is not recommended for MT to be further evaluated for treatment with antiviral therapy.
Fewer than 10% of cases are caused by atypical bacteria
5. Which of the following over-the-counter therapies may be
such as Bordetella pertussis, Chlamydophila pneumoniae, or
most hdpful in alleviating MT's symptoms?
Mycoplasma pneumoniae (rules out Answers A, B, C).3
A. Albuterol
B. Dextromethorphan 4. Explanation: The correct answer is D. Antibiotics such
C. Pseudoephedrine as amoxicillin/clavulanic acid and azithromycin are not
D. Diphenhydramine recommended since bacteria rarely cause acute bronchitis
(rules out Answers A and B). Osdtamivir is an antiviral
6. What is the desired goal of MT's pharmacotherapeutic plan?
but, since it is only active against the influenza virus, it
A. Resolution of cough
would not be recommended for this patient (Answer C is
B. Eradication of infection
incorrect). There is limited evidence of clinical benefit to
C. Preventing hospitalization
support the use of antibiotics in acute bronchitis. Studies
D. Preventing spread of infection into the bloodstream
indicate that antibiotics only modestly reduce severity and
and/or other organs
duration of symptoms (eg, duration or cough and impaired
7. Which of the following conditions is typically present in activity are reduced by only a fraction of a day). On the
patients with chronic bronchitis but absent in patients with other hand, antibiotic use can increase risk of antimicrobial
acute bronchitis? resistance, antibiotic-related adverse effects, and treatment
A. Presence of purulent sputum production cost Still, it is estimated that 70% to 90% of patients with
B. Past medical history of smoking acute bronchitis receive a prescription for an antiviral or
C. Past medical history of multiple respiratory infections antibiotic agent Antimicrobial therapy should be reserved
D. Presence of reversible airflow limitation on spirometry for cases where a bacterial pathogen is isolated or in high-
testing risk patients presenting with symptoms of influenza during
influenza season. If treatment is needed, the duration of
antimicrobial therapy depends somewhat on the antimi-
ANSWERS crobial agent but is typically between 5 and 10 days.3_.
1. .Explanation: The correct answer is A. Cough is the hall- 5. Explanation: The correct answer is B. Dextromethorphan
mark feature of acute bronchitis and tends to persist for may help this patient by relieving her cough symptoms.
CHAPTER 3 I ACUTE BRONCHITIS 11

Another reasonable option would be an expectorant such COPD exacerbations which, in contrast to acute bron-
as guaifenesin that has been shown to significantly reduce chitis, are usually precipitated by an infection. The most
cough frequency and sputum thickness when compared common organisms associated with COPD exacerbations
to placebo. Short-acting [32 agonists such as albuterol are Haemophilus influenzae, Moraxella catarrhalis, Strep-
have been found to only be beneficial in patients with tococcus pneumoniae, and Haemophilus parainfluenzae.
bronchial hyperresponsiveness or wheezing upon presen- Treatment with antibiotics is indicated if the patient has
tation as well as patients with evidence of airway obstruc- increased dyspnea, sputum volume, and sputum puru-
tion. Since MT has no evidence of wheezing or airway lence, or requires mechanical ventilation. Antibiotics for
obstruction, albuterol would not be indicated (rules out COPD exacerbations have been shown to decrease length
Answer A). Further, a nasal decongestant such as pseudo- of hospitalization, recovery time, and treatment failure.
ephedrine would also not be indicated since the patient's Purulent sputum production may be present in both acute
congestion has improved on its own (rules out Answer C). and chronic bronchitis (rules out Answer A). Past medi-
Lastly, there is no indication for an antihistamine such as cal history of respiratory infections varies widely between
diphenhydramine (rules out Answer D).2- 4 patients and is not a recommended parameter to distin-
guish acute and chronic bronchitis (rules out Answer C).
6. Explanation: The correct answer is A. Persistent cough
Presence of reversible airflow limitation on spirometry test-
is the chief complaint for this patient and can be man-
ing is typically associated with asthma (rules out Answer D).5
aged with supportive therapies such as over-the-counter
medications. Since the infection is caused by respiratory
viruses, no antimicrobial treatment for the infection is REFERENCES
available to eradicate it (rules out Answer B). The patient's
1. Blackford MG, Glover ML, Reed MD. Lower respiratory
infection is mild and her age and lack of serious comor-
tract infections. In: DiPiro JT, Talbert RL, Yee GC, Matzke
bidities put her at a very low risk for hospitalization (rules
GR, Wells BG, Posey L, eds. Pharmacotherapy: A Patho-
out Answer C). Acute bronchitis is typically limited to the
physiologic Approach. toed. New York, NY: McGraw-Hill.
respiratory tract with no risk for dissemination (rules out
2. Kinkade S, Long NA. Acute bronchitis. Am Fam Physician.
Answer D).
20 l 6;94(7) :560-565.
7. Explanation: The correct answer is B. Chronic bronchi- 3. Smith SM, Fahey T, Smucny J, Becker LA. Antibiotics
tis is broadly defined as presence of cough and sputum for acute bronchitis. Cochrane Database Syst Rev.
production lasting for 3 months or longer for two con- 20 l 7;6:CD000245.
secutive years or more. Chronic bronchitis is caused by 4. Tackett KL, Atkins A. Evidence-based acute bronchitis
chronic exposure to cigarette smoke or other noxious therapy./ Pharm Prac. 2012;25(6):586-590.
agents and is typically associated with chronic obstruc- 5. Global Initiative for Chronic Obstructive Lung Disease.
tive pulmonary disease (COPD). In contrast to acute Global strategy for the diagnosis, management, and
bronchitis, it is typically not curable but patients can slow prevention of chronic obstructive lung disease 2019
down disease progression with appropriate lifestyle modi- report. Available at https://goldcopd.org/wp-content/
fications, particularly smoking cessation. Patients with uploads/2018/ l l/GOLD-2019-vl. 7-FINAL-l 4Nov2018-
COPD and chronic bronchitis are at an increased risk for WMS.pdf. Accessed April 24, 2019.
This page intentionally left blank
4 Pharyngitis
Franks. Yu Jonathan C. Cho

PATIENT PRESENTATION ., General


Appears uncomfortable, tired, grimacing when swallowing
Chief Complaint
"Mommy, my throat is on fire!" ., HEENT
Anterior cervical lymph nodes enlarged and tender; tonsils
History of Present Illness moist, red, with white exudates
fI' is a 7-year-old Chinese American female, accompanied by
her mother, who presents to the community pharmacy with ., Point-of-Care GAS Rapid Antigen Detection Test
complaints of sore throat and fever, looking for medications Positive
to take to relieve her symptoms. She is fussy and describes
the pain when she swallows as feeling if her throat is "on fire:"
Her symptoms began yesterday morning, and she has only QUESTIONS
tried drinking a pei pa koa syrup containing medicinal herbs
1. What is the most common pathogen responsible for acute
(main active herb is ebn bark) and honey to relieve the sore
bacterial pharyngitis in children?
throat. This provided some relief but the pain has been getting
A. C-Ory~bacterlum diphtlurlae
worse. She did not have a temperature taken, but her forehead
B. Neisserla gonorrhoeae
was hot to the touch. She was not given any medications to
C. Group C streptococcus
relieve the fever. She was dressed with additional clothing and
D. Group A streptococcus
blankets to •sweat the fever out:' but the fever still persisted.
She reports that there may have been other sick classmates. 2. What signs and symptoms in this patient definitely discrimi-
She denies a prior history of sore throat nate betw~n GAS pharyngitis rather than viral pharyngitis?
A. Tonsils with white exudates
Past Medical History B. Temperature 101.9°F
Attention-deficit disorder, re<:Ul'rent otitis media (resolved) C. Pain on swallowing
D. All of the above
Surgical History 3. If GAS is suspected, what age range is typically excluded
None for testing for GAS!
A. Age <3 years
Famlly History B. Age 5-15 years
Non-contributory C. Age 16-64 years
D. Age >65 years
Social History 4. Which of the following antibiotic treatment regimens
Ear tubes at age 2 should be recommended for JT based on patient-specific
factors and general resistance patterns?
Allergies A. Cephalexin 500 mg orally twice daily x 10 days
B. Clindamycin 207 mg orally three times daily x 10 days
Amoxic:illin (throat swelling, difficulty breathing)
C. Azithromycin 354 mg orally once daily x 5 days
D. Clarithromycin 222 mg orally twice daily x 10 days
Home Medications
Methylphenidate ER 18 mg PO daily 5. Which of the following counseling points is not true for the
medication selected above!
Physical Examination A. Store the medication in the refrigerator
B. Rash
., Vital Signs C. Diarrhea
Temp 101.9°F (oral), Ht 4'4", Wt 29.55 kg D. Badly tasting medication

13
14 INFECTIOUS DISEASES: A CASE STUDY APPROACH

6. JT's parent asks what else can be given to help with her suggestive of viral pharyngitis. 1 Clinical signs and symp-
child's discomfort. What do you recommend? toms of pharyngitis may overlap between GAS pharyngitis
A. Aspirin 81 mg orally every 4 to 6 hours as needed for and viral pharyngitis, so diagnosis based on features alone
fever or pain is less accurate. Previous literature referred to scoring
B. Prednisolone 15 mg orally twice daily systems such as CENTOR to establish probability of GAS
C. Acetaminophen 400 mg orally every 4 to 6 hours as pharyngitis, but that is no longer recommended due to
needed for fever or pain the overlap of symptoms. Even in patients with all clinical
D. All of the above features, scoring high on having probable GAS, only 35%
to 50% had confirmed GAS, particularly in children.2- 5 For
7. JT presents back to your pharmacy two more times with
this reason, it is recommended to perform rapid antigen
complaints of sore throat, cough, raspy voice, and positive
detection test (RADT) or throat culture when GAS is sus-
GAS RADT over the next 12 months. What is the most
pected. Testing is not recommended for patients with overt
likely cause of these recurrent cases of GAS pharyngitis?
viral features.
A. Recurrent viral infections, with falsely identified GAS
presence 3. Explanation: The correct answer is A. GAS diagnostic test-
B. Inadequate antimicrobial therapy due to medication ing is not indicated for children < 3 years old due to unlike-
noncompliance lihood of GAS being responsible for pharyngitis in that
C. Recurrent viral infections, while carrying GAS population. However, testing may be warranted if the child
D. New infections from different GAS strains has contact with other children who have a GAS infection.
GAS is responsible for 20% to 30% of sore throat visits in
8. JT's parent asks if JT should get her tonsils taken out to children, most commonly in children age 5 to 15 years of
prevent this from occurring again in the future. What do age during the cold months of winter to early spring. It is
you recommend? less common in adults, being responsible for 5% to 15%
A. Yes of acute pharyngitis cases.6•7 Diagnostic testing by rapid
B. No antigen detection test (RADT) via throat swab is highly
specific (about 95%) and is available as a CUA-waived test
ANSWERS that can be utilized in pharmacies and prescriber offices.
1. .Explanation: The correct answer is D. Group A strepto- RADT has a primary advantage over throat culture in
coccus (GAS, most commonly Streptococcus pyogenes) is that treatment, if indicated, is not delayed. However, due
the most common pathogen responsible for acute bacterial to lower sensitivity of the RADT (70% to 90%) it may be
pharyngitis in children. C. diphtheriae and N. gonorrhoeae reasonable to confirm a negative RADT with throat culture
can cause acute bacterial pharyngitis and can be treated by in children.8•9
antimicrobial therapy, but are rare. Group C streptococcus 4. .Explanation: The correct answer is B. Due to the patient's
is a common cause of acute bacterial pharyngitis in college amoxicillin allergy with anaphylaxis, cephalosporins are
students and adults, but not usually present in children. not recommended for this patient. If her allergy was not
However, viruses such as adenovirus, influenza virus, anaphylactic, cephalosporins ( cephalexin and cefadroxil)
parainfluenza virus, and rhinovirus are the most common can serve as alternative agents. The treatment of choice
causes of acute pharyngitis cases in general. 1 GAS is the for GAS pharyngitis includes penicillin V, penicillin G IM
only pathogen that generally warrants antibiotic therapy (the only injectable option, at a single dose, which is help-
because pharyngitis due to other organisms has no proven ful for adherence), or amoxicillin, in patients who are not
benefit Preventing the exposure of patients to unnecessary allergic to penicillins. While clindamycin, azithromycin,
antimicrobial therapy due to non-GAS pharyngitis is good and clarithromycin are all reasonable alternatives, the
antimicrobial stewardship practice to reduce cost, adverse general resistance to clindamycin is about 1% and resis-
effects, and resistance. Appropriate treatment and eradica- tance to the macrolides is about 5% to 8%.1 Depending on
tion of GAS reduce the development of other conditions local resistance rates, clindamycin may be preferred in this
such as acute rheumatic fever with or without carditis, and patient. For pediatrics, clindarnycin is dosed 7 mg/kg/dose
post-streptococcal glomerulonephritis. 1 three times daily, at a maximum of 300 mg/dose. Based
2. .Explanation: The correct answer is D. Patients with GAS on the patient's weight of 65 lb or 29.55 kg, she requires
typically present with sudden-onset sore throat, pain on 207 mg/dose. Azithromycin is dosed at 12 mg/kg once
swallowing, and fever. Children may also develop head- (maximum dose of 500 mg) followed by 6 mg/kg (maxi-
ache, nausea, vomiting, and abdominal pain. Tonsillopha- mum dose of 250 mg) for the next four days. Clarithromy-
ryngeal erythema with or without exudates, and tender, cin is dosed at 7.5 mg/kg/dose (maximum dose of250 mg)
enlarged anterior cervical lymph nodes are usually present twice daily for 10 days. Note that treatment duration for all
on physical examination. Absence of fever, conjunctivitis, oral regimens is 10 days, except azithromycin.
coryza/rhinorrhea, cough, oral ulcers, discrete ulcerative 5. .Explanation: The correct answer is D. Clindamycin solu-
stomatitis, hoarseness, and viral exanthema are more tion should be stored at room temperature. Common side
CHAPTER 4 I PHARYNGITIS 15

effects include rash or diarrhea. However, these side effects pharyngitis. Tonsillectomy may be considered in rare cases
could also be indicators of more rare but severe adverse in which there are unexplainable, persistent, recurrent epi-
effects, such as erythema multiforme, exfoliative derma- sodes of GAS pharyngitis, though long-term benefits are
titis, Stevens-Johnson syndrome or Clostridioides dif!icile unknown. 1
infection. Adherence is important not only for adults, but
also for children. As pediatric medications are available
in oral liquid dosage form, taste is a factor to consider.
REFERENCES
Taste tests have shown that clindamycin is among the 1. Shulman ST, Bisno AL, Clegg HW, et al. Clinical prac-
most unpalatable of the liquid antimicrobials. Of the three tice guideline for the diagnosis and management of
viable antibiotics for this patient, azithromycin is the most group a streptococcal pharyngitis: 2012 update by the
palatable. 10 Infectious Diseases Society of America. Clin Infect Dis.
2012;55{10):e86-el 02.
6. Explanation: The correct answer is C. Adjunctive therapy
2. Bisno AL, Gerber MA, Gwaltney JM, et al. Practice guide-
with acetaminophen or a nonsteroidal anti-inflammatory
lines for the diagnosis and management of group a strep-
drug (NSAID) is reasonable to manage the fever and
tococcal pharyngitis. Clin Infect Dis. 2002;35(2):113-125.
pain associated with GAS pharyngitis. It is not, however,
3. Choby BA. Diagnosis and treatment of streptococcal
recommended to use aspirin in children due to the risk
pharyngitis. Am Fam Physician. 2009;79(5):383-390.
of Reye's syndrome, or corticosteroids due to potential for
4. Mcisaac WJ, Keller JD, Aufricht P, Vajanka A, Low
adverse effects and minimal efficacy in pain reduction.1
DE. Empirical validation of guidelines for the man-
Acetaminophen for this patient using the over-the-counter
agement of pharyngitis in children and adults. JAMA.
product, manufacturer dosing table, and supplied dosing
2004;291 :587-595.
cup is 400 mg (12.5 mL) given every 4 to 6 hours. It should
5. Kaplan EL, Top FH Jr, Dudding BA, Wannamaker LW.
not be given more than five times a day.11 Alternatively,
Diagnosis of streptococcal pharyngitis: differentiation of
the weight-based dosing is 10 to 15 mg/kg/dose using the
active infection from the carrier state in the symptomatic
160 mg/5 mL oral suspension. Additionally, JT should
child./ Infect Dis. 1971;123:490-501.
discontinue her herbal remedy, as it has not been effective
6. Bisno AL. Acute pharyngitis: etiology and diagnosis.
thus far, and the evidence regarding safety and efficacy is
Pediatrics. 1996;97:949-954.
insufficient.12
7. Ebell MH, Smith MA, Barry HC, Ives K, Carey M. The
7. Explanation: The correct answer is C. While medication rational clinical examination. Does this patient have strep
noncompliance and new GAS infections from community throat? JAMA. 2000;284:2908-2912.
contacts can occur, given the multiple infections while 8. Gerber MA, Shulman ST. Rapid diagnosis of pharyngi-
being GAS-positive as well as symptoms likely indicative tis caused by group A streptococci. Clin Microbiol Rev.
of a viral infection, JT may be a chronic pharyngeal GAS 2004;17:571-580.
carrier (hence the positive GAS RADT) experiencing 9. Tanz RR, Gerber MA, Kabat W, Rippe J, Seshadri R,
recurrent infections of viral origin. Patients identified as Shulman ST. Performance of a rapid antigen-detection
being a chronic pharyngeal GAS carrier generally do not test and throat culture in community pediatric offices:
require antimicrobial therapy as they are unlikely to trans- implications for management of pharyngitis. Pediatrics.
mit GAS pharyngitis to other close contacts and have little 2009;123:437-444.
risk of developing further complications. At times, it may 10. Gee SC, Hagemann TM. Palatability of liquid anti-
be difficult to differentiate new GAS infection from new infectives: clinican and student perceptions and practice
viral infections while being a GAS carrier, so signs and outcomes. J Pediatr Pharmacol Ther. 2007;12:216-223.
symptoms, local epidemiology, age, and season should be 11. Children's Tylenol• [drug facts]. Johnson & Johnson Con-
considered. 1 sumer Inc., Mcneil Consumer Healthcare Division; 2018.
8. Explanation: The correct answer is B. It is not recom- 12. Nin Jiom Pei Pa Koa [drug facts]. Nin Jiom Medicine
mended to perform a tonsillectomy for prophylaxis of GAS Manufactory (HK) Ltd; 2011.
This page intentionally left blank
5 Community-
Acquired
Pneumonia
Sean N. Avedissian Marc H. Scheetz

PATIENT PRESENTATION Home Medications


Chief Complaint Insulin (bolus/basal: -35 units total daily)
Aspirin 81 mg PO daily (cardiovucular protection)
"I have been coughing, have chest pain, and cannot breathe Atorvastatin 20 mg PO daily (cardiovascular protection)
for about 2 days now.·

History of Present Illness Physical Examination


WA is a 40-year-old Caucasian male who presents to the ~ V'lfal Signs
emergency department with a fever, cough. chest pain (wors- Temp 100.8°F, HR 110 bpm, RR 30, BP 125/75 mm Hg. p02
ening when breathing or coughing). and shortness of breath. 93%, Ht 5'9", Wt 70 kg
Normally. he has a fairly active lifestyle as he b:ains for mara-
thons. He has not traveled outside the United States recently.
He states he has "been taking cough medicine at night.. for ~ General
the past 4 days to help him sleep, but it has not been getting Slightly lethargic, mild-mode.rate distress
better. Also, he reports that he has been waking up at night
due to heavy sweating. He states all his symptoms have gotten
~ HEENT
worse in the last 2 days. After being assessed in the ED. WA is
admitted to the medicine unit for further workup. Normocephalic. atraumatic, PERRLA, EOMI. normal mucus
membranes and conjunctiva, adequate dentition
Past Medical History
Diabetes: Type 1 ~ Pulmonary
Diminished breath sounds and crackles (rales) bilaterally
Surgical History
NIA ~ Cardiovascular
NSR, no m.Jr/g
Famlly History
Father has diabetes: Type 1, history of heart attack; mother ~ Abdomen
has hypertension. Soft, non-distended, non-tender, bowel sounds hyperactive

Social History ~ Neurology


Married with no kids. Denies smoking and drinks alcohol Lethargic, oriented to place and person, (-) Brudzinski's sign,
occasionally (weekends, social events) (-) Kernig's sign

Allergies ~ Extremities
NKDA No significant findings

17
18 INFECTIOUS DISEASES: A CASE STUDY APPROACH

~ Vaccines 3. What empiric antibiotic(s) therapy would you start on WA


States he cannot remember all of them. He says he received all for community-acquired pneumonia (CAP)?
his age-related vaccines when younger. Has not received his A. Azithromycin
flu-shot this year as he always forgets to receive it. B. Ceftriaxone
C. Ceftriaxone plus azithromycin
D. Ceftriaxone plus vancomycin
Laboratory Findings E. Piperacillin-tazobactam plus azithromycin
Na = 148, mEq/L Hgb = 14g/dL Ca =8.5 mg/dL 4. Which of the following is not an adverse effects of
K= 4.3mEq/L Hct = 38% Mg= 2.3 mg/dL fluoroquinolones?
CL= 115 mEq/L Plt = 300 x 103 /mm 3 Phos = 4 mgldL A. QTc prolongation
B. Glucose abnormalities
C01 = 28 mEq/L WBC = 18 x 10 mm3 3
AST= 21 IU/L
C. Tendinopathy/tendon rupture
BUN = 20 mgldL Trop <0.01 ng/mL ALT= 35 IU/L D. Aplastic anemia
SCr = 0.9 mg/dL CK = 1.8 ng/mL T-Bili = 0.5 mg/dL
5. As per CAP guideline recommendations, 1 which of the
Glu = 140 mg/dL BNP = 30 pg/mL Alk Phos = 60 IU/L following is not a reason to switch WA to oral antibiotic
Procalcitonin = 0.8 mcg/L therapy?
A. Hemodynamic stability with clinical improvement
~ Chest X-ray B. Ability to ingest medications
Consolidation, pleural effusions present C. Failing IV therapy
D. Normally function gastrointestinal tract
~ Blood Cuhures 6. What would be an appropriate total duration of therapy
NGTD (days) of antibiotic(s) for WA if he is showing clinical
improvement?
A. 4 days
~ Sputum Culture (high-quality)
B. 5 days
Pending C. 7 days
D. 10 days
~ NARES MRSA PCR
7. What vaccine(s) would you recommend (including sea-
Negative
sonal vaccines) that WA receive (assuming he has not
already received them)?
~ Urine Antigen Test A. Annual influenza vaccine + 23-valent pneumococcal
Legionella (negative), Pneumococcal (negative) polysaccharide vaccine (PPSV23)
B. 23-valent pneumococcal polysaccharide vaccine
~ Respiratory Pathogen Panel (RPPJ (PPSV23)
C. 13-valent pneumococcal conjugate vaccine (PCV13)
Negative
D. Annual influenza vaccine + 13-valent pneumococcal
conjugate vaccine (PCV13)
QUESTIONS
1. What is W& CURB-65 score? ANSWERS
A. 1
1. .Explanation: The correct answer is B, i.e., 2. The CURB-
B. 2
65 is one example of a severity-of-illness index that can
c. 3 be useful for assessing the mortality risk and need for
D. 4
hospitalization in patients with community acquired pneu-
2. What findings does WA present with that will aid in the monia. For WA, his CURB-65 score is equal to 2 because
diagnosis of a patient positive for community-acquired he has an RR ~30 breaths/rain and an urea greater than
pneumonia (CAP)? 19 mg/dL.
A. Chest X-ray findings CURB-65 Criteria: 1
B. Blood culture findings Confusion, new onset
C. Urine antigen test findings Urea greater than 19 mg/dL
D. HisPMH Respiratory rate of 30 breaths/min or more
CHAPTERS I COMMUNITY-ACQUIRED PNEUMONIA 19

Blood pressure of90 mm Hg or less sy5tolic or diastolic of Summary of IDSA Guldellnes


60 mm Hg or less
Age 65 years or older MRSA-specific PNA risk Consider adding
Bach criterion counts as 1 point; score ofO = 0.7% mortal- factors vancomycin or linezolid
ity risk.1= 2.1%, 2 = 9.2%, 3 =14.5%, 4 = 40%. Consider • Dialysis (PD or HD)/ for MRSA coverage.
hospitalization if the CURB-65 score is 2 or more. end-stage renal disease
• Injection drug abuse
2. hplanation: The correct answer is A, chest X-ray find- • Prior inftuenza
ings. The consolidation with pleural effusions is consistent • Prior antibiotic therapy
with CAP. As the rest of the choices can provide essential (especially with
information in patients with CAP, they are not clinically fluoroquinolones)
significant in Wfi.s presentation. • Prior MRSA colonization
3. Explanation: The correct answer is C, cefuiaxone plus or infection
azithromycin. Given that the sputum Gram stain is still • Necrotizing or
pending. ceftriaxone would cover the potential typical cavitary PNA
pathogens (i.e., S. pneumoniae w. H. influenzae) for CAP. Gram-negative PNA risk Consider
CAP empiric treatment also includes coverage of atypical factors piperacillin-tazobactam
organisms (C. pneumoniae, M. pneumoniae, L. ptuiumoph-
• Severe COPD or cefepime +
ila). Thus, a macrolide such as azithromycin is recom- • Smoking azithromycin.
mended for added coverage. However, a macrolide should • Alcoholism
not be used as monotherapy due to high S. pneumoniae • Aspiration
resistance rates, especially in patients with comorbidities • Recent use of anbbiotics
such as type 1 DM. The current 2007 guideline lists levo- (within 3 months)
floxacin (tluoroquinolone) as an option (but this was not
a possible answer listed for this question). However, given MRSA-consistent clinical Patients who present
resistance development, adverse effects, and strengthened presentation of PNA with these symptoms
FDA warnings, the usage of fluoroquinolones should suspicious with gene are at high risk for
be reserved for patients who have no alternative treat- for Panton-Valentine toxin-producing MRSA
ment options. It is unclear how the new CAP guidelines leukocidin strains. Addition
(currently in development) will classify fluoroquinolone (necrotizing PNA) of dindamycin or
treatment for CAP. While W& Legionella antigen is nega- • Shock use of linezolid may
tive, you cannot rule out other atypical pathogens at this • Respiratory failure be considered for
time given cultures are still pending, and most atypical • Formation of abscesses and their potential toxin
bacteria are not detectable on Gram stain or cultivatable inlubito effects.u
on standard bacteriologic media. Further, adding van- Souru: Data from .references 1. 2, and 3.
co.mycin is not recommended given the patient does not
present with MRSA-specific risk factors/MRSA-consistent
clinical presentation. W& MRSA PCR is also negative, 4. Explanation: The correct answer is D, aplastic anemia.
which has a high negative predictive value for MRSA PNA. Options A through C are very serious and important
Finally. piperacillin-tazobactam would only be needed if adverse drug effects to be able to identify for fluoroquino-
the patient presented with gram-negative risk factors.3 As lones. Option Bis especially significant for WA given his
per guideline recommendations, it is also important to type 1 DM diagnosis. Option D is less specific for fluoro-
consider the floor the patient is admitted to when deciding quinolones and would be more commonly associated with
on empiric treatment because recommendations will differ an antibiotic such as chloramphenicol.
if treating outpatient vs. inpatient (non-ICU) vs. inpatient 5. .Explanation: The correct answer is C, failing IV therapy.
(ICU). Please refer to guidelines for more information. All other choices are criteria provided by the CAP
20 INFECTIOUS DISEASES: A CASE STUDY APPROACH

guidelines as reasons to consider switching a patient to recommend PPSV23. Annual vaccination remains the
oral therapy. If a patient is failing IV therapy, a discussion primary tool for influenza prevention; everyone at least
about if the antibiotic is appropriately covering the poten- 6 months of age should receive an annual vaccination.
tial pathogen(s) should happen with the health care team. Effectiveness of the influenza vaccination varies year by
The goal is to discharge patients as soon as they are clini- year, depending on how well matched the strains are to
cally stable, have no other active medical problems, and the vaccine components. The complete vaccination recom-
have a safe environment for continued care. Further, it is mendation table can be found in reference 5.
not necessary to keep the patient admitted while receiving
oral therapy if patient is clinical stable.
6. Explanation: The best answer would be B, 5 days. If the
patient is showing clinical improvement, which is specified REFERENCES
in the question, a shorter total days of therapy is recom- 1. Mandell LA, Wunderink RG, Anzueto A, et al.; Infectious
mended. Historically, patients received 7 to 10 days of Diseases Society of America, American Thoracic Society.
treatment for CAP. The 2007 IDSA CAP guidelines1 rec- Infectious Diseases Society of America/American Tho-
ommend that patients receive at least 5 days, depending racic Society consensus guidelines on the management
on clinical stability. Clinical stability includes temperature of community-acquired pneumonia in adults. Clin Infect
~37.8°C, heart rate ~100 beats/min, respiratory rate Dis. 2007;44(suppl 2):S27-S72.
~24 breaths/min, systolic blood pressure :2::90 mm Hg, 2. Gillet Y, Vanhems P, Lina G, et al. Factors predicting
arterial oxygen saturation ~90%, and ability to maintain mortality in necrotizing community-acquired pneumo-
oral intake and normal mental status. Further, patients nia caused by Staphylococcus aureus containing Panton-
should be afebrile for 48 to 7 hours with no more than 1 Valentine leukocidin. Clin Infect Dis. 2007;45:315-321.
CAP-associated sign of clinical instability. More recently, 3. Bernardo K, Pakulat N, Fleer S, et al. Subinhibitory con-
an RC'I04 evaluated decision making for the duration of centrations of linezolid reduce Staphylococcus aureus vir-
therapy in patients with CAP and found that 5 days of ulence factor expression. Antimicrob Agents Chemother.
therapy was safe in the studied population when patients 2004;48:546-555.
met criteria for clinical stability. There are many benefits 4. Uranga A, Espana PP, Bilbao A, et al. Duration of anti-
to shorter duration for CAP treatment, which include biotic treatment in community-acquired pneumonia:
decreased risk of developing colonization with penicillin- A multicenter randomized clinical trial. JAMA Intern
resistant S. pneumoniae when using f3-lactams, decreased Med. 2016;176:1257-1265.
risk of adverse drug effects, including C. difficile infection, 5. Bennett NM, Whitney CG, Moore M, Pilishvili T, Dool-
and improved adherence and decreased cost. ing KL. Use of 13-valent pneumococcal conjugate vaccine
7. Explanation: The correct answer would be A. Given that and 23-valent pneumococcal polysaccharide vaccine for
WA is diabetic (Type 1), the influenza vaccine (received adults with immunocompromising conditions: Recom-
annually) and the PPSV23 vaccine are recommended mendations of the Advisory Committee on Immunization
for those ~19 years. 5 As it is unclear if WA received his Practices (ACIP) (Reprinted from MMWR vol 40, pg 816,
PCV13 vaccine in the past, current ACIP guidelines only 2012). J Am Med Assoc. 2013;309:334-336.
6 Hospital-Acquired
and Ventilator-
Associated
Pneumonia
Stephanie E. Giancola Elizabeth B. Hirsch

PATIENT PRESENTATION Allergies


History of Present Illness NKDA
RW is a 67-year-old man who was mmsported to Acute Care
Medical Center (ACMC) after being struck by a car while Home Medications
crossing First Avenue on foot. He has been hospitalized at Lisinoprillhydroc.hlorothiazide 20/12.5 mg PO daily
ACMC for 6 days and is in the trauma ICU due to several Metformin 1000 mg PO BID
fractures sustained in the motor vehicle accident. The patient No recent antibiotic use within the past 6 months
has been intubated and on mechanical ventilation since
admission 6 days ago. During ICU roWlds the next morning, • Yitai Signs
the nurse notes he has had increased, yellow/green secretions Temp 101.3°F, BP 110/72 mm Hg. HR 80 to 90 bpm, RR 21
overnight. breaths per minute, Sp02 92% on mechanical ventilation
Ht 6'1.. Wt 98 kg
Past Medical History
• Genel'OI
Hypertension
Intubated, no acute distress
Type 2 diabetes mellitus
• HEENT
Family History Normocephalic, atraumatic, EOMI, PERRI.A, MMM, no
Mother died from a myocardial infarction at age 84. Father is adenopathy
still living in nursing home, has past history of prostate cancer.
• Cardiovascular
No m/r/g. normal Sl/S2
Soclal History
The patient is an engineering professor at the State University. • Pulmonary
He is married and lives at home with his wife. He has 2 college- Crackles in the right posterior base with dullness: left lung
age kids attending college outside of the metro area. As per clear
his wife. he has not traveled outside the local area for the past
6 months. but he traveled to Germany about a year ago. There • Abdomen
are no pets in the home. He drinks 2 to 3 beers per week.
Soft, non-tender, non-distended with positive bowel sounds

Current Medications • Genitourinary


Sodium chloride 0.9% (NS) @ 150 mL/hr Urinary (Foley) catheter is in place
Heparin 5000 units subcutaneously q8 h
Insulin infusion titrated to maintain BG 80-110 • Extremitiei
Fentanyl intravenous infusion at 25 mcg/hr LLE in ace wrap and post-op bandages, LUE in post-op
Dex:medetomidine intravenous infusion titrated to RASS 0 to -2 bandages

21
22 INFECTIOUS DISEASES: A CASE STUDY APPROACH

.... Neurology Which antibiotic regimen would be most correct for empiric
Alert and oriented x 2 upon admission (person and place, treatment in RW?
not date) A. Cefepime + tobramycin + vancomycin
B. Ciprofloxacin + ceftriaxone + vancomycin
C. Piperaclllin-tazobactam + vancomycin
Laboratory Findings D. Meropenem + levofloxacin
WBC 12.2 x 105/µ.L (8% bands}, platelets 156 >< 10'/µ.L, E. Meropenem
AST/ALT 54/32 units/L, SCr 1.5 mgldL, lactate 1.1 mmoUL, S. If the patient was not ventilated, what empiric antibiotic
albumin 2.2 g/d.L, INR 1.2, blood glucose (3 most recent q4h regimen would be recommended?
accuchecks = 110, 80, 90 mg/dL) and all other laboratory A. Cefepime + tobramycin + vancomycin
values are WNL B. Cefepime + vancomycin
C. Piperaclllin-tazobactam + levofloncln + linezolid
.... Pertinent Ventilator Settings D. Meropenem + levofloxacin
Fi02: 60% (increased from 40% yesterday) E. Meropenem
PEEP: 8 cm Hp (increased from 5 cm ~O yesterday) 6. Regardless of your recommendation above, the patient
is started on piperacillin-tazobactam 4.5 g IV q6b plus
levofloxacin 750 mg IV daily plw vancomycin 1500 mg IV
QUESTIONS q24h. and is clinically improving on this regimen with low
risk for mortality ( < 15%). Cultures return 48 hours later
1. What signs/symptoms does RW have that are consistent and reveal the following:
with the presentation of pneumonia?
Blood cultures: negative
A. Increased sputum production
Endotracheal aspirate culture: positive for 3+ Pseudo-
B. Increased ventilator settings (Fi02 and PEEP}
monas aeruginosa
C. Crackl~ on auscultation
D. AandB Amikacin s
E. A,B,andC
Cefepime s
2. Which diagnostic procedures would be indicated in RW at Ceftazidime s
the current time? R
Ciprofloxacin
A. Chest X-ray
B. Endotracheal culture Meropenem s
C. Bronchoalveolar lavage (BAL) Piperacillin-tazobactam s
D. AandB Tobramycin s
E. AandC
R = n:sistant; S = sUKeptlble.
3. A portable CXR is obtained on day 6, which shows a new
Which ofthe following definitive antibiotic regimens should
infiltrate in the right lower lobe. Which infectious syn-
now be recommended for RW?
drome would be highest on the differential diagnosis?
A. Discontinue piperacillin-tazobactam, levofloxacin, and
A. Community-acquired pneumonia (CAP)
vancomycin; start meropenem monotherapy
B. Hospital-acquired pneumonia (HAP)
B. Discontinue Ievofloxacin and vancomycin; continue
C. Ventilator-associated pneumonia (VAP)
piperacillin-tazobactam and add tobramycin
D. Ventilator-associated tracheobronchitis (VAT)
C. Discontinue piperacillin-tazobactam, levofloxacin, and
4. The hospital's antibiogram. shows the foll.owing susceptibil- vancomycin; start tobramycin monotherapy
ity data. D. Discontinue piperacillin-tazobactam, levofloxacin, and
vancomycin; start cefepime monothenpy
Percent 7. What total dunti.on of antibiotic therapy would be recom-
Orpniam Drug !U!CeJ?tible mended for RW's VAP?
Staphylococcus Oncillin 68% A. 5 days
aureus Vancomycin 100% B. 7 days
C. lOdays
Pseudomonas Piperacillin-tazobactam 84%
D. 14days
aeruginosa Cefepime 90%
Meropenem 93%
ANSWERS
Ciproftoxacin 78%
1. Explanation: The correct answer is E (i.e., all of the above).
Tobrarnycin 91% Increased sputum production. increased ventilator settings
CHAPTER6 I HOSPITAL-ACQUIRED AND VENTILATOR-ASSOCIATED PNEUMONIA 23

(Fi01 and PEEP), and cracltles on auscultation are all signs the patient's allergies should be verified. In this case, the
or symptoms of pneumonia. Additional signs/symptoms patient has no known drug allergies.
that are consistent with pneumonia include fever, chest
pain, cough, dyspnea, Ieukocytosis, and decreased oxygen
saturation.
TABLE 6.1. Risk Factors for Resistance for YAP
2. Ez:planation: The correct answer is D. A chest X-ray would
be essential for the diagnosis of pneumonia. Since the MDRVAP MDR
patient is hospitalized in the ICU, sputum cultures, blood (MRSA andMDR Pseudomonas
cultures, urinary antigen tests (UAT: for Streptococcus P. aeruginosa) MRSAVAP VAP
pneumoniae and Legionella spp.) could all be ordered. Since • PriorIV • Patients in • Patients in units
the patient is still intubated, an endotracheal culture is now antibiotic use units where where > 10% of
recommended as per the most recent 2016 Infectious Dis- within 90 days >10-20% of gram-negative
eases Society of America (IDSA) and American Thoracic • Septic shock at S. aureus isolates are
Society (ATS) guidelines.• Bronchoscopy would not be thetimeofVAP isolates are resistant to an
indicated at this time since there is no real reason to do this • ARDS methicillin agent being
invasive procedure if no sputum cultures have been done preceding VAP resistant considered for
yet. Rationale for this recommendation is based on the • Five or more • Patients in monotherapy
lack of evidence that invasive (ie., bronchoscopy) micro- days of units where the • Patients in units
biological sampling with quantitative cultures improves hospitalization prevalence of where local
clinical outcomes as compared to noninvasive sampling.1 prior to the MRSAisnot antimicrobial
Noninvasive sampling can also be done rapidly, with fewer occurrence of known susceptibility
complications and resources. VAP rates are
3. Explanation: The correct answer is C. CAP is defined as • Acute renal unknown
pneumonia that developed in the community with symp- replacement • Bronchiectasis
toms beginning prior to presentation and up to 48 hours therapy prior to • Cystic fibrosis
after hospital admission (rules out option A). HAP is YAP onset
defined as pneumonia that occurs 48 hours or more after
admission in non-ventilated patients (rules out option B).
VAT is defined as fever with no other recognizable cause,
with new or increased sputum production, positive endo- 5. Explanation: The correct answer is B. If the patient was
not ventilated, the patient's pneumonia would be classi-
tracheal aspirate culture, and no radiographic evidence
fied as HAP.. Similar to treatment of YAP, the choice of
of nosocomial pneumonia (rules out option D). VAP is
empiric antibiotics to treat HAP depends on risk factors for
defined as pneumonia that arises more than 48 hours after
endotracheal intubation. RW has been intubated for 6 days, resistance. However, risk factors for resistance in HAP are
slightly different from those in VAP, as shown in Table 6.2
has signs/symptoms of pneumonia, and has radiographic
evidence of pneumonia; therefore, answer C is correct. below.1 An antibiotic active against MRSA and 2 anboiot-
ics active against Pseudomonas are indicated if a risk factor
4. Esplanation: The correct answer is A. In patients with
suspected YAP, the empiric antibiotic regimen should
include coverage for Staphylococcus aureus, Pseudomonas
aeruginosa, and other gram-negative bacilli. The choice of TABLE 6.2. Risk Factors for Resistance for HAP I
antibiotics to cover these organisms depends on risk fac- MDRHAP MDR
tors for resistance.• An antibiotic active against methicillin- (MRSAandMDR PleudomotuU
resistant S. aureus (MRSA) is indicated if the patient has P. aeruginosa) MRSAHAP HAP
one of the risk factors listed in the Table 6.1.1 lf no risk
factors are present, then an agent active against methicil- • PriorIV • Patients in • Bronchiectasis
lin-susceptible S. aureus (MSSA) is recommended. Two antibiotic use units where • Cystic fibrosis
antibiotics from different antibiotic classes active against within 90 days >20%of
P. aeruginosa are recommended ifthe patient has one of the • Septic shock S. aureus
risk factors listed in Table 6.1. lf no risk factors are present, • Needfor isolates are
then one antibiotic active against P. aeruginosa is recom- ventilator methic:illin
mended. RW has been hospitalized for 6 days; therefore, support due to resistant
an agent active against MRSA (rules out options D and E) HAP • Patients in
and 2 antipseudomonal agents (rules out options B and C) units where the
are indicated. Answer A is the only option that includes prevalence of
2 antipseudomonal agents and an agent active against MRSAisnot
MRSA. Of note, before starting any antibiotic regimen, known
24 INFECTIOUS DISEASES: A CASE STUDY APPROACH

for resistance is present. RW is hospitalized in a unit where However, older data have shown conflicting results and
> 20% of S. aureus isolate are resistant to oxadllin. No increased frequency of recurrent pneumonia in patients
other risk factors for resistance are present. Therefore, an with VAP receiving short-course therapy due to non-
agent active against MRSA (rules out options D and E) plus lactose fermenting gram-negative bacilli, including Pseudo-
one agent active against P. aeruginosa (rules out options A monas spp. Therefore, the authors of the 2016 guidelines by
and C) are indicated for RW. Answer B is the only option the IDSA and ATS performed their own meta-analysis to
that includes one antipseudomonal agent and an agent answer this question.1 They found no difference in mortal-
active against MRSA. ity, clinical cure, or recurrent pneumonia, including in the
6. Explanation: The correct answer is D. When HAP or subgroup of patients with non-lactose fermenting gram-
VAP is due to P. aeruginosa, monotherapy with an anti- negative bacilli. Therefore, the shorter duration of 7 days of
biotic to which the isolate is susceptible is recommended therapy is recommended, making answer B correct.
rather than combination therapy unless the patient is in
septic shock or is at high risk for mortality, defined as
>25% (rules out option B).2 This patient has a low risk of REFERENCES
mortality (stated as <15% in question) and has a normal 1. Kalil AC, Metersky ML, Klompas M, et al. Management
MAP (85 mm Hg) and normal lactate level and is thus not of adults with hospital-acquired and ventilator-associated
in septic shock. Aminoglycosides are not recommended pneumonia: 2016 clinical practice guidelines by the Infec-
as monotherapy due to their poor lung penetration and tious Diseases Society of America and the American Tho-
lack of studies evaluating the effects of aminoglycoside racic Society. Clin Infect Dis. 2016;63(5):e61-elll.
monotherapy for VAP or HAP (rules out option C). Both 2. Kumar A, Safdar N, Kethireddy S, Chateau D. A survival
meropenem and cefepime will effectively treat this patient's benefit of combination antibiotic therapy for serious infec-
infection. To choose between these two options, antimicro- tions associated with sepsis and septic shock is contingent
bial stewardship principles should be considered. When only on the risk of death: a meta-analytic/meta-regression
recommending definitive therapy, de-escalating therapy to study. Crit Care Med. 2010;38(8):1651-1664.
the narrowest spectrum agent that will effectively treat the 3. Chastre J, WolffM, Fagon JY, et al Comparison of 8vs15 days
infection is generally recommended. In this case, cefepime of antibiotic therapy for ventilator-associated pneumonia in
is narrower in spectrum of activity than carbapenems and adults: a randomized trial. /AMA. 2003;290(19):2588-2598.
choosing meropenem carries higher risk for the develop- 4. Capellier G, Mockly H, Charpentier C, et al. Early-onset
ment of carbapenem resistance, which has limited treat- ventilator-associated pneumonia in adults randomized
ment options (rules out A). Therefore, answer D is correct. clinical trial: comparison of 8 versus 15 days of antibiotic
7. Explanation: The correct answer is B. Several studies have treatment. PLoS One. 2012;7(8):e41290.
assessed the optimal duration of therapy for VAP by com- 5. Pugh R, Grant C, Cooke RP, Dempsey G . Short-course
paring short durations (7-8 days) to longer durations versus prolonged-course antibiotic therapy for hospital-
(10-15 days). 3- 5 Most data showed no difference in mortal- acquired pneumonia in critically ill adults. Cochrane
ity, clinical cure/treatment failure, or hospital length of stay. Database Syst Rev. 2015(8):CD007577.
7 Cystitis
Kristy M. Shaeer

PATIENT PRESENTATION ... General


Thin, well-nourished female, lying in bed, NAD, AAO x 3
Chief Complaint
"I have painful and frequent urination:' """ HEENT
EOMI, PERRLA, normocephalic, no pharyngealexudate. Neclc,
History of Present Illness supple. Thyroid palpable, no nodules. No lymphadenopathy
VZ is a 19-year-old Hispanic female who presents to the urgent
care walk-in clinic with dysuria and polyuria for 3 days duntion. ... Pulmonary
CTAB without wheezing or crackles
Past Medical History
Seasonal allergies ... Cardiovascular
NSR. no mlr/g
Surgical History
... Abdomen
None
Soft, non-distended, non-tender, positive bowel sounds hyper-
active, no rebound or guarding
Family History
Mother has HTN; father has HTN, type 2 diabetes mellitus, """ Genitourinary
and dyslipidemia
Normal female genitalia. complaints of dysuria, denies hema-
turia. No malodorous discharge noted from vagina
Soclal History
Currently attending college as a sophomore. Smokes tobacco ... Neurology
and marijuana cigarettes socially on weekends for the past PERRLA. no focal deficits noted
2 years; drinks alcohol (beer and wine) socially on week-
ends for the past year. Sexually active with her boyfriend of ... Ex.tremltleJ
8 months; uses condoms
Edema present in lower extremity bilaterally. Pedal pulses
palpable
Allergies
Sulfas-rash and blisters """ Baclc
No tenderness to palpation on lower lumbar region
Home Medications
Ethinyl estradiol/etonogestrel 0.015 mg/0.12 mg unwrap and Laboratory Findings
insert one ring intravaginally and remove every 21 days then
Dipstick. Urinalysis:
repeat 7 days later
Cetirizine 10 mg PO daily PRN seasonal allergies Macroscopic: urine midstream, clean catch. Yellow, cloudy,
Triamcinolone acetonide 55 mcglspray 1-2 sprays EN PRN large leukocytes, positive nitrites, urine pH = 8, urine
seasonal allergies hemoglobin, protein, glucose, ketones, and bilirubin
Phenaz.opyridine hydrochloride 200 mg PO TID after meals negative. specific gravity = 1.012
Microscopic: WBCs >100, RBCs 0, squamous epithelial
Physical Examination cells 0, few WBC clumps
... Vital Signs
Temp 98.4°F, P 62, RR 10 breaths per minute, BP 112/82 mm ... Urine Gram Stain
Hg, p02 98%, Ht 5'6", Wt 52.3 kg Many gram-negative rods

25
26 INFECTIOUS DISEASES: A CASE STUDY APPROACH

QUESTIONS Cefotetan S4
Cefoxitin S4
1. Which of the following symptom(s) is/are suggestive of
cystitis? Select all that apply. Cefpodoxime S0.25
A. Flank pain Ceftazidime Sl
B. Dysuria
Ceftriaxone Sl
C. Polyuria
D. Fever Cefuro:xime axetil 4
Cefuroxime-Sodium 4
2. Which components of VZ's urinalysis are often suggestive
of a urinary tract infection (UTI)? Select all that apply. Ciprofloxacin S0.25
A. Specific gravity Ertapenem S0.5
B. Leukocyte esterase Gentamicin Sl
C. pH
Imipenem S0.25
D. Nitrites
E. RBCs Levofloxacin S0.12
Meropenem S0.25
3. Which empiric agent would be most appropriate for VZ's
cystitis? Nitrofurantoin Sl6
A. Amoxicillin Piperacillin/Tazobactam S4
B. Levofloxacin Tetracycline Sl
C. Aztreonam
Tobramycin Sl
D. Trimethoprim-sulfamethoxazole
E. Nitrofurantoin Trimethoprim/Sulfamethoxazole S20

4. How many day(s) would be the most appropriate duration A. Amoxicillin


of therapy? B. Nitrofurantoin
A. 1 C. Cefepime
B. 3 D. Cephalexin
c. 5 E. Trimethoprim-sulfamethoxazole
D. 7
7. VZ has been taking phenazopyridine for the last 24 hours.
E. 10
Which of the following are appropriate counseling points
5. The urinalysis and culture is suggestive of which of the fol- about this over-the-counter medication? Select all that apply.
lowing organisms? A. Avoid using for more than 48 hours longer with a total
Urine culture: gram-negative lactose-fermenting rods duration of 72 hours
A. Escherichia coli B. Avoid using for more than 24 hours longer with a total
B. Acinetobacter baumanii duration of 48 hours
C. Enterococcocus faecalis C. Produces a red to orange discoloration of the urine and
D. Staphylococcus aureus sclera leading to staining of clothing and contact lenses
E. Stenotmphomonas maltophilia D. Produces a bluish green discoloration of the urine and
sclera leading to staining of clothing and contact lenses
6. Although unnecessary, the provider sent the urine sample E. Not intended to treat a UTI and will help minimize
for culture and the following susceptibilities to various dysuria
agents. Which of the agents below is the most appropriate
option for treatment?
ANSWERS
Susceptibility Report 1. .Explanation: Correct answers are B and C. Dysuria,
Amikacin S2 polyuria, urgency, and suprapubic pain are symptoms of
cystitis. Flank pain and fever are commonly seen in pyelo-
Amoxicillin/Clavulanate S2
nephritis.1 Uncomplicated cystitis is defined as urgency,
Ampicillin S2 frequency, dysuria, suprapubic pain/tenderness, in an oth-
Ampicillin/Sulbactam S2 erwise healthy, nonpregnant woman who lacks fever, flank
pain, tenderness, and vaginal discharge. 1.2
Aztreonam Sl
2. .Explanation: Correct answers are B, C, and D. 3 Leukocyte
Cefazolin S4
esterase is an enzyme that is produced by neutrophils
Cefepime Sl and may indicate pyuria associated with a UTI. Urinary
Cefotaxime Sl pH can range from 4.5 to 8 but normally is slightly acidic
CHAPTER 7 I CYSTITIS 27

(eg, 5.5-6.5) because of metabolic activity. Alkaline urine (n = 240/263 women) vs. 95% (n = 232/245 women).6
can be seen in patients with a UTJ.2.3 Nitrites are not typi- Some beta-lactam agents can be prescribed for 5 to 7 days
cally fowid in urine and result when bacteria reduce urinary only if susceptibilities are known. 1 The 7- to 10-day dura-
nitrates to nitrites. Not all bacteria are capable of this con- tion is excessive and reserved for other types of UTis.1
version, thus a positive nitrite is helpful but a negative test Fluoroquinolones have also demonstrated equivalent cure
does not exclude a UTI. Organisms from the Enterobacte- rates for a 3 days vs. 7 days and with higher adverse effects
riaceae family (eg, E.coli, Klebsiella spp., Enterobacter spp., in the longer treatment group.1
Proteus spp.) and Staphylococcus spp., and Pseudomonas
5. Explanation: Correct answer is A. E. coli is characterized
spp. reduce nitrate to nitrite in the urine. Urinary specific
as a gram-negative, lactose fermenting rod or bacilli. It
gravity correlates with urine osmolality and gives important
is the most common pathogen (eg, 75-95%), associated
insight into a patient's hydration status. It also reflects the
with uncomplicated UTis followed by other organisms in
concentrating ability of the kidneys. Normal range is typi-
the Enterobacteriaceae family (eg, Proteus mirabilis and
cally 1.003 to 1.030. RBCs can be detected in a variety of
conditions and is not specific to diagnosis of a UTI.
Klebsiella pneumoniae). 1 A. baumanii and S. maltophilia
are characterized as nonlactose-fermenting gram-negative
3. Explanation: Correct answer is E.1 Nitrofurantoin is an rods and uncommonly associated with uncomplicated
appropriate first-line empirical agent for treatment of cys- UTis. S. aureus is characterized as coagulase-positive,
titis. Nitrofurantoin has many attractive characteristics that gram-positive cocci. The presence of S. aureus in the urine
make it a first-line option such as its low cost, availability as warrants further assessment since this is not a normal
an oral agent, tolerability, minimal resistance to common etiology for uncomplicated UTis and could represent a
urinary pathogens, and less likely to cause collateral dam- hematogenously spread infection. Remember, UTis are
age (eg, more focused spectrum of activity and less adverse caused by the ascending spread of an organism into the
effects on normal flora). Amo:xicillin and ampicillin should genitourinary tract. E. faecalis is characterized as gamma
not be used for empirical treatment given the relatively hemolytic gram-positive cocci in pairs and chains.
poor efficacy and high rates of resistance. Levofloxacin
is a broad-spectrum agent that is second-line and typi- 6. Explanation: Correct answer is B. Although susceptible,
cally unnecessary as empiric therapy for a healthy young the patient has an allergy to trimethoprim-sulfamethoxa-
woman with cystitis. Fluoroquinolones are second-line zole. Amoxicillin and cephale:xin also demonstrate suscep-
(eg, alternative) agents given rising rates of resistance, tibility; however, beta-lactams have lower rates of efficacy
risk of collateral damage (eg, ecological adverse effects and require longer durations of therapy that may lead to
of antimicrobial therapy), and reserved for more serious more adverse effects.1 In some scenarios it may be neces-
infections. Aztreonam is an intravenous agent that should sary to use amoxicillin or cephale:xin, but VZ does not
be reserved for a hospitalized patient with a severe beta- have any contraindications to nitrofurantoin. The goal is to
lactam allergy. The patient has a sulfa allergy; therefore, use a narrow-spectrum agent to treat the E. coli-associated
trimethoprim-sulfamethoxazole should be avoided. In cystitis and amoxicillin, nitrofurantoin, and cephalexin are
addition, empiric trimethoprim-sulfamethoxazole should viable options. The spectrum of activity increases from
be avoided if resistance prevalence is known to exceed 20% amoxicillin to cephalexin and nitrofurantoin. Nitrofuran-
or if the patient used it for UTI in previous 3 months. toin may demonstrate activity against multi-drug-resistant
organisms (eg, ESBL-producing Enterobacteriaceae, MRSA,
4. Explanation: Correct answer is C. 1,4-<; Nitrofurantoin is pre-
and VRE).7
scribed for 5 days. Trimethoprim-sulfamethoxazole can be
prescribed for just 3 days to eradicate cystitis. Hooton et al. 7. Explanation: Correct answers are B, C, and E. Treatment of
reported that after 6 weeks, women treated for cystitis had a urinary tract infection with phenazopyridine should not
82% (n = 32/39) cure with trimethoprim-sulfamethoxazole exceed 2 days because there is a lack of evidence that the
for 3 days compared with 61 % (n = 22/36) with nitrofu- combined administration of it with an antibacterial provides
rantoin (P = 0.04).4 Gupta et al compared a 3-day course greater benefit than administration of the antibacterial alone
of trimethoprim-sulfamethoxazole with a 5-day course after 2 days.a In addition, phenazopyridine is a urinary anal-
of nitrofurantoin in women with cystitis. The overall gesic used to provide relief for dysuria and are more ben-
clinical cure rate at 30 days was 79% (n = 117/148) in the eficial in patients with dysuria without a UTI.2.8 The use of
trimethoprim-sulfamethoxazole group and 84% among the phenazopyridine beyond the recommended 2-day duration
nitrofurantoin group (nonsignificant difference of-5%, 95% can increase the risk of side effects. Headache, rash, pruri-
confidence interval: 4-13%).5 Fosfomycin can be given as a tus, gastrointestinal disturbance, and an anaphylactoid-like
single dose and has comparable efficacy to other commonly reaction have been reported.a Methemoglobinemia, hemo-
used agents for cystitis (average: 93%; range: 84-95%).1.6 lytic anemia, and renal and hepatic toxicity have also been
A single dose of fosfomycin was compared to a 7-day course reported, but this was associated with extended or chronic
of nitrofurantoin, which demonstrated that the early clini- use. The use of phenazopyridine as an analgesic can hide
cal response rates (cure or improvement at 5-11 days after or mask symptoms of the UTI that may indicate it is not
starting therapy) were not significantly different at 91 % improving.
28 INFECTIOUS DISEASES: A CASE STUDY APPROACH

REFERENCES 5. Gupta K, Hooton TM, Roberts PL, et al. Short-course


nitrofurantoin for the treatment of acute uncomplicated
1. Gupta K, Hooton TM, Naber KG, et al. International clin-
cystitis in women. Arch Intern Med. 2007;167:2207-2212.
ical practice guidelines for the treatment of acute uncom-
6. Stein GE. Comparison of single-dose fosfomycin and a
plicated cystitis and pyelonephritis in women. Clin Infect
7-day course of nitrofurantoin in female patients with
Dis. 2011;52:e103-e120.
uncomplicated urinary tract infection. Clin Ther. 1999;
2. Sobel JD, Kaye D. Urinary tract infections. In Mandell
21:1864-1872.
GL, Bennett JE, Dolin R, eds. Mandelt Douglas, and Ben-
7. Huttner A, Stewardson A. Nitrofurans: Nitrofurazone,
nett's Principles and Practice of Infectious Diseases. Phila-
furazidine, and nitrofurantoin. In M. L. Grayson, ed.
delphia, PA: Churchill Livingstone; 2010. Available at
Kucers' The Use of Antibiotics. Boca Raton, FL: CRC
http://search.ebscohost.com/login.aspx? direct= true&db
Press; 2018. Available at https://online.vitalsource.com/#/
=nlebk&AN=45876l&site=eds-live.
books/9781351648158/cfi/6/260!/4/616/2@0:54.4
3. Simerville JA, Maxted WC, Pahira JJ. Urinalysis: a com-
8. Pyridium (phenoazopyridine) [package insert]. Bridge-
prehensive review. Am Fam Physician. 2006;74:1153-1162.
water, NJ: Gemini Laboratories, LLC; 2019.
4. Hooton TM, Winter C, Tiu F, et al. Randomized com-
parative trial and cost analysis of 3-day antimicrobial
regimens for treatment of acute cystitis in women. JAMA.
1995;273:41-45.
8 Pyelonephritis
Marylee V. Worley

PATIENT PRESENTATION ., General


Mild distress. nontoxic appearing
Chief Complaint
"I have severe back pain and it hurts when I urinate.•
., HEENT
Atraumatic, pupils equal round and reactive to light and
History of Present Illness
accommodation, moist mucosa. normal pharynx. normal ton-
KJ is a 58-year-old female who presents to the emergency sils and adenoids, normal tongue
department (ED) with complaints of fever, chills, dysuria,
urgency, and back pain. Upon physical exam CVA tender-
ness is noted; no other significant physical findings. She has ., Pulmonary
a fever of 101.2°F; however, she is hemodynamically stable Normal chest wall expansion; no rales, no rhonchi, no
in the ED. wheezing

Past Medical History ., Cardiovascular


Hypertension X 10 years, congestive heart failure, hyperlipid- Regular rate and rhythm, no murmurs, no gallops, normal Sl
emia, type 2 diabetes mellitus andS2

Surgical History ., Abdomen


None Soft. non-tender, non-distended. normal bowel sounds in all
quadrants. no hepatosplenomegaly
Social History
., Genitourinary
Married, livet at home with. husband and has 2 adult children
who do not live at home No incontinence, complains of dysuria

Allergies ., Neurology
Penicillins (reported a rash as a child) No headache, focal numbness or weakness, dizziness, or
seizures
Home Medications
., Nlusculoskeletal
Llsinop.ril 40 mg PO daily
Carvedilol 6.25 mg PO BID CVA tenderness noted. normal ROM in upper and lower
Furosemide 20 mg PO daily extremities, no swelling, no joint eryth.ema; lntegumentary:
Atorvastatin 40 mg PO daily warm, dry, pink, with no rash, purpura, or petechia
Metformin 500 mg PO BID
Laboratory Findings
Physical Examination Na= 140 mEq/L BUN = 26 mgldL Hgb = 13.2 g/d.L
., Vital Signs K = 3.8 mEq/L SCr =1.0 mg/dL Hct.=36%

Temp 10l.2°F, P 89, RR 18 breaths per minute, BP 139/73 mm.Hg, Cl= 98 mEq/L Glucose = 161 mgldL Ph =280 x 103/mm s
Ht 5'4", Wt 78 kg C02=26 mEq/L WBC = 14.2 x tc}'/ mm3

29
30 INFECTIOUS DISEASES: A CASE STUDY APPROACH

QUESTIONS
..
1. What laboratory tests are recommended for patients with
suspected pyelonephritis?
A. Urine culture
ll ;z .....
"" ~ ~ :a m~ ~ ~ ;.: Ii! - ~
B. CRP
C. ESR
D. Urine osmolality l i ...
"' "'
.. ; !il. . s:
..... i it ; - ~ i

2. What is a likely sign or symptom of pyelonephritis?


A. Bade pain i -. : § "'"" ; § §!

u
i - - - - :I
B. Dysuria
C. Fever
D. All of the above ....
..... : ~ ~ so ~ ; &I ~ s: ~ t -
3. Which of the following findings in Kfs urinalysis are con-
sistent with urinary tract infections?
A. Many epithelial cells
B. Negative nitrite
C. Large leukocyte esterase
D. Yellow color
I- ...... ~ - It <#:. :Iii ::it - .... - .... -

Urinalym from KJ J r!. § § § .....


"'
§ §! .
"'
§ s: § : !il.

Component Value Range&: Units 1


Color Yellow l ~ : g =: ~ ~ i::: ij! ~ ::i ~ ~

Transparency
Specific gravity
Cloudy
1.009 1.005-1.030 1......
~ ;:
0
~ ~ I:\ ;;;: s: ...... 51 I:\ - ....... ......
pH 5.0 5.0-8.0
Protein
Glucose
Ketones
Negative
Negative
Negative
Negative, mg/dL
Negative, mgldL
Negative, mgldL
I.. - - 8 § § ~ § § !t 0
~
0
~ !J: .... ~

Bilirubin
Blood
Negative
Negative
Negative
Negative
I.. - ~
§ "' . ..."' ...... s: "'.... ...... ~ §! - ......
Nitrite Negative Negative !
0.2-1.0 mg/dL s.. § ...
l
Urobilinogen 0.2 l:l ~ ;;;: s: "'..... ~
;D
"' ~ ' :a ~

Leukocyte esterase Large Negative


WBC
RBC
>SO
0-2
None seen/HPF
None seen/HPF
I iz
0
~ § § ~ .."' s: ~
...... ~ §! : ~
~
i
Bacteria
Epithelial cells
Many
Many
None seen/HPF
None seen/HPP
I - - Sr\' - - ~
....... ~
- i - - - iIi
~
0

~
~

Hospital Antiblogram J - ;a
§ "' . ... .- s: ..
..... 0
~ ;;: § .... .."' f
4. Which empiric antimicrobial therapy is most appropriate
for KJ based on the most likely pathogen and the hospital
II l~
antibiogram provided above?
A. Posfomycin 3 g oral once
so - -- - ~ ~ ~ - i -
~
B. Nitrofurantoin 100 mg orally twice daily for 5 days ?j ....
2 "' (t ...
"' !
..,~ ~
i i !:! I
....
Ill :cl
~
I'"

C. Levofloxacin 500 mg IV every 24 hours


D. Ceftriaxone 1 g IV every 24 hours
h 11d1 ~ 1 •~dn
Ill IJJi it i ffIiI JI!
CHAPTER 8 I PYELONEPHRITIS 31

5. If preliminary blood cultures report lactose-fermenting, (C-reactive protein) and ESR (erythrocyte sedimentation
gram-negative bacilli. what is the most common organ- rate) are both non-specific markers of inflammation that
ism associated with UTis which is known to have these can be detected in the blood; however, both are not uti-
characteristics? lized for the management of patients with pyelonephritis.
A. Eschmchia coli The response to therapy should be assessed and measured
B. Pseudomonas aeruginosa based on resolution of the patient's signs and symptoms of
C. Stenotrophomonas maltophilia the infection based on their original clinical presentation
D. Haemophilus injluenzae (eg, If the patient presented febrile initially, has the patient
defervesced?). Urine osmolality is not utilized for the man-
6. If the culture returns with the below susceptibility results,
agement of urinary tract infections and normally part of
which antimicrobial is the most appropriate option for KJ~
the diagnostic workup for certain electrolyte disturbances
A. Tigecycline
such as hyponatremia in a euvolemic patient.
B. Ertapenem
C. Fosfumycin 2. Explanation: The correct answer is D. Back pain, dys-
D. PiperacillinJTazobactam uria, and fever are all potential signs or symptoms of
pyelonephritis. Upper urinary tract infections involve
Miaobiology Results: Blood cultures (2/2 collected
the kidney and are referred to as pyelonephritis, which
on day 1): Escherichia coU
can lead to patients experiencing lower flank pain that is
MIC (mcglmL) Interpretation often expressed by patients more generally as back pain.
The physical exam finding of costovertebral angle (CVA)
Levofloxacin 0.25 s tenderness may be documented, which represents pain
Cefazolin >64 R around the kidneys. Symptoms from cystitis are often also
Ceftriaxone >64 R present in patients presenting with pyelonephritis as most
infections are ascending, meaning they travel from the
Cefepime >64 R
bladder to the kidneys. Lower tract infections including
ESBL + Positive cystitis (bladder}, urethritis (urethra}, prostati.tis (pros-
PiperadllinJTazobactam 8 s tate gland), and epididymitis will lead to symptoms such
as dysuria. urgency, frequency, nocturia, and suprapubic
Tigecycline 0.5 s heaviness. In elderly patients presenting with UTls, there
Meropenem 0.5 s may not be specific urinary symptoms, but they may pres-
Gentamicin 1 s ent with altered mental status, change in eating habits, or
gastrointestinal symptoms.
Urine Cultures: Eschmchia coli (same as above)
3. Explanation: The correct answer is C. The finding of large
7. Based on the same culture results above, which of the fol- leukocyte esterase is consistent with urinary tract infec-
lowing antimicrobial options would be appropriate if the tions as it is an indicator that the white blood cells are
patient was allergic to carbapenemsf actively making enzymes in response to a posSiole infec-
A. Moxifl.oxaci.n tion. Leukocyte esterase is found in primary neutrophil
B. Levofloxacin granules and indicates the presence of WBCs, which when
C. Meropenem detected in urine is called pyuria. The detection of WBCs
D. Ceftrlaxone upon microscopic examination is also indicative of pyuria.
The nitrite test is used to detect the presence of nitrate-
8. What is the appropriate duration of treatment for uncom-
plicated pyelonephritis? reducing bacteria in the urine (eg. E. coli). Nitrites can be
negative even if there is an infection; however, a positive
A. 7-10 days
nitrite would be more consistent with a UTI due to the fact
B. 14 days
C. 3 da}'! that the most common organisms that cause UTI (Entero-
bacteriaceae) are normally nitrite reducers. The finding of
D. Both A or B could be appropriate depending on the
many epithelial cells is indicative that this was not a •c1ean
antimicrobial prescribed
catch~ which is neither consistent nor nonconsistent with
a UTI as it normally represents a contaminated sample.
Obtaining a midstream clean catch is the preferred method
ANSWERS fur urine collection for urine cultures. Patients need to
1. Explanation: The correct answer is A. Urine samples that be instructed on the proper collection technique, which
are a clean-catch, mid-stream or from a catheterized urine involves cleaning (normally with a moist wipe) the ure-
sample should be sent for culture and susceptibility testing. thral opening area and discarding the initial 20 to 30 mL
This should be performed for all patients with pyelone- of urine. followed by collection of the urine specimen.
phritis in order to tailor the empiric therapy based on the The color of the urine is not normally considered with the
resistance pattern of the patient-specific uropathogen. CRP diagnosis of a UTI.
32 INFECTIOUS DISEASES: A CASE STUDY APPROACH

4. Explanation: The correct answer is D. Ceftriaxone is although reporting as susceptible, is not appropriate to
the most appropriate empiric therapy for treatment of treat infections in the urinary tract due to the pharma-
pyelonephritis based on the susceptibility rates. 1 The most cokinetic concerns with using tigecycline for UTI as only
common organism that causes uncomplicated UTis is 22% of the total dose is excreted unchanged. Fosfomycin is
Escherlchia coli, and according to this antibiogram, Esch- currently only available orally in the United States, and the
erichia coli is only 48% susceptible to levofloxacin and this oral formulation does not achieve high enough concentra-
is why it would not be the correct answer. Nitrofurantoin tions in the blood to treat bacteremia.
and fosfomycin orally are both only indicated for treatment
7. Explanation: The correct answer is B. Levofloxacin would
of cystitis due to the lack of sufficient concentrations in the
be appropriate for invasive ESBL infections that show sus-
kidneys that would be necessary to treat pyelonephritis.
ceptibility, in the absence of the option to treat with the
5. Explanation: The correct answer is A. Escherichia coli is drug of choice for this type of infection, which is a carbape-
the most common organism to cause an uncomplicated nem. Meropenem is a carbapenem, so this would not be
UTI. All four answer choices listed share the morphology appropriate in patients with a carbapenem allergy. Ceftri-
of gram-negative bacilli. Haemophilus influenzae is more axone is not listed as a susceptible option according to this
commonly associated with upper respiratory tract infec- patient's culture results and moxifloxacin is not appropriate
tions and not a normal pathogen for UTis. Stenotroph- for treatment of UTis due to low urinary concentrations.
omonas maltophilia is associated with infections related to
8. Explanation: The correct answer is D. Depending on the
devices or indwelling catheters due to biofilm formation.
antimicrobial prescribed, there is evidence to support a
Stenotrophomonas maltophilia is not commonly implicated
treatment duration with fluoroquinolones for 7 days of
in an uncomplicated UTI and can be rarely involved in
treatment and trimethoprim-sulfamethoxazole for 14 days
nosocomial UTis. Pseudomonas aeruginosa can cause
UTis; however, it is not one of the most common causes of or a beta-lactam for 10 to 14 days of treatment. 1
uncomplicated UTis, whereas empiric coverage for Pseu-
domonas aeruginosa could be considered in patients with REFERENCES
obstruction, foreign body, chronic indwelling catheter, or
1. Gupta K, Hooton TM, Naber KG, et al. International
complicated UTis.
clinical practice guidelines for the treatment of acute
6. Explanation: The correct answer is B. Ertapenem is the uncomplicated cystitis and pyelonephritis in women: a
correct answer based on the culture findings ofESBL-pro- 2010 update by the IDSA and ESMID. Clin Infect Dis.
ducing E. coli. This patient's allergy to penicillin, although 2011;52(5):e103-e120.
it was not reported as anaphylactic, would make piper- 2. Harris PNA, Tambyah PA, Lye DC, et al. Effect of piper-
acillin/tazobactam not an acceptable choice, in addition acillin-tazobactam vs meropenem on 30-day mortality for
to growing evidence that demonstrates inferiority com- patients with E. coli or Klebsiella pneumoniae bloodstream
pared to carbapenems for treatment of bloodstream infec- infection and ceftriaxone resistance: A randomized clini-
tions caused by ESBL-producing organisms.2 Tigecycline, cal trial. JAMA. 2018;320(10):984-994.
9 Bacterial Meningitis
Jonathan C. Cho

PATIENT PRESENTATION Citalopram 20 mg PO daily


Divalproex sodium 500 mg PO BID
Chief Complaint Purosemide 20 mg PO daily
"'I have severe headaches and fevers:' Levothyrmine 88 mcg PO daily
Levetiracetam 500 mg PO BID
History of Present Illness Lisinopril 20 mg PO daily
DJ is a 54-year-old Caucasian female who presents to the
emergency department with worsening headache. neck pain, Physical Examination
and back pain of2 days duration. She also complains oflow- ..- Vital Signs
grade fevers and chills that developed over the past 24 hours.
Temp 101.2°F, P 72, RR 23 breaths per minute, BP 162/87 mm
Her son, who is present during her exam, states that she seems
Hg, p02 91 %, Ht 5'3", Wt 56.4 kg
more lethargic and has difficulty maintaining her balance. In
addition, she reports 3 to 4 episodes of nausea and vomiting.
..- General
Lethargic, female with dizziness and in mild to moderate
Past Medical History distress.
CHF. COPD, HTN, epilepsy, stroke, hypothyroidism, am:iety
..- HEENT
Surgical History Normocephalic, atraumatic, PERRLA, EOMI, pale or dry
Hysterectomy, cholecystectomy mucous membranes and conjunctiva. poor dentition

Family History ..- Pulmonary


Father had HTN and passed away from a stroke 4 years ago; Diminished breath sounds and crackles bilaterally.
mother has type 11 DM and epilepsy; brother has HTN
..- Cardiovascular
Social History NSR, no rnJr/g
Divorced but lives with her two sons who are currently
attending college. Smokes Y.a ppd x 27 years and drinks alco- ..- Abdomen
hol occasionally. Soft. non-distended, non-tender, bowel sounds hyperactive

Allergies ..- Genitourinary


NKDA Normal female genitalia, no complaints of dysuria or
hematuria
Home Medications
..- Neurology
Advair 250 mcg/50 mcg 1 puff BID
Albuterol metered-dose-inhaler 2 puffs q4h PRN shortness Lethargic, oriented to place and person, (-) Brudzinski's sign,
of breath ( +) Kernig's sign
Alprazolam 0.5 mg PO daily
Aspirin 81 mg PO daily ..- Extremities
Atorvastatin 20 mg PO daily Pedal edema on lower extremities, petechial lesions on lower
Carvedilol 6.25 mg PO BID and upper extremities

33
34 INFECTIOUS DISEASES: A CASE STUDY APPROACH

~ Back 4. Which empiric antimicrobial therapy is most appropriate


Tenderness to palpation on lower lumbar region for DJ?
A. Ampicillin + cefotaxime
B. Vancomycin + ampicillin + ceftriaxone
Laboratory Findings C. Vancomycin + cefotaxime
Na =136 mEq/L Hgb = 14.5 g/dL Ca = 8.1 mg/dL D. Vancomycin + cefepime
K =4.1 mEq/L Hct = 38% Mg= 2.2 mg/dL 5. If the preliminary CSF Gram stain resulted in gram ( +)
Cl= 98m.Eq/L Plt = 132 X 103/mm3 Phos = 4.6 mg/dL bacilli, coverage against what pathogen should be included?
co2 = 26 mEq/L WBC = 8 x 103/mm3 AST= 24IU/L A. Streptococcus pneumoniae
B. Listeria monocytogenes
BUN = 26 mg/dL Trop < 0.01 ng/mL ALT= 22IU/L
C. Haemophilus influenzae
SCr = 1.66 mg/dL CK=3ng/mL T Bill= 1.8 mg/dL D. Neisseria meningitidis
Glu = 168 mg/dL BNP = 64 pg/mL AlkPhos = 76 IU/L
6. Once Listeria monocytogenes is isolated, what antimicro-
~ CTHead bial agent would you recommend for treatment of bacterial
meningitis in DJ?
Diffuse hypodensity in the left lentiform nucleus, possibly A. Ampicillin 2 g IV q4h
from a previous stroke B. Ceftriaxone lg IV q24h
C. Meropenem lg IV q8h
~ Blood Cultures D. Cefepime lg IV ql2h
Pending
7. What is the appropriate duration of treatment for bacterial
meningitis caused by Haemophilus influenzae?
~ LP/CSF Analysis
A. 7 days
Pending B. 10-14 days
C. 14-21 days
~ CSF Gram Stain D. ;;::21 days
Pending
8. Which is true regarding dexamethasone use in bacterial
meningitis?
QUESTIONS A. Should be administered empirically in patients with
suspected pneumococcal meningitis
1. Which laboratory test(s) help identify a patient with bacterial B. Reduces inflammatory responses in the subarachnoid
meningitis? space, decreasing risk of neurologic sequelae due to
A. Positive Gram stain of CSF bacterial meningitis
B. Low CSF glucose ( <40 mg/dL) C. May reduce antimicrobial penetration into CSF
C. High percentage of neutrophils in CSF D. All of the above
D. All of the above
2. DJ's CSF analysis revealed WBC 1750 cells/mm3 with 88% ANSWERS
neutrophils, 330 mg/dL protein, and glucose 40 mg/dL
1. .Explanation: The correct answer is D. A Gram stain
(plasma glucose 165 mgldL). What is the most likely diag-
examination of CSF allows for rapid identification of the
nosis for DJ?
causative bacterium with high rates of specificity. CSF
A. Community-acquired bacterial meningitis
analysis can also aid in the diagnosis of bacterial meningi-
B. Healthcare-associated bacterial meningitis
tis. Usual CSF findings in the setting ofbacterial meningitis
C. Viral meningitis
include low glucose ( <40 mg/dL), elevated white blood cell
D. Fungal meningitis
count (> 1000 cells/mm3), high percentage of neutrophils
3. What is a likely sign or symptom of bacterial meningitis in (>80%), and elevated protein (>200 mg/dL). In addition to
DJ? these findings, elevated CSF lactate concentrations can help
A. Diminished breath sounds and crackles to distinguish between bacterial and viral (aseptic) menin-
B. Pedal edema on lower extremities gitis. Blood cultures can also guide antimicrobial therapy;
C. Severe headaches especially in patients whom lumbar puncture cannot
D. Dyspnea be performed.
CHAPTER 9 I BACTERIAL MENINGITIS 35

Men1ngitia CSP Laboratory Findings


Normal Bacterial Viral Fungal
WBC (celWmm') <5 1000-5000 5-500 100-400
WBC dlil'mmtial Monocytes Neutrophils (>80%) Lymphocytes(>SO%) Lyinphocytes(>50%)
Protein (mgfdL) <50 Elevated (I00-500) Slightly elevated Elevated (60-150)
Glucose (mgldL) 45-80 Decreased ( <45) Normal Deaeased ( <45)
CSP/blood glucose ratio 0.5-0.6 Decreased ( <0.4) Normal Deaeased ( <0.4)

2. E:lplanation: The correct answer is A. Dfs CSF analysis considered a common pathogen in this age group. Answer
is consistent with bacterial meningitis with elevated WBC D is appropriate in patients with health care-associated
(neutrophil predominant), elevated protein, decreased glu- bacterial meningitis as resistant gram-negative bacilli (ie.
cose, and decreased CSF/blood glucose ratio (rules out Pseudomonas turuginosa) is a potential pathogen requiring
answers C and D).Health care-associated bacterial.meningi- the use of cefepime.1
tis is associated with head trauma, neurosurgical procedures,
5. E:splanation: The correct answer is B. Due to the increased
or placement of ventricular catheters (rules out answer B).
rates of pathogenicity, a CSF Gram stain showing gram
3. E:splanation: The correct answer is C. Fever, nuchal ( +) diplococci often suggests Streptococcus pneumonitu,
rigidity, headache, and altered mental status are common gram (-) diplococci suggests Neisseria meningitidis, gram
symptoms associated with bacterial meningitis. Other clas- (-) coccobacilli suggests Haemophilus influenzae, and
sic symptoms include Brudzinsk:i's sign (tlexion of the hips gram (+) bacilli suggests Listeria monocytogenes. L. mono-
when knees are flexed), Kernig's sign (inability to extend cytogenes can cause bacterial meningitis primarily in the
knee when hips are flexed). and purpuric and/or petechial elderly, newborns, and immunocompromised patients.
skin lesions. Presentation can vary with risk factors and
6. Bxplanation: The correct answer is A. Current guidelines
age, including clinical manifestations such as bulging fon-
recommend four antimicrobial options against isolated
tanelle (in infants) and neurologic complications (such as
ataxia or convulsions). DJ presents with symptoms consis-
L. monocytogenes in cases of bacterial meningitis: ampi-
tent with bacterial meningitis: headaches, fever, neck and
cillin or penicillin G as standards of treatment while tri-
methoprim-sulfamethoxazole or meropenem serve as
back pain, lethargy, and ( +) Kemigs sign. DJ's diminished
alternative regimens.1 Ampicillin has excellent activity
breath sounds and crackles are likely due to her COPD,
against L. monocytogenes and remains the preferred antimi-
pedal edema due to her CHF, and dyspnea due to her CHF
crobial option. Cephalosporins are not active against Listeria
or COPD (rules out answers A, B, and D).
sp. ruling out answers Band D. Meropenem is an option. but
4. Explanation: The correct answer is B. In patients more due to the drug-drug interaction with divalproex sodiwn
than 50 years of age. the recommended antimicrobial (meropenem lowers divalproex concentrations below thera-
therapy is vancomycin. ampicillin, and a third-generation peutic ranges; therefore, increasing the risk of breakthrough
cephalosporin (ceftriu:one or cefotu:ime). as the most seizures), it is not preferred. Additionally, higher doses of
common bacterial pathogens are Streptococcus p~umoniae. intravenous antimicrobials are needed for the treatment of
Neisseria meningiti.dis, aerobic gram (-) bacilli. and Listeria bacterial meningitis (ruling out answers B, C, and D).
monocytogenes. Ce:ftriaxone is active against S. pneumoniae
7. E:lplanation: The correct answer is A. Current guidelines
and gram (-) organisms, ampicillin is added for L. mono-
recommend 7 days of treatment for Ntisstria meningitidis
cytogenes coverage, and vancomycin is used to provide
and Haemophilus injluenzae, 10 to 14 days for Streptococ-
additional S. pneumoniae coverage until susceptibilities
cus pMumoniae, 14 to 21 days for Streptococcus agalactitu,
are known. Answer A is appropriate empiric treatment for
21 days for aerobic gram-negative bacilli. and ~21 days for
patients less than 1 month of age (ampicillin, in combina-
tion with either cefotaxirne or an aminoglycoside, is rec-
Listeria monocytogenes when isolated in the setting of bac-
terial meningitis.1 Although these recommendations are
ommended. as L. monocytogenes is a potential pathogen in
available. treatment duration should depend on individual
this age group). Ceftria:xone should not be used in patients
patient response.
younger than 1 month as it can cause biliary sludging and
hyperbilirubinemia. Answer C is appropriate in patients 1 8. Explanation: The correct answer is D. Dexamethasone ther-
month of age to 50 years of age as L. monocytogenes is not apy is recommended as adjunctive therapy to antimicrobial
36 INFECTIOUS DISEASES: A CASE STUDY APPROACH

therapy in patients with suspected or proven pneumococcal the benefits are known, clinical controversy regarding
meningitis. Dexamethasone should be administered 10 to adjunctive dexamethasone use exists as dexamethasone may
20 minutes before the first dose of antimicrobial therapy reduce inflammatory responses that aid in CSF penetration
and only continued in patients with bacterial meningitis due of certain antimicrobials, such as vancomycin.
to Streptococcus pneumoniae. In patients who have already
received antimicrobial therapy, use of dexamethasone is not
recommended, as it is not likely to improve patient outcomes. REFERENCE
Adjunctive dexamethasone therapy may decrease inflam- I. Thnkel AR, Hartman BJ, Kaplan SL, et al. Practice guide-
matory responses in the subarachnoid space and reduce lines for the management of bacterial meningitis. Clin
neurologic sequelae from bacterial meningitis. Although Infect Dis. 2004;39:1267-1284.
Viral Encephalitis
10 Ann Lloyd

PATIENT PRESENTATION .- Pulmonary


Chief Complaint Clear to auscultation bilaterally
Altered mental status and fever
.- Cardiovascular
History of Present Illness Tachycardic, regular rhythm, no m/r/g
GA is a 23-year-old Caucasian female who presents to the
.- Abdomen
emergency department with her parents who report that she
has been behaving abnormally over the past several days. They Soft, non-distended, no masses
report that she was withdrawn and would not speak. She has
complained of a sore throat. .- Neurology
Oriented to person and situation, follows some instructions
Past Medical History and nods head but does not speak
Depress.ion
.- Extremities
Surgical History Warm and dry, no edema. no rashes, ulcers. or lesions
None
Laboratory Findings
Family History Na= 129 mEq/L Hgb = 12.4g/dl Ca= 8.6 mg/dL
Mother with HTN and depression, father with type II DM K = 3.5 mEq/L Hct = 38% AST=26nJ/L
Cl = 97 rnEg/L Plt = 248 x 10'/mm' ALT = 15 nJ/L
Soclal History co2 =21 mEq/L WBC =13.5 x 10'/mm.S T Bill= 0.6 mg/dL
Smokes % ppd x 5 years and drinks alcohol socially BUN= 13 mgldL Alk Phos = 51 nJ/L
SCr = 0.88 mg/dL
Allergies Glu = 112 mg/dL
NKDA
.- CSF Analysis
Home Medications Colorless, hazy; RBC 0 cells/mm3, WBC 426 cell.s/mmi,
Fluoxetine 40 mg PO once daily 10% segs, 72% lymphs, 18% monos

Physical Examination .- CSF Protein


75 mgldL
.- Vital Signs
Temp 100.4°F,P 141,RR31 breathsperminute,BP151/89mm .- CSF Glucose
Hg, p02 98%, Ht 5'6", Wt 60 kg
61 mg/dL
.- General
Febrile, following limited commands, but in no acute distress
.- CSF Gram Stain/Culture
Pending
.- HEENT
+3 tonsils without exudate, normocephalic, atraumatic, .- Throat Cultun
EOMI, PERRI.A Pending

37
38 INFECTIOUS DISEASES: A CASE STUDY APPROACH

.,.. CSF Meningitis Panel by PCR {He table ftN C. Acyclovir 600 mg IV every 8 hours
contents of the panel} D. Acyclovir 900 mg IV every 8 hours
Pending 4. GA'.s meningitis PCR results return with no positive find-
ings. What is the most appropriate change to her anti.-
CSF Menlngitit Panel by PCR infective regimen?
Bacteria Viruses Yeast A. Continue acyclovir, ceftriaxone, and vancomycin
Escherichia coli Cytomegalovirus Cryptococcus B. Continue acyclovir, but discontinue vancomycin and
n.eoformans/ ceftriaxone
Haemophilus Enterovirus
gatti C. Discontinue acyclovir, but continue ceftriaxone and
in.fluenzae
vancomydn
Listeria Herpes simplex virus 1 D. Discontinue acyclovir, ceftriaxone, and vancomycin
monocytogenes
5. Three days later, GA'.s HSV PCR returns with a positive
Neisseria Herpes simplex virus 2
result confirming the diagnosis of HSV encephalitis. Her
meningitidis
CSP Gram stain is negative and there is no growth on the
Streptococcus Human herpesvirus 6 CSF culture. What changes should be made to her anti.-
agalactiae infective regimen now?
Streptococcus Human parech.ovirus A. Continue acyclovir, ceftriaxone, and vancomydn
pneumon.iae Varicella zoster virus B. Continue acyclovir, but discontinue vancomycin and
ceftriaxone
C. Discontinue acyclovir, but continue ceftriaxone and
.,.. CXR
vancomycin
Nonnal D. Discontinue acyclovir, ceftriaxone, and vancomycin

.,.. MR/Brain 6. What is the appropriate duration oftreatment for viral encep-
halitis caused by HSV?
Abnormal restricted diffusion, cortical edema, and vasogenic A. 5 days
edema involving the anterior temporal lobe, hippocampus, B. 7days
insular ribbon, and medial thalamus c. lOdays
D. 14-21 days
QUESTIONS 7. Which of the following adverse effects may be seen while
GA is receiving intravenous acyclovir?
1. Which laboratory findings in this patient make the diagnosis
A. Myopathy
of viral encephalitis most likely?
B. Nephroto:ricity
A. Lymphocyte predominance on the CSF differential C. Ototo.xicty
B. Slightly elevated CSF protein D. Thrombocytopenia
C. Normal CSF glucose
D. All of the above
2. Which empiric therapy is most appropriate for GA while ANSWERS
awaiting the CSF Gram stain and culture?
1. .Explanation: The correct answer is D. It is often difficult
A. Acyclovir
to distinguish viral encephalitis from bacterial meningitis
B. Acyclovir, ceftriaxone, and vancomycin
especially while awaiting the CSF Gram stain and culture.
C. Ampicillin, ceftriaxone, and vancomycin
However, in this case, the lymphocyte predominance on
D. Acyclovir, ampicillin, ceftriaxone, and vancomycin
the CSP differential along with the slightly elevated CSP
3. Which of the following acyclovir regimens should be started protein but normal CSP glucose all point more toward a
for GA? viral cause of this patient's meningitis. In the table below,
A. Acyclovir 400 mg PO five times daily the differences between bacterial and viral CSF findings are
B. Acyclovir 800 mg PO five times daily compared to normal CSF.
CHAPTER 10 I VIRAL ENCEPHALITIS 39

Mean Values of Components of Normal and Abnormal CSP


Variable Normal Bacterial VJral
Opening pressure 60-220mm~O Increased (>250mmH,O) Rarely increased
WBC (cells/mm') UPto5 1,000-5,000 100-500
Dominant cell type Lymphocytes Neutrophils Lymphocytes
Protein (mgldL) 20-60mg/dL Usually increased (100-500 mgldL) Normal/slightly increased
Glucose (mgldL) Two-thirds of serum Low ( <40 me/dL or 40% ofconcurrent serum) Normal

2. Ezplanation: The correct answer is B. While awaiting the lobe are strongly suggestive of HSY encephalitis. Further,
CSF Gram stain and culture results. empiric therapy should altered mental status is more common in encephalitis com-
be started as soon as possible providing coverage against pared to meningitis. Finally, the initial HSV PCR (which
both bacterial and vinl meningitis. Because this patient is is included in the meningitis PCR) may be negative and
between the ages of 1 month and 50 years old, she is at risk should be repeated. Until these results can be repeated,
for the following most common types of bacterial menin- all empiric therapy should be continued pending the CSF
gitis: Streptoc:occus pneumoniae, Neismia meningitJdis, and Gram stain and culture and the repeat HSY PCR.
Heumophiltu infounza.1 Additionally, based on her presen- 5. Explanation: The correct answer is B. Now that the HSY
tation and laboratory findings. there is a suspicion for viral PCR is positive and there is no bacterial growth on the CSF
causes of her infection. The most common viral pathogens culture, the vancomycin and ceftriaxone can be discontin-
include herpes simplex virus (HSV), West Nile virus, ued. Intravenous acyclovir alone should be continued to
enteroviruses, and other herpesv:iruses. In order to provide treat the HSY infection.
coverage against all possible organisms, the patient should 6. Explanation: The correct answer is D. According to cur-
be immediately started on acyclovir, ceft:riaxone, and van- rent guidelines, intravenous acyclovir should be continued
comycin (rules out answer A) pending the results of the for 14 to 21 days. 1 The actual duration of treatment should
CSP Gram stain and culture as well as the CSP meningitis be determined once individual patient characteristics are
panel. Ampicillin should be added to empiric therapy regi- taken into consideration and could be longer than 21 days
mens for patients younger than 1 month. older than age 50, in certain situations.
or immunocompromised patients to provide coverage
against Listeria monocytogenes {rules out answers C and D). 7. E:lplanation: The correct answer is B. Although oral
formulations of acyclovir are usually well tolerated, the
3. Explanation: The correct answer is C. For patients receiv- intravenous form requires specific monitoring. In addition
ing acyclovir for suspected HSY encephalitis, intravenous to neurotoxicity, practitioners should monitor patients for
acyclovir is the drug of choice (rules out answers A and B). changes in renal function (rules out answers A, C, and D).
For adult patients. the usual dose is 10 mg/kg IV every Serum creatinine levels may increase while on therapy.
8 hours (rules out answer D). In neonates. the dose is With high doses or in patients with renal insufficiency,
increased to 20 mg/kg IV every 8 hours to improve out- acyclovir may crystallize in the renal tubules. Dose adjust-
comes and prevent relapses. These same benefits are not ments should occur for patients with renal dysfunction,
seen when the dose is increased in adult patients. There is and prehydration with intravenous fluids may decrease the
some controversy surrounding the weight that should be risk of nephrotoxicity.
used to dose acyclovir in obese patients. Historically, the
ideal body weight was used to dose acyclovir. However.
there is concern that this may result in underdosing of REFERENCES
obese patients. Some clinicians may elect to use adjusted 1. Tunkel AR, Glaser CA. Bloch KC. et al. The manage-
bodyweight fo.r dosing in obese patients.2 ment of encephalitis: clinical practice guidelines by the
4. Explanation: The correct answer is A. Despite the menin- Infectious Diseases Society of America. Clin Infect Di5.
gitis PCR returning with no positive findings. there is still 2008;47:303-327.
a very high suspicion for a central nervous system infec- 2. Wong A. Pickering AJ. Potoski BA. Dosing practices of
tion} Therefore, all empiric therapy should be continued intravenous acyclovir fo.r herpes encephalitis in obe-
pending further laboratory studies. In this patient's case. sity: results of a pharmacist survey. J Pharm Pract.
the MRI findings which include edema in the temporal 2017;30(3):324-328.
This page intentionally left blank
11 Infective
Endocarditis
Rachel A. Foster P. Brandon Bookstaver

PATIENT PRESENTATION Infliximab 3 mg/kg IV every 2 months


Lisinopril 10 mg PO daily
Chief Complaint Pregabalin 75 mg PO BID
Headache, fevers, and chills Triamcinolone 0.1 % lotion topical BID
Vitamin D3 5,000 IU PO daily
History of Present Illness
GR is a 68-year-old woman with a notable past medi<:al history Physical Examination
of rheumatoid arthritis on infliximab and a prosthetic aortic .,. Yitai Signs
valve, who was brought to the emergency department (ED)
Temp 102.1°F (tympanic), HR 112 bpm, RR 19 breaths per
after her family found her extremely lethargic and confused at
minute, BP 91/52 mm Hg, Sp02 97% (on room air), Ht 165 cm,
home. She had been complaining of fevers, chills, headache,
Wt 91 kg, BMI 33.4 kg/m1
and neck pain for 2 days prior to presentation, and as per the
patient's family had steadily become less and less communica-
.,. Genel'tll
tive. Otherwise the patient has had no major medical issues in
the last year since her aortic valve replacement. Lethargic, acutely ill appearing, appears stated age

.,. HEENT
Past Medlcal History
No.rmocephalic. atraumati.c, PERRLA, EOMJ, faint conjunctival
CAD, depression, type JI DM, eczema. HTN, nbromyalgia.
hemorrhage, non-icteric sclera, poo.r dentition, no erythema or
severe aortic stenosis with valve replacement
swelling in the oropharym:
Surgical History .,. Neck
Bioprosthetic aortic valve replacement (10 months ago), S2-S4 No nuchal rigidity, tenderness to palpation on lower lumbar
diskectomy (4 years ago), tub al ligation (>15 years ago), cho- region
lecystectomy (> 15 years ago)
.,. Pulmonary
Family History Clear to auscultation bilaterally, no wheezes or cracldes
Father passed away from HP; mother has type II DM, HTN,
and h/o stroke; sister has type Il DM, COPD, and HTN .,. Cardiovascular
Regular rate and rhythm, faint systolic murmur over the right
Social History base
Widowed, lives by herself, never used alcohol, former smoker
(quit 10 years ago) .,. Abdomen
Soft, non-distended, no masses, no focal rebound or guarding,
Allergies tenderness in the epigastric region to palpation
Hydrocodone/acetaminophen (vomiting)
.,. Genitourinary
Home Medications No complaints of dysuria or hematuria
Aspirin DR tablet 81 mg PO daily
Atorvastatin 20 mg PO daily .,. Neurology
Fluoutine 40 mg PO daily AO x 1, no focal deficits, strength and sensation full and
Glimepiride 4 mg PO daily symmetric
41
42 INFECTIOUS DISEASES: A CASE STUDY APPROACH

~ Extremities ED Course
Intact distal pulses, no LE edema Blood cultures are drawn and GR undergoes a lumbar punc-
ture. She is started on vancomycin, ceftriaxone, ampicil-
~ Skin lin, acyclovir, and dexamethasone for empiric treatment of
meningitis/encephalitis while further workup is done in the set-
Warm and diaphoretic
ting of immunosuppression. The HSV PCR on the CSF returns
negative and the acyclovir is discontinued. Shortly thereafter,
~ Back her blood cultures (2/2 sets) flag positive and the Gram stain
Tenderness to palpation on lower lumbar region shows gram-positive cocci (GPC) in clusters. An hour later,
rapid diagnostic technology confirms the GPCs in the blood
cultures are methicillin-sensitive Staphylococcus aureus (MSSA).
Laboratory Findings The CSF analysis also returns and reveals a CSF clear/colorless
Na = 124 mmol/L BUN = 28 mg/dL T bill: 1.9 mgldL in appearance with glucose = 165 mgldL, protein = 45 mgldL,
K = 3.4 mmol/L SCr = 1.4 mg/dL AlkPhos = RBC = 10 cells/mm3, and WBC = 5 cells/mrn3, making men-
96 unit/L ingitis less likely.
Cl =88mmol/L Ca = 9.8 mgldL AST= 109 unit/L
C02 = 19 mmol/L Protein, total = ALT = 92 unit/L QUESTIONS
7.9 g/dL
1. GR is admitted and repeat blood cultures are ordered for
Glucose= Albumin= 3.8 g/dL
286mg/dL 48 hours from the previous sets drawn in the ED. With
the diagnosis of MSSA bloodstream infection, what is the
most optimal change in the antimicrobial regimen for this
Estimated CrCl: 36.47 mL/min; estimated GFR: 40 mL/min/
patient at this time?
1.73 m 2
A. Discontinue ceftriaxone, ampicillin, and dexametha-
sone; continue vancomycin
WBC = 21.3 K/µL Segs = 72% Bands =22%
B. Discontinue vancomycin, ampicillin, and dexametha-
Lymphocytes = 2% Monocytes = 4% Neutrophil, abs sone; continue ceftriaxone
= 18.1 K/µL C. Discontinue vancomycin, ceftriaxone, ampicillin, and
Lymphocyte, abs Monocyte, abs MCHC = 33.8 g/dL dexamethasone; start cefazolin
= 0.4K/µL = 0.8 K/µL D. Discontinue vancomycin, ceftriaxone, ampicillin, and
RBC = 3.74 x dexamethasone; start penicillin G
RDW SD = 42.4 fL Hemoglobin =
106/µL 11.5 g/dL 2. The medical intern asks if transesophageal echocardiog-
RDW= 13.5% Hematocrit = Platelets= 86 K/µL raphy (TEE) is warranted in the additional workup since
32.4% the patient has a prosthetic valve in the setting of MSSA
bacteremia. Which of the following is true regarding TEE
MCV = 86.5tl MPV= 10.8fL MCH= 29.3pg
for this patient? A TEE is:
Lactic acid, venous: 2.5 mmol/L A. Recommended because this patient has a prosthetic
valve
B. Not recommended because there is no evidence of
Imaging endocardial involvement
~ CTHead C. Not recommended because the patient already meets
Unremarkable, no evidence of infarct or hemorrhagic stroke pathologic criteria for definitive infective endocarditis
D. Not recommended since the TTE showed no signs of
vegetation and TTE is more sensitive than TEE in the
~ CJ<R detection of intracardiac vegetations
Indistinct central pulmonary vasculature, suggestive of pul-
3. A TEE is done the following morning which showed some
monary venous congestion but no radiographic evidence of
mild mitral and tricuspid valve regurgitation as well as a
pneumonia
1.5 x 0.4 cm linear echo density on her prosthetic aortic
valve, consistent with a vegetation. The patient is diagnosed
~ Transthoracic Echocardiogram (TTEJ with MSSA prosthetic aortic valve endocarditis. Cardiotho-
LVEF 73% (±3%), normal RV systolic function, AV appears racic surgery is consulted to determine surgical management.
to be well seated with normal functioning and trace regurgita- Which of the following clinical or echocardiographic features
tion, no pulmonary hypertension or pericardial effusion seen; suggest(s) the need for potential surgery? Select all that apply.
the study is of adequate quality A. Perivalvular abscess
B. Heart failure wuesponsive to medical therapy
CHAPTER 11 I INFECTIVE ENDOCARDITIS 43

C. A vegetation of any siu on the anterior mitral leaflet C. At least 6 weeks from first negative blood culture or
D. An embolic event during the first two weeks of antimi- surgical source control, whichever occurs first
crobial therapy D. At least 6 weeks from first negative blood culture or
surgical source control, whichever occurs last
4. Which of the following is/are true regarding Staphylococcus
aureus infective endocarditis (IE)? Select all that apply. 9. In applying the modified Duke IE criteria to this patient in
A. S. aureus is the most common causative organism of IE retrospect, how did this patient meet the criteria for IE?
in adults A. Two major criteria
B. Mortality rates associated with S. aureus IE range from B. One definitive histologic criteria
20% to 45% C. Five minor criteria
C. Blood cultures should be repeated to confirm if this is D. Without culturing the vegetation, this patient did not
a contaminant or a true pathogen technically meet the Duke IE criteria
D. All of the above are true
ANSWERS
5. Repeat blood cultures from hospital day (HD) 2 and HD 4 are
still positive for MSSA. The team obtains an Infectious Diseases 1. Explanation: The correct answer is C. Antistaphylococcal
(ID) consult and is specifically wondering about the addition of beta-lactams (eg, cefazolin, nafcillin, oxacillin) are the
gentamicin given the diagnosis of endocarditis. Which of the treatment of choice for MSSA infections and patients
following is true regarding the use of gentamicin in IE? should be switched as early as possible to one of these
A. Gentamicin should be added for all S. aureus IE, both agents once MSSA has been identified as the causative
native valve IE (NYE) and prosthetic valve IE (PYE) agent in bacteremia (answers A and B are incorrect).
B. Gentamicin is no longer indicated in the treatment of Although vancomycin does provide coverage of MSSA,
IE because risk of acute kidney injury is too high antistaphylococcal beta-lactams are considered superior to
C. If gentamicin is indicated, it should be added for the vancomycin, and early transition to definitive therapy with
first two weeks of the IE regimen one of these agents is associated with significantly lower
D. If gentamicin is indicated, dosing is the same as treat- treatment failure, shorter time to bacteremia clearance, and
ment dosing of gram-negative infections most importantly, lower mortality. The majority of staphy-
lococci (-85%) are resistant to penicillin. Because current
6. Gentamicin 1 mg/kg IV q8h (synergy dosing) is added by methods for detecting penicillin susceptibility have ques-
ID. On HD 6, the patient reports new headaches and is tionable reliability and may miss more than a one-third of
progressively more confused and somnolent. An MRI of beta-lactamase-producing S. aureus, penicillin should not
the brain was obtained that showed several small cortical be used to treat S. aureus (answer Dis incorrect). 14
ischemic lesions concerning for septic emboli. What is the
2. Explanation: The correct answer is A. Patients with infec-
most optimal therapy for this patient in addition to con-
tive endocarditis (IE) can present with highly variable and
tinuing gentamicin?
nonspecific symptoms making the initial differential diag-
A. Continue cefazolin
nosis very broad. Echocardiography should be performed
B. Change cefazolin to nafcillin
when IE is a reasonable diagnosis. TEE has been shown to
C. Change cefazolin to ceftriaxone
be much more sensitive than TTE in the detection of intra-
D. Change cefazolin to vanoomycin
cardiac vegetations, though TTE is generally performed
7. It is now HD 11 and the last set of repeat blood cultures first if IE is suspected given the noninvasive nature of the
show no growth for 4 days. The ID consult note says to add test and is also more sensitive in detecting right-sided
rifampin. Which of the following is true regarding the use vegetations. However, for patients in whom there is high
of rifampin in IE? clinical suspicion or risk, or those where patient-specific
A. Rifampin is only recommended for the first two weeks factors prohibit an optimal echocardiographic window,
of IE treatment a TEE should be performed first or as soon as possible
B. It is too late to add rifampin because it should have after TTE since the latter cannot definitively rule out IE
been started on day 1 of therapy or potential complications (answer Dis incorrect). TEE is
C. Rifampin therapy is benign with minimal side effects recommended in patients with prosthetic valves, patients
D. A drug interaction check should routinely be per- classified as "possible IE" by clinical criteria, or patients
formed when adding rifampin with complicated IE (eg, perivalvular abscess). TTE may be
considered sufficient if negative in patients with low clinical
8. What is the appropriate duration of treatment for PYE suspicion for IE or if the TTE shows vegetations, the likeli-
caused by MSSA? hood of complications is low, and obtaining a TEE will not
A. At least 4 weeks from first negative blood culture or subsequently alter management Furthermore, repeat TEE
surgical source control, whichever occurs first in 7 to 10 days is warranted if there is high clinical suspicion
B. At least 4 weeks from first negative blood culture or of IE and the initial TEE results are negative. Answer B is
surgical source control, whichever occurs last incorrect because this patient requires further imaging to
44 INFECTIOUS DISEASES: A CASE STUDY APPROACH

determine if there is endocardial involvement. Answer C gentamicin was previously considered in select cases of
is incorrect because pathologic criteria require culture or NVE, but growing evidence suggests that outside of the
histological examination of an intracardiac specimen."' presence of prosthetic valve/material, the risks of adding
gentamicin outweigh the benefits. Gentamicin dosed for
3. Explanation: The correct answers are A, B, and D. Several
synergy should be added for the first 2 weeks of therapy
features may indicate the need for potential surgery, includ-
provided high-level gentamicin resistance is not detected.
ing both clinical and prognostic factors. Indication for sur-
For prosthetic valve endocarditis caused by S. aureus, gen-
gery also depends on consideration of whether the patient
tamicin dosing is recommended to be 3 mg/kg/24h in 2 to
has prosthetic valve (PYE) vs. native valve (NYE) IE and
3 equally divided doses with target peaks of 3 to 4 mcg/mL
left- vs. right-sided IE. Generally, echocardiographic char-
and target troughs of <I mcglmL (answer Dis incorrect).
acteristics that suggest the need for surgery can be divided
Doses should be adjusted for changes in renal function and
into three main categories: vegetation characteristics, val-
based on therapeutic drug monitoring results. 2
vular dysfunction, and perivalvular extension. Answer C
is not correct as typically the greatest risk of embolic com- 6. Explanation: The correct answer is B. An antistaphylococ-
plications (and thus indication for surgery) occurs when cal beta-lactam is still treatment of choice in the setting of
mitral leaflet vegetations are larger than I cm in diameter. IE with associated embolic complications (answers C and
Additional vegetation characteristics that suggest the need D are incorrect). Nafcillin is recommended over cefazolin
for surgical intervention include an increase in vegetation and oxacillin in cases of IE complicated by septic cerebral
size despite appropriate antimicrobial therapy, persistent emboli or brain abscess, owing to its more consistent
vegetation after systemic embolization, or one or more blood-brain barrier penetration and greater level of evi-
embolic events during first two weeks of antimicrobial dence (answer A is incorrect). The IE guidelines also prefer
therapy (answer D). Valvular dysfunction includes acute nafcillin and oxacillin over cefazolin because cefazolin may
aortic or mitral insufficiency with signs of ventricular be more susceptible to type A beta-lactamase-mediated
failure, valve perforation or rupture, and heart failure hydrolysis than nafcillin, particularly in high-inoculum
unresponsive to medical therapy (answer B). Perivalvular infections. However, clinical data to substantiate this
extension includes a large abscess or extension of abscess "inoculum effect" are lacking and many clinicians may still
despite appropriate medical management, new heart bock, choose to use cefazolin for tolerability and cost reasons. 2.a
or valvular dehiscence, rupture, or fistula (answer A).
7. Explanation: The correct answer is D. Rifampin is used to
Additionally, many other clinical considerations may also
help with sterilization of the prosthetic material and should
drive the indication for surgery such as microbiologic fac-
be considered for the entire course of therapy (answer A
tors (eg, persistent bacteremia or IE caused by fungal or
is incorrect). However, rifampin resistance often develops
highly resistant organisms) or complicated right-sided IE.2
rapidly if used alone or in high-inoculum infections, thus
4. Explanation: The correct answers are A and B. Staphy- many experts recommend waiting to add rifampin until
lococci and streptococci account for 80% to 90% of IE blood cultures are clear to prevent treatment-emergent
when an identification is made. Streptococci, particularly rifampin resistance (answer Bis incorrect). Adverse effects of
the viridans group streptococci, are still the predominant rifampin include GI upset and hepatotoxicity, though typi-
pathogens in the developing world. However, staphylo- cally elevations in LFTs are transient and serious hepatotox-
cocci have assumed the role of the primary etiologic agent icity is limited to patients with underlying liver injury or on
in the industrialized world, which is thought to be resultant concomitant hepatotoxic drugs. More notably, patients and
of increased health care contact/exposure as a risk factor providers should be counseled that rifampin will turn body
for S. aureus bacteremia. Of further concern is the signifi- fluids a red-orange color, including urine, tears, sweat, and
cant morbidity and mortality associated with S. aureus bac- even contact lenses (answer C is incorrect). Drug interactions
teremia and IE. The primary foci of infection factors into with rifampin can be significant and are another important
the overall mortality rate of S. aureus bacteremia (SAB), consideration. Rifampin is a potent inducer of CYP450
but S. aureus IE-associated bacteremia are among the enzymes (especially CYP3A4) and most often results in
highest mortality rates, ranging from 20% to 45%. Mortal- decreased concentrations of drugs that are CYP450 sub-
ity rates are typically higher in non-intravenous drug use strates. It is important to note that enzyme induction requires
(IVDU)-associated IE (primarily left-sided) compared to upregulation or production of additional enzyme, thus the
!VDU-associated IE (primarily right-sided). Answer C is onset of drug interactions may not be immediate.2
incorrect because S. aureus should never be considered a
8. Explanation: The correct answer is D. Treatment of IE is
contaminant when isolated in the blood, especially given
typically prolonged because of the high bacterial inoculum
the presence of prosthetic heart valve. 5- 7
within vegetations, and duration is highly pathogen-specific.
5. Explanation: The correct answer is C. The addition of Other key factors that impact treatment duration include
gentamicin is only indicated in S. aureus IE when there the type of valve present (NYE vs. PYE), the location of the
is a prosthetic valve or other prosthetic material is pres- infection (left-sided vs. right-sided), and whether the case is
ent (answers A and B are incorrect). The addition of considered complicated or uncomplicated. In general, most
CHAPTER 11 I INFECTIVE ENDOCARDITIS 45

S. aureus IE should be treated for 6 weeks, with durations 2. Baddour LM, Wilson WR, Bayer AS, et al. Infective endo-
extended beyond 6 weeks for cases involving prosthetic carditis in adults: diagnosis, antimicrobial therapy, and
valves or other complicating factors such as perivalvular management of complications: a scientific statement for
abscesses. An exception may be made for uncomplicated healthcare professionals from the American Heart Asso-
right-sided MSSA NVE, in which treatment may be as short ciation. Circulation. 2015;132:1435-1486.
as 2 weeks. It is beyond the scope of this chapter to cover all 3. Stryjewski ME, Szczech LA, Benjamin DK, et al Use of
commonly encountered organisms/scenarios in IE, and the vancomycin or first-generation cephalosporins for the
authors encourage referencing the guidelines for full treat- treatment ofhemodialysis-dependent patients with meth-
ment recommendations.1 One important summary note in icillin-susceptible Staphylococcus aureus bacteremia. Clin
managing IE: Numerous studies have shown that adherence Infect Dis. 2007;44(2):190-196.
to quality-of-care indicators (QCis) in SAB is associated 4. Siegman-Igra Y, Reich P, Omi-WasserlaufR, Schwartz D,
with better outcomes and reduced mortality, particularly Giladi M. The role of vancomycin in the persistence or
when implemented as a bundled approach. The follow- recurrence of Staphylococcus aureus bacteraemia. Scand J
ing QCis should be considered standard of care in SAB: Infect Dis. 2005;37(8):572-578.
early removal of infectious foci (source control), follow-up 5. Fowler VG, Scheid WM, Bayer AS. Endocarditis and
blood cultures to assess clearance of SAB, early definitive intravascular infections. In: Bennett JE, Dolin R, Blaser
therapy for MSSA, echocardiography, and appropriate treat- MJ, eds. Principles and Practice of Infectious Diseases.
ment duration according to the complexity of the infection Philadelphia, PA: Saunders; 2015:990-1028e.11.
and/or host.9 6. Que Y, Moreillon P. Staphylococcus aureus (including
staphylococcal toxic shock syndrome). In: Bennett JE,
9. Explanation: The correct answer is A. In order to meet
Dolin R, Blaser MJ, eds. Principles and Practice of Infec-
the Dulce IE criteria for definitive diagnosis of IE, a patient
tious Diseases. Philadelphia, PA: Saunders; 2015:2237-
must have one of the following: a positive histologic result,
2271e.9.
two major criteria, five minor criteria, or one major and
7. van Hal SJ, Jensen SO, Vaska. Predictors of mortality
three minor criteria (answers B, C, and D are incorrect).
in Staphylococcus aureus bacteremia. Clin Micro Rev.
The two major criteria met in this patient are supportive
2012;25(2) :362-386.
laboratory evidence indicated by positive blood cultures
8. Schweizer ML, Furuno JP, Harris AD, et al Comparative
with an organism commonly associated with IE (S. aureus)
effectiveness of nafcillin or cefazolin versus vancomycin in
and a positive echocardiogram (eg, vegetation by TEE).
methicillin-susceptible Staphylococcus aureus bacteremia.
The full compilation of all modified Dulce criteria is a valu-
BMC Infect Dis. 2011;11:279.
able tool to review in full.2. 10
9. L6pez-Cortes LE, Del Toro MD, Galvez-Acebal J, et al.
Impact of an evidence-based bundle intervention in the
quality-of-care management and outcome of Staphylococ-
REFERENCES cus aureus bacteremia. Clin Infect Dis. 2013;57(9):1225-
1. Richter SS, Doern GV, Heilmann KP, et al. Detection 1233.
and prevalence of penicillin susceptible Staphylococ- 10. Li JS, Sexton DJ, Mick N, et al. Proposed modification to
cus aureus in the United States in 2013. J Clin Microbiol. the Dulce criteria for the diagnosis of infective endocardi-
2016;54:812-814. tis. Clin Infect Dis. 2000;30:633-638.
This page intentionally left blank
Sepsis
12 Emily L Heil

PATIENT PRESENTATION .- HEENT


Dry mucous membranes, neck supple, oropharynx clear.
Chief Complaint
Endotracheal tube and nasogastric tube in place
Unable to obtain due to clinical status
.- Respiratory
History of Present Illness Respirations are unlabored and ventilator dependent.
JG is a 72-year-old woman who was brought to the emergency Decreased breath sounds in bilateral bases. No rales
department by her daughter when she noticed the patient was
more confused than her baseline and was found to have a .- Cardiovascular
high fever with rigors. The daughter notes that the patient had Tachycardic, normal Sl-$2, regular rate and rhythm. No mur-
complained of fatigue and back/abdominal pain accompanied mur, rubs, or gallops
by nausea and vomiting 4 days prior to presentation. JG devel-
oped respiratory distress in the emergency department and .- Abdominal
required intubation. Soft, tender, non-distended, positive bowel sounds in all
quadrants
Past Medical History
Depression, hypertension, chronic kidney disease (baseline .- Genitourinary
SCr 1.9 mgldL). coronary artery disease-stable angina Foley catheter in place. Moderate suprapubic tenderness
Famlly History .- Slcln/EJCtl'emltles
Father passed away at age 80 from a stroke; mother had type 2 Dry, no rash. No peripheral edema
diabetes and passed secondary to breast cancer.
.- Neurology
Social History Pupils equal and reactive. alert and oriented X2
Widowed, lives with her daughter's family. Drinks alcohol
occasionally, never smoker Laboratory Findings
N~ 136 mEq/L Hgb: 9.5 g/dL Ca:7.2mg/L
Allergies
K:4.9mEq/L Hct:28% Mg:2.0mg/L
Penicillin (rash)
Cl: 100 mEq/L Plt: 90 Phos: 5.5 mEq/L
Home Medications co 1
: 11 mEq/L WBC: 21.2 x 10'/L AST:26W/L
Escitalopram 10 mg PO daily BUN: 29 mg/dL PT: 15.9 sec ALT:24IU/L
Amlodipine 10 mg PO daily SCr: 3.0 mg/dL JNR: 1.2 T bill: 2.0 mg/dL
Labetalol 600 mg PO qSh Glu: 140mg/dL PTT:62sec A1k Phos: 78 IU/L
Lisinoprll 40 mg PO daily
Nitroglycerin 0.4 mg sublingual PRN chest pain Lactate: 4.8 mmol/L
ABG: 7.17/30/80/11, prior to intubation
Physical Exam Urine output: 40 ml in 4 hours
.- Vital Signs Urinalysis:
Temp 39.4°C; BP 86/50 mm Hg; MAP 62; HR 123; RR 24 Appearance Cloudy
breaths per minute; 0 2 sat 90% on 50% Fi02, qSOFA =3, Wt Coler Yellow
68 kg, Ht 5' 4" pH 6.5
Specific gravity 1.0005
.- General Glucose Negative
Responsive, intubated. appears well nourished Kctones Negative
47
48 INFECTIOUS DISEASES: A CASE STUDY APPROACH

Leukocyte esterase +3 cultures. What would be the most appropriate change to her
Nitrites +1 antibiotic therapy?
WBC/hpf >SO A. No changes should be made to her antibiotic therapy
RBC/hpf 0-2 until full species identification and susceptibilities have
Bacteria Large been reported from the microbiology laboratory
Epithelial cells Negative B. Discontinue her vancomycin and continue cefepime
monotherapy
Urine and blood cultures pending
C. Discontinue her vancomycin and cefepime; start
ceftriaxone
D. Add empiric metronidazole for coverage of gram-negative
QUESTIONS
anaerobes
1. Based on the information provided, which of the following 7. Which of the following statements regarding duration of
is the most appropriate classification of JG's condition? antibiotic therapy for JG is true?
A. Sepsis A. All patients with septic shock should receive at least
B. Severe sepsis 7 days of antibiotic therapy
C. Septic shock B. Duration of antibiotic therapy should be chosen based
D. Uncomplicated urinary tract infection on the identified source of infection
2. Which of the following are included in the qSOFA score for C. Antibiotics should be discontinued after the patient's
identifying high-risk patients for in-hospital mortality with SOFA score decreases to <2
suspected sepsis? D. Antibiotics should be given for infection prophylaxis if
A. Heart rate >90 bpm with vasopressor therapy an alternate explanation for the shock is identified
B. Alteration of mental status
C. Mean arterial pressure ~65 mm Hg with vasopressor
therapy ANSWERS
D. Increased serum creatinine of 0.5 mg/dL from baseline 1. Explanation: The correct answer is C: Septic shock. Sepsis is
or 50% increase life-threatening organ dysfunction caused by a dysregulated
host response to infection. 1 The systemic inflammatory
3. All of the following interventions should be provided to response syndrome (SIRS) criteria are no longer considered
JG within her first hour of presentation according to the part of the definition of sepsis due to inadequate sensitiv-
Surviving Sepsis Campaign 2018, except: ity and specificity. Instead, sepsis is defined as a suspected
A. Obtain blood cultures prior to administration of or documented infection with either a "quick" Sequential
antibiotics Organ Failure Assessment (SOFA) score of at least 2 or an
B. Administer 30 mL/kg of crystalloid fluids for hypoten- acute increase of ~2 points on the full SOFA score. Severe
sion or lactate ~4 mmol/L sepsis was an old definition from a 1991 consensus confer-
C. Administer broad-spectrum antibiotics within 1 hour ence that was sepsis complicated by organ dysfunction. This
of identification of sepsis definition was sunset with the 2016 Sepsis-3 Consensus
D. Administer 30 mL/kg of 5% albumin Definitions. Septic shock is a subset of sepsis where under-
lying circulatory and cellular abnormalities are profound
4. Which empiric antimicrobial therapy is most appropriate
enough that the patient is at substantially increased risk of
to start for JG at this time?
mortality. Patients with septic shock meet the definition for
A. Ceftriaxone
sepsis and have hypotension requiring vasopressor therapy
B. Vancomycin + Ceftriaxone
to maintain a MAP ~65 mm Hg and having a serum lactate
C. Vancomycin + Cefepime
level greater than 2 mmol/L after adequate fluid resuscita-
D. Vancomycin
tion.1 While we do not have information on current medi-
5. Which of the following dosing strategies would best optimize cations for JG, we do know that her MAP is <65 mm Hg
the pharmacokinetic/pharmacodynamic principles and spe- and her lactate is >2 mmol/L indicating that she has septic
cific drug properties for JG's antibiotic regimen? shock.
A. Dosing vancomycin to achieve a trough of 5 to I 0 mg!L
B. Dosing vancomycin to achieve a trough of 20 to 25 mg/L 2. Explanation: The correct answer is B. The Quick Sequen-
C. Administering the cefepime as an extended infusion tial Organ Failure Assessment (qSOFA) criteria are altera-
after the first dose tion in mental status, systolic blood presmre ~ 100 mm Hg
D. Doubling the dose of cefepime and administering it less or respiratory rate 2::22 breaths/min. Patients with a score
frequently of 2 to 3 are at a 3-14-fold increase for in-hospital mortal-
ity. The tool was introduced by the Sepsis-3 task force as
6. Twenty-four hours after her presentation, JG's cultures a rapid bedside clinical score that could be used to iden-
return with gram-negative rods in both her blood and urine tify patients with a suspected infection at a greater risk
CHAPTER 12 I SEPSIS 49

for mortality. The score is meant to replace the systemic is added to provide broad-spectrum gram-positive cover-
inflammatory response syndrome (SIRS) criteria that were age including MRSA; however, vancomycin alone would be
previously used to identify sepsis as they are less sensitive inappropriate as it only provides gram-positive coverage.
and specific. •.z The options of ceftriaxone monotherapy or vancomycin
plus ceftriaxone alone are inadequate as ceftriaxone does
3. Explanation: The correct answer is D. The Surviving not provide coverage of P. aeruginosa. Once JG's cultures
Sepsis Campaign bundle was updated in 2018 to a single return, her antibiotic therapy should be promptly nar-
"hour-1 bundle" of resuscitation and management rec- rowed to eliminate any unnecessary coverage.
ommendations that should be completed within the first
hour of identification of sepsis or septic shock. The bundle 5. Explanation: The correct answer is C. Early optimization
elements include measuring a lactate level, obtaining of antimicrobial dosing is important to improve outcomes
blood cultures prior to the administration of antibiotics, in patients with severe infections, particularly as critically
administering broad-spectrum antibiotics, beginning rapid patients often have altered pharmacokinetics. Vancomydn's
resuscitation by administering 30 mLJkg of crystalloid (eg, efficacy is best described by the area under the concentra-
normal saline, lactated ringers) for hypotension or lactate tion time curve to minimum inhibitory concentration ratio
~4 mmoVL and to apply vasopressors if patients remain (AUC:MIC). Trough concentrations are typically used as a
hypotensive during or after fluid resuscitation to maintain surrogate marker with troughs of 15 to 20 mg/L used for
a MAP ~65 mm Hg.3 Administration of colloids such as serious infections. A trough concentration target of 5 to
albumin is not recommended as part of the bundle due to 10 mg/L is too low and risks the development of resistance,
an absence of data supporting a clear benefit to colloids whereas a trough concentration >20 mg/L puts the patient
compared to crystalloids for resuscitation in sepsis. at increased risk of vancomycin-associated nephrotoxicity.6
For beta-lactams, the pharmacokinetic/pharmacodynam-
4. Explanation: The correct answer is C. One of the most ics target is maximizing time that the plasma concentration
important interventions in the management of a patient of the drug is above the pathogen MIC relative to the dos-
with septic shock is prompt initiation of broad-spectrum ing interval. The best way to do this is to extend the infu-
antibiotics with coverage of gram-positive and gram- sion of the drug for several hours. The first dose should be
negative organisms. Antibiotics should ideally be given given as a bolus to rapidly achieve therapeutic blood levels
with I hour of the patient's presentation with septic shock with the infusion time extended for subsequent doses only.
but after appropriate cultures have been obtained. 3 Failure If extended infusions cannot be used due to things such as
to promptly initiate appropriate antibiotic therapy is asso- insufficient line access, increasing the frequency of dosing
ciated with increased morbidity and mortality in patients is a simple way to increase time above MIC.'
with sepsis and septic shock.• Antibiotic selection should
6. Explanation: The correct answer is B. Due to the increasing
be chosen based upon patient-specific factors such as the
frequency of antibiotic resistance worldwide, it is important to
suspected site of infection, chronic organ failures, recent
avoid unnecessary use of broad-spectrum antibiotics when-
infections or known colonization with other organisms,
ever possible. Given the high mortality associated with sepsis
and allergies. Other factors to consider are the preva-
and septic shock. it is appropriate to provide empiric broad-
lence of certain pathogens such as methicillin-resistant
spectrum therapy until a causative organism and its suscepti-
Staphylococcus aureus (MRSA) within the community
bilities are defined. 4 However, attempts to eliminate coverage
and the hospital, which can be found in local antibio-
that is no longer needed and to de-escalate broad-spectrum
grams.• Typically, therapy involved an agent with MRSA
agents to more narrow-spectrum agents tailored toward the
coverage plus an anti-pseudomonal beta-lactam such as
patient's cultures are an important antimicrobial stewardship
a broad-spectrum carbapenem (eg, meropenem, imipe-
initiative. At this point, we know that JG's cultures have gram-
nem/cilastatin), a higher-generation cephalosporin (eg,
negative organisms, so discontinuing her broad-spectrum
ceftazidime or cefepime), or an extended-range penicillin/
gram-positive coverage that was provided by vancomycin is
beta-lactamase inhibitor combination (eg, piperacillin/
appropriate. Since we do not have further identification of
tazobactam). Based on JG's symptoms, it is likely that she
the organism, it would not be appropriate to de-escalate to
has sepsis from a urinary source so the most likely patho-
ceftriaxone from cefepime, as anti-pseudomonal coverage
gens would be gram-negative bacteria; however, given
is still warranted. Adding metronidazole for gram-negative
the severity of her presentation, she should be provided
anaerobic coverage is not indicated at this time as these organ-
with broad-spectrum coverage of both gram-positive and
isms would be unlikely causative agents for a urinary source
gram-negative organisms until her cultures return. As JG
of infection.
has a reported allergy to penicillins, with limited history
on the reaction and timeline, use of the fourth-generation 7. Explanation: The correct answer is B. Most infections can
cephalosporin cefepime is unlikely to result in an allergic be adequately treated with a 7-day course of antimicrobial
reaction due to low rates of cross-reactivity and can be used therapy; however, some infections have data for shorter
for broad-spectrum coverage including Pseudomonas aeru- courses such as intra-abdominal sepsis with source control,
ginosa.5 Often in empiric treatment of sepsis, vancomycin and some require much longer courses such as complicated
50 INFECTIOUS DISEASES: A CASE STUDY APPROACH

S. aureus bacteremia. Therefore, the primary driver for 2. Seymour CW, Liu VK, Iwashyna TJ, et al. Assessment of
duration of therapy in patients with septic shock should clinical criteria for sepsis for the Third International Con-
be based on the underlying source of infection identified. sensus Definitions for Sepsis and Septic Shock (Sepsis-3).
For JG, since it has been determined that she had sepsis JAMA. 2016;315:762-774.
from a urinary source, her duration of therapy would be 3. Levy M, Evans LE, Rhodes A. The surviving sepsis
guided by the therapeutic guidelines for complicated uri- campaign bundle: 2018 update. Crit Care Med. 2018;
nary tract infections. While resolution of symptoms is an 46:997-1 OOO.
important factor in determining if it is appropriate to stop a 4. RhodesA,EvansLE,AlhazzaniW,etal. Surviving sepsis cam-
patient's antibiotics, use of the SOFA score to dictate dura- paign: International Guidelines for Management of Sepsis
tion is not appropriate. Additionally, antibiotics should be and Septic Shock: 2016. Crit Care Med. 2017;45:486-552.
promptly discontinued when patients have severe inflam- 5. Campagna JD, Bond MC, Schabehnan E, Hayes BD. The
matory states of non-infectious origin, and not be used for use of cephalosporins in penicillin-allergic patients: a lit-
prophylaxis.4 erature review./ Emerg Med. 2012;42:612-620.
6. Rybak MJ, Lomaestro BM, Rotschafer JC, et al. Vanco-
mycin therapeutic guidelines: a summary of consensus
REFERENCES recommendations from the Infectious Diseases Society of
1. Singer M, Deutschman CS, Seymour CW, et al. The Third America, the American Society of Health-System Phar-
International Consensus Definitions for Sepsis and Septic macists, and the Society of Infectious Diseases Pharma-
Shock (Sepsis-3). /AMA. 2016;315:801-810. cists. Clin Infect Dis. 2009;49:325-327.
13 Human
Immunodeficiency
Virus and
Opportunistic
Infections
Elizabeth Sherman

PATIENT PRESENTATION Social History


Chief Complaint Works as a cashier at a grocery store. Single, sexually active
with a monogamous male partner, does not regularly use con-
"I'm here for my follow-up visit to start HIV treatment and traception, lives alone. Never smoked, drinks oe<:asionally
fm concerned. about these white spots in my mouth. This is (2 to 4 drinks per week on the weekend).
the first time I have had them; they dont go away. I tried to
scrape them off and they just come back."
Allergles
History of Present Illness Sulfa (history of trimethoprim/sulfamethoxazole causing
Stevens-Johnson syndrome)
AS is a 25-year-old woman who comes to the family health
clinic for her routine HIV care, where she had deferred. starting
antiretroviral therapy beause she ·re1t fine" and because she Medications
was •not sure those drugs work anyway.' Six. weeks ago, while Atovaquone 1500 mg (10 mL) PO daily
on an extended family vacation she was admitted. to the hos- Omeprazole 20 mg PO daily
pital with Pneumocystis jirovecii pneumonia. Upon discharge.
she saw a physician friend of the family who convinced her to Physical Examination
start an antiretroviral regimen. Today the patient complains
of white, non-painful, plaques inside her mouth and on her .., Vital Signs
tongue for the past two weeks consistent with oropharyngeal Temp 98.9°F, P 87 beats per minute, RR 13 breaths per minute,
candidiasis. BP 125176 mm Hg, Ht 5 19 11, Wt 72 kg

Past Medical History ... General


HIV diagnosed 2 years ago; risk factor: heterosexual contact Pleasant, pale-looking female .in no acute distress
Pneumocystis pneumonia diagnosed 6 weeks ago
History of peptic ulcer 4 years ago, treated with antibiotic .._ HEENT
therapy and maintained on PPI Normocephalic, atraumatic, PERRLA, EOMI, visible white
plaques on the buccal area and tongue (no plaques seen in
Surgical History throat), poor dentition, L neck lymph node 0.8 cm in diameter
None .., Pulmonary
Effort normal and breath sounds normal
Family History
Father has hypertension and hyperlipidemia; mother has ... CardlovasculoT
breast cancer and hyperlipidemia Normal rate. regular rhythm, and normal heart sounds

51
52 INFECTIOUS DISEASES: A CASE STUDY APPROACH

~ Abdomen 3. For which opportunistic infection does AS require primary


Soft, non-distended, non-tender, bowel sounds normal prophylaxis?
A. Toxoplasma gondii encephalitis
~ Genitourinary B. Mycobacterium avium complex disease
C. Pneumocystis jirovecii pneumonia
Deferred
D. Oral candidiasis
~ Neurology 4. Which of the following describes a recommended initial
antiretroviral regimen for most people with HIV that
A&O X3
should be recommended to AS?
A. 1 integrase strand transfer inhibitor+ 2 nucleos(t)ide
~ Extremities reverse transcriptase inhibitors
Normal B. 1 boosted protease inhibitor+ 2 nucleos(t)ide reverse
transcriptase inhibitors
C. 1 non-nucleoside reverse transcriptase inhibitor + 2
Laboratory Findings
nucleos(t)ide reverse transcriptase inhibitors
Na = 141 mEq/L Hgb = 12 g/dL Ca = 9.3 mg/dL D. 1 entry inhibitor + 2 nucleos(t)ide reverse transcrip-
K= 4.2mEq/L Hct = 39% P04 = 3.8mEq/L tase inhibitors

Cl = 103 mEq/L 3
Plt = 206 x 10 /rrun 3
AST = 20 IU/L 5. Which one of the following is considered a recommended
C02 = 27 mEq/L WBC = 7.3 Xl03 mm3 ALT= 31 IU/L initial regimen for most people with HIV and represents
the best regimen for AS?
BUN = 12 mg/dL T Bili = A. Rilpivirine-tenofovir disoproxil fumarate-emtricitabine
0.7 mg/dL B. Darunavir-cobicistat plus abacavir-lamivudine
SCr = 1.1 mg/dL Allc Phos = C. Elvitegravir/cobicistat/tenofovir alafenamide/emtricitabine
76 IU/L D. Bictegravir-tenofovir alafenamide-emtricitabine
Glu = 95 mg/dL 6. After initiating antiretroviral therapy, which one of the
following approaches is recommended for monitoring AS's
CD4+ 78 cells/mm3 (6 weeks ago during hospitalization) initial response to treatment?
HIV RNA 123,000 copies/ml (6 weeks ago during A. Monitor the CD4 count monthly until greater than 500
hospitalization) cells/mm3
CD4+ 68 cells/mm3 (today) B. Check the HIV RNA within 2 to 4 weeks and then
HIV RNA 127,000 copies/ml (today) every 4 to 8 weeks until the HIV RNA level is below
Toxoplasmosis IgG ( +) the assays limit of detection
HLA-B""5701 ( +) C. Repeat the HIV RNA and CD4 count every 16 weeks
until steady state is reached
D. Monitor the HIV RNA every 4 weeks until below 200
copies/ml, then monitor every 12 weeks until below
QUESTIONS the assay's limit of detection
1. Which information (signs, symptoms, laboratory values) 7. AS has a urine pregnancy test performed in the clinic
from AS's case indicates an AIDS diagnosis? today and it is negative. She states that her male partner
A. Lymphadenopathy uses condoms "'most of the time." She inquires into using a
B. Oral candidiasis combination oral contraceptive containing levonorgestrel
C. HIV RNA above 100,000 copies/ml and ethinyl estradiol. Which one of the following antiretro-
D. CD4 + below 200 cells/mm3 viral regimens would present the lowest risk for drug-drug
2. What should AS be told regarding the need for antiretrovi- interactions with this oral contraceptive?
ral therapy? A. Efavirenz-tenofovir disoproxil fumarate-emtricitabine
A. Therapy is recommended for all persons with HIV B. Darunavir plus ritonavir plus tenofovir disoproxil
infection fumarate-emtricitabine
B. Therapy is recommended only for persons who have a C. Dolutegravir plus tenofovir alafenamide-emtricitabine
documented decline in CD4+ of at least 100 cells/mm3 D. Elvitegravir-cobicistat-tenofoviralafenamide-emtricitabine
C. Therapy can be considered but is not necessarily 8. Which one of the following is recommended for the initial
recommended treatment of AS's oral candidiasis infection?
D. Therapy is recommended only for persons who have an A. Liposomal amphotericin B 3 to 4 mg/kg IV daily for 21
HIV RNA above 30,000 copies/ml days
CHAPTER 13 I HUMAN IMMUNODEFICIENCY VIRUS AND OPPORTUNISTIC INFECTIONS 53

B. Micafungin 150 mg IV daily for 14 days 50 cells/mm3 after ruling out disseminated active MAC
C. Fluconazole 100 mg PO daily for 14 days infection and if the patient is not immediately initiating
D. Itraconazole capsules 200 mg PO daily for 21 days ART, is not receiving ART, or remains viremic while on
ART with no options for a fully suppressive ART regi-
9. AS asks if she can stop taking atovaquone. Which of the
men (rules out answer B).1 Because AS has already had
following is one of the recommended indications for discon-
Pneumocystis jirovecii pneumonia, she is not a candidate
tinuing Pneumocystis pneumonia secondary prophylaxis?
for primary prophylaxis, but rather she is a candidate
A. Normal chest radiograph 6 months after starting anti-
for secondary prophylaxis (rules out answer C). Routine
retroviral therapy
primary prophylaxis for oral candidiasis is not recom-
B. CD4+ increased from below 200 cells/mm3 to above
mended (rules out answer D). 1
200 cells/mm' for more than 3 months in response to
antiretroviral therapy 4. Explanation: The correct answer is A. According to
C. CD4+ above 100 cells/mm3 for more than 6 weeks in DHHS guidelines, the recommended initial regimen for
response to antiretroviral therapy most people with HIV is a two-drug nucleos(t)ide reverse
D. Undetectable HIV RNA for more than 6 months in transcriptase inhibitor (NRTI) backbone combined with
response to antiretroviral therapy, regardless of CD4+ an integrase strand transfer inhibitor (INSTI) anchor
drug.2 Regimens that include a boosted protease inhibi-
ANSWERS tor or a non-nucleoside reverse transcriptase inhibitor
are reserved only for certain clinical situations.1 Antiret-
1. Explanation: The correct answer is D. An individual is roviral regimens that include an entry inhibitor are not
diagnosed with AIDS when he or she has lilV and certain recommended in initial regimens for treatment-naive
laboratory values, signs, or symptoms defined by the U.S. patients.2 This information was up to date as of the writ-
Centers for Disease Control and Prevention (CDC).1 The ing of this chapter; however, these recommendations
CDC's definition of AIDS includes CD4+ below 200 cells/ change often. Please check for updates at aidsinfo.nih.gov.
mm3, a CD4 percentage below 14% of all lymphocytes,
5. Explanation: The correct answer is D. According to
or one or more AIDS defining conditions. AS's physical
DHHS guidelines the recommended initial regimen for
exam reveals lymphadenopathy and presumptive oral can-
most people with HIV is a two-drug NRTI backbone
didiasis, which are not AIDS-defining conditions. While
combined with an INSTI anchor drug (rules out answers
candidiasis of the esophagus, bronchi, trachea, or lungs
A and B which are NNRTI- and PI-based, respectively).2
are AIDS-defining conditions, candidiasis of the buccal
Additionally, AS's HLA-B*5701 test result was positive,
area and tongue are not (rules out answers A and B).
indicating that she is likely to develop a hypersensitivity
HIV RNA values do not indicate an AIDS diagnosis (rules
reaction to abacavir (rules out answer B). In this case,
out answer C). Therefore, the only criterion in AS's case indi-
abacavir should not be used and a true abacavir allergy
cating an AIDS diagnosis is her CD4+ below 200 cells/mm3•
should be recorded in AS's medical chart even though she
2. Explanation: The correct answer is A. According to the has never received the drug. Elvitegravir is an integrase
United States Department of Health and Human Services inhibitor; however, due to its need for pharmacokinetic
(DHHS) guidelines panel, antiretroviral therapy is recom- enhancement and lower genetic barrier to resistance, it
mended for all individuals with HIV infection, regardless is now only recommended for initial therapy in certain
of CD4 cell count or HIV RNA, to reduce morbidity and clinical situations (rules out answer C). The best answer
mortality associated with HIV infection and to prevent choice is D, which represents a guideline-preferred regi-
HIV transmission.2 men for most patients with HIV.
3. Explanation: The correct answer is A. Primary prophy- 6. Explanation: The correct answer is B. After initiating anti-
laxis is defined as a medication given to a patient with retroviral therapy, the recommended laboratory monitor-
no prior history or diagnosis of the opportunistic infec- ing approach is to recheck HIV RNA within 2 to 4 weeks
tion, but who is at risk of developing the opportunistic (and no later than 8 weeks) and then recheck every 4 to 8
infection. Secondary prophylaxis, also known as chronic weeks until the HIV RNA level is below the limit of detec-
maintenance therapy, is defined as a medication given tion; subsequently, the HIV RNA should be rechecked
to a patient to prevent a recurrence of a prior, success- every 3 to 4 months (rules out answers C and D).2
fully controlled infection. Primary prophylaxis against For individuals with consistently suppressed HIV RNA
toxoplasmosis is indicated if the CD4+ is below 100 levels for at least 2 years, clinicians may extend the inter-
cells/mm' and the patient is positive for toxoplasma IgG.1 val for monitoring HIV RNA.1 Additionally, although
Because AS meets both of these criteria, she is a candidate the CD4 count is a key factor in assessing the patient's
for Toxoplasma gondii encephalitis primary prophylaxis. overall immune status, frequent monitoring of the CD4
Primary prophylaxis for disseminated Mycobacterium cell count in the initial 12 weeks after starting antiretro-
avium complex disease is only indicated if CD4+ is below viral therapy does not provide meaningful information
54 INFECTIOUS DISEASES: A CASE STUDY APPROACH

(rules out answer A). It is recommended to check a CD4 as esophageal candidiasis (rules out answers A and B). 1
cell count 3 months after starting antiretroviral therapy. Itraconazole capsules are less effective than fluconazole
Thereafter, the CD4 count should be monitored every 3 to 6 because of their more variable absorption and are associ-
months for the first 2 years after starting antiretroviral ated with more drug-drug interactions than fluconazole
therapy. For patients who have persistently undetectable (rules out answer D).1 Additionally, itraconazole absorp-
HIV RNA levels and a CD4 count above 300 cells/mm3 , tion is dependent on gastric acidity and, therefore, the
monitoring of the CD4 cell count can be extended. patient's use of a proton pump inhibitor would interact
with itraconazole capsules.
7. Explanation: The correct answer is C. Both estrogen
and progestin are predominantly metabolized via the 9. Explanation: The correct answer is B. All patients diag-
cytochrome P450 system and can interact with multiple nosed with Pneumocystis pneumonia should receive
antiretroviral agents. Any major alteration in levels of oral chronic maintenance therapy (secondary prophylaxis)
contraceptive agents can result in contraception failure after completing the initial 21-day treatment course. 1 The
and unintended pregnancy. Dolutegravir does not have Opportunistic Infections Guidelines recommend dis-
any significant drug interactions with oral contracep- continuation of secondary prophylaxis for Pneumocystis
tives, and thus provides a safe option for coadministra- pneumonia if a patient has responded to antiretroviral
tion with oral contraceptives.2 Preliminary data suggest therapy with a sustained increase in CD4+ from below
an increased risk of neural tube defects in infants born 200 cells/mm3 to above 200 cells/mm3 for longer than
to women who were receiving dolutegravir at the time 3 months. 1 In addition, the Opportunistic Infections
of conception.2 For those using effective contraception, Guidelines note that clinicians can consider discontinu-
such as the hormonal contraceptive described above, a ing secondary prophylaxis for Pneumocystis pneumonia if
dolutegravir-based regimen can be considered. Dolute- the patient has a CD4+ 100 to 200 cells/mm3 and the HIV
gravir should not be prescribed to women who are RNA has remained below the limit of detection for at least
pregnant and within 12 weeks post-conception; or who 3 to 6 months.1 Note that for Pneumocystis pneumonia,
are of childbearing potential and planning to become the criteria for discontinuing and restarting secondary
pregnant; or who are of childbearing potential, sexually prophylaxis are the same as for discontinuing and restart-
active, and not using effective contraception. 2 Although ing primary prophylaxis.
efavirenz does not significantly alter ethinyl estradiol
plasma concentrations, it decreases levonorgestrel levels
by 83%; if efavirenz is used with an oral hormonal con- REFERENCES
traceptive, the recommendation is to use alternative or
1. Panel on Opportunistic Infections in HIV-Infected Adults
additional contraceptive methods (rules out answer A).l
and Adolescents. Guidelines for the prevention and treat-
Ritonavir-boosted protease inhibitor regimens cause a
ment of opportunistic infections in HIV-infected adults
moderate decrease in plasma levels of ethinyl estradiol and
and adolescents: recommendations from the Centers for
norethindrone, warranting the recommendation to con-
Disease Control and Prevention, the National Institutes of
sider using alternative or additional contraceptive methods
Health, and the HIV Medicine Association of the Infec-
or an alternative antiretroviral regimen (rules out answer B).2
tious Diseases Society of America. Available at https://
The effects of cobicistat on hormonal contraceptives have
aidsinfo. nih.govI guidelines/html/4/ ad ult-and-adoles-
not been well studied and experts recommend using alter-
cent-oi-prevention-and-treatment-guiddines/O. Accessed
native methods of contraception if a cobicistat-containing
March 25, 2019.
regimen is used (rules out answer D).2
2. Panel on Antiretroviral Guidelines for Adults and Ado-
8. Explanation: The correct answer is C. Oral fluconazole lescents. Guidelines for the use of antiretroviral agents
for 7 to 14 days is considered the drug of choice to treat in adults and adolescents living with HIV. Department
oropharyngeal candidiasis. 1 Intravenous antifungal agents of Health and Human Services. Available at https://aid-
are not warranted for oropharyngeal candidiasis, although sinfo. nih.govI guidelines/html/1/adult-and-adolescent-
they are considered alternative therapy to fluconazole for treatment-guidelines/0. AccessedMarch 25, 2019.
the treatment of more severe candida infections such
14 Febrile
Neutropenia
Wesley D. Kufel

PATIENT PRESENTATION diagnosis of acute myeloid leukemia (AMI.). His induction che-
motherapy regimen consists of 7 + 3 induction chemotherapy
Chief Complaint with cytarabine and daunorubicin. He was placed on neutrope-
"I have a fever and chills." nic precautions. started on appropriate antimicrobial prophyluis,
and a port was placed fur chemotherapy administration. Ten days
History of Present Illness after the completion of his induction chemotherapy (day 17), he
JP is a 34-year-old Caucasian male who is admitted to inpa- spikfd a fever of 38.S°C (10l.8°F) and complained of chills and
tient oncology service for induction chemotherapy for a recent nausea. Details of the hospital course by day are detailed below:

Day 1 2 3 4 7 9 12 17
WBC 4.5 4.1 3.7 3.2 1.4 0.7 0.2 0.1
Comments Daunorubicin. Daunorubicin Nausea Cytarabine Nausea Nausea Fever, chills,
and cytarabine completed; completed; nausea (current
started nausea nausea presentation)

Past Medical History Inpatient Medications


Epilepsy. depression Sertraline 1000 mg PO daily
Divalproex. sodium delayed-release 1000 mg PO BID
Surgical History Levetiracetam 1000 mg PO BID
Posaconazole delayed-release 300 mg PO BID x 1 day
Appendectomy (10 years ago)
followed by 300 mg PO daily {day 1 start)
Valacyclovir 500 mg PO BJD (day 1 start)
Family History Levofloxacin 500 mg PO daily (day 1 start)
Father had AML and passed away 8 years ago; mother has
hypertension and epilepsy; no siblings
Review of Systems (day 17)
Positive for fever, chills. and nausea; denies vomiting. cough,
Social History
diarrhea. or abdominal pain
Married with twins (S years old). Has never smoked and
denies illicit drug use. Drinks alcohol socially. He was previ-
ously enlisted in the Navy and is currently a gym teacher at a Physical Examination (day 17)
local elementary school .... Vital Signs
Temp 38.8°C, P 105 bpm, RR 16 breaths per minute,
Allergies BP 112/78 mm Hg. p02 98%, Ht 5' 8", Wt 74 kg
Penicillin (reaction: rash)
...., GeneTOI
Home Medications Male with fatigue and in mild to moderate distress
Sertraline 1000 mg PO daily
Divalproex sodium delayed-release 1000 mg PO BID ...., HEENT
Levetiracetam 1000 mg PO BJD PERRLA, EOMI, good dentition, no rhinorrhea or
Multivitamin PO daily mucositis
55
56 INFECTIOUS DISEASES: A CASE STUDY APPROACH

~ Pulmonary C. 14/mm3
Normal lung sounds; no wheezes, crackles, or rhonchi D. 5/mm3
2. Which of the following is TRUE regarding neutropenic
~ Cardiovascular fever?
Tachycardic but regular rhythm; no murmurs, rubs, or gallops A. Common signs and symptoms of infection are often
absent
~ Abdomen B. An oral temperature of 37.8°C represents a fever
Soft, non-distended, non-tender, normal bowel sounds C. Febrile neutropenia is more common in solid tumor
patients compared to acute leukemia patients
~ Genitourinary D. Chemotherapy-induced mucositis is an uncommon
cause of neutropenic fever
Normal male genitalia, no complaints of dysuria or hematu-
ria, prostate exam not performed 3. Which of the following is TRUE regarding infectious
pathogens in patients with neutropenic fever?
~ Neurology A. Fungi are the most frequent cause of neutropenic
A&O X3 fever
B. Gram-positive bacterial infections are more common
~ Extremities than gram-negative bacterial infections in neutropenic
fever
Warm and dry. No erythema or induration around port on
C. Infectious sources are frequently identified in febrile
the left chest. No rashes present
neutropenia cases
D. Viruses should not be considered in the workup of neu-
Laboratory Findings (day 17) tropenic fever
Na = 135 mEq/L Ca = 8.0 mg/dL Lymphs= 81% 4. Based on this patient's history, how should you categorize
K=4.0mEq/L Hgb = 7.8 g/dL AST= 18IU/L JP's neutropenic fever risk?
Cl= 97mEq/L Hct= 23.4% ALT= 22IU/L A. Low risk -+ Treat as outpatient with oral antibiotics
B. Low risk -+ Treat as outpatient with intravenous
C02 = 25 mEq/L Plt=23 x W /mm3 T Bili = 0.8 mg/dL antibiotics
BUN = 14 mg/dL WBC = 0.1 cells/ Alk Phos = 40 IU/L C. High risk -+ Treat as inpatient with oral antibiotics
mm3 D. High risk -+ Treat as inpatient with intravenous
SCr = 0.8 mg/dL PMNs= 14% HSV IgG = positive antibiotics
(day 1 lab) 5. Which of the following antibiotics should be started for JP
Gk = 84mg/dL Bands= 5% CMV IgG = negative on day 17 in response to his fever spike?
(day 1 lab) A. Discontinue levofloxacin prophylaxis and start
ertapenem
~ Blood Cuhures B. Discontinue levofloxacin prophylaxis and start
Periphery X2 (pending); port catheter (pending) piperacillin-tazobactam
C. Discontinue levotloxacin prophylaxis and start cefepime
~ Urinalysis D. Discontinue levofloxacin prophylaxis and start
0 WBC, leukocyte esterase negative, nitrite negative; urine meropenem
culture not performed 6. Intravenous vancomycin should be added to JP's empiric
antibiotic regimen.
~ Methidllin-Resistant Staphylococcus aureus (MRSA) A. True
Nasal Polymerase Chain Reaction (PCR) Saeen B. False
Negative
7. JP was started on appropriate empiric antibacterial therapy
based on the pharmacist's recommendations. Blood cul-
~ Chest X-ray
tures from the port and periphery were collected prior
Lungs are clear to antibiotic administration on day 17. On day 18, both
blood cultures were positive with gram-negative rods on
QUESTIONS Gram stain. On day 19, both blood cultures are growing
Pseudomonas aeruginosa with the following susceptibility
1. Which of the following is the correct absolute neutrophil results: piperacillin-tazobactam (susceptible); ceftazidirne
count (ANC) for JP on day 17? (susceptible); cefepirne (susceptible); meropenem (suscep-
A. 100/mm3 tible); ciprotloxacin (resistant); gentamicin (susceptible);
B. 19/mm3 tobramycin (susceptible). Which of the following would be
CHAPTER 14 I FEBRILE NEUTROPENIA 57

the MOST appropriate recommendation for management The most common contributing factors of neutropenic
in this patient? fever are the direct effects of chemotherapy on mucosal
A. Remove the catheter/port and continue cefepime barriers and the immune system as well as breeches in
B. Remove the catheter/port, discontinue cefepime, and host defense mechanisms. Mucositis occurs when normal
start meropenem flora (ie, from the gastrointestinal tract) crosses mucosal
C. Retain the catheter/port, start gentamicin lock therapy, membranes and translocates into the bloodstream caus-
and continue cefepime ing infection.
D. Remove the catheter/port, continue cefepime, and add
3. Explanation: The correct answer is B. Bacteria are the
gentamicin
most frequent infectious causes of neutropenic fever
8. Empiric antifungal therapy should be considered after 4 to 7 rather than fungi. 1.i The risk for invasive fungal infec-
days of appropriate empiric antibacterial therapy in high- tions increases with the duration and severity of neu-
risk neutropenic patients who continue to be febrile. tropenia, prolonged antibiotic therapy, and number or
A. True intensity of chemotherapy. Fungi are rarely identified as
B. False the cause of the first febrile episode, but rather after 1
to 2 weeks of neutropenia. This makes option A incor-
rect. Gram-negative bacteria were historically the most
ANSWERS common cause of neutropenic fever, but gram-positive
1. Explanation: The correct answer is B. The absolute neu- bacteria have become the most common pathogens.
trophil count (ANC) determines the degree of neutro- Many gram-positive bacteria are considered normal flora
penia.1.i The ANC is calculated by adding the percent of on the skin; however, less virulent bacteria (ie, coagulase-
polymorphonuclear cells (PMNs) (or the mature segs) to negative staphylococci) may become pathogenic in neu-
the percent of bands (immature form) and multiply this tropenic patients where host immunity is compromised.
percentage by the total white blood cell (WBC) count. In Additionally, oral levofloxacin is commonly used for
this case, the total WBC count is 100 and percent PMNs prophylaxis in neutropenic patients, and generally has
or segs are 14% and the percent bands are 5%. When greater activity against gram-negative bacteria compared
these values are added together, the total percentage is to certain gram-positive bacteria. This makes option B
19%. When the total percentage (19% or 0.19) is multi- correct. Unfortunately, an infectious source is often
plied by the total WBC count ( 100), the ANC is calculated only identified in approximately 20% to 30% of febrile
as 19/mm3 making option B correct. Option A represents neutropenia cases. This makes neutropenic fever chal-
the WBC count. Option C represents the WBC multiplied lenging for clinicians especially if the source of infection
by the percent PMNs only. Option D represents the WBC is not clearly identified. This makes option C incorrect.
multiplied by the percent bands only. The definition of Viral infections are common in high-risk patients with
neutropenia can vary between institutions, but is usually febrile neutropenia. Human herpesviruses are among the
defined as an absolute neutrophil count (ANC) < 1500 most common viral pathogens especially in patients who
or 1000/mm3 , and severe neutropenia is usually defined are seropositive. This makes option D incorrect. In this
as an ANC <500/mm3 or an ANC that is expected to case, JP's herpes simplex virus (HSY) IgG was positive
decrease to <500/mm3 over the next 48 hours. Profound and reactivation of latent infection is common in this
neutropenia is typically defined as an ANC <100/mm3• scenario. Therefore, patients typically receive an antiviral
This patient has an ANC < 100/mm3, which is considered (ie, valacyclovir) for HSY prophylaxis. Patients with AML
as profound neutropenia. who undergo induction chemotherapy are commonly
given an extended-spectrum azole antifungal with mold
2. Explanation: The correct answer is A. Option A is correct
coverage (ie, posaconazole) for prophylaxis against fungal
since common signs and symptoms of infection are often
infections as well.
absent in neutropenic fever because the magnitude of the
neutrophil-mediated component of the inflammatory 4. Explanation: The correct answer is D. Risk assessment is
response may be blunted in neutropenic patients. i.i These needed to determine the type of empiric antibiotic ther-
patients may not have the same clinical presentation apy (oral vs intravenous), location of treatment (inpatient
or signs and symptoms as immunocompetent patients. vs outpatient), and duration of antibiotic therapy.1.2 High-
Fever is defined as a single oral temperature of ~38.3°C risk patients are considered to be those with anticipated
(101.4 °F) or an oral temperature of38°C to 38.2°C (100.4 prolonged (>7 days duration) and profound neutrope-
to 100.9°F) persisting for one hour or longer. Therefore, nia (ANC :5:100/mm3) following cytotoxic chemother-
option B does not represent a fever. Febrile neutropenia apy and/or significant medical comorbidities including
is more common in acute leukemia patients and patients hypotension, pneumonia, abdominal pain, or neurologic
with hematologic malignancies compared to patients changes. These patients should be admitted to the hospi-
with solid tumors. Therefore, option C is incorrect. tal for empiric intravenous antibiotics. Risk classification
Chemotherapy-induced mucositis is a common cause may also be calculated using the Multinational Associa-
of neutropenic fever, making answer choice D incorrect. tion for Supportive Care in Cancer (MASCC) risk index
58 INFECTIOUS DISEASES: A CASE STUDY APPROACH

score. High-risk patients have a MASCC score <21, any specific factors that would preclude use. Cefepime is
whereas low-risk patients have a MASCC score ~21 and a fourth-generation cephalosporin with a dissimilar R-1
may be candidates for oral antibiotics as an outpatient. side chain compared to penicillin making the potential
JP is considered high risk given his profound neutrope- cross reactivity very low. Thus, option C is corre<:t. This
nia and anticipated prolonged duration of neutropenia patient also has a past medical history of epilepsy and is
seoondary to his chemotherapy regimen's high likelihood currently on divalproex sodium. A drug-drug interaction
of neutropenic duration. Therefore, he should remain as exists between divalproex sodiwu (i.e. valproic acid) and
an inpatient and receive intravenous empiric antibiotics. meropenem since c:arbapenem antibiotics can decrease
A MASCC score could also be used to help calculate JP's valproic acid serum concentrations resulting in a lower
risk. seizure threshold and increasing the risk of seizures.
Therefore, meropenem should be avoided in this case
5. Explanation: The correct answer is C. Neutropenic
making answer choice D incorrect.
fever is considered a medical emergency.1.1 In patients
that are considered high-risk, empiric intravenous (IV) 6. Explanation: The correct answer is B. Vancomycin is
antibiotic therapy should be initiated promptly within not routinely reoommended as part of the initial empiric
the first 60 minutes of presentation and directed against antibiotic regimen (i.e. in combination with an anti-
Pseudomonas aeruginosa after blood cultures and other pseudomonal beta-lactam). 1.a Vancomycin (or another
cultures have been obtained if suspecting infection at antibiotic targeted against gram-positive bacteria) is rec-
specific sites (ie, urine, sputum). Anti-pseudomonal beta- ommended in the following scenarios:
lactam antibiotics are preferred over fluoroquinolones • Hemodynamic instability
and aminoglycosides as monotherapy in neutropenic
fever. Several combination antibiotic regimens have been • Positive blood cultures for gram-positive bacteria
studied as initial empiric therapy in neutropenic fever, while awaiting speciation and susceptibility .results
but none have been shown to be clearly superior tooth- • Skin or soft tissue infection
ers or to monotherapy antibiotic .regimens. Ertapenem • Pneumonia
is a c:arbapenem antfbiotic that does not exhibit activity • Suspected central venous catheter-related infection
against P. aeruginosa and should not be recommended.
This makes option A incorrect. Piperacillin-tazobactam, • Severe mucositis if a patient was receiving
cefepime, meropenem, and imipenem-cilastatin are all Buoroquinolone prophylaxis lacldn.g activity
reasonable antibiotics to consider for empiric antibi- against streptococci and in those who are receiving
otic therapy in febrile neutropenia due to their activity ceftazidime as empiric therapy
against P. aeruginosa. Higher doses of intravenous anti- • MRSA colonization or prior history of MRSA infection
pseudomonal beta-lactams are typically reoonunended While JP has a port in place, there was no erythema or
assuming normal renal function given the severity of the tenderness noted on physical exam around the port site. It
infection (Table 14.1). Extended infusion or continuous is important to remember that common signs and symp-
infusion strategies may be implemented to optimize beta- toms of infection may commonly be absent due to the
lactam antibiotics' time-dependent pharmacodynamics. blunted inflammatory response in neutropenic patients.
This patient has a penicillin allergy with a reaction of Additionally. JP's MRSA nasal PCR results we.re negative.
rash and, therefore, piperaclllin-tazobactam should be
avoided. This makes option B incorrect Cefepime is a 7. El:planation: The correct answer is A. This demonstrates a
reasonable antibiotic to recommend for empiric antibi- catheter-related bloodstream. infection, which is common in
otic therapy for febrile neutropenia and JP does not have febrile neutropenia patients due to the presence ofindwell-
ing catheters for chemotherapy administration.1..a Blood
cultures from peripheral sites and from the port are both
growing Pseudonwnas aeruginosa, which is susceptible to
all ofthe antibiotict reported except ciprofloncin. Catheter
(ie, port) removal is recommended rather than salvaging
the catheter for patients with catheter-related bloodstream
Beta-Lactam infections caused by Staphylococcus aureus, P. aeruginosa,
Piperacillin-tazobactam fungi, or mycobacteria. Therefore, the catheter/po.rt should
be removed making option C incorrect since salvaging the
3.375 g IV every 8 hours
port and using antl'bioticloclc therapy are not recommended
extended infusion with P. aeruginosa catheter-related bloodstream infections.
Cefepime Option B is incorrect since we ha:ve susceptibility data
Ceftazidime Shours and there is no need to broaden therapy to meropenem
Meropenem 1 g IV every 8 hours to cover for extended-spectrum beta-Iactamase (ESBL)-
producing bacteria as well as the drug-drug interaction
lmipenem-cilastatin SOO mg IV every 6 hours with divalproex sodium. Option D is incorrect since we
CHAPTER 14 I FEBRILE NEUTROPENIA 59

have susceptibility data that reports cefepime as suscep- should be considered in patients who are currently receiv-
tible, so there is no need to add gentamicin and put the ing an antifungal agent for prophylaxis.
patient at an increased risk of nephrotoxicity. Therefore,
option A is the correct answer since cefepi.me is sus-
ceptible and the catheter/port should be removed. The REFERENCES
duration of therapy for uncomplicated catheter-related
1. Frefield AG, Bow EJ, Sepkowitz KA, et al. Clinical practice
infections as described in this case is typically 14 days; guideline for the use of antimicrobial agents in neutropenic
however, data suggest that shorter courses of antibiotics patients with cancer: 2010 update by the Infectious Dis-
may be sufficient.'
eases Society of America. Clin Infect Dis. 2011;52:e56-e93.
8. Explanation: The correct answer is A. In most cases, 2. Baden LR, Swaminathan S, Angarone M, et al. Preven-
alterations to the initial empiric antibiotic regimen should tion and treatment of cancer-related infections, version
be guided by clinical and microbiologic data. •.2 However, 2.2016, NCCN Clinical Practice Guidelines in Oncology.
empiric antifungal therapy and evaluation for inva- J Natl Compr Cane Netw. 2016;14(7):882-913.
sive fungal infections should be considered in high-risk 3. Fabre V, Amoah J, Cosgrove SE, Tamma PD. Antibiotic
patients with persistent fevers after 4 to 7 days of appro- therapy for Pseudomonas aeruginosa bloodstream infec-
priate antibiotics and when the duration of neutropenia tions: how long is long enough? Clin Infect Dis. 2019;
is expected to be > 7 days. An alternative antifungal agent 30882137. Epub ahead of print
This page intentionally left blank
15 Skin and Soft
Tissue Infection I
Michael Kelsch

PATIENT PRESENTATION Home Medications


Chief Complaint Duloxetine 60 mg PO daily (nonadherent)
Amitriptyline 50 mg PO at bedtime (nonadherent)
"My hand hurts really bad; it feels hot, like it is on fire."' Albuterol metered-dose-inhaler 1 to 2 puffs q4h PRN short-
ness of breath (nonadherent)
History of Present Illness
SL is 25-year-old man who presents to the emergency depart- Physical Examination
ment with worsening pain, redness, and swelling in his right
hand. He got a splinter of wood on the palmer surface of his .,. Yitai Signs
right hand near the base of his thumb 3 days ago while chop- Temp 98.8°F, HR. 80 beats per minute, RR 16 breaths per minute,
ping wood. He thought he successfully removed the splinter, BP 127/83 mm Hg. p01 97% on room air; Ht 6'2", Wt 67 kg
but the hand progressively became more painful and swollen
where the splinter had been. Today, the burning pain (rated ... General
8/10) hasspread through his forearm and he is now feeling the
He appears thin and not well nourished. Not significantly dis-
pain up to his armpit. He reports that chills started yesterday. tressed, but somewhat anxious.
No numbnets or tingling of the arm or hand, no weakness
noted. Denies any prior issues with the hand.
.,. HEENT
Normocephalic and atraumatic; no sderal icterus; normal
Past Medlcal History
range of motion; neck supple
Recurrent major depressive disorder; ulnar neuropathy ofleft
upper extremity secondary to motor vehicle accident 2 years .,. Pulmonary
ago; hepatitis C
Breath sounds normal. He has no wheezes or rales.
Surgical History ... Cardiovascular
Repair of anterior cruciate ligament in left knee in 2015; Normal rate and regular rhythm; no murmur heard
cholecystectomy in 2013; motor vehicle accident 2 years ago
resulted in anterior cervical disc fusion C6/7, and repair of
.,. Abdomen
cervical fractures
Soft; no tenderness
Famlly History
.,. Neurology
Father has COPD; mother is healthy
Alert and oriented to person, place, and time
Social History .,. Skin
Single; lives with friends; mechanic work occasionally in the Skin is warm and dry, with scarring on various areas of
past, but is currently unemployed. Smokes 1 ppd x 7 years
extremities
and drinks 1 ease of boor per week. Smokes marijuana. and
inhales or injects methamphetamine daily. ... Musculoskeletal
Entry wound on the palmar surface near the base of the right
Allergies thumb, scabbed over with surrounding erythema and swell-
Penicillin (rash); sulfa (hives); morphine (itching) ing. No open wound or discharge. Swelling present throughout

61
62 INFECTIOUS DISEASES: A CASE STUDY APPROACH

the right hand, including the dorsal surface. Hand very ten- 2. Which of the following risk factors does SL have for
der to palpation throughout. Brisk capillary refill. Pain with community-acquired methicillin-resistant Staphylococcus
active and passive movement of hand. Extension and flexion aureus (CA-MRSA)?
of fingers mildly impaired due to swelling. Pain to palpation A. Injection drug use
along forearm. Mild red streaking of the skin extending to the B. Cervical disc fusion 2 years ago
shoulder from the hand. C. Hepatitis C
D. Mechanic work history
3. Which of the following interventions is considered most
Laboratory Findings
essential to ensure optimal management of this type of
Na = 136 mEq/L Hgb = 13.9 gldL Ca = 8.8 mgldL infection?
K= 3.9mEq/L Hct=42.6% Mg= 1.9mg/ A. Incision and drainage of the abscess
dL B. Dressing changes of the infected area
C. Decolonization of nares with intranasal mupirocin, and
d = 101 mEq/L Plt = 226,000/mm3 Phos = 3.1 mg/ skin with chlorhexidine washes
dL D. Anaerobic bacterial culture
C02 = 29 mEq/L WBC = 13,500/mm3 AST= 20IU/L
4. Which empiric antimicrobial therapy is most appropriate
BUN =9mgldL MCV = 98.2 fl/red ALT= 39IU/L
for SL?
cell
A. Vancomycin
Cr = 0.83 mgldL ESR = 10 mm/hr T Bili = 0.4 mg/ B. Trimethoprim/sulfamethoxazole
dL C. Daptomycin
Glu = 90 mgldL CRP = 14.5 mg!L AlkPhos = 86 D. Ceftriaxone
IU/L
5. Regardless of which antibiotic you chose in question 4, the
CK=93 U/L Alb = 4g/dL attending physician would like to start vancomycin, phar-
macy to dose. Please choose the most appropriate starting
dose, and goal monitoring level, for SL.
~ Urine Drug Screen
A. 1350 mg IV q12h; trough level of 10 to 20 mcg/mL
Positive for amphetamines, methamphetamine, benzodiaz- prior to the 2nd dose
epines, marijuana, opiates, oxycodone B. 1350 mg IV q24h; trough level of 10 to 20 mcg/mL
prior to the 4th dose
~ Microbiology C. 2000 mg IV q24h; peak level of 30 to 40 mcg/mL after
the4th dose
Blood cultures x 2 (pending)
D. 1000 mg N q8h; trough level of 10 to 20 mcglmL prior
Culture (aerohie and anaerobic) of fluid from abscess on right
to the 4th dose
hand (pending)
6. After 24 hours, SI:s blood cultures show no growth. How-
Imaging ever, an aerobic culture of fluid from abscess drained on
MRI of right hand and lower right arm: (1) Abscess contact- the right hand indicates moderate WBC (10 to 25/LPF),
ing the skin surface volar aspect hand between the fourth no epithelial cells, moderate gram-positive cocci in clusters
and fifth digits at level of the metacarpophalangeal joint. (6 to 30 per OIF). An anaerobic culture of fluid from the
(2) Insinuating abscess dorsal hand and wrist. Connection abscess showed no growth. Which of the following bacteria
between these 2 abscesses cannot be confirmed but could will most likely be identified from this sample?
be too small to visualize. Possible concern for tenosynovitis. A. Streptococcus pyogenes
(3) No findings of osteoarthritis or septic arthritis within the B. Staphylococcus aureus
hand or wrist C. Bacteroides fragilis
D. Enterococcus faecalis
7. What is the appropriate duration of antimicrobial treat-
QUESTIONS ment for this purulent skin infection that underwent suc-
1. Which of the following findings from the clinical presenta- cessful incision and drainage?
tion correctly identifies the general type and severity of this A. No antibiotics are necessary
skin infection? B. 4to 7 days
A. Purulent, mild C. 7 to 14 days
B. Non-purulent, moderate D. 4 to 6 weeks
C. Purulent, moderate 8. The following chart represents the institution's outpatient
D. Non-purulent, severe antibiogram in relation to Staphylococcus aureus. Which
CHAPTER 15 I SKIN AND SOFTTISSUE INFECTION I 63

percentage represents this institution's CA-MRSA preva- C. Clindamycin


lence during this sampling period? D. Trimethoprim/Sulfamethoxazole

Staphyloc°"us aureus; total isolates = 1116


ANSWERS
Antibiotic Susceptibility (%)
1. Explanation: The correct answer is C. Purulent abscess
Benzylpenicillin 20 is discovered during physical examination. Purulent
Oxacillin 75 skin infections are predominately caused by Staphylo-
Gentamicin 99 coccus aureus, whereas nonpurulent infections are most
often caused by Streptococci sp.1 Patient's temperature on
Ciprofloxacin 79
= =
admission is 98.8°F; WBC 13,500, HR 80, RR = 16;
Levofloxacin 80 therefore, he exhibits a systemic sign of infection (WBC
Clindamycin 80 > 12,000), which qualifies for at least moderate disease.
Erythromycin 58 Other signs of systemic infection include temperature
> 100.4, HR >90, and RR >24; this patient does not
Linezolid 100 meet any of these criteria. It is moderate and not severe
Rifampin 99 because he has not failed previous incision and drainage
Tetracycline 95 and antibiotics. 1
Trimethoprim/ 97 2. Explanation: The correct answer is A. Injection drug use
Sulfamethoxazole is an identified risk factor for CA-MRSA, as are tattoos
Vancomycin 100 and piercings, homelessness, HIV infection, men who
have sex with men, and individuals who are routinely
A. 25% in close contact with others (athletic teams, dormitory
B. 75% occupants, military personnel, prison inmates, and child
c. 20% care centers).2 Although this patient had a hospital stay
D. 100% post motor vehicle accident, it was 2 years ago and no
longer is considered a risk factor for HA-MRSA since it
9. SL reports feeling much better, still has pain but is manage-
is past the 90-day timeframe for it to be considered a risk
able with pain medication, has been afebrile for the past
factor. Hepatitis C itself is not considered a risk factor for
72 hours, blood cultures remain negative, and his wound
CA-MRSA, nor is his occasional work as a mechanic.1
appears to be healing well after the incision/drainage, IV
antibiotics, and dressing changes. The physician would like 3. Explanation: The correct answer is A. Incision and
to discharge this patient with oral antibiotics to complete drainage is the recommended treatment for abscesses.1
the treatment regimen. Sensitivity results are displayed Although dressing changes may improve wound healing,
below. Which of the following oral antimicrobials is rec- they are not more important than incision and drain-
ommended for SL to complete his treatment regimen? age and antimicrobial therapy for skin abscess recovery.
Decolonization is not warranted in this case; this is not
Organism: Staphylococcus aureus a recurrent infection.2 Also, decolonization is not part
of acute infection management. 1 Anaerobic bacterial
Antibiotic Value Susceptibility cultures may be beneficial for pathogen identification;
Clindamycin S0.25 Resistant however, incision and drainage is essential for source
eradication and control.
Erythromycin ~8 Resistant
Levofloxacin ~8 Resistant 4. Explanation: The correct answer is A. The decision to
administer antibiotics directed against S. aureus as an
Linezolid S2 Sensitive
adjunct to incision and drainage should be made based
Oxacillin ~4 Resistant upon presence or absence of systemic inflammatory
Penicillin ~0.5 Resistant response syndrome (SIRS), such as temperature > 38°C
Rifampin so.s Sensitive or <36°C, tachypnea >24 breaths per minute, tachy-
cardia >90 beats per minute, or white blood cell count
Tetracycline Sl Sensitive > 12,000 or <400 cells/µL.1 An antibiotic active against
Trimethoprim/ SlO Sensitive MRSA is recommended for patients who have failed
Sulfamethoxazole initial antibiotic treatment or have markedly impaired
Vancomycin S0.5 Sensitive host defenses or in patients with SIRS and hypotension. 1
Vancomycin is most appropriate because of its coverage
A. Linezolid of gram-positive bacteria including MRSA, in addition
B. Doxycycline to the rapid spread of the infection demonstrated by the
64 INFECTIOUS DISEASES: A CASE STUDY APPROACH

patient symptoms and markedly elevated WBC count. rod-shaped, anaerobic bacteria which most commonly is
Trimethoprim/sulfamethoxazole is incorrect because, found in intra-abdominal infections, particularly those
although it covers many isolates of MRSA, its reliability with abscess formation. Cutaneous abscesses may be
is generally lower than vancomycin and is not endorsed polymicrobial, containing regional skin flora or organ-
by the IDSA MRSA treatment guidelines for hospital- isms from the adjacent mucous membranes, but S. aureus
ized patients with a complicated SSTI (eg, patients with alone causes a large percentage of skin abscesses, with a
deeper soft-tissue infections, surgical/ traumatic wound substantial number due to MRSA strains. 1.2
infection, major abscesses, cellulitis, and infected ulcers
7. .Explanation: The correct answer is C. The duration of
and burns).2 Ceftriaxone does not cover MRSA and is pri-
therapy for skin infections has not been well-defined,
marily used empirically for moderate nonpurulent skin
although no differences in outcome were observed among
infections because of its excellent coverage of Streptococci
adult patients with uncomplicated cellulitis receiving 5
sp. 1 Although daptomycin covers MRSA, its usage is typi-
versus 10 days of therapy in a randomized, controlled
cally reserved for patients with MRSA isolates resistant to
trial. In the FDA licensing trials for complicated skin
vancomycin, or when vancomycin is not tolerated (type 1
infections (as in this case), patients were typically treated
hypersensitivity reactions). 1
for 7 to 14 days. 1 Duration of therapy should be indi-
5. .Explanation: The correct answer is D. Vancomycin 15 to vidualized on the basis of the patient's clinical response.
20 mg/kg/dose (actual body weight) given IV every 8 to Four to 6 weeks is not warranted since this patient does
12 h, not to exceed 2 g/dose, is recommended in patients not exhibit septic arthritis or osteomyelitis on MRI. Four
with normal renal function. Some experts suggest van- to 7 days is an insufficient treatment duration, since the
comycin loading doses (25 mg/kg) for serious suspected patient is not exclusively managed as an outpatient, does
or documented MRSA infections (sepsis, meningitis, exhibit signs/symptoms of extensive disease (tenosynovi-
pneumonia, or endocarditis) to enable early achievement tis concern on MRI), had rapid disease progression, has
of target trough concentrations, although clinical data are a history of injectable drug abuse (comorbid condition),
lacking. 3 Trough vancomycin concentrations are the most and the abscess is on the hand area which may be difficult
practical method to guide vancomycin dosing, although to completely drain. 1 No antibiotics is incorrect because
an AUC/MIC of >400 has been promoted as the ideal of the extensive factors listed above. Incision and drainage
target to predict successful organism eradication.3 There alone may be warranted in mild outpatient cases where
is not currently a widely accepted method to operational- no signs and symptoms of systemic infection are present. 1
ize AUC/MIC monitoring. Serum trough concentrations
should be obtained at steady-state conditions, prior to the 8. .Explanation: The correct answer is A. Staphylococcus
fourth dose. This patient has normal renal function which aureus isolates are uncommonly susceptible to penicillin.
should allow for vancomycin to reach steady state around Penicillin resistance (MSSA) is conferred by penicillinase
the fourth dose. Patients with impaired renal function production, which can be overcome by the addition of
may not necessarily reach steady-state concentrations a beta-lactamase inhibitor (eg, amoxicillin/clavulanate,
after the fourth dose and will likely require more frequent ampicillin/sulbactam) or use of penicillinase-resistant
monitoring. Monitoring of peak vancomycin concentra- penicillin (eg, oxacillin, nafcillin). 1 Methicillin resis-
tions is not recommended as they do not correlate with tance is conferred by the presence of the mecA gene that
therapeutic outcomes or toxicity.3 For serious infections, encodes penicillin-binding protein 2a, an enzyme that has
such as bacteremia, infective endocarditis, osteomyelitis, low affinity for beta-lactams and thus leads to resistance
meningitis, pneumonia, and severe SSTI (eg, necrotizing to methicillin, oxacillin, nafcillin, and cephalosporins
fasciitis) due to MRSA, vancomycin trough concentra- in general except for ceftaroline.2 Resistance percentage
tions of 15 to 20 mcglmL is recommended; otherwise is calculated by taking 100% minus the susceptibility
trough concentrations of 10 to 15 mcg/mL are acceptable percentage of the particular antibiotic, in this case 100%
for nonserious infections. 1.3 sr.:s creatinine clearance is - 75% (oxacillin) = 25%. Oxacillin is commonly used by
estimated at 129 mL/min using the original Cockcroft- laboratories to represent methicillin.
Gault equation. sr.:s creatinine clearance and age warrant
9. .Explanation: The correct answer is B. Doxycycline is the
the empiric every 8-hour interval.
most correct answer in this case considering that this
6. .Explanation: The correct answer is B. Gram-positive MRSA isolate is susceptible. The isolate is resistant to
cocci in clusters represent Staphylococcus aureus, which clindamycin. Trimethoprim/sulfamethoxazole would be
is the predominant bacteria involved in purulent skin an option in this case based upon susceptibility results;
infections. Streptococcus pyogenes, a gram-positive bac- however, this patient has an allergy to this antibiotic
teria that is found in chains, is the predominant bacteria (rash). Linezolid would also be an option based upon
involved in nonpurulent skin infections.1 Enterococcus susceptibility results; however, this patient abuses meth-
faecalis is a gram-positive bacteria that is often found in amphetamine which increases neurotransmitters such
short chains or diplococci and is not commonly found as norepinephrine, dopamine, and serotonin. Linezolid
in skin infections. Bacteroides fragilis is a gram-positive, is a weak monoamine oxidase inhibitor and therefore an
CHAPTER 15 I SKIN AND SOFTTISSUE INFECTION I 65

interaction may exist which potentially could generate 2. Liu C, Bayer A, Cosgrove SE, et al. Clinical Practice
dangerous levels of neurotransmitters and potentially Guidelines by the Infectious Diseases Society of America
cause sequela such as serotonin syndrome and hyperten- for the Treatment of Methicillin-Resistant Staphylococcus
sive urgency/emergency.4 Additionally, linezolid is more aureus Infections in Adults and Children. Clin Infect Dis.
costly than doxycycline and trimethoprim/sulfamethoxa- 2011;52(3):el8-e55.
zole and should be reserved for infections that do not 3. Rybak M, Lomaestro B, Rotschafer JC, et al. Therapeu-
respond to first-line therapy. 1.2 tic monitoring of vancomycin in adult patients: A con-
sensus review of the American Society of Health-System
Pharmacists, the Infectious Diseases Society of America,
REFERENCES and the Society of Infectious Diseases Pharmacists. Am l
1. Stevens DL, Bisno AL, Chambers HF, et al. Guidelines for Health-Syst Phann. 2009;66:82-98.
the Diagnosis and Management of Skin and Soft Tissue 4. Quinn DK, Stern TA. Linezolid and Serotonin Syn-
Infections: 2014 Update by the Infectious Diseases Soci- drome. Prim Care Companion l Clin Psychiatry.
ety of America. Clin Infect Dis. 2014;59(2):e10-e52. 2009; 11( 6) :353-356.
This page intentionally left blank
16 Skin and Soft
Tissue Infection II
Madeline King

PATIENT PRESENTATION Fluticasone (Flonase) nasal spray, one spray in each nostril
twice daily during allergy season
Chief Complaint
"I fell while hiking this weekend and now my arm has really Vitals on Admission
a painful blister."
Temp 39.5°C, BP 111/54 mm Hg. P 86, RR 16 breaths per
minute. Wt 57.5 kg, Ht 5'5"
History of Present Illness
SP is a 36-year-old woman who presents to the emergency Physical Examination
room of he.r local hospital 3 days after falling during a hike
and sustaining an injury to her right upper extremity. She has ... General
a few scratches on he.r right lower extremity, but no punctures Well-developed, well-nourished appearing Caucasian woman
to the skin. scabs, or blisters. She has been taking acetamin- in no acute distress
ophen 500 mg a few times per day since the fall. but states
that it hasn't helped much. She also states that she washed the ... Skin
wound when she got home with soap and water, but didn't use Right upper arm: red, erythematous, warm, and tend.er to
hydrogen peroxide or apply any topical antibiotics. touch; localized fluid collection that appears fluctuant

Past Medical History .., HEENT


Seasonal allergies PERRLA; EOMI, oropharyn:x. clear

... Neck/Lymph Nodes


Surgical History Supple, no lymphadenopathy
None
... Lungs/Thorax
Famlly History Cl.ear to auscultation, no rales or wheaing
Mother with hypertension
Father with diabetes ... Cardiovascular
Maternal grandfather with heart failure Regular rate/rhythm, no murmurs/rubs/gallops

Social History ... Abdomen


No illicit drug use Soft. NT/ND; ( +) Bowel sounds
No tobacco use
Occasional alcohol use (2 to 3 times per month) ... Genitourinary/Rectal
No observed abnormalities
Allergies ... Musculoskeletal/Exttemities
NKDA
Upper extremities: erythematous area on RUE with 2 x 3 cm
abscess that is painful to touch
Home Medications Lower extremities: no observed abnormalities
Ethinyl estradiol and norgestimate (Sprintec), 1 tablet by mouth
daily ... Neurologlc
Femfenadine (Allegra) 180 mg by mouth daily for allergies A&OX3
67
68 INFECTIOUS DISEASES: A CASE STUDY APPROACH

Imaging B. Photosensitivity, esophagitis


X-ray of RUE: diffuse soft tissue swelling of upper arm; no sub- C. Worsening infection, bone marrow suppression,
cutaneous gas or foreign body; no fractures GI upset
D. Bone marrow suppression, acute kidney injury

Laboratory Findings
WBC 18.1 x 103 cells/µL Plt 250,000/µL Na 135 mEq/L ANSWERS
K 4.8 mEq/L Glu 80 mgldL BUN15 1. Explanation: The correct answer is A. Cellulitis can occur
SCr 0.8 mg/dL at any anatomical site and is characterized by redness and
pain or swelling, except in the case of purulent cellulitis,
which can also include abscesses and blisters. This is often
QUESTIONS the result of minor trauma, an abrasion, or after surgery.
Cellulitis typically affects the dermis, while erysipelas
1. Which structural component of the skin is MOST likely to
affects the epidermis (rules out answer choice B). If the
be affected by cellulitis?
fascia is affected, that would be a more severe condition
A. Dermis
affecting deeper structural layers and leading to necrosis
B. Stratum corneum
(eg, necrotizing fasciitis), thus ruling out answer choice C.
C. Fascia
Bone involvement would be the result of trauma going
D. Bone
through multiple structural layers or a progressive,
2. What organism is MOST commonly the cause of purulent untreated SST! which reached the bone, thus ruling out
skin/soft tissue infections? answer choice D.
A. Streptococcus pyogenes
2. .Explanation: The correct answer is B. S. pyogenes is a
B. Staphylococcus aureus
common cause of nonpurulent cellulitis, including nec-
C. Pseudomonas aeruginosa
rotizing fasciitis, and can produce toxins, thus ruling out
D. Peptostreptococcus spp.
answer choice A. P. aeruginosa is an uncommon cause of
3. How would you classify the severity of SP's infection? SSTI, except in some diabetic foot infections if the patient
A. Mild soaks his or her feet in water (as P. aeruginosa is commonly
B. Moderate found in water sources), thus ruling out answer choice C.
C. Severe Peptostreptococcus is an anaerobic gram-positive coccus,
D. Life threatening which is normal flora of the mouth and genitourinary
tract, and is more commonly seen in mixed infections,
4. What would be the MOST appropriate empiric therapy for
causing dental abscesses or genitourinary abscesses. It can
a patient with moderate purulent cellulitis?
cause cellulitis or other soft tissue infections, but these are
A. Incision and drainage plus doxycycline
less common, ruling out answer choice D. S. aureus is a
B. Incision and drainage plus dicloxacillin
common cause of purulent cellulitis. 1
C. Amoxicillin
D. Vancomycin 3. .Explanation: The correct answer is B. Moderate severity:
Purulent infection with signs of systemic inflammation
5. What definitive antibiotic therapy would be most appro-
(elevated WBC, fever, elevated HR, low BP). Other signs
priate for SP, assuming she does not need to be admitted to
of systemic infection would include temperature > 100.4°F
the hospital and cultures from the I&D result as methicillin
and RR > 24 breaths per minute. Since there are systemic
susceptible S. aureus?
signs of infection, this is not a mild infection, ruling out
A. Doxycycline
answer choice A. It is not severe because he has not failed
B. Vancomycin
previous therapy with incision and drainage (I&D) plus
C. Amoxicillin
antibiotics, nor does he have progressive signs of a sys-
D. Dicloxacillin
temic response ( eg, worsening temperature, blood pres-
6. What is the most appropriate treatment duration for SP? sure, heart rate, WBC), thus ruling out answer choices C
A. 5 days and D. 1 It is important to determine severity because that
B. 10 days will guide therapy choices. Mild infection requires only
C. 14 days I&D, whereas moderate and severe infections require I&D
D. 21 days with antibiotics given either orally or intravenously.
7. Regardless of the antibiotic chosen in question 5, which adverse 4. .Explanation: The correct answer is A. Assume S. aureus,
reactions would you need to counsel SP on if doxycycline was and the potential for methicillin-resistant S. aureus
prescribed? (MRSA) even in the community setting due to an increas-
A. C. difficile infection, Stevens-Johnson syndrome, ing prevalence. Empiric treatment may be broad but
anaphylaxis should be narrowed once culture results are available.
CHAPTER 16 I SKIN AND SOFTTISSUE INFECTION II 69

Because we have to consider MRSA as a potential patho- 7. Explanation: The correct answer is B. Any antibiotic
gen, we need an MRSA active agent empirically.2 Doxy- could potentially cause C. difficile infection; however, cer-
cycline and vancomycin are the only two choices that tain antibiotics and longer courses of antibiotics cause an
cover MRSA, ruling out answer choices B and C.1.2 For increased risk. Clindamycin has the highest risk of caus-
purulent cellulitis (or any infection with an abscess or ing C. difficile, followed by cephalosporins and fluoro-
other drainable/removable source, the first step is source quinolones.6 Although patients could develop an allergic
control). Therefore, vancomycin without I&D is not reaction or anaphylaxis to any agent, allergies are more
the correct answer, but would be acceptable if a patient commonly reported for beta-lactams and TMP-SMX
required intravenous (IV) antibiotics in addition to I&D (answer choice A). TMP-SMX or linezolid would be the
(rules out answer choice D). Vancomycin should be most likely antibiotics to cause bone marrow suppression
dosed as 15 mg/kg every 12 hours, with a goal trough of (answer choices C and D). TMP-SMX is also one of the
10 to 15 mcg/mL. I&D plus dicloxacillin would be correct most likely to cause Stevens-Johnson syndrome (answer
if the pathogen is known to be methicillin-susceptible choice A).7 Doxycycline can cause drug-induced esopha-
S. aureus (MSSA). Amoxicillin rarely covers S. aureus gitis and photosensitivity; patients should be counseled to
(most isolates are resistant to penicillin, ampicillin, amox- take doxycycline with a full glass of water, remain upright
icillin due to production of a penicillinase). Additionally, for at least 30 minutes after administration, and avoid
even if it were susceptible, the patient would first require prolonged sun exposure.8 Therefore, the correct answer is
l&D. Therefore, doxycycline 100 mg twice daily would B, photosensitivity, nausea, and esophagitis.
be the best choice.1 An alternative option, not listed here,
would be trimethoprim-sulfamethoxazole (TMP-SMX).
REFERENCES
5. Explanation: The correct answer is D. Because you know
1. Stevens DL, et al. Practice guidelines for the diagnosis
now that the patient has MSSA, she doesn't need MRSA
and management of skin and soft tissue infections: 2014
coverage any longer. It is most appropriate to use a nar-
update by the Infectious Diseases Society of America. Clin
rower spectrum agent when possible. Ofthe listed options,
Infect Dis. 2014:59.
vancomycin and doxycycline cover MRSA, which is not
2. Horseman M, Bowman JD. Is community-acquired meth-
necessary, so A and B are not correct. As discussed in
icillin-resistant Staphylococcus aureus coverage needed
question 4, amoxicillin rarely covers S. aureus, ruling out
for cellulitis? Infect Dis Ther. 2013;2:175-185.
option C. Susceptibilities were not provided, but most
3. Hepburn MJ, Dooley DP, Skidmore PJ, Ellis MW. Starnes
reports do not include penicillin or amoxicillin as options
WF, Hasewinkle WC. Comparison of short-course
because of the rarity of susceptibility. Therefore, dicloxacil-
(5 days) and standard (10 days) treatment for uncompli-
lin, as an oral MSSA active agent, is the best option, dosed cated cellulitis. Arch Intern Med. 2004;164(15):1669-1674.
at 500 mg 4 times daily. Oral cephalosporins (eg, cepha- 4. Prokocimer P, De Anda C, Fang E, Mehra P, Das A.
lexin) would also be viable options for MSSA. Doxycycline
Tedizolid phosphate vs linezolid for treatment of acute
or trimethoprim-sulfamethoxazole would be appropriate bacterial skin and skin structure infections: the ESTAB-
oral options, despite having broader (eg, MRSA) coverage,
LISH-I randomized trial. /AMA. 2013;309(6):559-569.
and may be useful for patients with a penicillin allergy. 1
5. Moran GJ, Fang E, Corey GR, Das AF, De Anda C, Pro-
6. Explanation: The correct answer is C. The recommended kocimer P. Tedizolid for 6 days versus linezolid for 10
duration of antibiotic therapy for moderate, purulent cel- days for acute bacterial skin and skin-structure infections
lulitis is 5 days unless there is a delay in clinical improve- (ESTABLISH-2): a randomised, double-blind, phase 3,
ment, in which case the treatment duration could be non-inferiority trial. Lancet Infect Dis. 2014; 14(8):696-705.
extended. Clinical assessment should be used to deter- 6. Brown KA. Meta-analysis of antibiotics and the risk of
mine why the patient is not improving and explore other community-associated Clostridium difficile infection.
reasons for continued disease. The benefits of shorter Antimicrob Agents Chemoth. 2013;57(5):2326-2332.
durations of therapy include a lower risk of adverse effects 7. Sulfamethoxazole-trimethoprim [package insert].
from antibiotics (eg, Clostridioides difficile infection), Kenner, LA: New Horizon Rx Group, LLC; 2013
and preventing the emergence of resistance by providing 8. Doxycycline [package insert]. Morgantown, WV: Mylan;
unnecessary antibiotics.3-6 If this were a mild infection, no 2019.
antibiotics would be required.
This page intentionally left blank
17 Necrotizing Fasciitis
Tianrui Yang Jonathan C. Cho

PATIENT PRESENTATION ., General


Well-developed, well-nourished male; appears lethargic and
Chief Complaint
in distress
"My right leg hurts:"
., HEENT
History of Present Illness
Normocephalic, atraumatic, PERRLA, EOMI, normal fundu-
WL is a 52-year-old Caucasian male who presents to the emer- scopic exam, normal visual fields
gency department with pain and redness of his lower right leg
as well as fever and malaise. He also noticed bulla formation on ., Pulmonary
his right shin which worsened within hours. He recalls acciden- Clear to auscultation bilaterally, no rales/rhonchi/wheezes
tally bumping his leg on the side of his bed and sustaining an
abrasion to his shin 2 days ago. He states he had to skip work ., Cardlovasculal'
the next day since it was so painful. When the pain got worse
overnight. he decided to come into the emergency department. NSR. no m/r/g

Past Medical History ., Abdomen


HTN, type 2 DM, HLD Soft. non-distended, non-tender, normal bowel sounds

Surgical History ~ Genitourinary


Appendectomy Deferred; no complaints of dysuria or hematuria

Family History ., Neurology


Father had CAD and passed away from a myocardial infarc- Lethargic, oriented to place, person, and time
tion at age 50; mother has type 2 DM and hypothyroidism.
., Extremities
5 x 12 cm erythematous patch with bullae on the front RLE,
Social History
tender to palpation
Married and living with his wife. Smokes ~ ppd x 30 years and
drinks alcohol socially.
Laboratory Findings
Allergies Nal40mEq/L Ca8.2mgldL WBC 16.3 gldL
NKDA K3.7mEq/L Mg2.lmgldL Hgb 12.7 gldL
Cll03mEq/L P04 4.lmg/dL Hct3896
Home Medications
HC01 26 mEq/L AST22U/L Plt 256 x 10'/µL
Lisinopril 40 mg PO daily
Atorvastatin 80 mg PO daily BUN 19mgldL ALTlSU/L
Metformin 1 g PO BID SCr 1.2 mgldL T Bill 0.9 mg/d.L
Empagliflozin 25 mg PO daily Glu 173 mg/dL Alk Phos 81 U/L
Alc7.5% Lactate 0.8 mmol/L
Physical Examination
., Vital Signs ~ CTofRLE
Temp 102.4°F, P 107, RR 24 breaths per minute, BP 124/70 mm Extensive gas formation in the soft tissues of the front leg
Hg. p02 97 %, Ht S'llH, Wt 100 kg along the fascia! planes toward the back ofthe leg and the knee
71
72 INFECTIOUS DISEASES: A CASE STUDY APPROACH

~ Blood Cuhures C. De-escalate to penicillin alone


Pending D. De-escalate to penicillin + clindamycin
8. What is the appropriate duration of treatment for WL?
QUESTIONS A. 4 to 6 weeks of IV antibiotics
B. A minimum of 2 weeks of IV antibiotics after last
1. Which of the following is NOT a risk factor that WL has for debridement
contracting necrotizing fasciitis? C. A minimum of 2 weeks of IV antibiotics followed by
A. Obesity 2 weeks of PO antibiotics
B. Type2DM D. Continued for a few days after patient is clinically
C. Injury improved, afebrile for at least 48 hours, and no longer
D. HLD needs debridement
2. Given WU. risk factors, which type(s) of necrotizing fasciitis 9. Which of the following is NOT a potential complication
is/are WL most at risk for contracting? that WL should be monitored for?
A. Type I A. Fournier gangrene
B. Type I &II B. Toxic shock syndrome
C. Type II C. Compartment syndrome
D. Type II & IV D. Septic shock
E. Type III
3. Which of the following is/are clinical features of necrotizing ANSWERS
fasciitis?
A. Tenderness only within the cutaneous erythema 1. .Explanation: The correct answer is D. Choices A through
B. Presence of crepitus C are all identified risk factors for necrotizing fasciitis.
C. Rapid response to IV antibiotic therapy Other risk factors to consider when screening patients
D. Mild pain without systemic signs or symptoms include IV drug use, immunocompromised state, and
chronic diseases such as renal failure, cirrhosis, and alco-
4. How is necrotizing fasciitis best diagnosed? holism.1 Diabetes is recognized as a significant risk factor
A. CT is the gold standard of diagnosis as it has great sensi- as 44.5% to 72.3% of necrotizing fasciitis patients have
tivity and specificity for showing edema along the fascia! been reported to have diabetes as an underlying disease.2
plane In most cases, clinical features of necrotizing fasciitis
B. MRI is the gold standard of diagnosis as it has great extend from a skin lesion. Injury ranges from large surgi-
sensitivity and specificity for showing edema along the cal incisions to small lesions such as insect bites or injec-
fascia! plane tion sites (ie, IVDU). However, a small percentage of the
C. Appearance of the subcutaneous tissues or fascia! patients may have no visible skin lesion.3
planes seen during surgical exploration
D. Clinical scoring systems such as the laboratory risk 2. .Explanation: The correct answer is B. There are 4 types of
indicator for necrotizing fasciitis (LRINEC) necrotizing fasciitis. 4 Type I, or polymicrobial necrotizing
fasciitis, is the most common type of the disease. Caus-
5. What is an appropriate management plan for WL? ative pathogens include a combination of gram-positive
A. Surgical debridement cocci, gram-negative rods, and anaerobes. Patients with
B. Surgical debridement along with antibiotics Type I necrotizing fasciitis are typically older patients
C. Surgical debridement along with antibiotics and aggres- with comorbidities such as diabetes and peripheral vas-
sive fluid administration cular disease. Trauma may or may not be present in this
D. Surgical debridement along with antibiotics, aggressive type of necrotizing fasciitis. Type II infections involve
fluid administration, and IVIG group A f3-hemolytic Streptococcus, with or without
6. Which of the following would be an appropriate initial anti- staphylococcal species. Type II infections tend to occur
biotic regimen? in younger patients. Risk factors for Type II infections
A. Piperacillin-tazobactam + vancomycin include a history of trauma, surgery, or IVDU. Type III
B. Meropenem infections are caused by marine organisms, most com-
C. Penicillin + clindamycin monly Vtbrio spp. or Clostridium spp. Vtbrio spp. are
D. Ceftriaxone + linezolid natural inhabitants of coastal waters in the southeastern
United States, Central and South America, and Asia.
7. WL is stable after debridement and empiric antibiotics. His Clostridium spp. are anaerobic bacteria that can result
finalized tissue culture was only positive for group A Strep- from intestinal surgical wounds or external wounds caus-
tococcus. What is the most appropriate next course of action? ing local devascularization. Type III infections typically
A. Continue all antibiotics as diabetic patients are at risk occur through contamination of a pre-existing wound or
for polymicrobial disease through an injury acquired while exposed to salt water.
B. De-escalate to vancomycin alone However, ingestion of raw seafood has also been reported
CHAPTER 17 I NECROTIZING FASCllTIS 73

to cause Type III infections. Type IV infections are caused as Clostridium and Peptostreptococcus spp. should be
by fungal organisms, typically Candida spp. Type IV initiated. If Type II infection is suspected, a combina-
infections are very rare and typically occur in patients tion of penicillin and clindamycin is recommended. For
with traumatic wounds and bums and in those who are Type III infections, a combination of third-generation
severely immunocompromised. Given WI.:s PMH of DM cephalosporin and doxycycline or minocycline should
and SH of IVDU, he is most at risk for developing Type I be started. Lastly, antifungal therapy, such as fluconazole
and Type II infections. and amphotericin B, should be initiated if the patient
has Type IV necrotizing fasciitis. The antifungal agent
3. Explanation: The correct answer is B. Choice A is incor-
selected should be based on the suspected/isolated
rect as patients with necrotizing fasciitis will typically
organism and susceptibility. Since fungal necrotizing
experience edema or tenderness beyond the cutaneous
fasciitis is rare and typically only occurs in patients who
erythema. Choice C is incorrect because patients with
are severely immunocompromised, there is no need to
necrotizing fasciitis will typically fail to respond to initial
add antifungal coverage for WL at this time. When there
antibiotic therapy (regardless of IV or PO) and develop
is not a clear suspicion for one type of infection, empiric
progressive redness and swelling despite antibiotics (in
therapy should be broad and provide coverage for
contrast to cellulitis). Choice D is incorrect because these
gram-positive, gram-negative, and anaerobes.3 MRSA
patients will typically experience severe pain that seems
coverage should be based on prevalence of MRSA in the
disproportional to the clinical findings. Other clinical
region. Choice B is incorrect because meropenem lacks
features include hard, wooden feel of the subcutaneous
MRSA coverage. Choice C is incorrect because this regi-
tissue, bullous lesion, and skin necrosis or ecchymoses.3
men does not provide sufficient gram-negative coverage.
4. Explanation: The correct answer is C. Although both Choice D is incorrect because this regimen does not
CT and MRI may show edema extending along the fas- cover anaerobic organisms.
cia! plane, the sensitivity and specificity of these studies
7. Explanation: The correct answer is D. Choice A is incor-
are not well defined and may delay definitive diagnosis
rect because while diabetes is a risk factor for polymicro-
and treatment and therefore choices A and B are incor-
bial necrotizing fasciitis, this is not the case with WL as
rect. Choice D is incorrect because while clinical scoring
evidenced by the tissue culture, and antibiotics should be
systems are useful aids for diagnosis, they should not be
de-escalated for stewardship purposes. While vancomycin
used as the gold standard for diagnosis as their accuracy
covers Streptococcus spp., choice B is incorrect since van-
in identifying necrotizing fasciitis produced conflict-
comycin is neither the narrowest option nor the treatment
ing results. In diagnosing necrotizing fasciitis, clinical
of choice for group A Streptococcus infections. Although
features are important in raising the initial suspicion. To
penicillin is the treatment of choice for infections due to
confirm the suspicion, the most important diagnostic fea-
group A Streptococcus, choice C is not the best answer in
ture is the appearance of the tissues seen during surgery.
this case since a combination therapy with clindamycin is
Findings include swollen and dull-gray appearance of the
recommended when treating necrotizing fasciitis caused
fascia, thin exudate without clear purulence, and easy
by group A Streptococcus. Clindamycin is able to suppress
separation of tissue planes by blunt dissection.3
toxin and cytokine production by group A Streptococcus
5. Explanation: The correct answer is C. Treatment of through the inhibition of bacterial protein synthesis and
necrotizing fasciitis will require surgical management. potentially decrease skin necrosis. 3
After initial debridement, patients should return to the 8. Explanation: The correct answer is D. There is no estab-
operating room daily until the surgical team finds no lished time period for the duration of treatment. Duration
further need for debridement. Antibiotics should also be is based on clinical status and the need for debridement as
administered in conjunction with surgical debridement to complete debridement is the only way to achieve source
help treat and prevent the spread of infection. Since these control.3
wounds can discharge large amounts of tissue fluid, IV
fluid replacement should be given to maintain adequate 9. Explanation: The correct answer is A. Fournier gan-
hydration.3 Lastly, while IVIG may be beneficial in treat- grene is a variant of necrotizing soft tissue infection that
ing streptococcal infections by neutralizing the toxins, it involves the scrotum and penis or vulva. It's a similar
should not be started empirically before the identification disease but it's not considered a complication from the
of the pathogen as data supporting use is inconsistent.5 current disease that WL has (Fournier gangrene is typi-
cally caused by facultative organisms and anaerobes).3
6. Explanation: The correct answer is A. Treatment of Toxic shock syndrome is an abrupt onset, life-threatening
necrotizing fasciitis should be based on the suspected complication characterized by vascular collapse and mul-
type of infection. If Type I necrotizing fasciitis is sus- tiorgan failure. WL is at risk for this because the causative
pected, broad-spectrum antibiotics effective against pathogen for his necrotizing fasciitis is group A Strep-
gram-positive organisms such as methicillin-resistant tococcus which can release toxins to cause toxic shock
S. aureus (MRSA), gram-negative organisms such as syndrome. Compartment syndrome is a painful condition
E.coli and Klebsiella spp., and anaerobic organisms such that typically results from trauma. Necrotizing fasciitis
74 INFECTIOUS DISEASES: A CASE STUDY APPROACH

may cause tissue hypoxia, edema, and altered oxygen 3. Stevens DL, Bisno AL, Chambers HF, et al. Practice guide-
tension that lead to an increase in intracompartmental lines for the diagnosis and management of skin and soft
pressure resulting in tissue ischemia, necrosis, and nerve tissue infections: 2014 update by the Infectious Diseases
damage. Septic shock is another life-tlueatening compli- Society of America. Clin Infect Dis. 2014;59(2):e10-e52.
cation characterized by vasodilatory hypotension despite 4. Davoudian P, Flint N. Necrotizing fasciitis. Anaesth Crit
fluid resuscitation and elevated lactate level and may arise Care Pain. 2012;12(5):245-250.
from many different infections including necrotizing 5. Parks T, Wilson C, Curtis N, Norrby-Teglund A,
fasciitis. 6 Sriskandan S. Polyspecific intravenous immunoglobulin
in clindamycin-treated patients with streptococcal toxic
shock syndrome: a systematic review and meta-analysis.
REFERENCES Clin Infect Dis. 2018;67(9):1434.
1. Puvanendran R, Huey J, Pasupathy S. Necrotizing fasci- 6. Centers for Disease Control and Prevention. Necrotizing
itis. Can Fam Physician. 2009 Oct;55(10):981-987. fasciitis: all you need to know. Available at https://www.cdc.
2. Cheng N, Tai H, Chang S, Chang C, Lai H. Necrotizing gov/groupastrep/diseases-public/necrotizing-fasciitis.html.
fasciitis in patients with diabetes mellitus: clinical char- Atlanta, GA, 2018.
acteristics and risk factors for mortality. BMC Infect Dis.
2015;15:417.
18 Diabetic Foot
Infection
Kayla R. Stover

PATIENT PRESENTATION Home Medications


Chief Complaint Allopurinol 100 mg PO daily
Amlodipine 10 mg PO daily
"My foot is red, draining fluid, and it stinks." Aspirin 81 mg PO daily
B-complex-C-folic acid (RENA-VITE): 1 tablet PO daily
History of Present Illness Calcitriol 0.25 mc:g PO daily
EH is a 58-year-old female who presents to the hospital with Carvedilol 25 mg PO BID with. meals
a wound to the right plantar surface of the foot that is edema- Cetirizine 10 mg PO daily
tous, erytltematous, and with foul-smelling drainage. She was Furosemide 20 mg PO daily
seen in the emergency department 6 days ago and received Insulin glargine 35 units subcutaneously qHS
clindamycin 300 mg every 8 hours for 5 days. She reports Insulin lispro 4 units subcutaneously TID before meals
compliance with. the medications, and has been soaking her Levothyroxine 50 mc:g PO daily
feet routinely in hot water. Since that time, her wound has Omeprazole 20 mg PO daily
only worsened. She denies fever or chills but reports frequent Pravastatln 80 mg PO qHS
feelings of hopelessness. Sevelamer 3200 mg (4 tablets) PO TIO with meals
Sertraline 100 mg PO daily
Past Medical History
Insulin-dependent type 2 diabetes, dyslipidemia, HTN, ESRD Physical Examination
(on dialysis). depression, hypothyroidism, and CAD ..,. Vital Signs
Temp 98.3°F. P 66, RR 18 breaths per minute, BP 161/94 mm Hg,
Surgical History sp02 9996, Ht 5'4H, Wt 81.1 kg
AV fistula placement, cholecystectomy, coronary artery
bypass graft, finger amputation, oophorectomy, tonsillectomy, ..,. General
and adenoidectomy Well-developed, well-nourished female oriented to person,
place, and time
Family History
Fath.er had diabetes, Alzheimer's disease, heart disease, H'IN, ..,. HEENT
high cholesterol, arthritis, and vision loss and passed away 2 Nonnocephalic:, atraumatic:, external ears normal, oropharynx
years ago; mother has diabetes, HTN, high cholesterol, and clear and moist; no discharge from eyes, no scleral icterus;
history of stroke neck supple

Social History ..,. Pulmonary


Smokes Y.i pack per da:y x 15 years; reports no alcohol or illicit Effort and breath sounds normal; no respiratory distress
drug use
..,. Cardiovascular
Allergies RRR, normal heart sounds; no m/r/g
Penicillin (lip swelling, tongue irritation; has tolerated
cephalosporins and carbapenems in the past); atorvastatin ..,. Abdomen
("spaghetti legs"; tolerates pravastatln) Soft. bowel sounds nonnal

75
76 INFECTIOUS DISEASES: A CASE STUDY APPROACH

~ Musculoskeletal 2. In addition to the evaluation of signs and symptoms on


Right lower extremity reddened and edematous from ankle presentation, which of the following tests would be most
downward; +2 pedal pulses bilaterally helpful to diagnose a patient with a diabetic foot infection?
A. C-reactive protein
~ Skin B. Erythrocyte sedimentation rate
C. Magnetic resonance imaging
Warm and dry. Foul smelling wound with yellow drainage D. White blood count
noted to right plantar surface of foot. Macerated edges noted
with surrounding redness and swelling. 3. Which of the following types of pathogen would you suspect
is responsible for EH's diabetic foot infection?
~ Neurology A. Aerobic gram-negative bacilli
Alert and oriented to person, place, and time B. Aerobic gram-positive cocci
C. Anaerobic organisms
~ Psychiatric D. All of the above
Normal mood and affect; behavior normal 4. Which of the following best describes the risk factors for
acquiring diabetic foot infections caused by Pseudomonas
Laboratory Findings aeruginosa?
A. Chronic, previously treated wounds now with severe
Na = 134 mEq/L WBC= 6 x ESR = 61.0 mm/hr infection
1Q3/mm3 B. High local prevalence, warm climate, frequent water
K= 5.0mEq/L Hgb = 10.3 g/dL CRP = 2.30 mg/L exposure
C. Nasal carriage, previous hospitalization, and local
Cl= 90mEq/L Hct = 31.6% PT 12.5 sec
resistance
C02 = 25 mEq/L Plt = 206 x INR 1.06 D. Prior long-term antibiotic use and long duration of foot
1Q3/mm3
wound
BUN = 57.0 mg/dL
5. Which specific factor is most likely to put EH at risk for a
Ser= 6.29 mg/dL drug-resistant pathogen?
Glu = 246 mgldL A. Complicated past medical history
B. History of smoking ~ pack per day
~ x-rayFoot C. Previous treatment with clindamycin
D. Previous visit to the emergency department
From ED visit: Medially prominent soft tissue swelling; ero-
sive changes in the medial head of the first metatarsal, medial 6. Which of the following regimens would be the best empiric
and lateral head of the second metatarsal, and lateral head therapy for EH?
of fifth metatarsal; erosive changes could be consistent with A. Amoxicillin-clavulanate orally
known neuropathic arthropathy. B. Ciprofloxacin plus metronidazole orally
C. Clindamycin plus penicillin intravenously
~ Blood Cultures D. Vancomycin plus piperacillin-tazobactam intravenously
Pending 7. EH responds well to initial therapy, with improvements in
the appearance of her foot. Three days into therapy, deep
~ Deep Wound/Tissue Cultures wound cultures grow methicillin-susceptible Staphylococcus
Pending aureus. Which agent would be best to recommend for EH?
A. Daptomycin
B. Dicloxacillin
QUESTIONS C. Levofloxacin
D. Vancomycin
1. Based on presentation, how would this patient be classified
according to the systems developed by the International 8. What would be the most appropriate duration of therapy
Working Group on the Diabetic Foot (PEDIS grade) and for a severe diabetic foot infection with residual underlying
the Infectious Diseases Society of America? bony involvement?
A. 1, uninfected A. 1 to 2 weeks
B. 2, mild B. 2 to 4 weeks
C. 3, moderate C. 4 to 6 weeks
D. 4, severe D. 2::3 months
CHAPTER 18 I DIABETIC FOOT INFECTION 77

ANSWERS treatment that failed (6 days on clindamycin from the


emergency department), she lilc:cl.y has frequent health
1. Explanation: The correct answer is C. This patient meets care exposure. which puts her at higher risk for polymi-
the criteria for infection (erythema + edema + discharge; crobial infection with gram-positive, gram-negative, and
answer A is incorrect). but has surrounding edema >2 cm anaerobic organisms (answers A. B. and Care incorreet).1
(per physical exam, reddened and swollen from ankle 4. E:q>lan•tion: The COlTect answer is B. According to the
downward), placing them into Grade 3 or moderate infu:- JDSA clinical practice guidelines, warm climates, fre-
tion (answer B is incorrect). This patient has no signs or
quent exposure to water, and high local prevalence of
symptoms of systemic infection, so would not be classi-
Pseudomonas aeruginosa increase the risk of infection
fied as severe (Grade 4) (answer Dis incorrect).1
with that pathogen. 1 Chronic wounds or those that have
been previously treated or that are with severe infection
Diabetic Poot Infedion Severity Classification by the are most likely representative of anaerobic organisms
IDSA and PEDIS Grading Scales (answer A is incorrect). Risk factors for methicillin-
IDSA PEDIS resistant Staphylococcus aureus (MRSA) include prior
Seftrity Grade Signs/Symptoms long-term or inappropriate use of antibiotics, previous
Uninfeded 1 No signs or symptoms of infection• hospitalizations, long duration of foot wounds, presence
Mild 2 Local .infection (no involvement of of osteomyelitis, and nasal carriage of MRSA (answers C
deeper tissues); erythema between and D are incorrect).
0.5 and2 cm. S. EJ:planatlon: The correct answer is C. Previous antibiotic
Local infection with erythema use or treatment failure is a specific risk factor for infec-
>2 cm or involvement of deeper tion with drug-resistant pathogens. including methicil-
Moderate 3 tissues/structures WTmOUT lin-resistant Staphylococcus aureus.1 Although previous
signs of systemic inflammatory hospitalizations are also risk factors, this does not include
response syndrome (SIRS)b outpatient visits (such as to the emergency department)
Severe 4 Local infection plus signs of SIRS" (answer D is incorrect). Although a complicated past
medical and smoking history are potentially risk factors
IDSA. Infedious Diseases Society of America; PEDIS, International for complications, they have not specifically been associ-
Working Group on the Diabetic Foot Infection.
ated with drug resistance (answers A and B are incorrect).
•To be defined a. an Infection, 2 or more of the following should be
present: local swelling or indw-ation, erythema, local tenderness or 6. Explanation: The correct answer is D. For moderate to
pain, local warmth, purulent discharge. severe infections, the guidelines suggest starting with
"To be defined as SIRS, 2 or more of the following should be intravenous therapy that is sufficiently broad to cover sus-
present: temperature >38°C or <36°C, heart rate >90 beats pe<:ted pathogens.1 This patient has potential risk factors
per minute, respiratory rate >20 breaths per minute or PaCO1
for both methicillin-resistant Staphyloroccus aureus and
<32 mm Hg. white blood count >12,000 or <4000 cellsJpL or
~10% immature forms.
Pseudomonas aeruginosa, so should be covered for both
Souru: Data from Lipsky BA. Berendt AR. Comia PB, et al.; empirically. For mild or some moderate infections, oral
Infectio1111 Disc:ue.il Society of America. 2012 InfcctloUI Diacaae.il therapy would be appropriate. This patient has previously
Society of America clinical practice guideline for the diagnosis failed oral therapy, so she should be started initially on
and treatment of diabetic foot infections. Cltn Infod Dis. intravenous therapy (answers A and B are incorrect). In
2012;54(12):el32-e73. addition, amoxicillin-clavulanate is a suggested therapy
only for mild infection. Although ciprofloxacin plus
2. Explanation: The correct answer is C. Although C-reactive metronidazole would cover gram-negative and anaerobic
protein. erythrocyte sedimentation rate. and white blood pathogens, that regimen would not cover Staphylococcus
counts are laboratory markers suggestive of infection, aureus. which may be a factor for this patient. Clindamy-
they are not speciftc to diabetic foot infection (answers A, cin is an option for mild infections, but this patient
B, and D are incorrect). In con1J:ast, radiography includ- already failed therapy. In addition. clindamycin plus
ing both plan films (X-ray) and magnetic resonance penicillin would not cover the most suspe<:ted pathogens
imaging (MRI) are recommended by the guidelines in (answer C is incorrect).
order to identify bony abnormalities, soft tissue gas. for- 7. E:J:planation: The correct answer is B. Dicloncillin
eign bodies, and/or deeper, underlying infections such as
provides specific, targeted therapy toward methicillin-
osteomyelitis.1 susceptible Staphylococcus au:reus (MSSA). This patient
3. Explanation: The correct answer is D. Based on EH's has responded well to initial therapy (defined as improve-
medical and surgical history and reports of previous ment in or resolution of signs or symptoms of infection
78 INFECTIOUS DISEASES: A CASE STUDY APPROACH

[redness, swelling, induration, pain, purulence] and/or REFERENCE


improvement or resolution of SIRS criteria [normaliza-
1. Llpsq BA, Berendt AR, Comia PB, et aL 2012 Infectious
tion of temperature, white blood count. heart rate, and
Diseases Society of America clinical practice guidelines
respiratory rate]}, and culture results are back, meeting
for the diagnosis and treatment of diabetic foot infections.
the criteria for conversion to oral therapy according to
Clin Infect Dis. 2012;54:132-173.
the diabetic foot infection guidelines.1 Daptomycin and
vancomyc.in would both provide intravenous options, but
are broader spe<:trum. agents than are needed for MSSA
(answers A and D are incorrect). Although levofloxac.in
provides a nice oral option, the guidelines suggest it is
suboptimal against Staphylococcus aureus and an alterna-
tive should be selected (answer C is incorrect).
8. EqJlanation: The correct answer is C. The guidelines re<:-
ommend 4 to 6 weeks of therapy for patients with diabetic
foot infection with retidual underlying bony involve-
ment.• Mild to moderate infections may be treated for
1 to 3 weeks (answer A is incorrect), while severe infections
without residual bony involvement may be treated for 2 to
4 wedcs (answer B is incorrect). Only patients with severe
infection with residual dead bone should be treated for 3
or more months (answer D is incorrect).

Route, Duration, and Treatment Location for Diabetic


Foot Infections
'Iieatment
Classiicatioa Route Duration Location
Topical or
Mild 1-2weeks Outpatient
oral
Moderate Oral or l-3weeks Outpatient or
initial IV Inpatient
Initial rv.
Inpatient
switch to
Severe 2-4weeks then
oral when
Outpatient
possible
Bone IV or oral 2-5 days
Inpatient
involvement then
with Outpatient
amputation
Bone
Initial rv.
involvement Inpatient
switch to
with residual 4-6weeks then
oral when
infected outpatient
possible
bone
Bone Initial rv. C?:3 months Inpatient
involvement switch to then
with residual oral when outpatient
necrotic possible
bone
Source: Data from Llpsky BA, Berendt AR, Cornia PB, et al;
Infectious Diseases Society of America. 2012 Infectious Diseases
Society of America. clinical practice guideline for the diagnmil
and treatment of diabetic foot infections. Clin Infect Dis.
2012;54(12):el32-e73.
19 Vertebral
Osteomyelitis
Meghan N. Jeffres

PATIENT PRESENTATION ~ General


He does not appear to be in any acute distress
Chief Complaint
Back pain ~ HEENT
Pupils equal/round, reactive to light, conjunctiva clear; poor
History of Present Illness dentition noted
CJ is a 62-year-oldmale who was referred to the hospital by his
primary care physician for further workup for back pain. His ~ Pulmonary
back pain started sii: weeks ago and has gotten progressively Clear to auscultation bilaterally, no wheezing, rhonchi, or rales
worse over that time. Stretching and exercise have not helped
with the pain. Lying flat in bed provides some relief. The pain ~ Cardiovascular
is localized to his lower back and gets worse throughout the Regular rate and rhythm; no appreciable murmurs, gallops,
day. The pain was severe during his physical exam this morn- or rubs
ing. which is why he was referred to the hospital for further
workup. He denies nausea, vomiting, fevers. chills, chest pain, ~ Abdominal
shortness of breath. and bowel or bladder incontinence. Soft, non-tender, non-distended; bowel sounds present
Past Medical History ~ Genital/Rectal
Hyperlipidemia, hypertension Exam deferred

Family History ~ Extremities


Noncontributory No edema; peripheral pulses intact; normal range of motion;
no evidence of injection sites or skin infection
Social History
~ Bade
Denies illicit substances; consumes EtOH socially-
He has reproducible pain in the lwnbar spine.
approximately three 12 oz beers weekly; lives with his wife of
35 years; employed as an electrician ~ Neurologk
Allergies Cranial nerves JI-XII are intact. Alert and oriented x 3;
mood, affect appropriate
NKDA
Laboratory Findings
Home Medications
Atorvastatin 40 mg PO daily Na 131 mEq/L Hgb 14.l gldL AST35IU/L
Lisinopril 40 mg PO daily K4.2mEq/L Hct42% AI.T40IU/L
Acetaminophen 500 mg PO PRN for back pain Cl97mEq/L Plt 498 x 103/mm' Alk Phos 80 IU/L
Ibuprofen 400 mg PO PRN for back pain; he has increased use COi24mEq/L WBC 12.3 x tol/mm3 T Bill 1.4 mg/dL
of APAP and ibuprofen over the past 6 weeks
BUN 19mg/dL Neutros77%
Physical Examination SCr 1.1 mg/dL Bands 1796 ESR 103 mm/hr
~ Vital Signs Glucose 89 mg/dL Lymphs 4% CRP76mg/L
Temp 98.4°F. BP 151/98 mm Hg, P 83bpm.RR14 breaths per Ca 9.4 mg/dL Monos 8%
minute. 9696 0 2 situation on room air. Ht 5'9". Wt 90 kg Mg2.l mgld.L
79
80 INFECTIOUS DISEASES: A CASE STUDY APPROACH

~ CTScan therapy is noninferior to intravenous antibiotic therapy.7


CT scan of abdomen and pelvis are unremarkable. Lumbar Based on the results of this study, would it be appropriate
spine shows degenerative disc disease. for CJ to receive oral antibiotics?
A. Yes
~ MRI
B. No
Tl-weighted-decreased signal intensity in lumbar vertebral 7. Which of the following oral antibiotic regimens performed
bodies and disc; T2-weighted-increased disc signal intensity. worst in the OVIVA study?
Impression: osteomyelitis of lumbar spine, no evidence of A. Penicillins
abscess B. Quinolones
C. Tetracyclines
~ Blood and Urine Cultures D. Macrolides
Pending 8. Which of the following selections accurately ranks antibi-
otics in the order of bioavailability? Rank is from highest
to lowest.
QUESTIONS A. Levofloxacin > Azithromycin > Amoxicillin
1. What is the mostlikelysource ofCJ's vertebral osteomyelitis? B. Doxycycline > Ampicillin > Penicillin
A. Skin infection C. Clarithromycin > Tetracycline > Moxiflo:xacin
B. IV drug use D. Minocycline > Amoxicillin > Ciproflo:xacin
C. Poor dentition 9. Culture and susceptibility results from CJ's bone biopsy
D. Diverticulitis flare have identified methicillin-sensitive Staphylococcus
2. What is CJ's primary symptom of vertebral osteomyelitis? aureus (MSSA). CJ is prescribed ceftriaxone 2 g IV q24h.
A. Fever What is the recommended antibiotic duration as per the
B. Back.pain IDSA Vertebral Osteomyelitis Guidelines?
C. Elevated ESR and CRP A. 4weeks
D. Hemodynamic instability B. 6 weeks
C. 8 weeks
3. A physician in the emergency room saw CJ's MRI results D. 12weeks
and initiated vancomycin and piperacillin/tazobactam.
While in the ED, CJ received 1 dose each of vancomy- 10. When should CJ have his ESR and CRP repeated to monitor
cin and piperacillin/tazobactam before a biopsy of the his response to treatment?
infected bone was performed. How will the receipt of A. Never, there is no utility in ESR and CRP as monitoring
antibiotics affect the likelihood of growing a pathogen parameters
from the biopsy? B. Daily
A. Reduces microbiology yield by 25% C. Weekly
B. Reduces microbiology yield by 50% D. After 4 weeks of antibiotic therapy
C. Reduces microbiology yield by 90%
D. No reduction in microbiology yield if biopsy is within
ANSWERS
3 days of antibiotic receipt
4. When should empiric antibiotics be initiated for CJ? 1. Explanation: The correct answer is C. Poor dentition is
A. As soon as osteomyelitis is seen or suspected in imaging noted in the physical exam. The three pathways of osteo-
(CT, MRI, X-ray) myelitis are hematogenous spread (through a blood stream
B. After blood cultures obtained infection), direct inoculation from trauma, or contiguous
C. After bone biopsy is performed spread from adjacent skin infection. Hematogenous spread
D. After bone biopsy, culture, and susceptibilities are is the most common cause and adults with degenerative
complete disc disease are at a higher risk. A systematic review of
clinical characteristics of pyogenic (bacterial) vertebral
5. What pathogen(s) is/are the most common cause of ver- osteomyelitis found that the most common source was
tebral osteomyelitis and should be covered in the setting the urinary tract, followed by skin infections, intravenous
of negative biopsy cultures or if biopsy is not performed? catheters, respiratory, gastrointestinal, or oral infection.1
A. Streptococci Choices A, B, and D are incorrect because CJ has no evi-
B. Staphylococcus aureus dence of skin infection ( erythema, edema, warmth), IV
C. Gram-negative bacilli drug use (visible tracks or injection site lesions), or symp-
D. Mycobacterium tuberculosis toms of diverticulitis on physical exam.
6. The OVIVA study (Oral versus Intravenous Antibiotics 2. .Explanation: The correct answer is B. The major clini-
for Bone and Joint Infectio'ii) inve;tigaid if oral antibiotic cal manifestation of vertebral osteomyelitis is back pain.
CHAPTER 19 I VERTEBRAL OSTEOMYELITIS 81

A patient's pain is typically Ioc:alized to the infected disc had a higher nwnerical. but not statistical, failure rate
space area and is exacerbated by phpical activity or than patients treated with quinolones, tetracyclines, or
percussion to the affected area. Answer A is incorrect macrolides.
because CJ did not have a fever (temperature ~38°C).
Answer C is incorrect beause elevated ESR and CRP are Failure, No Failure, Odds Ratio
associated with a diagnosis of osteomyelitis but are signs n(%) n(%) (95%CI)
not symptoms. Answer D is incorrect because CJ is not
Penicillins 17/57 9/48 1.6 (0.7-3.9)
hemodynamically unstable.
Quinolones 20/189 26/179 0.7 (0.4-1.4)
3. Explanation: The correct answer is D. A study of 173
Tetracyclines 7/40 7/30 0.8 (0.2-2.4)
with a diagnosis of vertebral osteomyelitis found no dif-
ference in microbial yield between patients who received Macrolides 5/53 6/44 0.7 (0.2-2.4)
antibiotics before or after a biopsy.:i One study found that
a duration ofantibiotic of 4 or more da:ys was a risk factor 8. bplanation: The correct answer is D. One of the poten-
for negative cultures.' tial reasons suggested for the difference in outcomes
4. Explanation: The correct answer is C. The IDSA Native among antibiotic classes in the OVIVA study is the dif-
Vertebral Osteomyelitis Guidelines recommend delaying ferences in bioavailability. While penicillin antibiotics
antibiotics until a microbiological diagnosis is made for have lower bioavailability as a class than qu.inolones or
patients with normal and stable neurologi.c examination tetracyclines. they are higher than macrolides. Dosing
and stable hemodynamic.• In patients with hemodynamic frequency is higher among penicillins and may lead to
instability. sepsis. septic shock. or severe or progressive higher failure rates. Of note. adherence rates between
neurologic symptoms, the guidelines suggest initiating the IV and PO groups were similar. Another difference
empiric antibiotics in conjunction with getting a micro- between antibiotics is bone penetration which may
biologic diagnosis. Of note. both recommendations are impact cure rates.• While the OVIVA trial is an impor-
weak and supported by low quality evidence. In a small tant trial and demonstrated that PO antibiotics can
study of nondrug users. blood cultures were positive in safely be used to treat bone and joint infections. we are
72% of patients and can aid in the microbiologic diagno- still in need of more information about specific antibiot-
sis in patients with vertebral osteomyelitis.5 ics and dosing regimens.

5. Explanation: The correct answer is B. Most patients have Bone


monomicrobial infections. The most common cause of concentration'
vertebral osteomyeliti& is Staphylococcus aureus. A study Antibiotic Bioaftilabllitf Frecniency (mgr/a)
in which 101 patients with an identified organism found Penicillin VK 60% 4 x daily unknown
that methicillin-sensitive Staphylococcus aureus (MSSA) Ampicillin SO% 4 x daily 12-20
was the most prevalent (63%).6 Gram-positive pathogens Amo:ddllin 77-80% 2-3 x daily 15-20
(staphylococci and streptococci) accounted for 87% of Ciprofioxacin 60-80% 2 x daily 0.7-1.4
cultured pathogens compared to 7% for gram-negative Mo:lifloxacin 9096 1 x daily 1.3-1.9
bacteria. Mycobaderium tuberculosis (MTB) can also cause Levoflo:a:ac.ln 99% 1 x daily 3-6
vertebral osteomyeliti&. Unlike pyogenic vertebral osteo- Tetracycline 8596 4 x daily unknown
myelitis, MTB infections are more subacute and diagnosis 2 x daily 0.1-2.6
Doxycycline 95%
is often not made until 4 to 6 months after symptom onset
Minocycline 95% 2 x daily 0.4-0.9
6. Bl.planation: The correct answer is A. The OVIVA Azithromycin 3896 1 x daily 1.9
study included 39 patients with acute or chronic verte- Clarithromycin SS% 2 x daily unknown
bral osteomyelitis.7 Important exclusion criteria include
patients with Staphylococcus aureus bloodstream infec-
9. E:iplanation: The correct answer is B. The IDSA Vertebral
tions and/or endocarditis. The OVIVA study results
Osteomyelitis Guidelines suggest 6 weeks of antibiotics
revealed that treatment failure at 1 year occurred in 15%
for all bacterial infections except Salmonella species for
(74/506) of patients in the IV group and 13% (67/509) in
which recommendations are 6 to 8 weeks of antibiotics.•
the PO group. These results demonstrated that that PO
A randomized trial of patients with pyogenic vertebral
antibiotics were noninferior to IV antibiotics for the treat-
osteomyelitis compared outcomes between 6 and 12 weeks
ment of bone and joint infections.
of antibiotics.10 Clinical cure at 1 year was 91% in both
7. Explanation: The correct answer is A. The OVIVA the 6- and 12-w~k treatment groups. Nearly half of the
study did not seek to compare specific antibiotic regi- patients in this trial received an oral fluoroquinolone and
mens or to stipulate which agents should or should not rifampicin. Contrary to these results, the median duration
be used.7 However. as per Figure S2 from the Supple- of antibiotics in the OVIVA study is 11 weeks in the IV
mentary Appendix patients treated with oral penicillins group and 10 weeks in the PO group.'
82 INFECTIOUS DISEASES: A CASE STUDY APPROACH

10. .Explanation: The correct answer is C. The IDSA Ver- guidelines for the diagnosis and treatment ofnative vertebral
tebral Osteomyelitis Guidelines suggest monitoring sys- osteomyelitis in adults. Clin Infect Dis. 2015;61(6):e26-e46.
temic inflammatory markers (ESR and/or CRP) after 5. Nolla JM, Ariza J, G6mez-Vaquero C, et al. Spontaneous
approximately 4 weeks of antibiotic therapy.4 Monitoring pyogenic vertebral osteomyelitis in nondrug users. Semin
ESR and CRP may help identify patients at greater risk for Arthritis Rheum. 2002;31(4):271-278.
treatment failure. Patients with a 50% reduction in ESR 6. Chong BSW. Brereton CJ, Gordon A, Davis JS. Epidemiol-
after 4 weeks rarely develop treatment failure. 11 Of note, ogy, microbiological diagnosis, and clinical outcomes in
most patients who do not experience a significant drop pyogenic vertebral osteomyelitis: a 10-year retrospective
in systemic inflammatory markers still have successful cohort study. Open Forum Infect Dis. 2018;5(3):ofy037.
outcomes. 7. Li HK, Rombach I, Zambellas R, et al. Oral versus intra-
venous antibiotics for bone and joint infection. N Engl J
Med. 2019;380(5):425-436.
REFERENCES 8. Spellberg B, Lipsky BA. Systemic antibiotic therapy
1. Mylona E, Samarkos M, Kakalou E, Fanourgiakis P, for chronic osteomyelitis in adults. Clin Infect Dis.
Skoutelis A. Pyogenic vertebral osteomyelitis: a systematic 2012;54(3):393-407.
review of clinical characteristics. Semin Arthritis Rheum. 9. IBM Micromedex', Micromedex Solutions. Ann Arbor,
2009;39(1):10-17. MI: Truven Health Analytics.
2. Saravolatz LD, Labalo V, Fishbain J, Szpunar S, Johnson 10. Bernard L, Dinh A, Ghout I, et al. Antibiotic treatment for
LB. Lack of effect of antibiotics on biopsy culture results 6 weeks versus 12 weeks in patients with pyogenic vertebral
in vertebral osteomyelitis. Diagn Microbiol Infect Dis. osteomyelitis: an open-label, non-inferiority, randomised,
2018;91(3):273-274. controlled trial. Lancet. 2015;385(9971):875-882.
3. Kim CJ, Song KH, Park WB, et al. Microbiologically and 11. Yoon SH, Chung SK, Kim KJ, Kim HJ, Jin YJ, Kim HB.
clinically diagnosed vertebral osteomyelitis: impact of Pyogenic vertebral osteomyelitis: identification of micro-
prior antibiotic exposure. Antimicrob Agents Chemother. organism and laboratory markers used to predict clinical
2012;56(4):2122-2124. outcome. Bur Spine J. 2010;19(4):575-582.
4. Berbari EF, Kanj SS, Kowalski TJ, et al. 2015 Infectious
Diseases Society of America (IDSA) clinical practice
20 Prosthetic Joint
Infection
Amelia K. Sofjan

PATIENT PRESENTATION Physical Examination


Chief Complaint ~ Vital Slgn1
"My right knee hurts really bad." Temp 100°F, P 78, RR 16 breaths per minute, BP 148/92 mm
Hg, p02 98% on room air, Ht 5'4", Wt 90.7 kg
History of Present Illness
~ General
AD is a 66-year-old female who had a right total knee arthro-
pla.sty (TKA) and has been experiencing right lmee pain that Well-groomed female in moderate distress
has increased gradually in severity since 2 weeks ago. The pain
is persistent and moderate to severe. She al.so has been having ~ HEENT
trouble walking around the block. Of note, the right knee is PERRLA, EOMI
swollen and red. She followed up in clinic with her surgeon yes-
terday and was told she needed to be admitted to the hospital. ~ Pulmonary
Clear to auscultation, no crackles or wheezing
Past Medlcal History
Obesity, type 2 diabetes. osteoarthritis. hypertension, hyper- ~ Cardiovascular
lipidemia. depression RRR. no m/r/g

~ Abdomen
Surgical History
Right TKA 3 weeks ago, C-section Normal BS, abdomen soft and tender

~ Neurology
Family History
A&O X3, CN II-XII grossly intact
Father died at age 62 from a heart attack; mother died at age
70 from breast cancer ~ Extremities
Right knee is edematous and erythematous; knee feels stable
Social History but warm to the touch
No alcohol, tobacco, or IV drug use; lives at home with her
husband; she is retired Laboratory Findings
Na = 140 mEq/L Hgb = 14.1 g/dl AST= 20IU/L
Allergies
K=4.3mEq/L Hct= 42% ALT=24W/L
Septra ("bad rash")
Cl = 102 mEq/L Plt= 167 x T Bill = 1.5 mgldL
103/mm1
Home Medications
Metformin 1000 mg PO BID
col= 27 mEq/L WBC = 9 x Alie Phos =86 W/L
103/mm'
Desvenlafuine 50 mg PO daily
Ibuprofen 600 mg PO TID PRN BUN = 24 mg/dl
Lisinopril 40 mg PO daily Ser = 1.01 mg/dl
Hydrochlorothiazide 25 mg PO daily Glu = 158 mg/dl
Simvastatin 80 mg PO daily

83
84 INFECTIOUS DISEASES: A CASE STUDY APPROACH

QUESTIONS C. 7 to 11 months
D. 12 to 24 months
1. Which of the following is the most common organism iso-
lated in PJI? 7. Two years later, AD is diagnosed with another episode of
A. Staphylococcus staphylococcal PJI and will undergo a 2-stage exchange.
B. Enterococcus How is the antibiotic treatment for this 2-stage exchange
C. Streptococcus similar or different compared to the treatment for DAIR
D. Gram-negative bacilli that AD received initially?
A. AD will need rifampin as part of combination therapy
2. AD's ESR was 40 mm/hr, and CRP was 15 mg/dL. An
just like in DAIR
arthrocentesis Ooint aspiration) revealed a leukocyte count
B. AD will need 3-6 months of antibiotic therapy just like
of 5,100 cells/µL (88% neutrophils). The synovial fluid
in DAIR
aerobic culture is positive for CONS. AD was taken to the
C. AD will need pathogen-specific therapy up until the
operating room for an inspection of her right knee, and
day of the reimplantation
the intra-operative note stated purulence surrounding the
D. AD will need pathogen-specific therapy for 4 to 6
prosthesis. Which of AD's findings is definitive for PJI?
weeks
A. Elevated ESR and CRP
B. Elevated leukocyte and neutrophil count of the synovial
fluid
C. Synovial culture positive for CONS ANSWERS
D. Purulence surrounding the prosthesis I. Explanation: The correct answer is A. Choices B, C, and D
are incorrect because the most common organisms isolated
3. The orthopedic surgeon opted for DAIR (debridement, in PJI are coagulase-negative Staphylococcus (CONS) and
antibiotics, and implant retention) to manage AD's PJI. Is Staphylococcus aureus (50% to 60%), followed by Entero-
DAIR a reasonable surgical strategy for AD? coccus and Streptococcus (10%). Gram-negative bacilli are
A. Yes, because the synovial leukocyte and neutrophil uncommon, and fungal and atypical bacteria have been
count were not extremely high typically isolated in patients with malignancy, autoim-
B. Yes, because the prosthesis was implanted 3 weeks ago mune or immunocompromising conditions, or prolonged
C. No, because her symptoms have lasted for 2 weeks
antibiotic use.1
D. No, because her synovial fluid is positive for CONS
2. Explanation: The correct answer is D. Choice A is incor-
4. AD's pre-operative synovial fluid and 2 intra-operative
rect because elevated ESR and CRP are supportive of a PJI
cultures are growing CONS with the following susceptibil-
diagnosis but nonspecific. Choice B is incorrect because
ity report: oxacillin (R), vancomycin (S), tetracycline (S),
elevated leukocyte and neutrophil count have good sensi-
TMP/SMX (S), clindamycin (R), rifampin (S), linezolid (S),
tivity and specificity for PJI but are not definitive. Choice
ciprofloxacin (S), daptomycin (S). Which of the following C is incorrect because one culture positive for a common
is the preferred initial antibiotic therapy for AD? AD is skin flora such as CONS is not definitive for PJI and must
willing to switch her non-infective drugs if needed except
be interpreted in the context of other findings. However,
her antidepressant.
two cultures (both intra-operative or one pre-operative
A. Vancomycin
and one intra-operative) growing the same organism is
B. Rifampin
definitive for PJI. Purulence surrounding the prosthesis
C. Vancomycin plus rifampin in the absence of other etiology, a sinus tract that com-
D. Linezolid
municates with the prosthesis, or acute inflammation in
E. Daptomycin
histopathological exam is definitive for PJI.1.2
5. AD was initiated on vancomycin plus rifampin for 4 weeks
3. Explanation: The correct answer is B. Please see table for
and is now being transitioned to an all oral regimen. Which
the different types of surgical options and their respective
of the following is the most optimal recommendation?
patient candidates.2 Synovial leukocyte and neutrophil
A. Ciprofloxacin 750 mg PO BID
count are not determinants of surgical management (rules
B. Doxycycline 100 mg PO BID
out choice A). Choice C is incorrect because AD's symp-
C. Levofloxacin 250 mg PO daily + rifampin 300 mg
toms lasted <3 weeks, which makes her a good candidate
PO BID
for DAIR. Choice D is incorrect because CONS is not viru-
D. Ciprofloxacin 750 mg PO BID + rifampin 300 mg
lent and typically not multidrug resistant, making DAIR a
PO BID
reasonable strategy. PJis due to more virulent organisms,
6. Which of the following is the recommended total antibiotic such as S. aureus, have been associated with increased rates
treatment duration for AD? of DAIR failure, and it may be worse with methicillin-
A. 4 to 6 weeks resistant S. aureus (MRSA).3 However, a recent study did
B. 3 to 6 months not find this association.• DAIR can still be appropriate for
CHAPTER 20 I PROSTHETIC JOINT INFECTION 85

MRSA PJls depending on other patient-specific factors as benefits outweigh the risks. Rif.unpin can cause hepatotox-
listed in the table. icity and interacts with many drugs including AD's si.m.v-
astatin. In this case the drug can be switched to another
Surgical management of PJI: deacription of procedure statin, but sometimes it may not be feasible to modify
and patient candidates the interacting medication, thus precluding rifampin use.
Debridement. antibiotics, and implant retention (DAIR) Linezolid (choice D) has a success rate of about 80% but
Debridement, exchange of modular components, and is an alternative agent due to less clinical experience and
prolonged antibiotic therapy risk of thrombocytopenia, peripheral neuropathy, and
• Symptom duration <3 weeks or prosthesis implantation optic neuritis with prolonged use.3.a Linezolid is also a
within past 30 days PLUS well-fixed prosthesis, no sinus weak monoamine-oxidase inhibitor and interacts with
tract, and organism susceptible to oral antibiotic agents desvenlafuine, putting AD at risk for serotonin syndrome,
• Elderly patients with comorbidities unable to do 1- or a rare but serious event~ Daptomycin (choice E) dosed at
2-stage exchange about 6 to 8 mg/kg/d has a success rate ranging from 58%
to 88% but is currently an alternative agent due to less clini-
I-stage euhaoge cal experience, although more data have been published
Removal ofimplant, debridement, and implantation of new recently.l.10.11 Daptomydn has been used as salvage therapy
prosthesis all in the same surgery or when vancomycin cannot be used due to allergy or
• Typically done in Europe toxicities. It can cause elevated CPI<, myopathy, and rarely
2-ttage euhange rhabdomyolysis. Twice weekly CPK monitoring is prudent
Removal ofimplant, debridement ± placement of in patients on concomitant statin and daptomycin to mini-
antibiotic-impregnated spacer, antibiotic therapy mize these adverse effects.
(lst surgery), then spacer exchange or prosthesis
5. Expl•n•tion: The correct answer is D. Because the implant
reimplantation (2nd surgery)
is retained, rifampin should be continued for its activity
• Commonly used in the United States if medically stable against biofilm organisms in staphylococcal PJI. The treat-
and able to undergo multiple surgeries ment of choice following IV therapy plus rifampin is an
Permanent raec:tion arthroplasty oral agent plus rifampin (rules out choices A and B).2.5-7
Removal ofimplant with or without arthrodesis Ooint Levofloxacin or ciprofloxadn is a first-line oral agent for
fusion) or limb amputation staphylococcal PJI based on good efficacy data.'-12 Choice
• Non-ambulatory, multiple comorbidities, or minimal C is incorrect because levofloxa.cin should be dosed at
750 mg PO daily.12
bone stock or soft tissue coverage
• Multi.drug-resistant organisms or refractory/relapsing 6. Explanation: The correct answer is B. IDSA guidelines
infection despite all efforts recommend 6 months for TICA and 3 months for total hip
• Failed prior 2-stage exchange and infection risk after arthroplasty (THA) based on older studies.2.5-7 However,
rei.m.plantation is unacceptable a recent observational study of S. aureus PJI managed
Source: Diits. from Osmon DR, Berbari BF, Berendt AR, et al. with DAIR reported a mean ± SD treatment duration of
Diagnosis and management of prosthetic joint infection. Clin Infect 12.6 ± 7 weeks, which was similar for TKA and THA.1] In
Dis. 2013;56:el-e25. this study, longer treatment duration was independently
associated with success at 2 years. Overall the evidence
4. Explanation: The correct answer is C. The initial treat-
suggests TKA may be potentially treated as short as 3
ment of choice following DAIR for oxacillin-resistant
months like THA and up to 6 months. Lastly, a recent
staphylococcal PJI is vancomydn plus rifampin (if the
randomized trial of patients with acute staphylococcal PJI
staphylococcus is susceptible to rifampin and rifam.pin
managed with DAIR showed no difference in cure rates
can be used safely) for 2 to 6 weeks.2 Vancomycin is pre-
between 8 weeks vs 3 months or 6 months of treatment
ferred due to established clinical experience but requires
duration for THA or TICA, respectively. However, the trial
therapeutic drug monitoring and can cause nephrotoxicity
had a small sample size and was not adequately powered;
and leukopenia. Rifampin is used for staphylococcal PJl
hence, the benefit of antibiotics beyond 8 weeks cannot
managed by DAIR because of its activity against biofilm
be ruled out.12 Nevertheless, shorter treatment duration
organisms which form in the presence of a foreign body.
should be evaluated in larger future trials. Current evi-
Rifampin should always be used in combination due to
dence does not support choice A, C, or D.
a high rate of resistance emergence if used alone (rules
out choice B). Two observational studies and a random- 7. Explanation: The correct answer is D. The recommended
ized controlled trial of patients with staphylococcal PJI, medical management in patients undergoing a 2-stage
in which 21% to 44% of patients had CONS (the rest had exchange is 4 to 6 weeks of pathogen-specific intravenous
S. aureus). showed the benefit of rifampin combination.s-7 or highly bioavailable oral antibiotic therapy. Because the
Although the case for adding rifampin is stronger for S. implant is removed in a 2-stage exchange, rifampin is not
aureus PJI, rifampin should be added for CONS PJI if generally recommended and the treatment duration is
86 INFECTIOUS DISEASES: A CASE STUDY APPROACH

shorter (4 to 6 weeks) than that of DAIR (rules out choices 7. Peel TN, Buising KL, Dowsey MM, et al. Outcome of
A and B). In contrast, rifampin and prolonged antibiotic debridement and retention in prosthetic joint infections
therapy are necessary in patients with staphylococcal PJI by methicillin-resistant staphylococci, with special refer-
undergoing DAIR because the implant is retained. Choice ence to rifampin and fusidic acid combination therapy.
C is incorrect because after a patient receives 4 to 6 weeks Antimicrob Agents Chemother. 2013;57:350-355.
of antibiotic therapy, antibiotics should be stopped for 2 to 8. Morata L, Tornero E, Martinez-Pastor JC, Garda-Ramiro
4 weeks prior to reimplantation to optimize the diagnostic S, Mensa J, Soriano A. Clinical experience with linezolid
yield of cultures during reimplantation.2 for the treatment of orthopaedic implant infections.
f Antimicrob Chemother. 2014;69(suppl l):i47-i52.
9. Karkow DC, Kauer JF, Ernst EJ. Incidence of serotonin
REFERENCES syndrome with combined use of linezolid and serotonin
1. Abad CL, Haleem A. Prosthetic joint infections. Gurr reuptake inhibitors compared with linezolid monotherapy.
Infect Dis Rep. 2018;20:15. l Clin Psychopharmacol. 2017;37(5):518-523.
2. Osmon DR, Berbari EF, Berendt AR, et al. Diagnosis and 10. Chang YJ, Lee MS, Lee CH, Lin PC, Kuo FC. Daptomycin
management of prosthetic joint infection. Clin Infect Dis. treatment in patients with resistant staphylococcal peri-
2013;56:el-e25. prosthetic joint infection. BMC Infect Dis. 2017;17:736.
3. Salgado CD, Dash S, Cantey JR, Marculescu CE. Higher 11. Malizos K, Sarma J, Seaton RA, et al. Daptomycin for the
risk of failure of methicillin-resistant Staphylococcus treatment of osteomyelitis and orthopaedic device infec-
aureus prosthetic joint infections. Clin Orthop Relat Res. tions: real-world clinical experience from a European reg-
2007;461:48-53. istry. Eur J Clin Microbial Infect Dis. 2016;35:111-118.
4. Fink B, Schuster P, Schwenninger C, Frommelt L, 12. Lora-Tamayo J, Buba G, Cobo J, et al. Short- versus long-
Oremek DA. A standardized regimen for the treatment of duration levofloxacin plus rifampicin for acute staphylo-
acute postoperative infections and acute hematogenous coccal prosthetic joint infection managed with implant
infections associated with hip and knee arthroplasties. retention: a randomized clinical trial. Int l Antimicrob
l Arthroplast. 2017;32(4):1255-1261. Agents. 2016;48:310-316.
5. Widmer AF, Gaechter A, Ochsner PE, Zimmerli W. 13. Lesens 0, Ferry T, Forestier E, et al. Should we expand
Antimicrobial treatment of orthopedic implant-related the indications for the DAIR (debridement, antibiotic
infections with rifampin combinations. Clin Infect Dis. therapy, and implant retention) procedure for Staphylo-
1992;14:1251-1253. coccus aureus prosthetic joint infections? A multicenter
6. Zimmerli W, Widmer AF, Blatter M. Role of rifampin retrospective study. Eur f Clin Microbiol Infect Dis.
for treatment of orthopedic-implant related staphylo- 2018;37:1949-1956.
coccal infections: a randomized controlled trial. JAMA.
1998;279:1537-1541.
21 Intra-Abdominal
Infections
Jamie L Wagner

PATIENT PRESENTATION Physical Examination


Chief Complaint ~ General Appearance
"'My belly kups filling with fluid, and I don't feel well." Cachectic appearing female in NAD, supine in bed

~ Vital Signs
History of Present Illness
SB is a 49-year-old female with a PMH of HCV who pre- Temp 101.5°F, HR 80, BP 108/72, RR 16 breaths per minute,
sented with complaints of sharp, constant abdominal pain Sp02 98% RA, Ht 5'4H, Wt 46 kg
and swelling.
~ Skin, Hair, and Nalls
Past Medical History Jaundice. no rashes or bruises

• Childhood lllneun ~ HEENT


Measles at age 5, recurrent OM AT, bitemporal wasting, PERRL bfl, no ptosis, + scleral
icterus, nares patent, MMM
• Travel
Recent trip to Arizona ~ Neck
Supple, NROM, + JVD
• Habits
+ EtOH, (-) tobacco, (-) illicits ~ Chest and Lungs
Ct.AB, no w/r/r
• Immunizations
Up-to-date on all childhood immunizations ~ Heart
RRR. no m/r/g, severe pitting ederna to level of abdomen
• Surgical History
TWo pins in R great toe s/p bunion repair ~ Abdomen
Taut, distended, fluid wave, midline large hernia, caput medu-
• Family History sae, RUQ guarding. hypoactive bs, massive ascites
Mother: MI, DM2, dyslipidemia, obesity
Father: DM2, obesity, deceased at age SO ~ Extremities
3+ pitting edema BLLE
• Social History
SB currently lives with her mother. She works a full-time job ~ Neurologk
as a teller at the local credit union. Sensation intact, CN II-Xll intact, strength 5/5 in UE and LE,
+ asterW.s, + ataxia
• Allergln
NKDA • Psychiaftic
Appears confused, somnolent
• Home Medic.ations
None

87
88 INFECTIOUS DISEASES: A CASE STUDY APPROACH

Laboratory Findings
Chemistry, Coags, CBC
Sodium Potassium Chloride co2 (20-30) BUN Creatinine Glucose Calcium
(136-146) (3.5-5.1) (98-108) mmol/L (7-17) (0.5-1.2) (74-106) (8.4-10.2)
mmol/L mmol/L mmol/L mg/dL mg/dL mg/dL mg/dL
122 4.1 96 20 20 1.87 86 9.1
T. Protein Albumin T. Bili D. Bili (0-0.3) I. Bili Allc Phos AST (0-40) ALT (0-41)
(6.6-8.7) g/dL (3.5-5.5) g/dL (0-1) mg/dL (0-1.1) (40-129) U/L U/L U/L
mg/dL mg/dL
6.2 2.3 18.6 10.5 8.1 263 108 52

PT ( 11-13.5) INR (<1.1) Ammonia WBC RBC Hgb Hct Platelets


sec (15-45) (4.0-11.0) (4.5-5.4) (12.5-15.5) g/dL (39.0-45.0) % (130-400)
mcg/dL th/cmm m/cmm th/cmm
17.8 1.84 153 13.5 4.5 12.1 36.5 42

CTAbdomen C. Vancomycin
D. Ceftriaxone
~ ABDOMEN
4. The preliminary ascitic fluid Gram stain resulted in gram
Nodular contour of the liver which demonstrates diffuse
low attenuation of the hepatic parenchyma. No gallbladder ( - ) bacilli. Which of the following pathogens is most likely
wall thickening and/or presence of pericholecystic fluid. No to be cultured?
A. Salmonella enterica
biliary dilatation. There is presence of portosystemic vari-
ces within the anterior abdominal wall and upper abdomen. B. Citrobacter freundii
Splenomegaly with findings of portal venous hypertension. C. Klebsiella pneumoniae
D. Pseudomonas putida
Large amount of ascites present.
5. Escherichia coli was isolated from the ascitic fluid culture.
~ IMPRESSION How long would you treat SB?
Diffuse low-attenuation of the hepatic parenchyma with nod- A. 3 days
ular contour favoring steatosis/cirrhosis. Splenomegaly and B. 5 days
extensive varices with findings of portal venous hypertension. C. 7 days
D. 10 days
~ Cultures 6. SB is discharged from the hospital with prophylaxis
Blood Cx pending against another episode of SBP. What is the recom-
mended antibiotic for SBP prophylaxis in a non-hospital-
ized patient?
QUESTIONS A. Ciprofloxacin 500 mg PO daily
1. Which laboratory test(s) help identify a patient with spon- B. Doxycycline 100 mg PO daily
taneous bacterial peritonitis? C. Amoxicillin 875 mg PO daily
A. Positive blood culture D. Cefdinir 300 mg PO daily
B. Ascitic cell count and differential
C. Ascitic albumin 7. SB is readmitted to the hospital 3 weeks later with a
D. Serum ammonia variceal hemorrhage. Which of the following should
be started to provide prophylaxis for SBP for this
2. What is a sign or symptom of spontaneous bacterial peri- admission?
tonitis in SB? A. Oral trimethoprim/sulfamethoxazole
A. Demonstration of abdominal pain B. Oral ciprofloxacin
B. Presence of portosystemic varices C. IV ceftriaxone
C. Explainable encephalopathy D. IV ampicillin
D. All of the above
8. SB should receive long-term prophylaxis against SBP based
3. SB's ascitic fluid analysis revealed PMN count of 729 cells/ upon which of the following risk factors?
mm3• The diagnosis of spontaneous bacterial peritonitis is A. Ascitic fluid total protein < 1.5 g/dL
made. Which empirical therapy is most appropriate for SB? B. Previous variceal hemorrhage
A. Cefazolin C. Previous SBP episode
B. Piperacillin/tazobactam D. All of the above
CHAPTER 21 I INTRA-ABDOMINAL INFECTIONS 89

ANSWERS SBP in patients who have already experienced at least 1


episode.3 Routine prophylaxis for gut decontamination in
1. Explanation: The correct answer is B. Answer A is incor-
cirrhosis patients does not select for resistant bacteria.'
rect because infection within the peritoneum does not
always infiltrate the bloodstream; therefore, ascitic fluid 7. Explanation: The correct answer is C. While PO trim-
cultures would be more accurate and appropriate. Answer ethoprim/sulfamethoxazole and PO ciprofloxacin can be
C is incorrect because ascitic albumin is used to help diag- used to prevent SBP on an outpatient basis, during an
nose the presence of portal hypertension, not spontaneous acute variceal bleed, IV therapy is recommended. 1 There-
bacterial peritonitis. Answer D is incorrect because serum fore, answers A and B are incorrect. Answer D is incorrect
ammonia is used to help diagnose hepatic encephalopathy, because the spectrum of activity is not adequate to cover
not spontaneous bacterial peritonitis. Answer B is correct all the suspected pathogens and has been proven to be
because spontaneous bacterial peritonitis ( eg, primary inferior to third-generation cephalosporins.5 Answer C is
peritonitis) is diagnosed by an elevated ascitic fluid abso- correct because IV ceftriaxone for 7 days has demonstrated
lute polymorphonuclear leukocyte (PMNs) ~250 cells/ efficacy in preventing SBP in patients with cirrhosis and
mm3 without an evident intra-abdominal, surgically treat- GI-related hemorrhages.5
able infection. 1
8. Explanation: The correct answer is D. According to the
2. Explanation: The correct answer is A. Answer B is incor- guidelines, answers A, B, and C were identified as risk fac-
rect because the presence of varices is not indicative of tors most closely associated with subsequent episodes of
SBP, but rather, portal hypertension. Answer C is incorrect SBP. Other patient characteristics (eg, number of previous
because SBP is related to unexplainable encephalopathy SBP episodes, age, sex, etiology of cirrhosis, liver function
(eg, ammonia within normal limits) instead of explainable tests, previous treatment for SBP, previously isolated organ-
encephalopathy. Answer A is correct because pain within isms) are not used to guide prescribing prophylaxis, as this
the abdominal cavity in a patient with ascites, in addition would lead to a more liberal use of antibiotics, eventually
to fever and unexplained encephalopathy, are indicators leading to colonization and subsequent infection by resis-
that an infection is most likely present and empiric therapy tant flora. 1.6
should be started. 1
3. Explanation: The correct answer is D. Answer A is
incorrect because the spectrum of activity is not reliable
REFERENCES
to provide coverage against all the suspected gram- 1. Runyon BA. Management of adult patients with ascites
negative pathogens (eg, Enterobacteriaceae). Answer B due to cirrhosis: update 2012. Hepatology. 2013;57(4):
is incorrect because the spectrum of activity is unneces- 1-27. Available at https ://www.aasld.org/sites/default/
sarily broad for treating the suspected gram-negative files/2019-06/AASLDPracticeGuidelineAsciteDueto
pathogens by providing coverage against anaerobes and CirrhosisUpdate2012Edition4_.pdf.
Pseudomonas aeruginosa. Answer C is incorrect because 2. Runyon BA, McHutchison JG, Antillon MR, Akriviadis
no gram-negative coverage is provided. Answer D is EA, Montano A. Short-course vs long-course antibiotic
correct because the spectrum of activity of ceftriaxone treatment of spontaneous bacterial peritonitis: a random-
can cover 95% of the suspected infecting organisms. 1 ized controlled trial of 100 patients. Gastroenterology.
1991;100:1737-1742.
4. Explanation: The correct answer is C. Answer C is cor- 3. Gines P, Rimola A, Planas R, Vargas V, Marco F, Almela
rect because K. pneumoniae is one of the 3 most common M, et al. Norfloxacin prevents spontaneous bacterial peri-
isolates, which also include E. coli and S. pneumoniae. tonitis recurrence in cirrhosis: results of a double-blind,
Answers A, B, and D are all possible options for causing placebo-controlled trial. Hepatology. 1990;12:716-724.
SBP; however, the most commonly isolated pathogen out 4. Fernandez J, Acevedo J, Castro M, Garcia 0, Rodriquez
of the answer choices provided is answer C. 1 de Lope C, Roca D, et al. Prevalence and risk factors of
5. Explanation: The correct answer is B. According to the infections by resistant bacteria in cirrhosis: a prospective
guidelines, 1 a randomized controlled trial of 100 patients study. Hepatology. 2012;55:1551-1561.
5. Felisart J, Rimola A, Arroyo V, Perez-Ayuso RM,
demonstrated that 5 days of therapy was as effective as 10
days of therapy for treatment of SBP.2 Therefore, answers A, Quintero E, Gines P, et al. Randomized comparative study
C, and D are incorrect. of efficacy and nephrotoxicity of ampicillin plus tobramy-
cin versus cefotaxime in cirrhotics with severe infections.
6. Explanation: The correct answer is A. Answers B, C, and D Hepatology. 1985;5 :457-462.
are incorrect because spectrum of activity and/or probabil- 6. Tit6 L, Rimola A, Gines P, Llach J, Arroyo v, Rodes J.
ity for adequate target attainment into ascitic fluid are not Recurrence of spontaneous bacterial peritonitis in cir-
sufficient for prophylaxis. Answer A is correct based upon rhosis: frequency and predictive factors. Hepatology.
previous data with norfloxacin success rates in preventing 1988;8:27-31.
This page intentionally left blank
22 Clostridioides
difficile Infection
Rebecca L. Dunn Jonathan C. Cho

PATIENT PRESENTATION _. General


Chief Complaint Mild distress, obese
Watery diarrhea _. HEENT
History of Present Illness Normocephalic, atraumatic, PERRLA, EOMI, pink/moist
mucous membranes and conjunctiva, no headache, no neck
AH is a 52-year-old Caucasian male with a re<:ent history of
stiffness/pain, no photophobia
hospitalization for cellulitis. He presents to the emergency
department with watery diarrhea. He reports approximately 5 _. Pulmonary
unformed stools per day over the last 3 days. He also complains
of fever, chills, abdominal cramping, and general fatigue. He CTAB
states that he was admitted to the hospital 10 days prior for
cellulitis of the right knee and was treated with clindamycin. _. Cardiovascular
He was discharged, after a 2-day admission, on 5 additional NSR. no m/r/g, DOE
days of clindamycin therapy.
_. Abdomen
Past Medical History Soft, non-distended, mildly tender to palpation, positive
T2DM, diabetic neuropathy, H1N, HLD, chronic venous sta- guarding, bowel sounds hyperactive
sis, obesity, GERD, cellulitis (resolved)
_. Genitourinary
Family History
Normal male genitalia, no complaints of dysuria or hematuria
Father with T2DM, obesity, and deceased at age 71 secondary
to a myocardial infarction. Mother with HTN (living). Sibling _. Neurology
with T2DM, H1N (living)
A&O X3, cranial nerves intact
Social History
_. Extremities
Tobacco 1/4 ppd x 45 years (quit 8 years prior}; 1 (12 oz) beer
per day. No illicit drug use Normal range of motion, no edema, changes consistent with
venous stasis
Allergies
Penicillin (swelling of the face) Emergency Department Labs
Na= 140mEq/L Hgb = 14.6 g/dL AST= 36IU/L
Home Medications
K=4.2mEq/L Hct=42% ALT = 28 IU/L
Metformin 1000 mg PO BID
Glipizide XL 10 mg PO daily Cl= 106 mEq/L Plt = 160 X l~/ Alk.Phos= 69
Pregabalin 100 mg PO TID mm' IU/L
Lisinopril/HCTZ 20 mg/12.5 mg. 2 tablets PO daily COl = 26 mEq/L WBC = 18.l x 103/ Albumin = 4.2 g/
Amlodipine/atorvastatin 10 mg/80 mg PO daily mm3 dL
ASA 81 mg PO daily BUN= 21 mg/dL Neutros = 75% T Bill = 1.7 mg/dL
Omeprazole 20 mg PO daily
Clindamycin 450 mg PO QID (recently completed a 7-day SCr = 1.7 mg/dL Bands= 9% Mg= 2.0 mg/dL
course of therapy) Glu = 209 mg/dL Lymphs= 15% Phos = 4.1 mg/dL
Monos= 1% Ca= 9.8 mg/dL
Physical Examination
_. VdalSigns _. C. diffidle Toxin Enzyme lmmunoauay (EIAJ
Temp 102.7°F, P 88, RR 18 breaths per minute, BP 158/ AJB toxin assay positive
88 mm Hg. pO:a 94%, Ht 5'10", Wt 141 kg
91
92 INFECTIOUS DISEASES: A CASE STUDY APPROACH

~ Blood Cuhures B. Handwashing with soap and water


Pending C. Hand antisepsis with alcohol-based products
D. Isolation of asymptomatic carriers of C. difficile
~ Urine Cuhures E. Antibiotic stewardship
Pending
ANSWERS
~ Fecal Occult Blood Test (FOBTJ
1. .Explanation: The correct answer is B. To increase com-
Negative parability of CDI cases and improve surveillance efforts
between clinical settings, the use of standardized defmi-
tions is recommended. CDI can be defined in several ways.
QUESTIONS Community-associated (CA) cases are those occurring in
1. According to the clinical practice guidelines by the Infec- patients with no inpatient stay in the previous 12 weeks.
tious Diseases Society of America (IDSA) and Society for Community-onset health care facility-associated (CO-
Healthcare Epidemiology of America (SHEA), how is AH's HCFA) cases are of CDI occurring within 28 days after
Clostridioides difficile infection (Cm) defined? discharge from a health care facility. Due to AH's recent
A. Community-associated (CA) CDI hospital admission, his episode most closely matches this
B. Community-onset, health care facility-associated definition. Health care facility-onset (HO) CDI are cases
(CO-HCFA) cm occurring >3 days after admission to a health care facility.
C. Health care facility-onset (HO) cm Finally, recurrent CDI is defined as an episode with posi-
D. Recurrent Cm tive symptoms and diagnostic assay following a confirmed
episode within the previous 2-8 weeks. 1
2. What risk factors does AH have for the development of
Cm? (select all that apply) 2. .Explanation: The correct answers are C and D. C. difficile
A. Age (52 years old) is the most common cause of infectious diarrhea in health
B. Serum creatinine (1.7 mgldL) care settings, and represents a significant burden on the
C. Recent antibiotic exposure health care system and affected individuals. Although the
D. Recent hospitalization body of associative risk factors is not well defmed, under-
standing risk factors can aid in prevention and diagnosis.
3. What finding qualifies AH for C. difficile testing?
The most important risk factor for the development of
A. 5 unformed bowel movements per day
cm is exposure to antibiotics. Antibiotics suppress nor-
B. PPI use
mal bowel flora, creating an environment for C. difficile
C. Recent antibiotic exposure
to grow. Third- and fourth-generation cephalosporins,
D. Recent hospitalization
fluoroquinolones, carbapenems, and clindamycin carry
4. What laboratory methods are available to diagnose patients the highest risk. However, almost all antibiotics have been
with suspected Cm? implicated. Multiple antibiotics and duration of exposure
A. Stool toxin test (toxin A and B enzyme immunoassay) also compound the risk of cm. The risk increases during
B. Glutamate dehydrogenase (GDH) enzyme immunoassay therapy and for approximately 3 months following therapy.
C. Nucleic acid amplification test (NAAT) AH has just completed a course of clindamycin; therefore,
D. All of the above this is his primary risk factor, making answer C appropri-
ate. Additionally, AH has recently been discharged from
5. What is the most appropriate treatment for AH?
the hospital, which increases his risk further (the longer
A. Vancomycin 125 mg PO QID for 10 days
the duration of hospitalization the higher the risk). While
B. Metronidazole 500 mg PO TID for 10 days
renal dysfunction is a risk for complicated disease, it is
C. Fecal microbiota transplantation
not a risk factor for the development of CDI on its own
D. Vancomycin 500 mg PO QID + metronidazole 500 mg
(rules out option B). Finally, AH's age is not considered
IVQ8H
of advanced age (rules out option A). Other risk factors
6. What follow-up and monitoring is most appropriate in the development of cm include chemotherapy, human
for AH? immunodeficiency virus, manipulation or surgery of the
A. Resolution of clinical signs and symptoms gastrointestinal tract, inflammatory bowel disease, immu-
B. Repeat NAAT nosuppression, and renal failure. Use of proton pump
C. Stool toxin test (toxin A and B enzyme immunoassay) inhibitors (PPis) has also been implicated. While the
D. Glutamate dehydrogenase (GDH) enzyme immunoassay risk associated with PPis is a source of debate, they may
decrease the protective effects of stomach acid, allowing
7. When providing care for patients with CDI, what infection
C. difficile to take hold.1
prevention and control measures should be used? (select all
that apply) 3. .Explanation: The correct answer is A. Clinically, cm
A. Donning of gloves and gowns by health care providers should be suspected in those with new-onset diarrhea,
CHAPTER 22 I CLOSTRID/OIDES DIFFICILE INFECTION 93

with current or recent antimicrobial use. However, the has a role in therapy, has fallen out of favor as a first-line
evidence surrounding appropriate testing for CDI is weak. agent due to inferior clinical response rates compared to
According to the IDSNSHEA practice guidelines, patients vancomycin (rules out option B). Additionally, repeated
with ~3 unformed stools, in 24 hours, that are both unex- or prolonged courses of metronidazole pose the risk of
plained and new-onset are the preferred target population cumulative and irreversible neurotoxicity; therefore, met-
for CDI testing. Other factors that contribute to the deci- ronidazole use should be limited. CDI cases are classified
sion to test include likelihood of infection, risk factors for as initial (new episode of symptoms and positive assay
CDI, available testing methods, and potential confounders with no symptoms and positive assay within the previous
(eg, underlying disease, laxative or other medication use) 8 weeks) or recurrent (episode with positive symptoms
(rules out options B, C, and D).Health care providers can and diagnostic assay following a confirmed episode
improve testing results by testing only those with diarrhea within the previous 2 to 8 weeks). Furthermore, initial
that cannot be explained by another cause. Laboratories episodes are stratified as non-severe, severe, or fulminant
can improve testing by rejecting stool specimens that based on clinical findings. Leukocytosis > 15,000 cells/ml
are not liquid or soft, and by collaborating with quality or a serum creatinine of > 1.5 mg/dL are the accepted
improvement teams to ensure the appropriate patient criteria used to define severe CDI. Fulminant cases are
population is being tested. 1 defined by hypotension, shock, ileus, or megacolon.
These criteria are based on expert opinion rather than
4. Explanation: The correct answer is D. The best testing
firm evidence and may require revision as more evidence
recommendation is based on whether the institution has
is obtained. Treatment selection is based on this classifi-
preagreed criteria for submission of patient stool samples
cation and stratification. Because AH's CDI is classified
(commonly submitted stool samples versus stool samples
as an initial episode, and is severe based on his clinical
from patients likely to have CDI based on clinical symp-
parameters (WBC = 18.1 x 10 3 mm3; serum creatinine =
toms). There are several testing methods that can aid
1.7 mg/dL), options C and D can be ruled out. Other
in a diagnosis of CDI and vary in their sensitivity and
treatment strategies include discontinuing therapy with
specificity. Additionally, testing methods and protocols
antibiotic agents, if possible, and starting empiric antibi-
are institution specific and based on laboratory capabili-
otics in situations where diagnosis may be delayed. 1
ties, costs, and other factors. Options include those that
detect the organism or one of its toxins (option A or B) 6. Explanation: The correct answer is A. Repeat testing
in the stool. Stool toxin enzyme immunoassays (EIAs), (within 7 days) during the same episode of diarrhea is not
once a mainstay of therapy, have variable performance, recommended unless there are significant changes in the
high costs, take a long time for completion, and have patient's clinical presentation due to false-positive results,
been replaced by newer methods with greater reliability. unnecessary retesting, and low diagnostic yields. In the
Stool toxin tests still have utility in testing algorithms, setting of recurrent CDI, repeat testing is warranted via
but are no longer used alone. Glutamate dehydrogenase a toxin test. Retesting to establish cure is discouraged
(GDH) enzyme immunoassay detects a common antigen as many patients will remain C. difficile positive after
in all isolates of C. difficile, but must be combined with a successful treatment. Therefore, the most appropriate
toxin test since it doesn't discriminate against toxigenic monitoring and follow-up is to evaluate for resolution
and nontoxigenic strains. Nucleic acid amplification of diarrhea and other signs and symptoms of ongoing
tests (NAAT), such as polymerase chain reaction (PCR), infection. 1
are more sensitive than toxin EIAs and GDH. Based on
7. Explanation: The correct answers are A, B, and E.
data, PCR is the most reliable diagnostic method when
C. difficile transmission, in the health care setting, occurs
applied to patients who meet the clinical criteria for CDI
as a result of person-to-person contact (via the fecal-oral
testing (with ;;:::: 3 unformed stools, in 24 hours, that are
route) on the contaminated hands of health care provid-
both unexplained and new-onset). This test is quick to
ers. As such, health care providers must use gloves, gowns,
perform, and is becoming the test of choice. Therefore,
and appropriate hand hygiene when caring for this patient
in patients meeting clinical criteria, and when there are
population. Hand hygiene should be performed before
preagreed institutional criteria for patient stool submis-
and after the care of patients with CDI and after glove
sion, the use of NAAT alone or a multistep algorithm
removal. The methods for hand hygiene include washing
(ie, GDH + toxin; GDH + toxin, arbitrated by NAAT;
with either soap and water or an alcohol-based hand rub.
or NAAT + toxin) is preferred. If there are no preagreed
However, hand hygiene with soap and water is preferred
criteria, a stool toxin test should be employed as part of a
over alcohol-based products given superior spore removal
multistep algorithm rather than NAAT alone.1
with soap and water (C. difficile spores are highly resistant
5. Explanation: The correct answer is A. Current guide- to killing with alcohol), especially during outbreaks or if
lines recommend oral vancomycin or fidaxomicin as the there is direct contact with feces or an area where fecal
mainstay of therapy for CDI. Oral administration is pre- contamination may have occurred (rules out option C).
ferred due to negligible absorption and the potential for Environmental contamination is also thought to contribute
few systemic adverse effects. Metronidazole, which still to the spread of CDI, and precautions should be taken to
94 INFECTIOUS DISEASES: A CASE STUDY APPROACH

safeguard against environmental spread of disease, includ- at the patient and ward-level is critical to helping control
ing: isolation, dedicated/disposable patient care equipment, outbreaks ofCDI.1
and appropriate environmental cleaning. Isolation of CDI
patients is recommended in patients with suspected or
confirmed CDI, and for 48 hours following resolution REFERENCE
of diarrhea. At this time, there is no evidence to sup- 1. McDonald LC, Gerding DN, Johnson S, et al. Clinical
port contact precautions in asymptomatic carriers (rules practice guidelines for Clostridium difficile infection in
out option D). Finally, antibiotic stewardship has good adults and children: 2017 update by the Infectious Dis-
evidence and may be one of the most helpful methods eases Society of America (IDSA) and Society for Health-
for controlling rates of CDI. Limiting the frequency and care Epidemiology of America (SHEA). Clin Infect Dis.
duration of antibiotics (especially high-risk antibiotics) 2018;66:e 1-e48.
23 Traveler's Diarrhea
Amber B. Giles

PATIENT PRESENTATION _. General


Male with dizziness and in mild distress
Chief Complaint
"'I can't stop going to the bathroom, and rm starting to get _. HEENT
dehydrated."
Normocephalic, PERR.LA. EOMI, dry mucous membranes
History of Present Illness and conjunctiva, fair dentition
JB is a 32-year-old Caucasian male who presents to his primary
care physician with complaints of bloody diarrhea approxi- _. Pulmonary
mately 5 times per day, abdominal pain, and nausea for the past Normal breath sounds
4 days. He also complains ofintennittent fevers and dry mouth.
He states that he recently traveled to India on a medical mis- _. Cardiovascular
sion trip with other students in his medical school program. Of NSR, no m.Jr/g
note, JB did not receive any vaccinations or medications prior
to travel and has not received antibiotics in the past 5 years. He _. Abdomen
is up-to-date on all routine childhood vaccines.
Slightly distended, positive for abdominal pain. bowel sowids
hyperactive
Past Medical History
Seasonal allergies, depression, anxiety _. Genitourinary
Normal male genitalia, no complaints of dysuria or hematuria
Surgical History
Tonsillectomy and adenoidectomy in primary school _. Neurology
Oriented to place and person
Family History
Father has hyperlipidemia and type 2 diabetes; mother has no _. Extremitie5
significant medical history
Negative for pain or rash
Social History _. Bade
Student in his third year of medical school. married, lives with Negative for back pain
wife, and drinks alcohol occasionally. .Reports no illicit drug
or tobacco use Laboratory Findings
Allergies Na =148 mEq/L K=3.0mEq/L d= 107mEq/L
Ciprofloxacin (hives/shortness of breath) co 2
= 24 mEq/L BUN = 32 mg/dL SCr = 1.5 mgldL
Glu = 90 mgldL Ca = 8.4 mg/dL Mg = 1.5 mgldL
Home Medications Phos = 4.8 mgldL Hgb = 15.0 gldL Hct= 5096
Cetirizine 10 mg PO daily Pit = 200 x 10'/mm. WBC = 15 x
3
AST= 18IU/L
Sertraline 100 mg PO daily lO'mm'
ALT= 20IU/L T Bill = 1.4 mg/dL Alk. Phos =
Physical Examination 62IU/L
_. VdalSigns
_. Stool Cultures
Temp 102.3°F, P 89, RR 24 breaths per minute, BP 110/69 mm
Hg, p02 94%, Ht 6'2", Wt 89 kg Pending

95
96 INFECTIOUS DISEASES: A CASE STUDY APPROACH

QUESTIONS 7. Which of the following is true in regard to prophylaxis


against typhoid fever?
1. Which of the following organisms are most likely to cause
A. Typhim Vi is a live attenuated oral vaccine that should
bloody diarrhea and fever in an international traveler such
be given prior to travel in areas of moderate to high risk
as JB?
for exposure
A. Norovirus
B. Vivotif is an inactivated oral vaccine that should be
B. Clostridioides difficile
given prior to travel in areas of moderate to high risk
C. Salmonella enterica
for exposure
D. All of the above
C. Either vaccine will be >90% effective in preventing
2. What is the most appropriate management of JB's moder- typhoid fever
ate dehydration? D. Food and hand-washing precautions are recommended
A. Oral intake of fluids such as Gatorade or apple juice in addition to either of the available vaccines
B. Oral rehydration solution (ORS)
C. Intravenous normal saline
D. No intervention is needed at this point ANSWERS
3. Which empiric antibiotic regimen would you recommend 1. .Explanation: The correct answer is C. Norovirus typically
for JB? presents as a watery diarrhea. Additionally, C. difficile
A. Azithromycin 1,000 mg PO daily for one dose does not usually present as a bloody diarrhea, and JB has
B. Ciprofloxacin 500 mg PO daily for one dose not been exposed to antimicrobials in the recent past.
C. Amoxicillin-clavulanate 875-125 mg PO q12h for Common causes of bloody diarrhea and fever in imrnu-
one day nocornpetent patients who have recently travelled include
D. Empiric antibiotic therapy is not appropriate in this Salmonella enterica, Shigella, and Campylobacter spp. 1
patient 2. Explanation: The correct answer is B. Current guidelines
4. JB is concerned about his wife being exposed to his and the CDC recommend ORS as first-line management
infection. He asks if she needs to be prescribed an antibi- of mild to moderate dehydration caused by diarrhea.1.i If
otic as well for empiric treatment. He states that she cur- JB is unable to tolerate oral intake, ORS may be adminis-
rently has no signs of infection and has no known drug tered via nasogastric routes as well. 1 Beverages with high
allergies. What is the most appropriate response to this sugar content, such as apple juice, may actually cause
inquiry? osmotic diarrhea if consumed in large quantities. Intrave-
A. Recommend an additional dose of azithromycin nous solutions such as normal saline and lactated ringer's
1,000 mg for JB's wife should be reserved for severe cases of dehydration,
B. Recommend ciprofloxacin 500 mg PO once altered mental status, and/or shock or in cases where ORS
C. Recommend appropriate infection prevention mea- has failed. Once rehydration has been achieved, ongoing
sures, such as regular hand hygiene losses should be replaced with ORS until the diarrhea has
D. Recommend both B & C resolved.1

5. Based on JB's presentation, what should be recommended 3. .Explanation: The correct answer is A. Current guidelines
in regard to adjunctive antidiarrheal medications? and the CDC recommend empiric antibiotic therapy in an
A. Initiate loperarnide 4 mg PO once, followed by 2 mg irnmunocompetent adult with bloody diarrhea plus a tem-
PO after each subsequent stool perature > 101.3°F and/or signs of sepsis who have trav-
B. Initiate diphenoxylate-atropine 5 mg-0.05 mg 4 elled internationally recently. 1.i JB's recorded temperature
times daily is 102.3°F and recently travelled to India. Azithromycin or
C. Initiate bismuth subsalicylate 525 mg PO 4 times daily fluoroquinolone antibiotics are recommended empirically
D. Antidiarrheal medications are inappropriate in this in cases of travelers' diarrhea;' however, JB is allergic to
patient ciprofloxacin (hives and shortness of breath). Additionally,
azithromycin is the preferred treatment for inflammatory
6. Which of the following preventative measures should be
diarrhea due to increasing ciprotloxacin resistance. 2 Taking
taken to prevent traveler's diarrhea in a patient traveling to into account all of this information, azithromycin would be
a resource-limited country? the most appropriate agent for this patient.
A. Avoid eating raw or undercooked foods, including sea-
food and meats 4. .Explanation: The correct answer is C. Current guidelines
B. Avoid drinking tap water (including ice) state that contacts of patients with infectious diarrhea
C. Wash hands frequently using soap and water or hand should not be prescribed empiric antibiotics if contacts are
sanitizer, if needed currently asymptomatic1; therefore, answers A and B are
D. All of the above incorrect. Appropriate infection control measures should
CHAPTER 23 I TRAVELER'S DIARRHEA 97

be followed, such as contact precautions and appropriate patients traveling to moderate- to high-risk international
hand hygiene. To prevent transmission of the infectious destinations.3 Typhim Vi is an inactivated intramuscular
agent, infected persons should refrain from preparing food injection that should be given once at least 2 weeks prior to
or working in childcare settings during periods of diarrheal travel, and a booster should be administered every 2 years
illness. 1 in patients at continued risk. Vivotif is a live attenuated oral
vaccine that should be given every other day for 4 doses
5. Explanation: The correct answer is D. Adults with bloody
and completed at least 1 week prior to travel. This vaccine
diarrhea and fever are not candidates for antimotility
should be boosted every 5 years in those at continued risk.
therapy.1 Slowing of fecal transit time may worsen symp-
Each of the vaccines are only 50-80% effective in prevent-
toms or lead to gastrointestinal damage and complications
ing typhoid fever, so food and hand-washing precautions
in patients with bloody, inflammatory diarrhea. In cases of
are still of utmost importance when traveling.3
acute watery diarrhea in immunocompetent adult patients,
antimotility agents may be considered to reduce the num-
ber of stools; however, it is important to note that these REFERENCES
medications should never be used as a substitute for fluid 1. Shane AL, Mody RK, Crump JA, et al. 2017 Infectious
and electrolyte therapies. 1
Diseases Society of America clinical practice guidelines
6. Explanation: The correct answer is D. "Boil it, cook it, peel for the diagnosis and management of infectious diarrhea.
it, or forget it" is a common saying in regard to preparing Clin Infect Dis. 2017;65(12):1963-1973.
foods in resource-limited countries. 2 All meat should be 2. Connor BA. Travelers' diarrhea: CDC health informa-
cooked well, and fruits and vegetables should be peeled or tion for international travel. Centers for Disease Control
cooked by the traveler. Any raw fruits or vegetables that and Prevention; 2017. Available at https://wwwnc.cdc.
cannot be peeled should not be consumed. Additionally, it gov/travel/yellowbook/2018/the-pre-travel-consultation/
is important for travelers to avoid drinking tap water, ice, travelers-diarrhea. New York, NY, 2018.
beverages prepared with tap water such as tea or reconsti- 3. Judd MC, Mintz ED. Typhoid & paratyphoid fever: CDC
tuted fruit juice, or unpasteurized milk and dairy products. Health Information for International Travel. Centers for
Good personal hygiene should be practiced in order to Disease Control and Prevention; 2017. Available at https://
prevent travel-related illnesses as well. Hand washing with wwwnc .cdc .govI travel/yellow b o ok/20 18/infe ctious-
soap and water or alcohol-based sanitizers should be used disease s-related-to-travel/typhoid-paratyphoid-fever.
regularly, which is extremely important before handling or New York, NY, 2018.
preparing foods.2
7. Explanation: The correct answer is D. Two vaccines are
indeed available for protection against typhoid fever in
This page intentionally left blank
24 Hepatitis C
Lindsey Childs-Kean

PATIENT PRESENTATION Physical Examination


Chief Complaint ~ Yitai Signs
"I want to get this Hepatitis C cured~ Temp 98.8°F, P 82 bpm. RR 18 breaths per minute, BP
127/78 mm Hg. p02 99%, Ht 6', Wt 101 kg
History of Present Illness
~ Genel'OI
PD is a 45-year-old Caucasian male who presents to the
infectious diseases clinic seeking treatment for his chronic Normal appearing male in no apparent distress, well groomed
hepatitis C infection. He states that he was diagnosed with
~ HEENT
the disease a few years ago. but he's not sure when he con-
tracted it. He doesn't remember having any symptoms Nomocephalic. atraumatic, PERRI.A, EOMI. b/l sclera anic-
related to the infection. He has never received any treat- teric, moist mucous membranes. poor dentition
ment for his hepatitis C.
~ Pulmonary
Past Medical History Cl.ear to auscultation, no use of accessory muscles. no cracks
HTN, anxiety, dyslipidemia, GERO orwhea:es

~ Cardiovascular
Surgical History
NSR, no m.Jr/g
None
~ Abdomen
Family History Soft. non-distended, non-tender, normal bowel sounds
Father and mother both alive with HTN and dyslipidemia;
sister is alive with no known medical issues. ~ Genitourinary
Normal male genitalia. no complaints of dysuria or hematuria
Social History
Former IV drug user and alcohol drinker, but clean and sober ~ Neurology
for 7+ years. No tobacco. Works repairing wheelchairs and Alert and oriented x 3, CN 2-12 grossly intact
scooters. Married with 2 children (12 and 4 years). Enjoys
playing sports and doing outdoor activities with wife and ~ Extremities
children. No edema, cyanosis, or clubbing

Allergies ~ Skin
NKDA Intact, w/o rashes or lesions, or erythe.ma

Home Medications Laboratory Findings


A.mlodipine 10 mg PO daily WBC:6 x 109 Glucose: Albumin: 3.8 mg/di.
Alpraz.olam. 1 mg PO TID PRN for anxiety cells/L 80mg/dl.
Si.mvastatin 20 mg PO daily
Omepraz.ole 40 mg PO daily Hgb: 15 gldL BUN: 15 mgldL T bill: 0.8 mgldl.
Multivitamin PO daily Hct:45% SCr: 0.9 mg/di. AST/ALT: 50 IU/L/
Acetaminophen 325 mg PO PRN for headache 50IU/L
Pits: 102,000/mmol K: 4 mEq/L Alk Phos: 100 IU/L

99
100 INFECTIOUS DISEASES: A CASE STUDY APPROACH

Hgb AlC: 5% Na: 140 mEq/L Total Chol: 180 mg/dL 6. What is the recommended duration of treatment if PD is to
be treated with elbasvir/grazoprevir?
TSH: 2mIU/L C02: 22 mEq/L HDL: 65 mg/dL
A. 12weeks
INR: 1.3 Ca: 8.5 mgldL LDL: 90 mgldL B. 16 weeks
Cl: 100 mEq/L Triglycerides: C. 24weeks
150 mg/dL D. Not enough information to answer
Hepatitis C Genotype= la
7. In addition to the hepatitis C viral load, what laboratory
Hepatitis C Viral Load= 8,237,625 IU/mL
findings should be monitored as PD undergoes treatment
Hepatitis A Antibody Total = Negative
for his hepatitis C? (limit to laboratory findings relevant to
Hepatitis B sAg Screen = Negative
hepatitis C disease and antiviral treatment)
Hepatitis B cAb Total = Negative A. Serum creatinine, albumin, total bilirubin, ASTI ALT,
Hepatitis B sAb =Negative
alkaline phosphatase
Liver biopsy (2016) = Cirrhosis, inflammation grade 3 out of 4
B. CBC, serum creatinine, AST/ALT
Resistance-Associated Substitutions= None detected C. CBC, serum creatinine, albumin, total bilirubin, ASTI
ALT, alkaline phosphatase
QUESTIONS D. CBC, TSH, albumin, total bilirubin
1. What is the most likely route from which PD contracted 8. Based on PD's hepatitis A and B serologies, what, if any;
hepatitis C? hepatitis vaccinations does he need to receive?
A. Alcohol use A. No vaccinations needed
B. Sexual activity B. Hepatitis A only
C. Ndruguse C. Hepatitis B only
D. Eating contaminated food D. Hepatitis A and B
2. What is this patient's Child-Pugh score? Correspondingly,
how would PD's degree ofliver fibrosis/cirrhosis be staged?
ANSWERS
A. Score: 0, No fibrosis
B. Score: 5, Bridging fibrosis 1. Explanation: The correct answer is C. Hepatitis C is most
C. Score: 5, Compensated cirrhosis commonly transmitted by blood to blood contact. Since
D. Score: 8, Decompensated cirrhosis the blood supply has been able to be screened (mid-1990s),
the most common route of transmission is IV drug use.
3. At what time point does the viral load need to be undetect- Since the patient has a history of IV drug use, this is likely
able in order for the patient to achieve a sustained virologi- the way he was infected with hepatitis C. Hepatitis C can
cal response, the goal of treatment, which is regarded as a be transmitted sexually, but it is very rare in heterosexual
"virological cure"? monogamous couples. Alcohol use and eating contami-
A. At the end of treatment nated food are not routes of transmission of hepatitis C.
B. 4 weeks after treatment ends
C. 8 weeks after treatment ends 2. Explanation: The correct answer is C. The patient's liver
D. 12 weeks after treatment ends biopsy shows cirrhosis, which is stage 4 of 4 fibrosis (Stage
1 is no fibrosis and Stage 3 is bridging fibrosis). Accord-
4. Which of the patient's current medications does NOT cause ing to Child-Pugh, which looks at INR, total bilirubin,
any drug-drug interactions with the four recommended albumin, presence or absence of ascites, and presence or
hepatitis C regimens for treatment na'ive genotype la absence of hepatic encephalopathy, the patient has com-
patients• (sofosbuvir/ledipasvir, sofosbuvir/velpatasvir, gle- pensated cirrhosis with a Child-Pugh score of 5.
caprevir/pibrentasvir, and elbasvir/grazoprevir)?
A. Amlodipine 3. .Explanation: The correct answer is D. Sustained virologi-
B. Alprazolam cal response is defined as an undetectable viral load at least
C. Simvastatin 12 weeks following the end oftreatment for chronic hepatitis C.
D. Omeprazole
4. .Explanation: The correct answer is B. Simvastatin is a
5. Which regimen would you recommend for the treatment substrate of the OATP lBl/3, BCRP, and P-gp transporters,
of PD's chronic hepatitis C without having to change any of so the plasma levels of simvastatin are going to increase
his other medications? significantly with an inhibitor of those transporters, like
A. Elbasvir/grazoprevir glecaprevir/pibrentasvir, sofosbuvir/ledipasvir, and sofos-
B. Glecaprevir/pibrentasvir buvir/velpatasvir. Amlodipine is a substrate of the P-gp
C. Sofosbuvir/ledipasvir transporter, and ledipasvir is an inhibitor of that trans-
D. Sofosbuvir/velpatasvir porter. Omeprazole reduces the acidity of the stomach,
CHAPTER 24 I HEPATITIS C 101

and absorption of ledipasvir and velpatasvir is significantly 7. Explanation: The correct answer is A. A complete blood
reduced with less acidity in the stomach. Alprazolam is the count (CBC) is only recommended if a patient is receiving
only drug listed free of drug-drug interactions with the ribavirin as a part of treatment due to its risk of anemia.
common hepatitis C regimens. PD is not receiving ribavirin, so a CBC does not need to be
monitored. Serum creatinine and a hepatic function panel
5. Explanation: The correct answer is A. Glecaprevir/pibren-
(including albumin, total bilirubin, AST/ALT, alkaline
tasvir is contraindicated with simvastatin use. PD's current
phosphatase) are recommended at 4 weeks into treatment
dose of omeprazole is above the recommended max dose
and then as indicated.
(20 mg) that is to be given concurrently with ledipasvir or
velpatasvir. Elbasvir/grazoprevir does potentially interact 8. Explanation: The correct answer is D. PD has a negative
with simvastatin, but only monitoring of side effects is hepatitis A antibody, so he needs to receive the hepatitis A
recommended. vaccination series. He is also negative for both the hepatitis
B core and surface antibodies, so he needs the hepatitis B
6. Explanation: The correct answer is A. Patients who have
vaccination series as well.
genotype la (as PD does) and are going to receive elbasvir/
grazoprevir need to have baseline resistance testing done,
looking for resistance-associated substitutions (RASs) to REFERENCE
the NSSA inhibitor (elbasvir in this case). Since PD has
1. AASLD-IDSA. Recommendations for testing, manag-
no RASs, then the patient should receive 12 weeks of
ing, and treating hepatitis C. Available at http://www.
treatment. If he had had at least one RAS, then the patient hcvguidelines.org. 2019.
should have received 16 weeks of treatment.
This page intentionally left blank
25 Syphilis
Trent G. Towne

PATIENT PRESENTATION Home Medications


Chief Complaint Ibuprofen 200 mg PO PRN pain (has taken 4 doses in the last day)
"I think I am allergic to something~
Physical Examination
History of Present Illness .,. Vital Signs
JS is a 27-year-old man who presents to a free health clinic Temp 101°F, P 72,RR 16breathsperminute, BP 141/85 mm Hg,
at the county hospital. He states he was in his usual state of p02 94%, Ht 5'7", Wt 60 kg
health until about 3 days ago when he began developing a
rash on his stomach that is now on the palms of his hands .,. General
and soles of his feet. The rash is not painful or itchy. He NAD, awake, alert. slightly underweight man
states that he had this strange little "ulcer-like"' thing on
his penis a couple weeks ago but it went away and never .,. Skin
really hurt. He is single and sexually active with two to Diffuse mucocutaneous rash noted on abdomen, bade, upper
three concurrent male partners. He has had unprotected extremities (including palms of hands) and soles of feet; mac-
sex with "at least one of his partners• in the past couple ules are easy to blanch and are not associated with any area of
of weeks. He doesn't know the sexual histories of his cur- fluctuance
rent or past sexual partners, and admits to over 15 lifetime
partners. He endorses rectal and oral sex. He doesn't ever .,. HEENT
recall being tested for HIV, and knows he received all his
childhood vaccines, "cause my mom told me."' He has never PERRLA; EOMI; mucous membranes are moist and neck is
been vaccinated against HPV stating, "that's a woman's dis- supple without any evidence oflymphadenopathy
ease:' and is unsure if he ever has received a hepatitis A
vaccine.
.,. Pulmonary
Clear auscultation with no wheezing or rhonci
Past Medical History
.,. Cardiovascular
None
NSR; no m/r/g

Surgical History .,. Abdomen


None NTND with no rebounding or guarding; ( +) BS; noted rash
as described above
Famlly History
Father had HTN and passed away from a stroke 4 years ago; .,. Genitourinary
mother is still living and has type 2 DM Abdominal rash extends to genital region and base of penis;
noted healing wound/ulcer on the dorsal aspect of the penis
Social History .,. Neurology
MSM with multiple sexual partners; ( +) EtOH, (-) Tobacco.
CN Il-XII intact
( +) Marijuana, (-) Illicit drugs
.,. Extremities
Allergies Well-perfused and warm to the touch; noted rash as described
NKDA above

103
104 INFECTIOUS DISEASES: A CASE STUDY APPROACH

Laboratory Findings 6. After completing the correct treatment for his syphilis,
what is the most appropriate recommendation for follow-
Na =136 mEq/L Hgb = 11.4 gldL
up of his infection?
K = 4.1 mEq/L Hct = 35% A. No further follow-up is required
Cl = 98 mEq/L Pit = 141 x 103/mm3 B. He should have a follow-up RPR test at 6 months
C. He should have a follow-up FTA-ABS test at 12 months
C02 = 26 mEq/L WBC = 10.2 x 103 mm3 D. He should return to a primary care provider for evalu-
BUN= 14 mg/dL AST = 54 IU/L ation of his clinical response in 1 week
SCr = 0.96 mg/dL ALT= 85 IU/L
7. What additional screening should be performed for JS
Glu = 101 mg/dL T. Bili = 4.9 mg/dL because of his diagnosis of syphilis?
A. HIV
~ Additional Testing B. Urine drug screen
C. HPV
RPR: Positive with a tiler of 1:128
D. Tuberculosis
TP-EIA: Positive
CSF-VDRL: Non-reactive 8. If JS had presented with a type-I allergy to penicillin, what
Hepatitis A: HAV IgM Ab: Negative would be the most appropriate antibiotic regimen to consider
Hepatitis B: HBsAb: Positive; HBsAg: Negative for his therapy
Hepatitis C: RNA negative A. Doxycycline 100 mg PO BID x 10 days
B. Ceftriaxone lg IV daily x 14 days
C. Levofloxacin 750 mg PO daily x 10 days
QUESTIONS D. Rifampin 300 mg PO TID x 14 days
1. Which pathogen is most likely causing this patient's current
symptoms?
A. Mycoplasma genitalium ANSWERS
B. Chlamydia trachomatis 1. .Explanation: The correct answer is C. Treponema pallidum
C. Treponema pallidum is the causative pathogen for the sexually transmitted infec-
D. Staphylococcus aureus tion, syphilis. In this case, JS has several symptoms that
2. Which additional testing could be done to confirm JS's diag- point toward a diagnosis of syphilis including the muco-
nosis of syphilis? cutaneous rash on the abdomen, fever, and prior history
A. Darkfield microscopy of the ulcer on his penis. Additionally, he has one of the
B. Acid-fast bacilli staining most common risk factors for syphilis, being a man who
C. KOH testing has sex with men (MSM). This correlates with the positive
D. Gram staining RPR and TP-EIA results, two tests used in the diagnosis
of syphilis. Serologic testing is the mainstay of diagnostic
3. What is the clinical stage of JS's syphilis? testing for syphilis and comes in two different antibody
A. Primary tests, nontreponemal and treponemal tests. Rapid plasma
B. Secondary reagin test (RPR) is the primary nontreponemal test that
C. Tertiary can be used in screening for syphilis and can serve as a
D. Neurosyphilis marker to gauge response to treatment. Another non-
treponemal test that is utilized in practice is the Venereal
4. What would be the most appropriate therapeutic choice for
Disease Research Laboratory (VDRL) test. While still a
JS's syphilis?
A. Benzathine Penicillin G 2.4 million units IM X 1 dose quantitative assay, VDRL results cannot be compared to
the RPR, as RPR titers are usually higher than VDRL.
B. Benzathine Penicillin G 2.4 million units IM x 3 doses
VDRL testing has been largely replaced by RPR for general
separated by one week
C. Aqueous Penicillin G 20 million units IV continuous screening of patients with suspected syphilis, but it still is
widely used to evaluate CNS disease. Several treponernal
infusion daily for 10 days
D. Aqueous Penicillin G 24 million units IM daily x 3 doses tests are also available for making the diagnosis of syphilis.
The role of treponemal testing as either a screening tool
5. One day after starting therapy for syphilis, JS begins to or conformation test is still evolving in modern practice.
experience fever, muscle pain, and headache. What is the Nontreponemal tests include fluorescent treponemal anti-
most likely explanation for these findings? body absorbed test (FTA-ABS), microhemagglutination
A. Allergic reaction test for antibodies to T. pallidum (MHA-TP), T. pallidum
B. Stevens-Johnson syndrome particle agglutination assay (TP-PA), T. pallidum enzyme
C. Jarisch-Herxheimer reaction assay (TP-EIA), and chemiluminescence immunoassay
D. Treatment failure (CIA). The conventional treponernal tests (FTA-ABS and
CHAPTER 25 I SYPHILIS 105

MHA-TP) have been largely replaced by TP-PA, EIA, and also can cause genital ulceration. Left untreated, primary
CIA testing due to ease of use and automated technologies. syphilis would progress to the symptoms JS is currently
Regardless, the results for treponemal testing will remain observing as secondary syphilis. At either the primary or
positive for life, even with effective treatment.1.z Answer secondary state of syphilis a patient may develop neuro-
choices A and B both represent pathogens capable of caus- syphilis. While many patients will remain asymptomatic
ing STis; however, JS's symptoms do not correlate with and even resolve their neurosyphilis, those who do develop
the presentation of those organisms. Additionally; answer disease may exhibit a wide array of syndromes including
choice D would be incorrect because S. aureus is usually meningitis, meningovascular disease, cranial neuritis, and
associated with fluctuant skin and soft tissue infections occular disease that may overlap. JW's treponemal serum
rather than generalized cellulitis. titers are positive, but his CSF-VDRL is negative and he is
exhibiting no signs or symptoms of meningitis (eliminat-
2. Explanation: The correct answer is A. Direct detection
ing answer choice D).Patients who are not treated, or who
of T. pallidum is another methodology that can be used
are undertreated, in the secondary phase of syphilis may
in the diagnosis of syphilis, however for many reasons
also simply resolve their signs and symptoms of infection
serologic testing is preferred. Examples of direct meth-
progress to latent syphilis. Latent syphilis can be further
ods of detection of syphilis include darkfield microscopy,
subdivided into early latent ( < 1 year from infection)
direct fluorescent antibody test for T. pallidum (DFA-TP),
and late latent (> 1 year from infection) types. During
silver staining, and PCR testing. Darkfield microscopy
early latent syphilis, relapses of infection with or without
is a rapid, direct method of confirming the presence of
spirochetemia are common; however, in late latent syphilis
T. pallidum. Its use is limited by the need to investigate
these relapses and bloodstream infections are uncommon.
only motile treponomes, which requires special equip-
Tertiary syphilis usually produces illness > 5 years after the
ment and highly trained personnel to utilize. Much like
initial infection (eliminating answer choice C). Much like
darkfield microscopy, direct fluorescent antibody test for
the other stages of syphilis, tertiary syphilis can be subdi-
T. pallidum (DFA-TP) also requires highly specialized
vided into cardiovascular, gummatous, and some forms of
equipment and is technically complex to conduct. Silver
neurosyphilis.1.z
staining of T. pallidum in tissue fixed with formalin also
represents a traditional method for direct detection; how- 4. Explanation: The correct answer is A. Penicillin G is the
ever, it lacks specificity for only T. pallidum and has limited correct antibiotic choice for this patient; however, route
sensitivity. PCR testing represents the final example of of administration, dose, and duration are the defining
direct detection of T. pallidum, and while PCR has been features for this answer as syphilis treatment is based on
successfully employed in a number of infectious disease stage of disease. For patients with primary, secondary, or
diagnostic tools, in the setting of syphilis there is no com- early latent syphilis, a single dose of benzathine penicillin
mercially available testing. In addition, this method will G 2.4 million units, given as a one-time dose intramuscu-
amplify the DNA ofliving and dead bacteria, leading to the larly, is the most appropriate therapy. It should be noted
potential for false-positive results. Answer choice B would that Bicillin L-A (standard benzathine penicillin G) and
be correct only for mycobacterial species (T. pallidum Bicillin C-R (benzathine-procaine penicillin G) are not
is a spirochete). Answer choice C would be correct for a interchangeable. Only Bicillin L-A should be utilized in
number of STI-associated vaginal discharge, but since JS is the management of primary, secondary, or early latent
a man this would not be an incorrect answer. Answer D is syphilis as using Bicillin C-R may result in not fully treat-
not correct because although a bacterium T. pallidum does ing the patient Answer choice B, while correct in dose and
not stain by traditional bacterial Gram-staining methods. •.z route, is incorrect with regard to the duration of therapy
of 3 weeks. Titis duration would be more consistent with
3. Explanation: The correct answer is B. This patient has
the diagnosis of tertiary or late, latent syphilis. Aqueous
secondary syphilis as is evidenced by the systemic nature
penicillin G given as IM or IV is recommended only for
of his symptoms including the mucocutaneous rash on
the treatment of neurosyphilis (eliminating answer choices
his abdomen, palms of his hands and soles of his feet, and
C andD).1
his fever. While secondary syphilis can impact all organ
systems, over 95% of patients will experience lesions of 5. Explanation: The correct answer is C. This patient is
the skin and mucous membranes. Patients with primary experiencing a Jarisch-Herxheimer reaction which can
syphilis will present with an ulcer, or chancre, that is usu- occur with the initiation of treatment for syphilis at any
ally painless, has a raised border, and is without exudate. stage of the disease, but most commonly in the primary
JW did identify a healed, non-painful ulcer on his penis and secondary phases. It most commonly occurs within
that resolved 2 weeks ago, making it likely that was the 6 to 8 hours of the initiation of treatment and lasts for 12 to
primary stage of syphilis {eliminating answer choice A). 24 hours. The reaction is thought to be due to a cytokine
The absence of pain from the chancre is also helpful in the storm in response to the lysing and death of the trepono-
differentiation of primary syphilis from other STis such nemes. While it can vary in severity, symptom onset is
as Haemophilus ducreyi and herpes simplex virus which usually abrupt and can include fever, chills, headache,
106 INFECTIOUS DISEASES: A CASE STUDY APPROACH

myalgias, mild hypotension, vasodilation with flushing, answer choice B). HPV infection is a widely dissemi-
hyperventilation,andtachycardia.Broadlyspeaking,symp- nated STI; however, there are no recommendations for
toms can be managed by anti-inflammatory agents and/or routine screening of this infection in men, regardless
antipyretics. Though commonly confused with an allergic of diagnosis of another sexually transmitted infection
reaction (especially in patients with secondary syphilis (eliminates answer choice C).Tuberculosis, while a com-
who have a rash), it is not an allergic reaction (eliminating mon co-infection in patients with HIV, is not directly
answer choice A). Stevens-Johnson syndrome can occur associated with syphilis; therefore, the screening is not
as a result of drug therapy; however, this does not usually necessary. If the patient is tested for HIV and found to be
occur until 1 to 3 weeks after the initiation of therapy, and positive, then tuberculosis screening would be warranted
is usually associated with skin blisters that easily slough off (eliminates answer choice D).1
(eliminating answer choice B). Since the patient has only
8. Explanation: The correct answer is A. In patients with a
been on therapy for one day, failure of therapy cannot be
type 1 penicillin allergy, all beta-lactam antibiotics should
assessed at this point (eliminating answer choice D). 1.z
be avoided (eliminating answer choice B).This makes the
6. Explanation: The correct answer is B. The patient should use of a tetracycline antibiotic the preferred agent. While
have serology, with non-treponemal testing (eg, RPR), tetracycline would be an okay choice for this patient, using
completed at 6 and 12 months. Treponemal testing would doxycycline for 10 days minimizes the frequency of dosing
not provide any additional information as the patient will to promote compliance (ie, BID vs QID). Neither answer
likely have a positive result for the rest of his life and treat- C nor D is routinely used in the management of sexually
ment will not result in any decline in the positivity of this transmitted infections, including syphilis. 1
result (eliminating answer choice C). Evaluation of the
clinical response is not necessary until the 6-month mark
unless the patient has not observed a significant resolution REFERENCES
in the signs and symptoms of his infection (eliminating
1. Workowski KA, Bolan GA. Sexually transmitted diseases
answer choices A and D). 1
treatment guidelines 2015. MMWR Recomm Rep. 2015;
7. Explanation: The correct answer is A. Patients who 64(No. RR-3):1-140.
are diagnosed with any sexually transmitted infection 2. Radolf JD, Tramont EC, Salazar JC. Syphilis (Treponema
should be screened for HIV. While he does have a his- pallidum) in principles and practice of infectious disease. 8th
tory of drug use (marijuana), simply being diagnosed ed. In: Bennett JE, Dolin R, Blaser MJ, eds. Philadelphia, PA.
with syphilis does not merit screening for this (eliminates Elsevier Saunders; 2015:2684-2709.
26 Herpes
Elias B. Chahine

PATIENT PRESENTATION Physical Examination


Chief Complaint ~ Yitai Signs
"I have painful sores in my genital area and I have been feel- Temp 100.04°F; BP 118/74 mm Hg; HR 80 beats/minute; RR
ing achy:' 18 breaths/minute; Ht 5'6n; Wt 52.3 kg

~ General
History of Present Illness
Well nourished, mild acute distress
JJ is a 20-year-old Caucasian female who presents to the com-
munity health center for evaluation of genital lesions that have ~ HEENT
been present for two days. The lesions are painful. vesicular, and
Normocephalic, atraumatic, PERRLA, EOMI, moist mucus
accompanied by .inguinal lymphadenopathy. nhas never expe-
membranes, good dentition
rienced these symptoms before. Upon further questioning. she
stated that she experienced pain. tingling, itching, and burning ~ Cardiovascular
sensation before the appearance ofthe lesions. She admits to hav-
ing multiple male sex partners within the past six months while NSR, no mlr/g
she was studying abroad and stated that her last sexual encounter ~ Abdomen
was a week.ago. She and her partners occasionally but not always
use barrier methods during sexual intercourse. JJ is also com- Soft. non-distended, non-tender, normal bowel sounds
plaining of nasal congestion and sneezing.
~ Genitourinary
Past Medical History Small painful unruptured vesicular lesions in the labia majora,
redness and inflammation in the surrounding area, tender
Vulvovaginal candidiuis, allergic rhinitis local inguinal lymphadenopathy
Surgical History ~ Neurology
Appendectomy at age 18 Alert and oriented X3

Family History ~ Extremities


Father has HTN, mother has T2DM, sister has PCOS No edema, no petechiae

Social History Laboratory Findings


IJves at home with her parents; enjoys swimming and run- Na =138 mEq/L Hgb =15 g/dL Ca= 8.2 mg/dL
ning; drinks alcohol occasionally; denies tobacco and illicit K =4 mEq/L Hct =4596 Mg= 2.3 mg/dL
drug use
Cl= 100 mEq/L Pit= 250 x Phos = 4.6 mg/dL
103/mm3
Allergies
Penicillin (hives)
col= 28 mEq/L WBC = 8.2 x AST=24ID/L
10'/mm'
BUN= 20 mgldL ALT= 22ID/L
Home Medications
SCr = 0.9 mg/dl T bili = 1.2 mgldL
Cetirizine 10 mg PO daily
Ethinyl estradiol 0.02 mg and drospirenone 3 mg. one tablet Glu = 99 mgldL A1k phos = 78 IU/L
PO daily
Fluticasone furoate (27.5 mcg/spray), one spray in each nostril ~ Other Labs
once daily HSV PCR vaginal swab: (+)for HSV-2; HIV antibodies non-
Multivitamin, one tablet PO daily reactive; hCG test negative
107
108 INFECTIOUS DISEASES: A CASE STUDY APPROACH

QUESTIONS about what others will think if she has a breakout of cold
sores during graduation and wants to know if there is any
1. Which of the following is a risk factor for acquiring genital
medication that she can obtain from the pharmacy without
herpes infection?
a prescription from the doctor's office. What is the best
A. Drinking alcohol
recommendation for JJ at this time?
B. Having multiple sex partners
A. Initiate acyclovir 5% cream and apply 5 times daily for
C. Swimming in a community pool
4days
D. Taking birth control pills
B. Initiate acyclovir 5% ointment and apply 6 times daily
2. What signs and symptoms does JJ have that are consistent for 7 days
with a genital herpes infection? C. Initiate docosanol 10% cream at first sign of cold sore
A. Fever and nasal congestion and apply 5 times daily until healed
B. Fever and painful vesicular genital lesions D. Initiate penciclovir 1% cream at first sign of cold sore
C. Nasal congestion and painful vesicular genital lesions and apply every 2 hours while awake for 4 days
D. Nasal congestion and sneezing
3. What is the best treatment regimen for JJ's initial episode
of genital herpes? ANSWERS
A. Acyclovir 5 mg/kg IV q8h x 7 days 1. .Explanation: The correct answer is B. Genital herpes is
B. Acyclovir 400 mg PO TID x 7 days usually acquired through sexual contact with an indi-
C. Famciclovir 125 mg PO BID x 5 days vidual infected with HSV-2. Having multiple sex partners
D. Valacyclovir 1 g PO daily x 5 days increases the risk for acquiring sexually transmitted dis-
4. Three months later, JJ calls the community health center eases including genital herpes. Drinking alcohol, swim-
complaining of painful lesions in her genital area that ming in the community pool, and taking birth control pills
look and feel the same as her previous episode from her do not increase the risk of acquiring genital herpes in the
initial visit. However, there are fewer lesions and they are absence of sexual intercourse. 1
less painful. What is the best treatment for JJ's lesions at 2. .Explanation: The correct answer is B. Although the clini-
this time? cal manifestation of genital herpes is highly variable, initial
A. Acyclovir 400 mg PO TID x 7 days presentation may include painful vesicular lesions, tender
B. Famciclovir 250 mg PO TID x 10 days local inguinal lymphadenopathy, dysuria, fever, and head-
C. Valacyclovir 1 g PO daily x 5 days ache. Nasal congestion and sneezing are likely the symp-
D. Valacyclovir 1 g PO BID x 7 days toms of her allergic rhinitis.'

5. One year later, JJ returns to the community health center 3. .Explanation: The correct answer is B. According to the
complaining of frequent genital herpes recurrences. She is 2015 CDC guidelines for the treatment of sexually trans-
wondering if there is anything that can be done to decrease mitted diseases, acyclovir 400 mg PO TID for 7 to 10 days
the frequency of her recurrences. JJ is currently asymptom- is one of the recommended regimens for the treatment
atic. What is the best recommendation for JJ at this time? of the initial episode of genital herpes. Treatment can be
A. Acyclovir 200 mg PO BID extended if healing is incomplete after 10 days of therapy.
B. Famciclovir 250 mg PO daily Intravenous therapy is recommended for patients with
C. Valacyclovir 1 g PO daily severe herpes disease that requires hospitalization such as
D. Valacyclovir 500 mg PO BID disseminated infection and CNS manifestations. Famciclo-
vir 125 mg PO BID x 5 days and valacyclovir I g PO daily
6. JJ is now in a relationship with a man she met at recent x 5 days are two of the recommended regimens for the
birthday party. JJ is wondering if there is anything that episodic treatment of recurrent genital herpes, not initial
can be done to prevent transmission of genital herpes to episodes. All antivirals are considered equally effective and
her new boyfriend. What are some important counseling the choice of an agent is influenced by cost, frequency of
points for the couple to decrease the risk of transmission administration, and duration of therapy. The duration of
of genital herpes? therapy is usually longer for an initial episode of genital
A. Abstain from sexual intercourse during symptomatic herpes. 1• 4
episodes 4. .Explanation: The correct answer is C. According to the
B. JJ should continue to take suppressive antiviral therapy 2015 CDC guidelines for the treatment of sexually trans-
C. Use condoms consistently and appropriately
mitted diseases, valacyclovir 1 g PO daily for 5 days is one
D. All of the above
of the recommended regimens for the episodic treatment
7. JJ returns to the community health center complaining of recurrent genital herpes. Acyclovir 400 mg PO TID x 7
of a burning, tingling, and itching sensation around her days, famciclovir 250 mg PO TID x 10 days, and valacy-
mouth since yesterday. She has had cold sores in the past clovir 1 g PO BID x 7 days are three of the recommended
and was told that she has HSV-1 infection. She is worried regimens for the treatment of the initial episode of genital
CHAPTER 26 I HERPES 109

herpes. It is recommended to start treatment as early as unaware of their status and should consider receiving treat-
possible after symptom onset. All antivirals are considered ment if they are infected or if they become symptomatic. 1
equally effective and the choice of an agent is influenced by 7. Explanation: The correct answer is C. Docosanol 10%
cost, frequency of administration, and duration of therapy. cream is the only topical antiviral product available over-
The duration of therapy is usually shorter for a recurrent the-counter for the treatment of herpes labialis. Penciclovir
episode of genital herpes.1-4 1% cream and acyclovir 5% cream are used for the treat-
5. Explanation: The correct answer is C. According to the ment of herpes labialis; however, they require a prescrip-
2015 CDC guidelines for the treatment of sexually trans- tion. Acyclovir 5% ointment is used for the treatment of
mitted diseases, valacyclovir 1 g PO daily is one of the genital herpes and requires a prescription.5- 7
recommended regimens for the suppressive treatment
of recurrent genital herpes. Acyclovir 200 mg PO BID,
famciclovir 250 mg PO daily, and valacyclovir 500 mg PO REFERENCES
BID are incorrect regimens for the suppressive treatment
1. Centers for Disease Control and Prevention. Sexually
of recurrent genital herpes. Valacyclovir 500 mg daily is
transmitted diseases treatment guidelines, 2015. MMWR.
considered less effective than other regimens in patients
2015;64(3):1-137.
with 10 or more episodes per year. The choice of an agent
2. Zovirax (Acyclovir) [package insert]. Research Triangle
is influenced by cost and frequency of administration.1--4
Park, NC: GlaxoSmithKline; June 2005.
6. Explanation: The correct answer is D. In order to decrease 3. Famvir (Famciclovir). [package insert]. East Hanover, NJ:
the risk of transmission of genital herpes, the couple should Novartis Pharmaceuticals Corp; February 2006.
abstain from sexual intercourse during symptomatic epi- 4. Valtrex (Valacyclovir) [package insert]. Research Triangle
sodes and use condoms consistently and appropriately. Park, NC; GlaxoSmithKline; September 2008.
Latex condoms or polyurethane condoms in the presence 5. Usatine RP, Tinitigan R. Nongenital herpes simplex virus.
of allergy to latex provide better protection against sexually Am Fam Physician. 2010;82:1075-1082.
transmitted diseases than "natural"/lambskin condoms. In 6. Abreva (Docosanol) [package insert]. Research Triangle
addition, JJ should continue to take suppressive antiviral Park, NC: GlaxoSmithKline; June 2006.
therapy to decrease the risk of genital herpes recurrences. 7. Denavir (Penciclovir) [package insert]. Newtown, PA:
Transmission rates can increase during symptomatic peri- Prestium Pharma, Inc; September 2013.
ods. JJ should inform her sexual partners about her infec-
tion. Sexual partners should consider testing if they are
This page intentionally left blank
27 Chlamydia and
Gonorrhea
Takova D. Wallace-Gay Jonathan C. Cho

PATIENT PRESENTATION Physical Examination


Chief Complaint ~ Yitai Signs
"I've noticed some discharge in my underwear:" Temp 98.6°F. P 68, RR 18 breaths per minute, BP 116/70 mm Hg,
pOi 97%, Ht 5'8", Wt 72.7 kg
History of Present Illness
~ Geneml
TP is a 23-year-old black female who presents to the
family medicine clinic with complaints of unusual discharge. Well developed, well-nourished fem.ale in no acute distress
She reports a frequent sensation of urination and mentions
~ HEENT
that she has been having odorless vaginal discharge that is
different than usual. Upon questioning she mentions being Non-contributory
nervous that she may have an STD because she had unpro-
tected vaginal intercourse with a young man after a fraternity ~ Pulmonary
party about 2 weeks ago. Since then, she heard rumors that Denies shortness of breath
this particular gentleman has a history of multiple sexual part-
ners, but she was not concerned until noticing the discharge ~ Cardiovascular
2daysago. NSR, no m.Jr/g

Past Medical History ~ Abdomen


Asthma, prior ankle injury Soft. non-distended, non-tender, (+) bowel sounds

Surgical History ~ Genitourinary


Tonsillectomy Normal female genitalia, denies abnormal bleeding. (+) vagi-
nal discharge
Family History
~ Neurology
Mother and father with HTN
(+)headaches 1-2 times a week, A&O X3
Social History ~ Extremities
Active as a college basketball player; occasionally smokes Noncontributory
marijuana; (-) tobacco; (+) EtOH (beer and Uquor most
weekends)
Laboratory Findings
Allergies Na= 136mEq/L Hgb = 12.9 g/dL Ca= 8.5 mg/dL
Doxycycline (swelling oflips and tongue as a teenager) K=4.lmEq/L Hct= 37.3% Mg= 2.1 mEq/L
Cl=98mEq/L Plt = 256 x 103/ Phos = 3.5 mgldL
Home Medications rrunS
Acetaminophen 500 mg, two tablets PO q4-6h PRN pain COl = 26 mEq/L WBC = 6.3 x 103/ AST= 18U/L
Ibuprofen 200 mg PO q4-6h PRN pain mmi
Albuterol metered-dose-inhaler 2 puffs q4h PRN shortness of BUN= 12 mgldL ALT=22U/L
breath and prior to exercise
Ethinyl estradiol and norgestimate (Ortho 'In-Cyclen Lo) one SCr = 0.9 mgldL T Bill = 0.2 mgldL
tablet PO daily Glu = 108 mg/dL Alk Phos = 40 IU/L

111
112 INFECTIOUS DISEASES: A CASE STUDY APPROACH

7. Regardless of recent sexual activity, how often should


women under the age of 25 be screened for chlamydia?
A. Every year
Vaginal discharge: Whiff test (-); yeast (-); wet mount: B. Twice a year
pending C. Every 3 years
Urine nucleic acid amplification testing (NAAT): pending D. Only when symptomatic

Serology: pending 8. Which of the following is true regarding gonorrhea and


chlamydia infections?
A. Partners of the co-infected individual only need to
undergo treatment if interoourse occurred less than 60
QUESTIONS days prior to the diagnosis ofchlamydia and gonorrhea.
1. Which sexually transmitted infection is TP mostly likely to B. Partners of the co-infected individual only need to
have? undergo treatment if he/she is experiencing symptoms
A. Neisseria gonorrhoeae oonsistent with c.hlamydia or gonorrhea.
C. Test-of-cure is not needed for persons with a diagnosis
B. Candida albicans
uncomplicated urogenital gonorrhea and chlamydia
C. Treponema paUidium
D. Test-of-cure is needed for all patients with co-infection
D. GardnereUa vaginal#
of gonorrhea and chlamydia
2. Which laboratory test is considered to be the most sensi-
tive and is recommended for detecting N. gonorrhoeae? 9. Considering that this patient was initiated on azithromycin
A. Wet mount on genital discharge 1 g PO once, what would be the most important potential
B. Serologic (blood antibody) test adverse reaction to include in your counseling session?
C. Rapid plasma region (RPR) card test A. Photosensitivity
D. Nucleic acid amplification test (NAAT) B. Ede.ma
C. Hyperglycemia
3. If preliminary urine NAAT is positive for N. gonor- D. Loose stools
rhoeat, what other sexually transmitted infection should be
assumed?
A. Syphilis ANSWERS
B. Chlamydia 1. Explanation: The correct answer is A. Due to the TP's
C. Genital herpes clinical findings of abnormal. odorless vaginal discharge. it
D. Bacterial vaginitis is most likely that patient has gonorrhea (N. gonorrhoeae).
4. What empiric antimicrobial therapy is most appropriate Female patients infected with gonorrhea may complain of
forTP? abnormal wginal discharge or uterine bleeding as well as
A. Doxycycline 100 mg PO BID X 7 days dysuria and urinary frequency. 1 Although the timing could
B. Azithromycin 1 g PO once + ciprofl.oncin 500 mg be appropriate for syphilis/T. pallldium (10 to 90 days,
BID X Sdays mean 21 days), patients typically present with single. pain-
C. Ceftria.xone 250 mg IM once + azithromycin 1 g PO less ulcer in the groin area.1 Answers B and D are incor-
once rect since a negative Whiff test and the absence of yeast
D. Azithromycin 2 g PO once reduce the likelihood of bacterial vaginosis (G. vaglnalis)
and vulvovaginal c:andidiasis (C. albicans), respectively. G.
5. Which of the following is the most appropriate sec- vaginalls is present in a diagnosis of bacterial vaginosis and
ond-line treatment option for this patient's diagnosis of patients typically have thin white or gray vaginal discharge
c.hlamydia? in addition to pain. itching, burning, and a strong fish-like
A. Doxycycline 100 mg PO BID x 7 days odor.i
B. Cephalexin 1000 mg PO BID x 7 days
2. Explanation: The oorrect answer is D. RPR card is used
C. Levofloncln 500 mg PO daily x 7 days
to screen for syphilis and serologic tests are used to detect
D. Fluconazole 150 mg PO once
blood antibodies against STis such as syphilis, HIY, and
6. Which of the following is true regarding symptoms of hepatitis. Serology and RPR are not used for N. gonor-
chlamydia? rhoeae or Chlamydia. Wet mount/wet preparations of
A. Genital discharge is a cardinal symptom that is always genital secretions are convenient and inexpensive and can
present provide clinicians with good insights regarding overall
B. Many men and women are asymptomatic vaginal health; however, the sensitivity of vaginal specimen
C. Men and women frequently complain of urinary wet mount is low (51 % to 65%). Nucleic acid amplification
retention test (NAAT) such as polymerase chain reaction (PCR) can
D. Men and women oomplain of painful genital blisters be performed on endocervical swab, vaginal swab, and
CHAPTER 27 I CHLAMYDIA AND GONORRHEA 113

urine specimens and sensitivity is superior to cultures for among men and women, making screening an important
diagnosing gonorrhea and/or chlamydia. NAATs offer a aspect for diagnosis. Urethral and vaginal discharge may
simplistic, efficient, and accurate testing option.3 be present in persons with chlamydia but is more com-
3. Explanation: The correct answer is B. Due to the high mon with gonorrhea (rules out answer A).3 Additionally,
frequency of N. gonorrhoeae and Chlamydia trachomatis infected persons may complain of dysuria or urinary fre-
(chlamydia) co-infection, treatment guidelines recom- quency rather than retention (rules out answer C). Painful
mend that individuals treated for gonococcal infection genital blisters have not been associated with chlamydia
also be treated with a medication regimen effective against and are more common in persons with genital herpes.'
uncomplicated C. trachomatis infection (this fact rules out 7. Explanation: The correct answer is A. According the most
answer choices A, C, and D). recent guideline recommendations, annual screening is
4. Explanation: The correct answer is C. Answer A is suggested for all women under the age of 25 years and
incorrect since doxycycline alone does not provide good for older women with increased risk (eg, more than one
coverage for gonorrhea. Doxycycline may be used as a sex partners, sex partner with an STI, or those with a new
second-line agent to treat chlamydia. Fluoroquinolone- sex partner) of infection (this recommendation rules out
resistance against N. gonorrhoeae has been emerging in answer choices B, C, and D).3
the United States over the past decade; therefore, the CDC 8. Explanation: The correct answer is C. According to the
no longer recommends using this class of antibiotics to most current guidelines, test-of-cure is not necessary for
treat gonorrhea (rules out answer B). Additionally, dual patients with uncomplicated diagnoses. Test-of-cure is
therapy has been recommended for the treatment of gon- recommended in certain situations such as pharyngeal
orrhea in order to improve treatment efficacy and slow gonorrhea, suspected treatment failure, special populations
any emergence of cephalosporin resistance. With this dual (pregnancy and neonatal), questionable adherence, and
therapy recommendation, the CDC recommends using persistent symptoms among others (rules out answer D).
two antimicrobials with differing mechanisms of action Partners of infected persons should be referred for evalu-
with the primary agents being ceftriaxone and azithro- ation, testing, and treatment if sexual intercourse took
mycin. Oral cefixime and doxycycline may be alternatives place in the 60 days preceding the symptoms or diagnosis
to ceftriaxone and azithromycin, respectively. Answer of chlamydia or gonorrhea, regardless of symptoms since
choice D has had some clinical controversy recently since many individuals are asymptomatic (rule out answer B). In
data has shown high effectiveness (approximately 99%) the instance that the 60-day interval has passed, it is still
against uncomplicated urogenital gonorrhea; however, the recommended the most recent sex partner should still be
treatment guidelines no longer support this regimen due evaluated and treated (rules out answer A). Although test-
to concerns regarding N. gonorrhoeae resistance against of-cure is not necessary, it is recommended that all men
macrolides.3 and women who have been treated for chlamydia or gon-
5. Explanation: The correct answer is C. According to a meta- orrhea be retested roughly 3 months following treatment
analysis consisting of 12 RCTs, azithromycin (recommend since reinfection may be present due to sexual partners
therapy) versus doxycycline for urogenital chlamydia treat- neglecting to be treated or due to the individual engaging
ment was shown to be equally efficacious; however, this in sexual activity with a new infected partner. Regardless of
patient has reported an allergy to doxycycline with lip and if the person believes their sexual partner(s) were treated,
tongue swelling which makes it an inappropriate option retesting at 3 months is recommended.3
(rules out answer A).4 The extent of this allergy to doxycy- 9. Explanation: The correct answer is D. According to drug
cline (swelling oflip and tongue) makes it an inappropriate information resources loose stools are a common and
option; therefore, it is important to collect accurate allergy unfavorable occurrence with azithromycin use and the
history (timing and extent). Current guidelines endorse incidence may be higher with single-dose regimens. Pho-
that levofloxacin (SOO mg PO daily x 7 days) and ofloxa- tosensitivity is a potential adverse event with doxycycline,
cin (300 mg PO BID X 7 day) are effective treatments, but the same risk is not present with azithromycin (rules
although they may not be as cost-effective and offer no out answer A). Answer choices B and C are potential coun-
benefit regarding dosing regimen (Answer C). Additional seling points, but these effects have less than 1% incidence
alternative regimens include erythromycin 500 mg PO four and would not be the most important points to cover for
times daily for 7 days or erythromycin ethylsuccinate 800 single-dose regimens with azithromycin.6
mg PO four times a day x 7 days. 3 Fluconazole is an anti-
fungal and is effective against Candida infections (rules out
answer D). Cephalexin is a first-generation cephalosporin REFERENCES
and has poor coverage against intracellular bacteria such as
1. Knodel LC, Duhon B, Argamany J. Sexually transmitted
chlamydia.5
diseases. In: Dipiro JT, Talbert RL, Yee GC, et al. Phar-
6. Explanation: The correct answer is B. According to the macotherapy: A Pathophysiologic Approach. lOth ed. New
current guidelines, asymptomatic infection is common York, NY: McGraw-Hill.
114 INFECTIOUS DISEASES: A CASE STUDY APPROACH

2. Hainer BL, Gibson MV. Vaginitis: Diagnosis and treat- 5. Gallagher JC, MacDougall C. Antibiotics Simplified, 3rd
ment. Am Fam Phys. 2011;83(7):807-815. ed. Sudbury, MA: Jones and Bartlett Learning.
3. Workowski KA, Bolan GA. Sexually transmitted dis- 6. Azithromycin. In: Wolters Kluwer Clinical Drug Infor-
eases treatment guidelines, 2015. MMWR Recomm Rep. mation, Inc. (Lexi-Drugs); April 4, 2019.
2015;64(No. 3):55-68.
4. Lau CY, Qureshi AK. Azithromycin versus doxycycline for
genital chlamydia! infections: a meta-analysis of random-
ized clinical trials. Sex Transm Dis. 2002;29(9):497-502.
28 Bacterial Vaginosis,
Vulvovaginal
Candidiasis, and
Trichomoniasis
Elizabeth A. Cook Jessica Wooster
Jonathan C. Cho

PATIENT PRESENTATION Home Medications


Chief Complaint .Metform.in 1000 mg PO BID W/F
Empagliflozin 25 mg PO daily
"I'm having some issues with my private parts:' Lisinopril 20 mg PO daily
.At.orvastatin 20 mg PO daily
History of Present Illness Copper IUD inserted 2 months ago
SP is 42-year-old Caucasian female presenting to her primary care
provider for the above complaint. For the past seven days. she bas
Physical Examination
experienced moderate vaginal discomfort, described as a burn-
inglitclling sensation around her vulvovaginal area, particularly ..,. Vital Signs
when she urinates. She reports abnormal amounts of thin dis- Temp 98.5°F, P 86 beats per minute, RR 16 breaths per minute,
c::harge from her vagina throughout the day. She is most perturbed BP 124172 mm Hg, p02 99%, Ht 5'8", Wt 89.8 kg
by the odor and color of the discharge, which is dark yellow and
foul smelling. She is recently divorced and is GOncemed that these ..,. General
symptoms will impact her social life, as she has began to experi-
ence pain during vaginal intercourse and is self-conscious of the Slightly overweight, but well-groomed woman
odor of her discharge. Her last period was 2 weeks ago.
..,. Genital/Rectal
Past Medlcal History Labia minora inflamed and red. No discharge issuing from
T2DM, HTN, HLD uretlua. Vagina exhibiting small amounts of frothy, thin yel-
low mucus, which emotes a putrid odor. Cervix completely
Surgical History visualized upon speculum insertion; appears pink with red
"strawberry1' lesions. Purulent discharge observed around
Adenoidectomy endocervi.cal canal.

Famlly History
Laboratory Findings
Noncontributory
Na= 136 mEq/L HbAic=6.8%
Social History K=4.lmEq/L TC= 126 mg/dL
G3P2Al. Recently divorced after a 20-year marriage. Cl = 100 mEq/L LDL = 68 mgldL
Currently dating several men and has multiple male sexual C01 = 27 mEq/L HDL = 54 mg/dL
partners. Does not use condoms during sexual intercourse.
BUN= 10 mgldL TG=98mg/dL
Denies tobacco use. Drinks 4 to 5 alcoholic mixed drinks on
the weekends when out with friends. SCr = 0.6 mg/dL
Glu = 106 mg/dL
Allergies Ca=9mg/dL
NKDA
11 s
116 INFECTIOUS DISEASES: A CASE STUDY APPROACH

~ Urinalysis 5. SP states that she has had sexual intercourse with several
Color-light yellow, Clarity/turbidity-clear, pH-6, Specific partners since symptom presentation 7 days ago. She is
gravity-1.005, Glucose-300mg/dL, Blood-none, Ketones- concerned that she may have transmitted her condition
none, Nitrites-negative, Leukocyte esterase-negative to these individuals. Which of the following statements/
recommendations is true?
~ Serum Pregnancy Test A. Presumptive therapy should be initiated to all recent
sexual contacts regardless of symptom presentation.
Negative
B. Therapy should only be initiated if similar symptoms
are present in the patient's sexual partners.
~ "'Whiff0 Test
C. This condition is not transmitted through sexual
Negative contact; thus, treatment is not required of sexual
partners.
~ pH of Vaginal Secretions D. This condition only manifests in female patients, thus
6 only treatment of female sexual partners is required.

~ Vaginal Wet Mount 6. SP's primary care provider asks whether confirmatory test-
ing for disease eradication will be necessary after comple-
Several motile pear-shaped, flagellating organisms, many leu- tion of therapy. Which of the following monitoring plans is
kocytes and minimal squamous epithelial cells are visualized most appropriate?
in field. Clue cells and lactobacilli absent A. Confirmatory testing for disease eradication is recom-
mended for both the patient and her sexual partners.
Present to clinic within 3 months for follow-up.
QUESTIONS B. Confirmatory testing for disease eradication is only
1. Based on the above signs, symptoms, and laboratory find- recommended for the patient. Present to clinic within
ings, which of the following conditions is/are most likely 3 months for follow-up.
afflicting SP? C. Confirmatory testing for disease eradication is only
A. Bacterial vaginosis recommended for patient's sexual partners. Present to
B. Trichomonal vaginitis clinic within 3 months for follow-up.
C. Vulvovaginal candidiasis D. Confirmatory testing for disease eradication is not
D. Urinary tract infection warranted for the patient nor her partners.
2. Which of the following pathogens is associated with caus- 7. SP calls your clinic 2 weeks later complaining of pruritus
ing the above diagnosis? and erythema isolated to her vulvovaginal area. These
A. Gardnerella sp. symptoms are accompanied by scant vaginal discharge
B. Candida sp. that is "thick and white like cottage cheese and doesn't
C. Prevotella sp. smell bad like last time." She states that these symptoms are
D. None of the above vastly different from those she experienced previously. She
3. Which of the following treatments would be most appro- informs you that she has not had sexual intercourse since
priate for treatment of SP's current condition? she saw you last month. Which of the following conditions
A. Clindamycin cream 2%, one full applicator PV QHS x is/are most likely afflicting the patient?
7days A. Bacterial vaginosis
B. Metronidazole gel 0.75%, one full applicator PV daily X B. Trichomonal vaginitis
7days C. Vulvovaginal candidiasis
C. Metronidazole 500 mg PO x 7 days D. Urinary tract infection
D. Metronidazole 2000 mg PO x 1 day
8. SP inquires whether there are any over-the-counter treat-
4. Which of the following counseling points should be con- ments that she can purchase at her local pharmacy to
veyed to individuals managed with the therapeutic agent treat her new infection. Which of the following would be
selected above? most appropriate over-the-counter treatment for SP's new
A. Avoid drinking alcohol while actively taking this condition?
medication and for 24 hours following completion of A. Clindamycin cream 2%, one full applicator PV QHS x
therapy. 7days
B. Separate this therapy from foods, drinks, medications, B. Metronidazole gel 0.75%, one full applicator PV QHS
and supplements containing polyvalent metallic ions. x 5 days
C. Sunscreen should be utilized while taking this C. Miconazole 4% cream, once full applicator PV QHS X
treatment. 3 days
D. Use of this medication may result in orange-red discol- D. Terconazole 80 mg vaginal suppository PV QHS x 3
oration of feces, saliva, sweat, tears, and urine. days
CHAPTER 28 I BACTERIAL VAGINOSIS, VULVOVAGINAL CANDIDIASIS, AND TRICHOMONIASIS 117

ANSWERS vaginosis. Nitroimidazoles have sufficient activity in ewm·


nating anaerobic bacteria seen in bacterial vaginosis, which
1. Explanation: The correct answer is B. The patient's symp· may demonstrate utility in the treatment of individuals
toms and laboratory findings do not align with vulvo· presenting with both trichomoniasis and bacterial vagi·
vaginal candidiasis, bacterial vaginosis. or a urinary tr.lct nosis. It is important to note that only oral nitroimidazole
infection (UTI), making trichomonal vaginitis the only regimens are viable options for the treatment of tricho·
correct answer. The odor and physical characteristics of monas, as intravaginal metronidazole gel does not reach
her vaginal discharge, presence of cervical •strawberry sufficient therapeutic levels in the urethra or perivaginal
spots"' on her pelvic exam, vaginal pH, and flagellating glands for full eradication. Additionally, oral metroni·
protozoa visible in her vaginal wet mount are all supportive dazole dispensed as a single-dose regimen should be the
of a diagnosis of trichomonal vaginitis.1 Uncomplicated preferred treatment for trichomonal vaginitis. as several
UTI typically presents as lower urinary symptoms includ· clinical trials have demonstrated its superiority in con·
ing dysuria. frequency. and urgency in otherwise healthy veying parasitologi.cal cure compared to extended 7-day
nonpregnant women.:a Table 28.1 is provided below to treatment regimens. Intr.avaginal metronidazole. which
compare and contrast characteristics of bacterial vaginosis, is eirective in the treatment of bacterial vaginosis, is still
tricb.omonal vaginitis, and vulvovaginal ca:ndidiasis. The considered a viable treatment option for that disease. Table
presence of the key characteristics listed in this table, are 28.2 compares and contrasts the treatment of trlchomonia·
utilized by providers in the diagnosis and exclusion of sis and bacterial vaginosis to highlight potential errors that
common vaginal infections.1 may occur in treating the two distinct conditions.1
2. Explanation: The correct answer is D. Trichomoniasis is 4 • .Explanation: The correct answer is A. Alcohol consump·
associated with only one causative protozoan, llichomona.s tion should be avoided during treatment with nitro.imid·
vaginalis. Gardnerella vaginalis and other anaerobic bac· azoles due to the potential for disulfuam-.like reactions (eg,
teria (eg, Bacteroides sp., PrevoteUa sp., Peptostreptococ· severe nausea and vomiting, flushing. tachycardia, hypo·
cus sp.) are all common causative pathogens for bacterial tension). Due to the different half.Jives of metronidazole
vaginosis. Candida spp. are implicated in vulvovaginal and tinidazole. alcohol abstinence should be observed for
candidiasis.1 24 and 72 hours, respectively, after completion of therapy
3. Explanation: The correct answer is D. Only medications to ensure medication is eliminated from the body. Metro·
belonging to the nitroimidazole class (metronidazole and nidazole does not chelate with polyvalent ions (eg, calcium,
tinidazole) have activity against trichomonas. Therefore. iron, magnesium). nor does it increase photosensitivity like
clindamycin is not a viable option for the treatment of tetracycline or quinolone antibiotics. Metronidazole use
trichomoniasis, but is effective in the treatment of bacterial does not result in orange-red discoloration of body fluids,

TABLE 28.1. Characteristics ofVaglnal Infections


Cauative Pathogen
Charaderi8tk Bac:terlal Candl4o 7Wchomonas
Wet mount Mycelia 6bers, minimal Pear·shaped, 1lagellating protozoa,
findings aue cells, minimal leukocytes leukocytes increased leukocytes
pH Elevated (>4.5) Normal (3.5-5.0) Elevated (>S.O)
PeMcenm Vulva and/or vagina may or may Vulva and/or vagina is/are Vulva and/or vagina may or may
not be inftamed or erythematous. frequently inftamed or not be inflamed or erythematow.
Abnormal volume of discharge erythematow. Abnormal "Strawberry spot/" visible on cervix.
is frequently present Positive volume of discharge is Abnormal volume of discharge may
"'whiff test': frequently present or may not be present.
Odor of Fishy None Malodorous
discharge
Colorof Gray White White. yellow. or green·gray
discharge
Viscosity of 1hin,homogenous 1hick. clumpy Thick or thin, frothy
discharge
Scmrce: Data from Workowski KA, Bolan GA; Center& for Dilea&e Control and Prevention. Sexually transmitted di&eases treatment guidelines,
2015. MMWR. Recomm Rep. 2015 Jun 5;64(RR-03):1-137.
118 INFECTIOUS DISEASES: A CASE STUDY APPROACH

TABLE 28.2. Preferred and Alternative Regimens TABLE 28.3. Over-the.COunter and Prasaiption
for the Treatment of Bacterial Yaginosis and Treatment Regimens forVulvovaginal Candidiasis
Tridtomoniasis Prescription
aver.the-Counter
Treatment Clotrimazole 1% cream, one full Butoconazole 2%
Regimens Baderlal. Vaglnoais Trichomoniuis applicator (5 g) PV daily x 7-14 cream (single
Prefured Metronidazole 500 mg Metronidazole da}'5 OR dose bioadhesive
PO BID x 7 days OR 2000mgPO x 1 Clotrimazole 2% cream, one full product) 5 g PV x
Metronidazole gel dose OR applicator (5 g) PV daily x 3 1 day OR
0.75%, one full Tinidazole 2000 da}'5 OR Terconazole 0.4%
applicator mg PO x 1 dose Miconazole 2% cream, one full cream, one full
(S g) PV once daily x applicator (S g) PV daily x 7 applicator (S g) PV
5daysOR da}'5 OR daily x 7 days OR
Clindamycin cream 2%, Miconazole 4% cream, one full Terconazole 0.8%
one full applicator (S applicator (5 g) PV daily x 3 cream, one full
g) PV QHS x 7 days days OR applicator (5 g) PV
Alternative Tinidazole 2000 mg PO Metronidazole 500 Miconazole 100 mg one daily x 3 days OR
daily x 2 days OR mg PO BID x 7 suppository PV daily x 7 days Terconazole 80 mg
Tinidazole 1000 mg PO days OR one suppository PV
daily x 5 days OR Miconazole 200 mg one daily x 3 days OR
Clindamycin 300 mg suppository PV daily x 3 days Fluconazole 150 mg
PO BID x 7 days OR OR PO x 1
Clindamycin ovules 100 Miconazole 1200 mg one
mg PV QHS x 3 days suppository PV daily x 1 day
Source: Data from Workowski KA, Bolan GA; Centera for Diseaae OR
Control and Prevention. Sexually transmitted diseases treatment Tioconazole 6.5% ointment, one
guidelines, 2015. MMWR Recomm Rep. 2015 Jun 5;64{RR-03): full applicator (S g) PV x 1 day
1-137. Source: Workowski KA, Bolan GA; Centers for Disease Control
and Prevention. Scmally transmitted diseases treatment guidelines,.
2015. MMWR Recomm Rep. 2015 Jun 5;64(RR-03):1-137.
as seen with agents such as rifampin, but does have the
potential to cause brownish discoloration of the urine due 17% by some estimates, confirmatory testing for eradica·
to excretion of metabolites of the parent compound.1 tion is suggested for all females within 3 months following
5. Explanation: The correct answer is A. Presumptive treat- completion of initial treatment Testing for disease eradi-
ment should be initiated in all sexual partners to prevent cation can be completed by wet mount, culture. eney:me
transmission or reinfection with Trichomonas vaginalis immunoassay, nucleic add probe test, or nucleic acid
regardless of symptom presentation. Individuals should ampli&ation. Data are currently insufficient to recom-
abstain from sexual intercourse until they and their part- mend retesting for males following completion of treat·
ners have completed treatment and have achieved com- ment for trichomoniasis. If not already performed at the
plete symptom resolution. Despite the name Trichomonas initial visit, testing for other sexually transmitted diseases,
vaginalis, trichomoniasis can manifest in both females and including human immunodeficiency virus, should be
males, so both sexes must be treated if they have engaged completed by all parties. If the patient had been diagnosed
in intercourse with an infected individual Males are fre- with either bacterial vaginosis or vulvovaginal candidiasis,
quently asymptomatic. If the patient had been infected follow·up visits would not be considered necessary should
with either bacterial vaginosis or vulvovaginal ca.ndidiasis, her symptoms resolve following completion of medica-
recommendations for the treatment of sexual partners dif. tion therapy. Confirmatory eradication testing in females
fer. Evidence suggests that response to therapy and/or dis· treated for either disease state has not shown evidence
ease relapse remains unchanged when partners of women in reducing recurrence of uncomplicated infections or
with bacterial vaginosis are provided with treatment. clinical outcomes. Additionally, follow·up testing would be
Therefore, presumptive treatment of bacterial vaginosis considered unwarranted for partners considering sexual
is not recommended. Additionally, uncomplicated vulvo· transmission is unlikely for both bacterial vaginosis and
vaginal candidiasis is typically not transmitted through vulvovaginal candidiasis.1
sexual intercourse and data do not support treatment of 7. Explanation: The correct answer is C. Based on the infor·
sexual partners.1 mation provided by the patient, the most likely diagnosis
6. Explanation: The correct answer is B. Due to relatively is vulvovaginal candidiasis. The close proximity of her
high reinfection rates with Trichomonas vaginalis, up to symptoms in relation to completing metronidazole therapy
CHAPTER 28 I BACTERIAL VAGINOSIS, VULVOVAGINAL CANDIDIASIS, AND TRICHOMONIASIS 119

may be indicative of an imbalance in normal vaginal flora, requires a prescription. Both clindamycin cream and
resulting in overgrowth of Candida sp. The signs/symp- metronidazole gel would be effective in the treatment of
toms that the patient presents with do not correlate with bacterial vaginosis, but would not be effective in the treat-
her previous diagnosis of trichomonal vaginitis, making ment of vulvovaginal candidiasis. Furthermore, both the
recurrent infection, or treatment failure, unlikely. The clindamycin cream and metronidazole gel would require a
physical characteristics, absence of odor from the vaginal prescription to be dispensed to the patient for treatment 1
discharge, and lack of urinary symptoms also make it
unlikely that the patient is presenting with bacterial vagi-
nosis (Table 28.1). Uncomplicated UTI typically presents REFERENCES
as lower urinary symptoms including dysuria, frequency, 1. Workowski KA, Bolan GA; Center for Disease Control
and urgency in otherwise healthy nonpregnant women, and Prevention. Sexually transmitted diseases treatment
making this an unlikely diagnosis as well.2 guidelines, 2015. MMWR Recomm Reports. 2015 June
8. Explanation: The correct answer is C. The preferred treat- 5;64(RR-03):1-137. Erratum in: MMWR Recomm Rep.
ment of uncomplicated vulvovaginal candidiasis includes 2015 Aug 28;64(33):924.
a short course of topical azole antifungal therapy, such as 2. Gupta K, Hooton TM, Naber KG, et al. International clin-
miconazole or terconazole. However, of those two agents, ical practice guidelines for the treatment of acute uncom-
only the miconazole cream is available without a prescrip- plicated cystitis and pyelonephritis in women: a 2010
tion. Table 28.3 highlights over-the-counter and prescrip- update by the Infectious Diseases Society of America and
tion treatment modalities for vulvovaginal candidiasis. the European Society for Microbiology and Infectious
Of note, oral treatment with £1.uconazole is available, but Diseases. Clin Infect Dis. 2011;52(5):e103-e120.
This page intentionally left blank
29 Superficial Fungal
Infections
Winter J. Smith Jonathan C. Cho

PATIENT PRESENTATION Physical Examination


Chief Complaint ..- Vital Signs
"I have an itchy rash on my stomach and feet that won't go aw.ay." BP 118/78 mm Hg, P 60, RR 18 breaths per minute, Ht 5'11",
Wt82kg
History of Present Illness
RP is a 16-year-old male who presents to his primary care ..- General
provider's clinic at his mother's insistence because of a per- Healthy-looking teenager, muscular build, well-developed,
sistent pruritic rash on his trunk and feet He is a high school well-nourished. in NAD
sophomore and a member of the wrestling team. He reports
having two areas on his trunk that are circular, red. and itchy. 111- HEENT
He says they have increased in size since he first noticed them Normocephalic, atraumatic, PERRI.A, EOMI, moist mucous
about 3 weeks ago. He says his feet have always smelled bad,
membranes
but he first started to notice redness and "extreme itching"
between his toes on both feet about 2 weeks ago. He has used
over-the-counter hydrocortisone cream on the rash on his ..- Pulmonaty
trunk without improvement. He also started applying deodor- Normal breath sounds, no crackles or wheezes
izing spray to his feet a week ago without improvement
..- Cardiovascular
Past Medical History NSR, no mlr/g
Seasonal allergies; asthma as a young child (asymptomatic for
10 years) ..- Abdomen
Soft. non-distended, non-tender, normal bowel sounds
Surgical History
None ..- Genitourinary
Normal male genitalia without erythema or scaling
Family History
Father has diabetes, diagnosed at age 45; mother has HTN, ..- Neurology
diagnosed at age 38; both parents are living AAOX4

Social History ..- Skin


Lives at home with parents and elder sister; denies tobacco, Two round, erythematous, and scaly areas visible on the
alcohol, and illegal drug use trunk; each measures about 1 to 2 inches in diameter; each
with a well demarcated, raised border and area of central
Allergies clearing
NKDA
..- Extremities
Home Medications Foul-smelling, dry, scaling feet with localized redness and
Loratadine 10 mg PO daily white flaking between the toes. Toenails on both feet appear
Occasional ibuprofen for muscle pain/sports injuries normal and without discoloration.

121
122 INFECTIOUS DISEASES: A CASE STUDY APPROACH

QUESTIONS ANSWERS
1. Which infections are consistent with RP's presentation? 1. .Explanation: The correct answers are B and D. Based on
Select ALL that apply. RP's complaints and the location ofhis symptoms, the most
A. Tinea capitis likely diagnoses are two different dermatophyte infections:
B. Tinea corporis tinea corporis (ringworm) and tinea pedis (athlete's foot). 1
C. Tinea cruris In addition, the use of a topical steroid without symptom
D. Tinea pedis improvement suggests the presence of a superficial fun-
E. Tinea unguium gal infection. Tinea corporis generally appears as round,
scaly lesions with a well-defined raised border and an area
2. What predisposing risk factor for infection does
of central clearing. Use of topical steroids can alter the
RP have?
appearance of these lesions such that the central clearing
A. Age
and raised borders are not visible. This can make diagnosis
B. Male sex
difficult, and KOH microscopy may be needed for an accu-
C. Contact sport
rate diagnosis. Tinea pedis often affects the skin between
D. Asthma history
the toes, but can also impact other parts of the foot. Red-
3. Which empiric therapy is most appropriate for RP? ness, flaking, and skin break.down can also be observed.
A. Topical clotrimazole Despite the use of topical steroids, the characteristics of RP's
B. Oral terbinafine trunk lesions and feet are classic presentations of tinea corpo-
C. Topical nystatin ris and tinea pedis, respectively. Tinea capitis (answer A) is a
D. Oral griseofulvin dermatophyte infection of the scalp, tinea cruris (Answer C)
affects the groin area (jock itch), and tinea unguium affects
4. Which is the most appropriate duration of empiric therapy
the toenails (onychomycosis). None of the other answer
for RP?
choices is consistent with RP's complaints or physical
A. 1 day
examination findings.
B. 5 days
C. 14 days 2. .Explanation: The correct answer is C. Participation in
D. 56 days contact sports is a well-known risk factor for dermato-
phyte infections due to close contact with other potentially
5. What is the goal of therapy for RP's infections? infected athletes and the potential for transmission via sur-
A. Negative fungal cultures faces and objects (eg, locker room floors, showers, towels,
B. Immune function restoration clothing, equipment). 1 -~ Because of this, athletes should
C. Resolution of systemic signs of infection take precautionary measures to reduce the risk of infection
D. Rash and symptom resolution and transmission.24 Other risk factors for dermatophyte
6. What nonpharmacologic measures can help prevent infections include excessive sweating (hyperhidrosis), liv-
infection persistence/recurrence in RP? Select ALL that ing in close contact with others (eg, military housing),
apply? obesity, and diabetes. 5 Answers A and B are incorrect
A. Change socks every 3 days because tinea corporis and pedis can affect individuals of
B. Wear the same shoes each day any age and sex.1 (However, tinea capitis is most common
C. Keep affected body areas clean and dry in pediatric patients. 1) RP's distant asthma history does not
D. Wear footwear in shared showers and locker rooms predispose him to dermatophyte infections (Answer D is
E. Share clothing and towels with teammates incorrect).
F. Wear absorbent socks 3. .Explanation: The correct answer is A. Topical antifungal
7. If RP completes his course of therapy but his infection agents are first-line for tinea corporis and tinea pedis
persists without significant improvement, which is the cases that are not severe or extensive. 1 Topical options
most appropriate next step? include azoles (eg, clotrimazole, miconazole, econazole),
A. Wait and watch, no antifungal therapy allylamines (naftifine, terbinafine). one benzylamine agent
B. Topical antifungal therapy (butenafine), and one thiocarbamate agent (tolnaftate).1•6
C. Oral antifungal therapy Most topical antifungal preparations are available without
D. Intravenous antifungal therapy a prescription (over-the-counter [OTC]). Patients may
select a specific formulation based on the site of infection,
8. Which infection would RP be at the highest risk for, in cost, and their preference. 7 Patients should be instructed to
addition to his current infection, if he is not treated? apply the selected product to the skin lesion(s) and one to
A. Tinea capitis two inches around the lesion(s).7 Topical antifungal agents
B. Tinea versicolor are well tolerated, only infrequently causing transient and
C. Tinea cruris mild skin irritation.6 Although oral (systemic) terbinafine
D. Tinea unguium (Answer B) is effective against dermatophyte infections,
CHAPTER 29 I SUPERFICIAL FUNGAL INFECTIONS 123

it is usually reserved for more severe/extensive infections Answer E is incorrect because sharing clothing and towels
or those that do not respond to topical therapy.1.6 Another with teammates can promote the transmission of dermato-
indication for systemic therapy, such as oral terbinafine and phyte infections.
oral itraconazole, is fungal nail infections. Conventional
topical antifungals do not penetrate the nail, and thus are 7. Explanation: The correct answer is C. Tinea corporis
not effective for treatment. Topical preparations designed and tinea pedis infections that do not respond to initial
specifically for fungal nail infections, such as ciclopirox, are topical therapy should be treated with oral (systemic)
also associated with low cure rates.1 Answer C is incorrect therapy.1 Answer A is incorrect; oral (systemic) therapy
because nystatin is ineffective against the organisms that is necessary if topical therapy fails. Answer B is incorrect
cause dermatophyte infections. Nystatin is, however, an because failure of topical therapy warrants the use of oral
option for treating cutaneous Candida infections. Answer (systemic) therapy. Examples of oral therapy options for
D, griseofulvin, is incorrect because topical therapy should tinea corporis and tinea pedis include terbinafine 250 mg
be used first in this case. Also, oral griseofulvin requires PO daily for 2 to 4 weeks, itraconazole 200 to 400 mg PO
longer treatment durations than many other oral antifun- daily for 1 week (400 mg dose divided twice daily), and
gal agents used for dermatophyte infections. It is generally fluconazole 150 mg PO once weekly for 2 to 4 weeks.6.11
reserved for the treatment of tinea capitis. Terbinafme is the most effective oral (systemic) therapy
for onychomycosis treatment, with cure rates around
4. Explanation: The correct answer is C. The frequency of 45%.9 Common and significant side effects associated
topical antifungal application and duration of therapy vary with terbinafine include headache, gastrointestinal (GI)
based on the agent, but they are typically applied 2 to 3 times disturbances, and hepatotoxicity.6.1 Hepatic transaminase
daily for 1 to 4 weeks. 1•6 While certain topical azole monitoring is recommended at baseline and periodically
preparations may be effective after one day of treatment during therapy. Terbinafine use is not recommended in
(Answer A) for vulvovaginal candidiasis, this is not the case patients with liver disease or injury. Drug interactions
for dermatophyte infections. Answer B is incorrect because are possible, as terbinafine is metabolized via CYP450
the minimum duration of therapy is 7 days for topical ter- and is also a CYP2D6 inhibitor. Itraconazole (labeled
binafine and 14 days for topical azoles.1.6 It is important to indication) and fluconazole (off-label use) are less effective
note that the duration of therapy depends on the type of for the treatment of onychomycosis, with cure rates around
dermatophyte (tinea) infection as well as the severity and 20%.9 Itraconazole is available as an oral capsule (two
spread of the lesion(s). Answer D is incorrect for empiric formulations) and an oral solution.6.11 The dosage form
therapy in cases that are not severe/extensive. However, must be selected carefully due to differences in absorption.
longer durations of therapy may be necessary in cases that Itraconazole can cause GI disturbances, namely diarrhea
are severe/extensive, recurrent episodes, or do not respond and nausea, particularly in the oral solution form. Wors-
to initial therapy. ening of heart failure and new-onset heart failure have
5. Explanation: The correct answer is D. Tinea corporis been reported with itraconazole use; it is contraindicated
and tinea pedis are most often diagnosed based on signs, in patients with a history of heart failure. ltraconazole is
symptoms, and physical examination. 1 Therefore, the goal/ both a substrate and an inhibitor of CYP3A4 and several
endpoint of therapy is clinical resolution (rash and symp- drug interactions are possible. Like terbinafine, hepatic
toms). Answer A is incorrect, because cultures are not transaminase monitoring is recommended at baseline and
typically obtained for these infections. Answer Bis incor- periodically during therapy with itraconazole. Fluconazole
rect because dermatophyte infections can occur in both is well tolerated with a low risk of liver toxicity.6•8 With
immunocompetent and immunocompromised patients. weekly dosing, fluconazole drug interactions are usually
The development of a dermatophyte infection does not minimal.6.B Exceptions to this include narrow therapeutic
provide information about a patient's immune function. index drugs like warfarin, phenytoin, and cyclosporine.
Answer C is incorrect because RP has no systemic signs of Systemic azole antifungals (including itraconazole and flu-
infection, only local symptoms. conazole) are contraindicated in pregnancy, and treatment
of onychomycosis in pregnancy is not recommended since
6. Explanation: The correct answers are C, D, and F. Clean- therapy can be delayed until after delivery.10 Breastfeeding
ing the affected areas gently with soap and water as well should be avoided during terbinafme therapy. Answer D is
as drying them thoroughly (Answer C) will help avoid the incorrect; the next step after topical therapy failure is oral
optimal environment for fungal growth (moist, warm).24 therapy, not intravenous therapy.
Similarly, wearing absorbent socks (Answer F) can help
keep the feet dry. Wearing footwear (eg, flip-flops) in 8. Explanation: The correct answer is D. Tinea unguium
shared showers and locker rooms (Answer D) can help ( onychomycosis) can develop as a result of untreated tinea
prevent transmission via surfaces. Answer A is incorrect pedis.1 Onychomycosis often warrants oral (systemic)
because socks should be changed at least daily. Answer B is therapy and requires treatment for 3 to 6 months. 1•9
incorrect, because shoes should be rotated daily to ensure Alternatively, some milder cases of onychomycosis may
that each pair dries completely before it is worn again. be treated with topical antifungal preparations developed
124 INFECTIOUS DISEASES: A CASE STUDY APPROACH

specifically for nail infections (ie, not the same preparations 3. Centers for Disease Control and Prevention. Ringworm
used for cutaneous fungal infections). Answers A and C risk & prevention. Available at https://www.cdc.gov/
are incorrect. While different dermatophyte infections fungal/diseases/ringworm/risk-prevention.html.
often coexist, as seen in this case, due to the proximity of 4. American Academy of Dermatology. Athlete's foot: How to
the feet and toenails, RP is at highest risk of developing prevent. Available at https://www.aad.org/public/diseases/
tinea unguium from untreated tinea pedis. Additionally, contagious-skin-diseases/athlete-s-foot-how-to-prevent.
tinea capitis most often occurs in children. Answer B is 5. American Academy of Dermatology. Ringworm: Who
incorrect because tinea versicolor (now named pityriasis gets and causes. Available at https://www.aad.org/public/
versicolor) is a superficial fungal infection that is not diseases/contagious-skin-diseases/ringworm#causes.
considered a dermatophyte infection because it is not 6. Gupta AK, Cooper EA. Update in antifungal therapy of
caused by a dermatophyte organism. 1 Dermatophytes are dermatophytosis. Mycopathologica. 2008; 166:353-367.
organisms in three genera, Trichophyton, Microsporum, 7. PL Detail-Document. Topical antifungal agents for tinea
and Epidermophyton. Pityriasis versicolor is caused by infections. Pharmacist's Letter/Prescriber's Letter. May
yeasts in the genus Malassezia. 2014.
8. PL Detail-Document. Comparison of therapies for ony-
chomycosis. Pharmacist's Letter/Prescriber's Letter. May
REFERENCES 2013.
9. Gupta AK, Versteeg SG, Shear NH. Onychomycosis in the
1. Ely JW, Rosenfeld S, Seabury Stone M. Diagnosis and
21st century: an update on diagnosis, epidemiology, and
management of tinea infections. Am Fam Physician.
treatment./ Cutan Med Surg. 2017;21:525-539.
2014;90:702-710.
10. Pilmis B, Jullien V. Sobel J, Lecuit M, Lortholary 0,
2. Likness LP. Common dermatologic infections in athletes
Charlier C. Antifungal drugs during pregnancy: an
and return-to-play guidelines. l Am Osteopath Assoc.
updated review./ Antimicrob Chemother. 2015;70:14-22.
2011;111 :373-379
Cryptococcus
30 Paul 0. Gubbins

PATIENT PRESENTATION Physical Examination


Chief Complaint ~ Yitai Signs
"My dad has been complaining of headaches and is now act- Temp 101.3°F, pulse 70, RR 20 breaths per minute, BP 130/92
ing confused:' mm Hg, p02 92%, Ht 5'9", 70 kg

~ Geneml
History of Present Illness
JK is a 68-year-old male, who was brought to the emergency Lethargic, irritable gentleman with dizziness and confusion in
department with a headache, nausea, and episodic vomit- moderate distress
ing that started about a week ago, and recent onset of diz-
~ HEENT
ziness, confusion, and irritability that was noticed today by
his son while visiting with him for breakfast. He has a fever Normocephalic, PERRI.A, EOMI, (-) papilledema, photopho-
(101.3°F) of unknown duration. "A recent 4th generation bia. or blurred vision
HIV Ag-Ab test was negative:' His son states that JK lives
alone and started complaining of not feeling well about a ~ Pulmonary
week.ago. Diminished breath sounds

Past Medical History • Cardiovascular


Type 2 diabetes, controlled with medications NSR, no mlr/g

~ Abdomen
Surgical History
Soft, non-distended, non-tender, bowel sounds hyperactive
Appendectomy 20 years ago
~ Genitourinary
Family History No complaints
Father had lung cancer and passed away 40 years ago; mother
had HTN and passed away from stroke 15 years ago; no ~ Neurology
siblings Lethargic, oriented to place and person, (-) Brudzinski's sign,
(-) Kernig's sign, (-) nuchal rigidity
Social History
~ Extremities
He is a retired salesperson, widowed (wife passed away from
metastatic breast cancer 10 years ago). who lives alone. He Normal
has one son who visits him regularly. His hobby is golf and
~ Back
tending to a pet cockatoo; never smoked, but drinks alcohol
occuionally Tenderness to palpation on lower lumbar region

Allergies Laboratory Findings


NKDA Na= 139 Hgb = 13.5 Ca=9.2mg/ CK=2.0ng/
mmol/L gldL dL mL
Home Medications K=4.2 Hct=40% Mg=2.2mg/ TBili = 1.8
Metformin (Glucophage XR•) 2000 mg PO daily with evening mmol/L dL mgldL
meal Cl= 100 Plt=227 Phos = 4.4 mg/ Chol= 190
Multivitamin 1 tablet daily mmol/L dL mgldL

125
126 INFECTIOUS DISEASES: A CASE STUDY APPROACH

C03 =27 WBC= AST=23 U/L Trigly= 144 4. How would recommended induction therapy differ from
mmol/L 13.0 x 10' mg/dL the case above if JK were a 35-year-old infected with HIVI
AIDS diagnosed with cryptococcal meningoencephalitis?
BUN = 40.0 Neut = 2596 ALT= 32U/L HDL= 54 A. The duration ofhis induction therapy would be shorter.
mg/dL mgldL B. His induction combination therapy would include flu-
SCr =0.8 Lymphs= AllcPhos = LDL= 90mg/ conazole instead of an amphotericin B formulation.
mg/dL 7096 82U/L dL C. His induction combination therapy would include an
Glu = 180 Monos= Tot Prot = 6.8 BNP= 70pg/ echinocandin instead ofan amphotericin B formulation.
mg/dL 596 g/dL mL D. He would receive fluconazole monotherapy as induc-
tion therapy instead of combination therapy.
... Fourth-Generation Ag-Ab HIV 5. Assume JK is a 35-year male with HIV/AIDS, diagnosed
Negative with cryptococcal meningoencephalitis and that he has
successfully completed his induction and consolidation
.,. CTHead periods of treatment. What should be recommended for
Unremarkable maintenance therapy?
A. Fluconazole 200 mg PO daily indefinitely
... Blood Cultures B. Itraconazole 400 mg PO daily indefinitely
C. Fluconazole 200 mg PO daily for at least a year, then
Pending
reassess based on HIV markers
D. Itraconazole 400 mg PO daily for at least a year, then
... Lumbar Puncture (LPJ/CSF Analysis
reassess based on HIV markers
CSFGlu= CSF Prot= CSFWBC= 13
6. How would recommended therapy differ from the case
35mg/dL 150mg/dL (Neut = 2096; Lymphs 6796)
above if JK were a 40-year-old organ transplant recipient
diagnosed with cryptococcal meningoencephalitis?
Crypto antigen-pending, opening pressure: 30 cmH20 ;
A. He would receive an amphotericin B lipid formulation
Gram stain-no organisms observed; CSF cultures:
instead of the deoxycholate formulation.
pending.
B. He would receive fluconazole as induction therapy
instead of an amphotericin B lipid formulation.
QUESTIONS C. The duration of induction therapy would be prolonged.
1. What sign(s) or symptom(s) is/are characteristic of crypto- D. After completing consolidation therapy, JK would
receive fluconazole 200 mg PO daily as primary
coccal meningoencephalitis in JK?
A. Nonspecific symptoms that developed over approxi- prophylactic therapy indefinitely.
mately a week 7. Which statement regarding the detection of relapsed infec-
B. The lack of an organism on CSF Gram stain tion is TRUE?
C. The presence of CSF lymphocytic pleocytosis A. Serial cryptococcal antigen testing should be performed
D. All of the above to monitor therapeutic response and detect relapsed
2. What is the recommended initial antifungal induction infection.
B. Persistence of a positive CSF India ink stain is sufficient
therapy for JK?
A. Amphotericin B deoxycholate 50 mg IV plus 5-FC to detect relapsed infection.
1,750 mg PO QID X 4 weeks C. Detecting relapse only requires evidence ofviable cryp-
tococci from a previously checked sterile site.
B. Amphotericin B deoxycholate 50 mg IV plus flucon-
azole 800 mg PO daily x 2 weeks D. In detecting relapse, clinicians must assess the patient's
C. Amphotericin B deoxycholate 50 mg IV plus 5-FC adherence with consolidation or maintenance therapy.
1,750 mg PO QID x 6 weeks
D. Amphotericin B deoxycholate 50 mg IV plus flucon- ANSWERS
azole 400 mg PO daily x 2 weeks
1. Explanation: The correct answer is D. Cryptococcal
3. How should increases in intracranial pressure be managed meningoencephalitis manifests as a subacute meningo-
in JK during induction therapy? encephalitis with nonspecific symptoms including fever,
A. Administer mannitol 20 mg as an intravenous bolus headache, lethargy, confusion, nausea, and vomiting that
dose develop progressively over 1 to 4 weeks before a patient
B. Relieve by CSF drainage via lumbar puncture seeks medical attention. Although his presentation appears
C. Administer acetazolamide 500 mg PO BID nonspecific, JK exhibits several findings including a CSF
D. Administer prednisone 40 mg PO daily, and taper to WBC <20/mm3, presenting with altered mental status,
response an opening pressure ~25 cmHp, and hypoglycorrhachia
CHAPTER30 I CRYPTOCOCCUS 127

(a low CSF glucose level), that carry a poor prognosis. patient is symptomatic, then daily lumbar puncture may
Cryptococcus possesses a capsule with immunosuppres- be performed until the CSF pressure and symptoms have
sive properties. It cannot be detected by Gram stain, but been stabilized for 12 days. In such cases, clinicians should
can be visualized with an India ink stain. The detection of consider temporary percutaneous lumbar drains or ven-
Cryptococcus spp. by India ink stain is highly specific, but triculostomy (ie, a device that drains excess CSF from the
it is associated with a low sensitivity (:S50%). Therefore, head) for patient comfort. If these measures do not control
serologic testing of blood and CSF should be done when- increased intracranial pressure, then permanent ventricu-
ever cryptococcal CNS infection is suspected. Leukocyte loperitoneal (VP) shunts should be placed.'
count in the CSF is abnormally increased (lymphocytic
pleocytosis), with a lymphocyte predominance. 4. Explanation: The correct answer is A. Given the lack of
immune function and the likelihood of a large fungal bur-
2. Explanation: The correct answer is C. The correct dose for den in individuals with HIV infection, antifungal therapy
flucytosine (5-FC) is 100 mg/kg/d orally in four divided should rapidly and consistently sterilize the CNS through
doses. Experts recommend at least 4 weeks for induction rapid fungicidal activity. A sterile CSF culture 2 weeks after
therapy for cases like JK.1 However, the 4-week induction initiating therapy indicates a successful fungicidal induc-
therapy (ie, initial therapy to reduce fungal burden) is tion regimen and is associated with favorable outcomes.1
reserved for cases with meningoencephalitis without neu- Several trials demonstrate the combination of ampho-
rological complications and cerebrospinal fluid ( CSF) yeast tericin B formulations with 5-FC for 2 weeks of induction
culture results that are negative after 2 weeks of treatment.1 therapy produced the most rapid fungicidal effects and
JK has moderate evidence of neurological complications cleared the CSF significantly faster than other regimens,
(eg, recent onset of dizziness, confusion, lethargy, and irri- including amphotericin B deoxycholate monotherapy, or
tability) so the initial plan should be for 6 weeks of induc- amphotericin B deoxycholate combined with fluconazole
tion therapy. A reasonable approach in this case would be or therapy with all three agents together. Fluconazole is
to reassess after 2 weeks of treatment by examining CSF not an option for primary therapy due to a lack of fungi-
yeast cultures and assessing clinical response (ie, improve- cidal activity. Data demonstrate higher early mortality and
ment in symptoms, and laboratory findings, normal- reduced ability to clear the CSF rapidly with fluconazole
izing intracranial pressures). If cultures are negative and alone. The echinocandins lack activity against Cryptococ-
response excellent, clinicians in this case may opt to initiate cus neoformans.
consolidation therapy (ie, clearance of infected space) with
fluconazole, while others may continue induction therapy 5. Explanation: Correct answer is C. Comparative trials
for another 2weeks, shortening it from the initial plan of clearly demonstrate that fluconazole is the most effective
6 weeks to a total of 4 weeks. Currently all induction regi- maintenance therapy.1 In a clinical trial comparing fluco-
mens consist of 5-FC. If 5-FC cannot be administered (eg, nazole to itraconazole as maintenance therapy, fluconazole
significant myelosuppression, liver failure, renal failure, was proven superior. The trial was terminated prematurely
history of hypersensitivity), then induction therapy with after interim analysis revealed that significantly more
amphotericin B monotherapy for a minimum of 6 weeks is itraconazole-treated patients had a relapse, compared to
warranted. 1 If JK had been AmBd intolerant in the past or the fluconazole-treated patients.1 Historically, life-long
had renal impairment, a lipid AmB formulation could have maintenance therapy to prevent disease relapse was rec-
been used. ommended for all patients with AIDS after successful
completion of primary induction therapy for cryptococcal
3. Explanation: The correct answer is B. Control of CSF pres- meningoencephalitis. However, more contemporary data
sure is a critical determinant of outcome for cryptococ- indicate the risk of relapse is low in patients meeting the
cal meningoencephalitis. Opening pressures in excess of following criteria: (a) They have successfully completed
25 cmHp suggest a high fungal burden in the CSP and their primary therapy (including at least a year of main-
carry a poor prognosis. Thus, they must be addressed tenance therapy). (b) They remain asymptomatic and free
promptly. Mannitol has no proven benefit and should not of any signs of active cryptococcosis. (c) They are receiv-
be recommended.1 Prednisone could be used to treat IRIS, ing highly active effective antiretroviral therapy, and their
which may occur in HIV-infected patients receiving highly response to that has been stable [ie, a sustained CD4 cell
active antiretroviral, but it has no role in treating increases count > 100 cells/mL and an undetectable viral load (HIV
in intracranial pressure in the absence of such symptoms. RNA) for at least 3 months].
Similarly acetazolamide should be avoided as well. 1 Data
in HIV-negative patients are sparse, but if the CSF pres- 6. Explanation: The correct answer is A. In addition to CNS
sure is ~25 cmHp and the patient is symptomatic, cryptococcosis, organ transplant recipients can develop
CSF drainage via lumbar puncture should be performed pulmonary or disseminated infections. Regardless of the
to reduce the opening pressure by at least 50% if it is type of solid organ transplant, the immunosuppressive
extremely high or to a normal pressure :S20. 1 If elevations regimen following transplantation typically consists of a
in intracranial pressure persist (ie, ~25 cmHp), and the calcineurin inhibitor and approximately 25% of transplant
128 INFECTIOUS DISEASES: A CASE STUDY APPROACH

recipients have renal dysfunction when diagnosed. Thus, rising CSF titer several months after initial diagnosis should
amphotericin B deoxycholate is not recommended as raise clinical suspicion enough to warrant obtaining a culture.
first-line therapy in this patient population. Fluconazole Similarly, India ink stain by itself is not sufficient for deter-
is not recommended for induction therapy. The lipid mining relapse, as the presence of lingering nonviable yeast
formulations are less potent on a weight basis, but are in CSF is common. Positive India ink stains may be observed
equally efficacious to the deoxycholate formulation when for up to a year following diagnosis despite treatment Positive
administered in recommended doses. Although most CSF culture for C. neofonnans is diagnostic for microbiologic
cases of cryptococcal disease in transplant recipients rep- relapse or treatment failure. In order to detect relapse there
resent reactivation of an existing subclinical infection, must also be re-emergence of signs and symptoms at that
such infections are difficult to detect and progress to infection site. In a relapsed infection, cultures and symptoms
disease in an unpredictable manner. Thus, routine pro- have normalized and then recurred Most cases of relapse are
phylaxis for cryptococcosis in transplant recipients is not due to inadequate primary therapy (dose and/or duration) or
recommended. failure of compliance with consolidation or maintenance of
fluconazole dose.
7. Explanation: The correct answer is D. Serum and CSF
cryptococcal antigen titers are important in establishing the
presumptive diagnosis and assessing the prognosis of infec-
tion. The test measures cryptococcal polysaccharide capsule
REFERENCE
antigens, thus it does not differentiate viable from nonviable 1. Perfect JR, Dismukes WE, Dromer F, et al. Clinical practice
organism. CSF antigen titers are not precise indicators for early guidelines for the management of cryptococcal disease: 2010
relapse or for making therapeutic decisions. Although serial update by the Infectious Diseases Society of America. Clin
antigen determinations are not useful, a single unchanged or Infect Dis. 2010;50:291-322.
Aspergil losis
31 Ashley H. Marx

PATIENT PRESENTATION Home Medications


Chief Complaint Cyclosporine (NeoraJ•) 75 mg PO each morning, SO mg PO
each evening
"I am coughing and very short of breath." Mycophenolate mofetil 500 mg PO twice daily
Prednisone 5 mg PO daily
History of Present Illness Cholecalclferol (Vitamin D3) 1,000 units PO twice daily
UL is a 47-year-old Caucasian male who presents to the emer- Sodiwn bicarbonate 650 mg PO twice daily
gency department via EMS with complaints of recent onset Furosemide 20 mg daily as needed for swelling
of fevers, shortness of breath, cough, and pleuritic cllest pain.
The patient reports that his symptoms began 3 days ago when Physical Examination
he woke up in the middle of the night with shortness of breath
and noticed sharp right-sided chest pain. The chest pain was ..,. Vital Signs
provoked primarily by deep inspiration, and he was not able Temp 102.2°F, P 110, RR 24, BP 136/78 mm Hg, p02 95%, Ht
to go back to sleep. Last night. he began developing worsening 51611, Wt 62.4 kg
chills, and also began to have wheezing and progressive short-
ness of breath. He also developed a productive cough, bring- ..,. General
ing up thick sputum eventually culminating in an episode of Middle-aged. thin gentleman in no acute distress
dry heaving and emesis. He states his emesis was originally
dark blood. and toward the end of throwing up had some ..,. HEENT
flecks of bright blood mi.ud in. Anicteric sclera, pale conjunctiva, erythematous posterior
When EMS arrived at his home, UL was hypoxic (70% 0 2 oropharynx. Neck supple without lymphadenopathy
sat), tachypnic (RR 30), and tachycardic (HR llOs). He was
administered nebul.ized albuterol and ipratropium and 1 L ..,. Pulmonary
normal saline on the way to the hospital.
Inspiratory rales most prominent anterolaterally on both
sides, few rales posteriorly
Past Medlcal History
Lupus erythematosus (inactive since 1992), coronary artery dis- ..,. Cardiovascular
eue. deceased donor renal transplant 1994 secondary to lupus NSR, no mlr/g
nephritis. CKD 4 (GFR 15-29); baseline SCR 2.5-3 mgldL
..,. Abdomen
Surgical History Soft, non-distended, non-tender, bowel sounds present
Renal transplant 1994
..,. Bade
No tenderness over renal allograft
Family History
Father has HTN and type 2 DM; mother alive and well ..,. Genitourinary
Deferred
Soclal History
Married and lives with wife. No history of smoking or illicit ..,. Neurology
drug use; drinks alcohol occasionally AOX4

Allergies ..,. Extremities


NKDA Normal, no lesions or edema

129
130 INFECTIOUS DISEASES: A CASE STUDY APPROACH

Laboratory Findings response and therapeutic drug monitoring will be guided by


current national guidelines.1
Na= Ca= K= Mg=
136 mEq/L 8.4 mg/dL 4.1 mEq/L 1.6 mgldL
QUESTIONS
Cl= Phos = co 2
= AST=
1. Which risk factor for invasive pulmonary aspergillosis
107 mEq/L 3.3 mg/dL 21 mEq/L 23 IU/L
(IPA) is present in UL?
BUN= ALT= SCr = 2.46 T Bill= A. High dose steroids for > 1 month
45 mgldL 32 IU/L mgldL 1.8 mgldL B. Solid organ transplantation
Glu= AlkPhos = WBC=S.2 x C. Stem cell transplantation
88 mg/dL 130 IU/L 103 nun3 D. Environmental exposure from construction sites
RBC = 2.19 x 2. Voriconazole was selected for therapy of invasive pulmo-
103 nun3 nary aspergillosis. Voriconazole is likely to pose a signifi-
Hgb= HCT= Pit= 33 x 103 cant drug interaction with which of his home medications?
6.7 g/dL 20.0% mm3 A. Cyclosporine
B. Mycophenolate mofetil
~ Rapid Antigen Test C. Furosemide
D. Cholecakiferol
Influenza negative; RSV negative
3. Assuming the patient clinically responds to therapy, what
~ Legionella Urinary Antigen Test is the most appropriate minimum duration of therapy for
treatment of IPA recommended in IDSA guidelines?
Negative
A. 1 to 2 weeks
~ Chest X-ray B. 2 to 4 weeks
C. 4 to 6 weeks
Multifocal infiltrates, predominantly in the upper lobes. D. 6 to 12 weeks
Bilateral pleural effusions
4. Which therapy is considered an alternative first-line therapy
~ CT of the Chest to voriconazole for the treatment of invasive aspergillosis?
A. Caspofungin
Upper lobe predominant, groundglass and consolidative alve-
B. Itraconazole
olar opacities bilaterally, left greater than right. Small bilateral
C. Isavuconazole
pleural effusions
D. Amphotericin B
~ Initial Management 5. For which antifungal(s) should therapeutic drug monitor-
UL is administered 1 unit PRBCs, lactated ringer's fluid ing be recommended?
500 mL bolus x 1, then 100 mL/hr x 10 hours. Community- A. Itraconazole
acquired pneumonia therapy was initiated with ceftriaxone B. Voriconazole
1 g IV plus azithromycin 500 mg IV daily. Based on UCs status C. Isavuconazole
as an immunocompromised host, a bronchoscopy was sched- D. Both A and B
uled. Specimens from bronchoalveolar lavage (BAL) were sent 6. You are reviewing respiratory sputum culture results of a
for culture, galactomannan, Mycoplasma pneumoniae PCR, patient with cystic fibrosis who is currently experiencing an
and direct fluorescent antibody (DFA) for P. jirovecii. exacerbation. The most recent results are: 3+ methicillin-
susceptible S. aureus, 3+ Pseudomonas aruginosa, and 1
~ Bronchoscopy Results colony of Aspergillus fumigatus. Which represents the best
BAL gram stain: 1+ polymorphonuclear cells; no organisms assessment and plan for this patient?
BAL culture: no growth A. Invasive pulmonary aspergillosis; initiate voriconazole
BAL galactomannan: 1.2 (normal <0.5) B. Chronic necrotizing aspergillosis; initiate isavuconazole
DFA: negative for P. jirovecii C. Allergic broncho-pulmonary aspergillosis; initiate
M. pneumoniae PCR: negative itraconazole
D. Possible Aspergillus colonization; do not initiate therapy
Based on U:Ls compatible signs and symptoms, CT find-
ings, and galactomannan result, he was diagnosed with prob- 7. Which statement regarding antifungal susceptibility testing
able invasive pulmonary aspergillosis. The decision was made is true?
to stop antibacterial therapy and initiate voriconazole 6 mg/kg A. Susceptibility testing is readily available and standard-
IV ql2h x 2 doses, followed by 4 mg/kg IV ql2h. His myco- ized in the same way as bacterial testing
phenolate will be stopped to lighten his immune suppression B. Susceptibility testing is available, but standardized
during this period of active infection. Assessment of clinical interpretations are generally not
CHAPTER 31 I ASPERGILLOSIS 131

C. Susceptibility testing and interpretation is available therapies. Itraconazole is effective for prophylaxis and treat-
only at reference labs ment of less severe forms of aspergillosis such as brocho-
D. Susceptibility testing is not needed as resistance to anti- pulmonary aspergillosis; however, it is not appropriate for
fungal drugs has not been described invasive forms of aspergillosis.
8. Which are common adverse effects associated with vori- 5. Explanation: The correct answer is D. Both itraconazole
conazole therapy? and voriconazole exhibit significant intra-patient vari-
A. Visual disturbances and hepatotoxicity ability in serum concentrations, due to absorption (for
B. Neutropenia and nephrotoxicity itraoonazole) and metabolism (for voriconazole). The
C. Hypokalemia and hypomagnesemia recommended timing for concentration monitoring is
D. Infusion-related reactions of fever and rigors within 60 minutes prior to the next dose (trough), usu-
ally after steady-state conditions have been reached.
Therapeutic concentration ranges have been established
ANSWERS for efficacy of itraconazole and voriconazole for the pre-
1. Explanation: The correct answer is B, solid organ trans- vention and treatment of fungal infections; voriconazole
plantation. Although UL has been on chronic steroids toxicity is more often observed among patients with
(which is an additional risk factor), his dose is not "high" supratherapeutic concentrations. Relationships between
or supra-physiologic at 5 mg daily. All listed choices are isavuconazole concentrations and efficacy or toxicity have
risk factors for the development of invasive pulmonary not been established, and therefore concentration moni-
aspergillosis. Additional risk factors for IPA include toring is not recommended.
chronic immune suppression (present in UL), hemato-
6. Explanation: The correct answer is D. Colonization of
logic malignancy, prolonged neutropenia, critical illness,
airways with environmental molds like Aspergillus is
hemodialysis, liver disease, COPD and other chronic lung
common, especially among patients with cystic fibrosis
diseases, and chronic granulomatous disease.
who cannot effectively clear respiratory secretions. The
2. Explanation: The correct answer is A, cyclosporine. low quantity of mold in relation to high burdens of other
Voriconazole is metabolized by CYP4502C19, 2C9, and common pathogens associated with cystic fibrosis exac-
3A4, and exhibits complex, dose-dependent drug interac- erbations also support possible colonization. Choices A,
tions. Voriconazole is also a potent inhibitor of CYP450 B, and C represent diagnoses on the spectrum of diseases
3A4, which is involved in the metabolism of cyclosporine. caused by Aspergillus, but signs and symptoms, and addi-
Empirical reductions in cyclosporine doses of about 50% tional diagnostic tests would be required to establish one
are recommended when voriconazole is initiated; this of these diseases.
dose reduction should be recommended for UL Sig-
7. Explanation: The correct answer is B. Antifungal sus-
nificant drug interactions would not be predicted with
ceptibility testing methods are described and must be
furosemide, cholecalciferol, or mycophenolate, as none of
validated according to the Clinical Laboratory Standards
these agents is metabolized by the CYP450 system.
Institute in the United States; other regulatory bodies
3. Explanation: The correct answer is D. The minimum govern this process at international sites. The interpreta-
duration of therapy recommended for IPA is 6 to 12 tion of susceptibility results is not as well-established as
weeks. In order to discontinue therapy, the patient should for bacterial pathogens, and breakpoints for susceptibility
demonstrate good clinical response based on symptoms have been determined for relatively few pairs of fungal
and radiographic improvement. Other durations listed pathogens and drugs. Consultation with a microbiologist,
are too short for invasive infection with Aspergillus. For infectious diseases specialist, and pharmacist is recom-
UL, because he has an indication for on-going immune mended when selecting therapy for clinical situations
suppression, his course may be extended, or he may when breakpoints do not exist
receive prophylaxis after active therapy. Another situa-
tion when prolonged therapy would be indicated would 8. Explanation: The correct answer is A. Voriconazole
causes visual disturbances, described as blurry vision
include dissemination to the central nervous system.
or changes in color perception in up to 20% of patients.
4. Explanation: The correct answer is C, isavuconazole. Isa- Up to 5% of patients experience hepatotoxicity. Choice
vuconazole was found to be non-inferior to voriconazole B describes toxicities of flucytosine; choices C and D
for the treatment ofIPA. Amphotericin B is effective for the describe adverse effects of amphotericin B.
treatment of invasive aspergillosis, but is associated with
significantly more adverse effects than azole therapy; further,
amphotericin B formulations are associated with greater REFERENCE
patient mortality due to IPA compared to voriconazole. As
1. Patterson TF, Thompson GRIII, Denning DW, et al.
such, amphotericin B is considered a second-line therapy.
Practice guidelines for the diagnosis and management of
Caspofungin, and other echinocandins, are used as salvage
aspergillosis: 2016 update by the Infectious Diseases Soci-
therapy (alone or in combination with other agents) for
ety of America. Clin Inf Dis. 2016;63(4):el-60.
patients intolerant to or who have not responded to first-line
This page intentionally left blank
32 Protozoans
Lisa Avery

PATIENT PRESENTATION .- General


Well-nourished, well-developed, patient appearing stated age
Chief Complaint
in no acute distress
Fever for almost a week
.- Eyn
History of Present Illness
Pupils equally round and reactive to light, no conjunctiva!
AB is a 58-year-old male with. history oflatent TB (s/p INH x injectionlscleral icterus, extraocular motions intact
9 months), recent month-long trip to Kenya, returned 1 week
ago and developed fevers, malaise, body aches, abdominal
pain, and headaches. He received multiple mosquito bites .- Eats/Nosenhroat/Mouth
while in Kenya. and did not take malarial prophylaxis. His Atraumatic external nose and ears. Moist mucous mem-
fevers occur every 24 to 48 hours. branes; no pharyngeal erythema, no evidence of any
acute otitis media or external canal infection/mastoiditis
Past Medical History bilaterally. No nasal congestion noted. Neck: Supple and
non-tender. Trachea midline. No anterior cervical lymph-
Hypertension, hyperlipidemia, diabetes, depression, latent adenopathy. No JVD
tuberculosis: s/p INH x 9 months
.- Cardlovasculal'
Surgical History
Sinus tach.ycardia. no murmurs or gallops. Peripheral pulses
None 2+ and equal in all extremities

Family History .- Pulmonary


Father alive with type 2 diabetes, mother died at 72 of MI Unlabored respiratory effort. No wheezing. rhoncbi, or rales
noted. No chest wall tenderness noted
Social History
.- Abdomen
Married with 2 adult children, nonsmoker, drinks occasionally
Abdomen is soft with nonspec:ifi.c abdominal pain, is protuber-
ant and not distended No masses or hernias appreciated. No
Allergies rebound, guarding, or peritoneal signs noted. Splenomegaly
NKDA present

Home Medications .- Musculoskeletal


Citalopram 20 mg PO daily Extremities without deformity, edema or tenderness to
Docusate 100 mg PO BID palpation
Gabapentin 100 mg PO TID
Ibuprofen 400 mg PO q8h PRN fever .- Skin
Lisinopril 20 mg PO daily Warm, dry; no rashes or lesions
Acetaminophen 650 mg PO q6h
.- Neurologkal
Physical Examination
Normal gait and balance; sensation grossly intact
.- Vital Signs
Temp 103.3°F (oral), pulse 114 bpm, RR 18 breaths per min- .- Psychlattlc
ute, BP 112/80 mm Hg (sitting position), Sp01 97%, Ht 5'10", Awake, alert, and oriented to date and time and person; appro-
Wt87kg priate mood and affect

133
134 INFECTIOUS DISEASES: A CASE STUDY APPROACH

Laboratory Findings B. This patient has uncomplicated disease and should be


treated with oral therapy
Na = 134 mEq/L Hgb = 12.6 g/dL Ca = 8.4 mgldL
C. This patient has severe disease and should be treated
K = 3.9 mEq/L Hct = 46% Mg = 2.2 mgldL with intravenous therapy
Cl= 98 mEq/L Plt = 25 x 10 /mm3 Phos = 4.6 mg/dL
3 D. The treatment is the same and does not depend on the
severity of illness.
C02 = 24 mEq/L WBC = 2.5 x 103 AST= 51 IU/L
mm3 4. Which of the following statement is TRUE regarding Plas-
= 109 IU/L modium falciparum?
BUN = 16 mg/dL MCV = 81.6 L/cell ALT
A. It causes a rapidly progressive disease
SCr = 0.86 mg/dL MCH = 27 g/cell T Bili = 2.6 mg/ B. The dormant form remains in the liver and causes
dL relapsing disease
Glu = 364 mgldL MCHC = 33 g/L Allc Phos = 223 C. There is little reported resistance to commonly pre-
IU/L scribed medications
AlC = 14.3% PT= 10.3 Albumin = 2.5 D. It is not detected with rapid malaria diagnostic tests
g/dL 5. AB recently traveled to Kenya where there is chloroquine
Lactate = 2.13 aPTT = 28.5 resistance. Based on his disease severity, species, and
Lipase= 226 D-dimer = negative potential resistance what would be the most appropriate
therapy?
~ Chest X-ray A. Atovaquone-proguanil (250 mg/100 mg) 4 tablets PO
daily x 3 days + primaquine phosphate 30 mg base PO
No acute disease daily x 14 days
B. Hydroxychloroquine 620 mg base PO x 1, then 310 mg
~ Microscopic Examination of Blood base PO at 6, 24, and 48 hours. Total dose = 1550 mg
Peripheral smear under microscopy shows ring forms base
C. Artemether-lumefantrine (20 mg/120 mg) 4 tablets,
~ Blood Parasite repeat 8 hours later, then twice daily x 2 days
Plasmodium falciparum parasitemia 2.5% D. Atovaquone-proguanil 250 mg/100 mg 4 tablets PO
daily x 3 days
~ HIVTest 6. AB's 22-week pregnant daughter is planning a trip to Kenya
Nonreactive in 2 weeks. What would be the most appropriate malaria
chemoprophylaxis regimen?
~ EKG A. Atovaquone-proguanil 1 tablet daily, start 1 to 2 days
QTc = 492ms before traveling
B. Doxycycline 100 mg PO daily, start 1 to 2 days before
traveling
QUESTIONS C. Mefloquine 228 mg base weekly, start 1 to 2 weeks
before traveling
1. Which clinical symptoms are consistent with the diagnosis D. Tafenoquine 200 mg PO weekly, start 3 days before
of malaria? traveling
A. Fever
7. In addition to chemoprophylaxis, what nonpharmacologic
B. Muscle aches
measures can be employed to decrease AB's daughter's risk
C. Abdominal pain
of malaria transmission?
D. All of the above
A. Pure oil of lemon eucalyptus on skin not covered by
2. Which of the following laboratory tests are consistent with clothing
the diagnosis of malaria? B. DEET on skin not covered by clothing
A. Leukopenia, hyperglycemia, and hypokalemia C. Permethrin directly on skin
B. Elevated hepatic enzymes, thrombocytopenia, and D. Since she is pregnant she should NOT use any insect
anemia repellants
C. Macrocytic anemia
D. Elevated bicarbonate and elevated lactate
3. Which of the following statements are true regarding this
ANSWERS
patient's disease severity? 1. .Explanation: The correct answer is D. Adult patients
A. This patient is asymptomatic and does not require any who present with malaria may present with nonspe-
treatment cific symptoms, which include fever, muscle aches, and
CHAPTER 32 I PROTOZOANS 135

abdominal pain (nausea, vomiting, and diarrhea). These 4. Explanation: The correct answer is A. Both P. falciparum
symptoms are characteristic of uncomplicated diseases.1 and P. knowles are associated with rapidly progressive,
Severe malaria or complicated disease occurs when the severe disease if not treated promptly. P vivax and P.
diseases progress to involve organ dysfunction including ovale have intrahepatic forms that may remain dormant
neurologic symptoms (dizziness, confusion, disorienta- as hypnozoites anywhere from 2 weeks to greater than 1
tion, seizures, and coma), acute respiratory distress syn- year and may cause relapses. It is important to check the
drome, and renal dysfunction, which can be permanent. resistance patterns for P. falciparum and P. vivax since
The typical incubation period for P. falciparum is 10 to there are different patterns of resistance depending on
14 days, which coincides with this patient's return from the geographically region where the patient was infected.
Kenya 1 week ago. The fever associated with malaria Current rapid diagnostic testing (Binax Now) does detect
typically includes high fever, shaking chills, and sweats P. falciparum since it detects histidine-rich protein-2
(paroxysm) and can every 24 to 72 hours be dependent on antigen that is specific to P. falciparum in addition to a
the parasite species. These fever spikes correspond to the common malaria antigen.
burst of red blood cells and the release of parasites into
5. Explanation: The correct answer is D. This patient has
the bloodstream.
uncomplicated malaria with P. falciparum in an area
2. Explanation: The correct answer is B. In most cases with chloroquine resistance. The CDC malaria guidelines
hepatic enzymes will be elevated with thrombocytopenia recommend atovaquone-proguanil, arthemether-lumefan-
common in uncomplicated malaria. Patients with a nor- trine oral, quinine sulfate in combination with doxycycline,
mal platelet count should be evaluated for an alternative tetracycline, or clindamycin, or mefloquine monotherapy
diagnosis. Patients may also experience a normochromic, as treatment options for this patient.2 Artemether-lume-
normocytic anemia as seen in this patient. This is second- fantrine is considered first-line therapy per the 2015 WHO
ary to red blood cell hemolysis in addition to decreased guidelines, but would not be an appropriate choice in this
production of erythropoietin. Patients with severe disease patient since it is associated with QTc prolongation and the
may experience acidosis with a compensatory decrease in patient has an elevated QTc, electrolyte abnormalities, and
bicarbonate. Hypoglycemia may occur, although this is is on concomitant citalopram.3 Quinine is also associated
more common in children. with QTc prolongation. Mefloquine would also not be an
optimal choice, since this patient has active depression.
3. Explanation: The correct answer is B. It is important to
Mefloquine can cause severe anxiety. paranoia, hallucina-
classify the severity of illness to help determine whether
tions, depression, restlessness, and unusual behavior. Ato-
intravenous or oral therapy is necessary. If a patient has
vaquone-proguanil would be the correct answer and needs
one or more of the following criteria, he/she would have
to be taken with food or whole milk to increase absorp-
severe disease and would require hospitalization and
tion and decrease adverse reactions. The most common
intravenous therapy. Severe malaria is a medical emer-
adverse reactions are stomach pain, nausea, vomiting, and
gency and requires prompt and aggressive treatment.2
headache. If the patient vomits within 30 minutes of the
• Impaired consciousness or coma dose, repeat dose. The addition of primaquine phosphate
• Severe normocytic anemia (hemoglobin <7 gldL) is not necessary for this patient; since it treats the dormant
hypnozoites in the liver that occurs with P vivax or P. ovale
• Renal failure
infections. Hydroxychloroquine is used only in the treat-
• Acute respiratory distress syndrome (ARDS) ment of chloroquine-sensitive isolates.
• Hypotension
6. Explanation: The correct answer is C. Mefloquine can be
• Disseminated intravascular coagulation (DIC) used during pregnancy and should be started 1 to 2 weeks
• Spontaneous bleeding before departure and continued for 4 weeks after leaving
• Acidosis Kenya. None of the other choices can be used during
pregnancy. AB should contact her physician if she experi-
• Hemoglobinuria
ences signs of severe anxiety, feelings of mistrust toward
• Jaundice others, seeing or hearing things that are not present,
• Repeated convulsions depression, restlessness, or confusion. The most common
• Parasitemia ~5% side effects include nausea, vomiting, diarrhea, abdomi-
nal pain, dizziness, headache, and sleeping problems.
This patient does not meet any of the above criteria for
severe disease and is diagnosed with uncomplicated dis- 7. Explanation: The correct answer is B. Pregnant women
ease. He should receive oral therapy. The clinical symp- can use insect repellants approved by the Environmental
toms associated with uncomplicated disease are generally Protection Agency (EPA). DEET has been approved. The
nonspecific and may be confused with the common cold. CDC recommends DEET concentrations greater than or
That is why an accurate travel history is important in equal to 20%.4 If the concentration is less than 10%, there
nonendemic regions. is only 1 to 2 hours of protection. Concentrations greater
136 INFECTIOUS DISEASES: A CASE STUDY APPROACH

than 50% provide no further benefit. Pure oil of lemon 2. Centers for Disease Control and Prevention. Guidelines
eucalyptus has not been tested by the EPA and should not for treatment of malaria in the United States. Available at
be used. Permethrin should not be used directly on the https://www.cdc.gov/malaria/resources/pdf/Treatment-
skin, it should be sprayed on clothing as another line of Table_2018.pdf. Accessed September 26.
defense against mosquitoes.3 Counseling points for DEET 3. World Health Organization. Guidelines for the Treat-
application include the following: apply repellent only to ment of Malaria. 3rd ed. Available at https://www.who.
exposed skin or clothing, do not apply under-clothing, do int/malaria/publications/atoz/9789241549127/en/. April
not use on open skin, do not apply directly on face, wash 2015. Accessed September 26.
hands after application, heavy application of repellent 4. Centers for Disease Control and Prevention. Insect repel-
does not improve efficacy, and after returning indoors lents help prevent malaria and other diseases spread by mos-
wash skin thoroughly with soap and water. quitoes. Available at https://www.cdc.gov/malaria/resources/
pdf/fsp/repellents_2015.pdf. Accessed September 26.

REFERENCES
1. Centers for Disease Control and Prevention. Malaria.
Available at https://www.cdc.gov/parasites/malaria/index.
html. Accessed September 26.
33 Nematodes
Jessica Robinson

PATIENT PRESENTATION ... General


Energetic, talkative 5-year old in no obvious distress
Chief Complaint
Restlessness at night """ HEENT
Normocephalic, atraumatic, PEERLA, good dentition
History of Present Illness
LT is a 5-year-old male who presents to your clinic today for ... Pulmonary
a wellness che<:k. His mother states that over the last week Lungs clear. no abnormal breath sounds
he has been restless while sleeping. Otherwise, LT is healthy.
While giving him a bath. she has noted mild perianal irrita- ... Cardiovascular
tion but no change in bowel frequency or diarrhea. LT has
complained of being "itchy1" at night on several oc:casions, but NSR. no m/r/g
has no other complaint. He has no recent history of travel or
... Abdomen
walking barefoot, and the family currently does not own pets.
There has been no change in laundry detergent or introduc- Soft, non-distended, non-tender, bowel sounds normal
tion of new fabrics or soaps. LT's mother is not overly con-
cerned but does want to ensure nothing is wrong. ... Genitourinary
Normal male genitalia, scratch marks present near rectum
Past Medical History
Seasonal allergic rhinitis ... Neurology
Oriented to person, place, time
Surgical History
'fympanostomy tube placement ... ExtremltleJ
Range of motion intact, no evidence of skin rashes
Family History
Mother has depression; father has type 1 DM; sister (age 3) Laboratory Findings
is healthy
Na: 138 mEq/L Hgb: 14.5 g/dL
Social History K.: 4.1 mEq/L Hct: 37%
Mother reports he eats a balanced diet and is active throughout Cl: 105 mEq/L Plt 200 x 103/mm3
the day. co2: 25 mEq/L WBC: 7.2 x 103/mm3
Glucose: 68 mgldL
Allergies
SCr: 0.6 mg/dL
NKDA
BUN: 10 mg/dL
Home Medications
Loratadine 5 mg PO daily QUESTIONS
Physical Examination 1. What is the lilc:cly cause of LT's infection?
A. Ascariasis
... Vital Signs B. Strongyloidias.is
Temp 98.1°F, P 95 bpm, RR22 breaths per minute, BP 100/60 mm C. Hookworm disease
Hg. Ht 3'2", Wt 19 kg D. Enterobiasis

137
138 INFECTIOUS DISEASES: A CASE STUDY APPROACH

2. What diagnostic testing and/or imaging will be the most Ascariasis, strongyloidiasis, and hookworm disease are
helpful in confirming this diagnosis? often asymptomatic. Ascariasis does have the potential
A. Microscopic examination of stool to cause gastrointestinal or pulmonary symptoms. Stron-
B. Cellulose acetate tape test gyloidiasis may cause abdominal pain similar to that of a
C. Abdominal X-ray peptic ulcer, but can disseminate to other organ systems.
D. Abdominal CT Hookworm may also cause gastrointestinal symptoms,
and chronic infection may cause iron deficiency anemia.2
3. What is the most appropriate initial drug therapy for LT?
A. Albendazole 2. .Explanation: The correct answer is B. While microscopic
B. Itraconazole examination of stool is the preferred diagnostic method
C. Praziquantel for other infections caused by nematodes, pinworms are
D. Metronidazole not released in the feces. Therefore, cellulose acetate tape
must be applied to the rectum and examined microscopi-
4. What special instructions should be given when prescribing
cally to detect the presence of eggs. Abdominal images
albendazole for pinworm infection?
have no role in diagnosis.2
A. Take it with an acidic beverage to facilitate absorption.
B. Recommend a dose now, and a second dose 2 weeks later. 3. .Explanation: The correct answer is A. Three treatment
C. This mediation cannot be crushed. options currently exist for enterobiasis: albendazole,
D. Counsel on the severe gastrointestinal side effects often mebendazole, and pyrantel parnoate. 1.2 All are given ini-
seen with this medication. tially as one-time doses. Albendazole (dosed as 200 mg
PO x 1 in children <2 years, 400 mg PO x 1 in children
5. LT and his sister co-sleep several nights a week. She is cur-
>2 years) and mebendazole (100 mg PO x 1) are avail-
rently not exhibiting any signs or symptoms of illness, but
able only via prescription (mebendazole is only available
her mother is concerned she could also be infected. What
from compounding pharmacies in the United States),
is the most appropriate recommendation?
while pyrantel parnoate (11 mg/kg PO X 1) is available
A. Do not treat her at this time, pinworm infection is gen-
over the counter.3 Itraconazole, praziquantel, and metro-
erally not contagious.
nidazole have no role in therapy for pinworm infection.
B. Do not treat her at this time, you only treat patients if
they are symptomatic. 4. Explanation: The correct answer is B. Albendazole
C. Treat her today with albendazole, pinworm infections should be given as a one-time dose now, and then again
are highly contagious and it is generally advisable to 2 weeks later due to concern for reinfection, as albendazole
treat all members of the household. does not reliably kill pinworm eggs and a second dose
D. Treat her today with albendazole, pinworm infections can kill any adult worms that have hatched after the ini-
may be contagious, but because she co-sleeps with LT tial treatment. Albendazole can be crushed if the child is
she is likely infected. unable to swallow pills.3 It is generally very well tolerated.

6. LT's mother is currently pregnant with her third child (2nd 5. .Explanation: The correct answer is C. It is generally
trimester). She also reports no current symptoms of infec- recommended to treat all household members if one has
tion. Would you recommend treatment for her? been diagnosed with a pinworm infection. While you may
A. Yes, recommend treatment with albendazole. withhold treatment in certain situations, most health care
B. Yes, recommend treatment with mebendazole. professionals will treat the entire household at one time
C. Yes, recommend treatment with pyrantel pamoate. to prevent reinfection.1 Medications for the treatment of
D. No, do not recommend treatment at this time. pinworm are generally safe and well-tolerated.

7. Which of the following environmental precautions should 6. Explanation: The correct answer is D. All three agents are
not be recommended? pregnancy category C. As per CDC recommendations, a
A. Shower every night to help prevent the spread of pin- woman may be treated in the third trimester if the infection
worm and to prevent reinfection. is complicating the pregnancy. Otherwise, these medications
B. Follow good hand hygiene practices, especially after should be avoided due to lack of safety data in pregnant
using the bathroom. patients.1
C. Avoid biting fingernails and be sure to cut fingernails 7. Explanation: The correct answer is A. Once ingested, pin-
regularly. worms migrate to the colon and lay eggs around the anus
D. Wash all clothes and bedding each morning in hot water. at night. Therefore, it is best to take a shower each morning
to wash away any eggs that may be present. All clothing and
ANSWERS bedding should also be washed in the morning. Because
perianal irritation is common, eggs may be present under
1. .Explanation: The correct answer is D. The most com- the fingernails due to scratching. Therefore, it is important
mon manifestation of enterobiasis, or pinworm, infection to not bite fingernails to prevent any possible reinfection.
is perianal irritation. 1.2 This is often the only symptom. Appropriate hand hygiene is always recommended. 1
CHAPTER 33 I NEMATODES 139

REFERENCES Loscalzo J, eds. Harrisons Principles of Internal Medicine,


20 ed. New York, NY: McGraw-Hill. Available at http://
1. Centers for Disease Control and Prevention. Enterobia-
accessmedicine.rnhmedical.com/content.aspx?bookid=21
sis. Available at https://www.cdc.gov/parasites/pinworm/.
29&sectionid=192027581. Accessed September 27, 2019.
Accessed September 27, 2019.
3. Lexicomp Online. Pediatric Drug Information. Hudson,
2. Weller PF, Nutman TB. Intestinal nematode infections. In:
OH: Wolters Kluwer Clinical Drug Information; 2019.
Jameson J, Fauci AS, Kasper DL, Hauser SL, Longo DL,
This page intentionally left blank
34 Tuberculosis
David Cluck

PATIENT PRESENTATION Aspirin 81 mg PO daily


Atorvastatin 40 mg PO daily
Chief Complaint Budesonide/formoterol 160 mcg/4.5 mcg 2 inhalations BID
"I have a cough that wont go away:' Clonazepam 0.5 mg PO three times daily PRN anxiety
Lisinopril 20 mg PO daily
History of Present Illness Metoprolol succinate 100 mg PO daily
Tiotropium 2 inhalations once daily
A 63-year-old male presents to the emergency department with
Venla&.xine 150 mg PO daily
complaints ofcough/shortness of breath which he attributes to a
Warfarin 7.5 mg PO daily
*nagging cold~ He states he fears this may be something worse
after experiencing hemoptysis for the past 3 days. He also admits
to waking up in the middle of the night "drenched in sweat" for Physical Examination
the past few weeks. When asked, the patient denies ever having a
~ V'ttal Signs
positive PPD and was last screened "several years ago~ His chart
indicates he was in the emergency department last week with Temp 100.S"F, P 96, RR 24 breaths per minute, BP 150/84 mm
similar symptoms and was diagnosed with community-acquired Hg, p02 92%, Ht 5'10", Wt 56.4 kg
pneumonia and disclw:ged wilh azithromycin.
~ General
Past Medical History Slightly disheveled male in mild-to-moderate distress
Hypertension, dyslipidemia, COPD, atrial fibrillation, gener-
alized anxiety disorder ~ HEENT
Normocephalic, atraumatic, PERRLA, EOMI, pale/dry mucous
Surgical History membranes and conjunctiva, poor dentition
Appendectomy at age 18
~ Pulmonary
Family History Bronchial breath sounds in RUL
Father passed away from a myocardial infarction 4 years ago;
mother had type 2 DM and passed away from a ruptured
~ Cardiovascular
abdominal aortic aneurysm
NSR. no m/r/g
Social History
Retired geologist recently moved from India to live with his ~ Abdomen
son who is currently in medical school in upstate New York. Soft, non-distended, non-tender, ( +) bowel sounds
Smoked % ppd x 40 years and drinks 6 to 8 beers per day,
recently admits to drinking Y.a pint of vodka "every few days'" ~ Genitourinary
since the passing of his wife 6 months ago.
No complaint of dysuria or hematuria
Allergies
~ Neurology
Sulfa (hives); penicillin (nausea/vomiting); shellfish (itc.hing)
Oriented to person. place. and time
Home Medications
Albuterol metered-dose-inhaler 2 puffs q4h PRN shortness ~ Extremities
of breath No lesions or edema present

141
142 INFECTIOUS DISEASES: A CASE STUDY APPROACH

Laboratory Findings C. Pyrazinamide


D. Rifampin
Na =140 mEq/L Hgb = 14.5 g/dL Ca = 8.8 mg/dL
5. The patient is started on a regimen of rifampin/ethambutoV
K = 4.5 mEq/L Hct = 38% Mg = 2.2 mg/dL
pyrazinamide/isoniazid (RIPE). Shortly after starting his
Cl= 98mEq/L Pit = 332 x 10' /mm3 Phos = 4.6 mg/dL therapy the patient presents to an urgent care with concern
3 his "blood is too thin" due to overt red-tinged urine; however,
C02 = 26 mEq/L WBC = 12 x 103 mm AST = 24 IU/L
BUN = 26 mg/dL Trap <0.01 nglmL ALT= 22 IU/L a urinalysis is negative for blood The provider recognizes the
drug-drug interaction with warfarin and rifampin and asks
SCr = 1.48 mg!dL CK= 3ng/mL T Bill = 1.8 mg/dL for a recommendation to manage this drug-drug interaction.
Glu = 188 mg/dL BNP = 64 pg/mL A1k.Ph05 = 76 IU/L Which recommendation is most appropriate?
INR= 2.4 A. Switch warfarin to rivaroxaban
B. Continue warfarin with close monitoring
~ Chest X-ray C. Switch rifampin to rifapentine
D. Add enoxaparin while taking rifampin
Hilar lymphadenopathy visualized with RUL apical and pos-
terior segment infdtrate 6. Three weeks into therapy his liver enzymes are noted to
be 1Ox the ULN. What is the most appropriate manage-
~ Blood Culture ment of this adverse effect?
Negative A. Add milk thistle to the current regimen
B. Change rifampin to rifabutin
~ Sputum Culture C. Discontinue ethambutol, continue all other agents
D. Suspend all agents and reintroduce sequentially
Normal flora
7. The patient admits to stopping his medications after
~ Sputum AFB Cuhure experiencing the acute spike in his liver enzymes in addi-
Pending tion to fearing serious bleeding due to a change in the
color of his urine. Which recommendation is most likely
~ Influenza Screen to result in a positive outcome?
A. Change his current regimen to moxifloxacin/
Negative
cycloserine/isoniazid/linezolid
B. Obtain susceptibilities prior to reinitiating therapy
QUESTIONS C. Restart all therapies with twice weekly dosing in the
continuation phase
1. Which risk factor places this patient at highest risk of devd- D. Restart all therapies and add bedaquiline
oping tuberculosis disease?
A. Being foreign born 8. The patient is found to have positive sputum cultures at
B. Failure of CAP treatment 2 months. What is the optimal (total) duration of therapy
C. Hemoptysis given this finding?
D. History of COPD A. 3 months
B. 6 months
2. Which diagnostic criteria would suggest highest likelihood C. 9 months
of tuberculosis disease? D. 12 months
A. Positive AFB sputum smears plus negative NAA
B. Positive IGRA only 9. Three years after arriving in the United States the patient's
C. Positive IGRA plus chest radiography son, a 32-year-old male with type I DM, reports to his
D. Positive AFB sputum smear plus positive NAA primary care physician to have his PPD read. It is noted
to be 12 mm. Which regimen is most appropriate for
3. Which empiric regimen is most appropriate for active latent TB?
tuberculosis disease? A. Isoniazid for 9 months
A. Ethambutol plus clarithromycin B. Isoniazid for 3 months
B. Rifapentine plus isoniazid C. Pyrazinamide plus rifampin for 2 months
C. Rifabutin plus ethambutol D. Rifampin for 4 months
D. Rifampin plus ethambutol plus pyrazinamide plus
isoniazid 10. Which supplement should be added to the patient's latent
TB regimen if isoniazid is included?
4. In a fully active tuberculosis regimen, which agent may be dis- A. Ascorbic acid
continued if the organism is susceptible to all first-line agents? B. Cyanocobalamin (B 12)
A. Ethambutol C. Pyridoxine (B6)
B. lsoniazid D. Thiamine (B1)
CHAPTER 34 I TUBERCULOSIS 143

ANSWERS result in a lower INR and thus the urine discoloration is


not a result of a drug-drug interaction but a consequence
1. Explanation: The correct answer is A. Foreign birth and of rifampin use. Patients who require concurrent warfarin
immigration are considered risk factors for developing and rifampin should have close INR monitoring with
tuberculosis. Other risk factors include diabetes mellitus, adjustments being on an individual patient basis. Switch-
prolonged use of corticosteroids, immunosuppressive ing to rivaroxaban (answer choice A) would be inappro-
therapy, injection drug use, and chronic renal failure. 1 priate as this drug and other DOACs are also susceptible
Answer choices B and C (failure of CAP treatment and to drug-drug interactions with rifampin. Switching to
hemoptysis) are supportive in including tuberculosis in rifapentine (answer choice C) would also not obviate the
the differential diagnosis, but do not place the patient at drug-drug interaction as rifapentine is also a rifamycin
increased risk. Answer choice D (history of COPD) has capable of CYP enzyme induction. Enoxaparin (answer
some association with tuberculosis, but is not a univer- choice D) would not interact with rifampin, but is not
sally accepted risk factor. necessary given the INR.
2. Explanation: The correct answer is D. Confirmatory 6. Explanation: The correct answer is D. The optimal
diagnosis is established by isolation of Mycobacterium approach to reintroducing tuberculosis treatment after
tuberculosis from sputum or other bodily secretion. hepatotoxicity is not known; however, most tuberculo-
However, a positive AFB smear could also represent sis programs use sequential reintroduction of drugs.'
the presence of non-tuberculous mycobacteria (rules Because rifampin is much less likely to cause hepatotoxic-
out answer choice A). Interferon-gamma-release assays ity than isoniazid or pyrazinamide, it is typically restarted
(IGRA) and chest radiography are considered supportive first followed by isoniazid. Pyrazinamide may be started
diagnostic tools (rules out answer choice C). IGRAs have 1 week after isoniazid if ALT does not increase. If
some utility in diagnosis but are not helpful in differen- rifampin and isoniazid are tolerated and hepatitis was
tiating between latent tuberculosis and tuberculosis dis- considered severe, pyrazinamide can be assumed to be
ease (rules out answer choice B).2 In addition, due to the responsible and should be discontinued.' Answer choice
infectiousness, if active tuberculosis is in the differential A (adding milk thistle) is incorrect as this is unlikely to
diagnosis the patient should be placed in an airborne prevent ongoing drug-induced hepatotoxicity. Answer
infection isolation room. Precautions can be discon- choice B (changing to rifabutin) is unlikely to be benefi-
tinued once active tuberculosis has been ruled out, an cial as clinically apparent liver injury is likely to be similar
alternative diagnosis has been established, or initiation to rifampin. Answer choice C {discontinuing ethambutol)
of therapy results in three subsequent negative sputum is unlikely to be helpful as the predominant toxicity with
smears.3 ethambutol is optic neuritis.
3. Explanation: The correct answer is D. The regimen of 7. Explanation: The correct answer is B. When interrup-
choice for patients with newly diagnosed pulmonary tions are due to an interim loss of follow-up, at the time
tuberculosis consists of a 4-drug intensive phase consist- the patient is returned to treatment, additional sputum
ing of ethambutol, pyrazinamide, rifampin, and isoniazid should be obtained for repeat culture and drug suscep-
for 2 months followed by a 2-drug continuation phase tibility testing. Susceptibility data should be obtained
consisting of rifampin and isoniazid for 4 to 7 months. 4 prior to any change in the regimen (rules out answer
Answer choice A (ethambutol plus clarithromycin) and choice A); moreover, adding a single agent {rules out
answer choice C (rifabutin plus ethambutol) would be answer choice D) to a potentially failing regimen is dis-
useful in treating non-tuberculous mycobacterial infec- couraged and likely to result in suboptimal outcomes.
tion. Answer choice B (rifapentine and isoniazid) would Twice weekly dosing {rules out answer choice C) has
be useful in treating latent tuberculosis, but not active lesser efficacy and is more appropriate in an adherent
disease.5 patient.4
4. Explanation: The correct answer is A. If susceptibilities 8. Explanation: The correct answer is C. In patients treated
indicate lack of resistance to any of the first-line agents, for 6 months, having either cavitation on chest imaging
ethambutol may be discontinued. Ethambutol is useful or a positive culture at completion of 2 months of therapy
initially as it protects against rifampin resistance in the has been associated with rates of relapse of approximately
event there is unrecognized isoniazid resistance.4 20% compared to 2% among patients with neither factor.
For these patients, guidelines recommend extending the
5. Explanation: The correct answer is B. A well-known
continuation phase with isoniazid and rifampin for an
adverse effect of rifampin is discoloration of bodily fluids
additional 3 months! The other listed answer choices are
(usually a red or orange color), which is a key counseling
either inappropriately short or long.
point for patients being newly started on rifampin. In this
case the patient is also on warfarin, which has a clinically 9. Explanation: The correct answer is D. The interpretation
significant drug-drug interaction with rifampin; how- of the PPD is based on the measurement in millimeters of
ever, rifampin is a CYP enzyme inducer, which would induration and not the degree of inflammation/erythema.
144 INFECTIOUS DISEASES: A CASE STUDY APPROACH

An induration of 5 mm or more is considered positive C (pyrazinamide plus rifampin) is associated with higher
in people living with HIV, close contact with a person rates of liver injury and death.
with active TB disease, or in persons with immuno-
10. Explanation: The correct answer is C. Patients with a pre-
suppression (eg, receipt of TNF blocking agents). An
disposition for neuropathy. such as those with diabetes,
induration of 10 mm or more is considered positive in
should receive 25 to 50 mg of pyridoxine (B6) to prevent
patients who have recently immigrated, have a history
new onset or exacerbation of peripheral neuropathy.5
of injection drug use, and reside in high congregate
settings or mycobacteriology laboratory personnel. An
induration of greater than 15 mm is considered positive
in any individual including those without known risk REFERENCES
factors. 2 An induration of 12 mm in this case is sugges- 1. Zumla A, Raviglione M, Hafner R, Fordham von Reyn C.
tive of positivity due to recent immigration as well as Tuberculosis. N Engl l Med. 2013;368(8):745-755.
contact with a person known to have active tuberculosis 2. Lewinsohn DM, Leonard MK, LoBue PA, et al. Official
disease (father and son live in same household). The American Thoracic Society/Infectious Diseases Society
tuberculin skin test is easy to perform; however, trained of America/Centers for Disease Control and Prevention
personnel are needed for administration. In addition, clinical practice guidelines: diagnosis of tuberculosis in
the interpretation of the test is subject to reader vari- adults and children. Clin Infect Dis. 2017;64:1-33.
ability. IGRAs, similar to the tuberculin skin test, are 3. Jensen PA, Lambert LA, Iademarco MF, Ridzon R;
helpful in diagnosing infection. Moreover, IGRAs can Centers for Disease Control and Prevention. Guide-
be advantageous in certain situations including patients lines for preventing the transmission of Mycobacterium
who have received BCG either as a vaccine or as a cancer tuberculosis in health-care settings. MMWR Recomm
therapy or patients unlikely to return for PPD interpre- Rep. 2005;54(RR-17):1-141.
tation. The utility is partially offset by cost, inconsistent 4. Nahid P, Dorman SE, Alipanah N, et al. Official American
test reproducibility, and increased use of laboratory Thoracic Society/Centers for Disease Control and Pre-
resources. 2 vention/Infectious Diseases Society of America clinical
While there are several options for treating latent practice guidelines: treatment of drug-susceptible tuber-
tuberculosis, adherence and tolerability should be taken culosis. Clin Infect Dis. 2016;63:147-195.
into consideration when selecting any regimen. Rifampin 5. World Health Organization. Latent TB infection: Updated
for 4 months has a shorter treatment duration resulting and consolidated guidelines for programmatic manage-
in higher likelihood of adherence and better safety when ment, 2018. Available at http://www.who.int/tb/publica-
compared to other listed treatment options.6 Answer tions/2018/latent-tuberculosis-infection/en/. Accessed
choice A (INH for 9 months) could place the patient at September 27, 2019.
higher risk of peripheral neuropathy given he is diabetic. 6. Menzies D, Adjobimey M, Ruslami R, et al. Four months
Answer choice B (INH for 3 months) is inappropriate of rifampin or nine months of isoniazid for latent tuber-
due to the truncated duration of therapy. Answer choice culosis in adults. N Engl f Med. 2018;379:440-453.
INDEX

A cryptococcus, 126-128
A. baumanii, 27 cymtis, 26-27
Abacavir, 53 diabetic foot infection, 77- 78
Absolute neutrophil count (ANC), 57 febrile neutropenia, 57-59
Acetarninophen,6, 15 hepatitis C, 100-101
Acetazolamide, 127 herpes,108-109
Acquired immunodeficiency syndrome (AIDS). see AlDS hospital-acquired and ventilator-associated pneumonia,
Acute bronchitis, 9-11 22- 24
answers, 10- 11 human immunodeficiency virus, 53-54
patient presentation, 9 infective endocarditis, 43-45
questions, 10 influenza,2-4
references, 11 intra-abdominal infections, 89
Acute otiti.s media, 5-8 necrotlzing fasciltis, 72-74
answers, 6- 7 neinatodes,138-139
initial management, 7 pharyngitis, 14-15
patient presentation, 5 prosthetic joint infection, 84- 86
questions, 5-6 protozoans, 134-136
references, 7- 8 pyelonephrltls,31-32
Acyclovir, 39, 108-109 sepsis, 48-50
Adamantane antivirals, 3 skin and soft tissue infection, 68-69
Advisory Committee on hnmunization Practices (ACIP). 4 skin and soft tissue infection I, 63-65
AIDS, diagnosis of, 53 superficial fungal infections, 122-124
Albendazole, 138 syphilis, 104-106
Alcohol consumption, 117 traveler's diarrhea, 96-97
Allergies trichomonias.is, 117-119
to antibiotics, 14, 32, 49, 58, 64, 106, 113 tuberculosis, 143-144
to sulfa drug1, 27 vertebral osteomyelitls, 80- 82
Alprazolam, 101 viral encephalitis, 38-39
Altered mental atate, 39 vulvovaginal candidiasis, 117- 119
American Academy of Pediatrics (AAP), 6 Antibiotics. see also specific antibiotics
American Thoracic Society (ATS), 23, 24 acute bronchitis, 10
AmJnoglycosides, 24 acute otitis media, 6-7
Amlodipine, 100 alcohol comumption and, 117
Amoxicillin. 7 bacterial meningitis, 35
acute bronchitis, 10 bioavailability at: 81
cystitil, 27 chronic bronchitis, 11
MSSAand,69 community-acquired pneumonia, 19
pharyngitis, 14 cystitis, 27
Amphotericin B, 131 diarrhea and, 96
Amphotericin B deoxycholate, 127, 128 febrile neutropenia, 57-58
Ampicillin, 27, 35, 39 hospital-acquired and ventilator-associated pneumonia,
Anemia, 135 23- 24
Answers infective endocarditil, 44
acute bronchitis, 10-11 intravenous,58, 77
acute otitis media, 6-7 necrotizing fasciltis, 73
aspergillosia, 131 pharyngitis, 14
bacterial meningitis, 34-36 pyelonephrltls,32
bacterial vaginosis, 117-119 as risk factor for C. difficik., 92
chlamydia and gonorrhea, 112-113 sepsis, 49
Clostridioides difficile infection, 92- 94 skin and soft tissue infection, 68
community-acquired pneumonia, 18-20 skin and soft tissue infection I, 63-64

145
146 INDEX

Antibiotics (Contd.) patient presentation, 115-116


stewardship, 2, 14, 24, 49, 73, 94 questions, 116
tuberculosis, 143 references, 119
vertebral osteomyelitis, 81 Bacteroides fragilis, 64
viral encephalitis, 39 Baloxavir, 3
Antifungal therapy, 59, 73, 127 Bicillin L-A, 105
Antigen detection test, 3, 23 Bioavailability of antibiotics, 81
Antihistamine, 11 Breastfeeding, antifungal therapy and, 123
Antimicrobial resistance, 10, 14 Bronchitis. see Acute bronchitis; Chronic bronchitis
diabetic foot infection and, 77 Bronchoscopy, 23
rifampin and, 44 Brudzinski's sign, 35
risk factors for, 23
sepsis, 49 c
Antimicrobial therapy, 10, 11 C. diphtheriae, 14
Antimotility therapy, 97 Candida spp., 73, 117
Antipseudomonal beta-lactams, 58 Candidiasis, 53, 54
Antiretroviral therapy, 53-54 Carbapenem,32,49
Antistaphylococcal beta-lactams, 43, 44 Caspofungin, 131
Aplastic anemia, 19 Catheter-related bloodstream infection, 58
Ascariasis, 138 Cefdinir, 7
Aspergillosis, 129-131 Cefepime,24,58,59
answers, 131 Ceftriaxone, 19,32,35,39,49,64,89, 113
patient presentation, 129-130 Cellulitis, 68
questions, 130-131 CENTOR, 14
references, 131 Cephalexin, 27, 113
Aspirin, for children, 15 Cephalosporin, 35, 49, 69, 73
Athletes, and fungal infections, 122 Cerebrospinal fluid
Athlete's foot, 122, 123 antigen titers, 128
Azithromycin, 14, 19 gram stain examination of, 34-35, 38-39
acute bronchitis, 10 pressure of, 127
chlamydia and gonorrhea, 113 Chemoprophylaxis, 3-4
diarrhea and, 96 Chemotherapy, febrile neutropenia and, 57-58
side effects of, 113 Chest X-ray, 19, 23, 143
Aztreonam, 27 Chlamydia, 111-114
answers, 112-113
patient presentation, 111-112
B questions, 112
Back pain, 80-81 references, 113-114
Bacteria. see also specific bacteria Chronic bronchitis, 11
febrile neutropenia, 57 Chronic obstructive pulmonary disease (COPD), 11
gram-negative vs. gram-positive, 57 Chronic pulmonary disease, 4
pharyngitis, 14 Chronic wounds, 77
Bacterial coinfection, influenza and, 3 Ciprofloxacin, 77, 85, 89
Bacterial meningitis, 33-36 Cirrhosis, 89, 100
answers, 34-36 Clarithromycin, 14
patient presentation, 33-34 Clavulanate, 7
questions, 34 Clavulanic acid, 10
references, 36 "Clean catch;' 31
signs and symptoms, 35 Clindamycin, 14-15, 69, 73, 77, 117, 119
symptoms, 35 Clindamydin, 14-15
vs. viral encephalitis, 38 Clostridioides difficile infection, 69, 91-94. see also Diarrhea
Bacterial vaginosis, 115-119 answers, 92-94
answers, 117- 119 classification of, 93
INDEX 147

patient presentation, 91-92 Debridement, 73


questions, 92 Decolonization, 63
references, 94 DEET, 135-136
risk factors for, 92 Dentition, 80
transmission of, 93-94 Dexamethasone, 35-36
Clostridium spp., 72 Dextromethorphan, 10
Coagulase-negative Staphylococcus (CONS), 84, 85 Diabetes, necrotizing fasciitis and, 72
Cobicistat, 54 Diabetic foot infection, 75-78
Colloids, 49 answers, 77-78
Combination therapy, 24, 49 classification of, 77
Community-acquired pneumonia, 17-20 patient presentation, 75-76
answers, 18-20 questions, 76
definition of, 23 references, 78
IDSA guidelines, 19 Diagnosis
patient presentation, 17-18 acute bronchitis, 10
questions, 18 acute otitis media, 6
references, 20 of AIDS, 53
Community-associated infections, 92 Clostridioides difficile infection, 93
Community-onset health care facility-associated (COHCFA) cryptococcus, 126-127, 128
infections, 92 infective endocarditis, 45
Compartment syndrome, 73 necrotizing fasciitis, 73
Condoms, 109 nematodes, 138
Corticosteroids, 3, 15 pharyngitis, 14
Costovertebral angle (CVA), 31 syphilis, 104-105
Cough, 10, 11 tuberculosis, 143
C-reactive protein (CRP), 31 Diarrhea, 92-93, 95-97, 113
Cryptococcus, 125-128 Dicloxacillin,69,77-78
answers, 126-128 Direct detection, 105
patient presentation, 125-126 Direct fluorescent antibody test, 105
questions, 126 Divalproex sodium, 58
references, 128 Docosanol, 109
CTscan, 73 Dolutegravir, 54
Cultures Doxycycline, 64
anaerobic bacterial, 63 chlamydia and gonorrhea, 113
blood, 81 MRSAand,69
pneumonia and, 23 necrotizing fasciitis, 73
prosthetic joint infection, 84 skin and soft tissue infection, 69
CURB-65 score, 18-19 Drug interactions, 58
Cutaneous abscesses, 64 Dual therapy, 113
Cyclosporine, 131 Duration of therapy
CYP450 enzymes, 44 acute otitis media, 7
Cystitis, 25-28 aspergillosis, 131
answers, 26-27 bacterial meningitis, 35
patient presentation, 25 community-acquired pneumonia, 20
questions, 26 cryptococcus, 127
references, 28 cystitis, 27
diabetic foot infection, 78
D hepatitis C, 101
DAIR (debridement, antibiotics, and implant retention), hospital-acquired and ventilator-associated
84-85,86 pneumonia, 24
Daptomycin, 64, 78, 85 infective endocarditis, 44-45
Darkfield microscopy, 105 intra-abdominal infections, 89
Day care attendance, 6 necrotizing fasciitis, 73
148 INDEX

Duration of therapy (Contd.) Fluoroquinolones, 27, 32


oropharyngeal candidiasis, 54 Fluzone High-Dose, 4
pharyngitis, 14 Fosfomycin,27,32
prosthetic joint infection, 85-86 Fournier gangrene, 73
pyelonephritis, 32 Fulminant cases, 93
sepsis, 49-50 Fungi, 57, 59. see also Superficial fungal infections
skin and soft tissue infection, 64, 69
superficial fungal infections, 123 G
syphilis, 108-109 Gardnerella vaginalis, 117
viral encephalitis, 39 GAS pharyngitis, 14, 15
Dysuria, 26 Genital herpes. see Herpes
Gentamicin, 44, 59
E Glecaprevir/pibrentasvir, 101
E.coli,27,32,89 Glutamate dehydrogenase (GDH) enzyme immunoassay, 93
E. faecalis, 27 Gonorrhea, 111-114
Echocardiography, 43, 45 answers, 112-113
Efavirenz, 54 patient presentation, 111-112
Elbasvir/grazoprevir, 101 questions, 112
Elvitegravir, 53 references, 113-114
Empiric therapy, 39, 57 Gram stain examination, 34-35
Encephalopathy, 89 Group A 6-hemolytic Streptococcus, 72, 73
Enterobiasis, 138 Group A streptococcus (GAS), 14
Enterococcus, 84 Group C streptococcus, 14
Enterococcus faecalis, 64
Enteroviruses, 39 H
Environmental contamination, 93-94 Haemophilus influenzae, 7, 32, 35, 39
Environmental molds, 131 Hand hygiene, 93, 97
Enzyme imIDunoassays (EIAs), 93 Health care facility-onset (HO) infections, 92
Ertapenem, 32, 58 Health care-associated bacterial meningitis, 35
Erythrocyte sedimentation rate (ESR), 31 Hepatitis C, 99-101
Erythromycin, 113 answers, 100-101
Estrogen, 54 patient presentation, 99-100
Ethambutol, 143 questions, 100
Expectorants, 11 references, 101
Extended-spectrum beta-lactamase (ESBL)-producing Hepatotoxicity, 85, 143
bacteria, 58 Herpes, 107-109
answers, 108-109
F patient presentation, 107
Failing IV therapy, 19-20 questions, 108
Famciclovir, 108 references, 109
Febrile neutropenia, 55-59 Herpes simplex virus (HSY), 39
answers, 57-59 Hookworm, 138
patient presentation, 55-56 Hospital-acquired and ventilator-associated pneumonia,
questions, 56-57 21-24
references, 59 answers, 22-24
symptoms, 57 definition of, 23
Fever, 57, 63 patient presentation, 21-22
Fidaxomicin, 93 questions, 22
Fluconazole, 54, 123, 127, 128 references, 22-24
Flucytosine, 127 resistance risk factors for, 23
FluMist Quadrivalent, 4 HPV infection, 106
Fluoroquinolone, 96, 113 Human herpesviruses, 57
INDEX 149

Human immunodeficiency virus (HIV), 51-54 L


answers, 53-54 Latent syphilis, 105
patient presentation, 51-52 Legionella antigen, 19
questions, 52-53 Leukemia, 57
references, 54 Leukocyte esterase, 26, 31
screening for, 106, 118 Levofloxacin,27,32,57
Hypoglycemia, 135 chlamydia and gonorrhea, 113
Hypotension, 48 diabetic foot infection, 78
prosthetic joint infection, 85
Levofloxacin (fluoroquinolone), 19
Imipenem-cilastatin, 58 Linezolid,64-65,85
Immunotherapy, 3 Listeria monocytogenes, 35
Incision and drainage, skin and soft tissue infection, 63, 64, Live attenuated influenza vaccine (LAIV), 4
68
India ink stain, 127, 128 M
Induration, 143-144 Macrolides, 19
Infectious Diseases Society of America (IDSA), 23, 24, 81, 82, Magnetic resonance imaging. see MRI
85 Malaria, 134-135
Infective endocarditis, 41-45 Mannitol, 127
answers, 43-45 Marine organisms, 72-73
patient presentation, 41-42 Medication noncompliance, 15
questions, 42-43 Mefloquine, 135
references, 45 Meningoencephalitis,126
symptoms, 43 Meropenem,24,32,35,58
Influenza, 1-4 Methamphetamine, 64
acute bronchitis and, 10 Methicillin-resistant Staphylococcus aureus. see MRSA
answers, 2-4 Methicillin-susceptible Staphylococcus aureus. see MSSA
patient presentation, 1-2 Metronidazole, 49, 117
questions, 2 Clostridioides difficile infection, 93
references, 4 diabetic foot infection, 77
symptoms, 3 vulvovaginal candidiasis, 119
Influenza cell culture, 2 Middle ear effusion (MEE), 6
"Inoculum effect," 44 Minocycline, 73
Integrase strand transfer inhibitor (INSTI), 53 Molecular assay, for influenza, 3
Interferon-gamma-release assays (IGRA), 143, 144 Monotherapy, 24, 49
Intra-abdominal infections, 87-89 Moraxella catarrhalis, 7
answers, 89 Mortality rates, 44, 49
patient presentation, 87-88 Moxifloxacin,32
questions, 88 MRI,73,77
references, 89 MRSA, 3, 19, 58
Intravenous drug use (IVDU), 44, 63, 100 antibiotics and, 63-64
Invasive pulmonary aspergillosis (IPA), 131 diabetic foot infection, 77
Isavuconazole, 131 hospital-acquired and ventilator-associated pneumonia,
Isolation, 94, 143 23-24
Itraconazole, 54, 123, 131 necrotizing fasciitis and, 73
IV therapy, 19-20 prosthetic joint infection, 84-85
risk factors for, 63
J sepsis and septic shock, 49
Jarisch-Herxheimer reaction, 105-106 skin and soft tissue infection, 64, 68-69
MSSA,23,43,45,64,69,77-78,81
K Mucositis, 57
K. pneumoniae, 89 Multinational Association for Supportive Care in Cancer
Kernig's sign, 35 (MASCC) risk index, 57-58
150 INDEX

Mycobacterium avium complex disease, 53 p


Mycobacterium tuberculosis (MTB), 81, 143 P. aeruginosa, 68
P. falciparum, 135
N P. knowles, 135
N. gonorrhoeae, 14 P. ovale, 135
Nafcillin, 44 P. vivax, 135
Nasal congestion, 10 Pain, intra-abdominal infections, 89
Nasal decongestant, 11 Pain management, 6
Native valve (NVE) infective endocarditis, 44 Patient presentation
Native Vertebral Osteomyelitis Guidelines, 81 acute bronchitis, 9
Necrotizing fasciitis, 71-74 acute otitis media, 5
answers, 72- 74 aspergillosis, 129-130
patient presentation, 71-72 bacterial meningitis, 33-34
questions, 72 bacterial vaginosis, 115-116
references, 7 4 chlamydia and gonorrhea, 111-112
risk factors for, 72 Clostridioides difficile infection, 91-92
types of, 72, 73 community-acquired pneumonia, 17-18
Neisseria meningitidis, 35, 39 cryptococcus, 125- 126
Nematodes, 137-139 cystitis, 25
answers, 138-139 diabetic foot infection, 75-76
patient presentation, 137 febrile neutropenia, 55-56
questions,137-138 hepatitis C, 99-100
references, 139 herpes, 107
Neural tube defects, 54 hospital-acquired and ventilator-associated pneumonia,
Neuropathy, 144 21- 22
Neurotransmitters, 64-65 human immunodeficiency virus, 51- 52
Nitrites, 31 infective endocarditis, 41-42
Nitrofurantoin, 27, 32 influenza, 1-2
Nitroimidazole class, 117 intra-abdominal infections, 87-88
Noncompliance, medication, 15 necrotizing fasciitis, 71- 72
Nonsteroidal anti-inflammatory drug (NSAID), 15 nematodes, 137
Norovirus, 96 pharyngitis, 13
Nucleic acid amplification test (NAAT), 112-113 prosthetic joint infection, 83
Nucleic acid amplification tests (NAAT), 93 protozoans, 133-134
Nucleos(t)ide reverse transcriptase inhibitor {NRTI), 53 pyelonephritis, 29
Nystatin, 123 sepsis, 47-48
skin and soft tissue infection, 67-68
0 skin and soft tissue infection I, 61-62
Obesity, acyclovir dosage and, 39 superficial fungal infections, 121
Ofl.oxacin, 113 syphilis, 103-104
Omeprazole, 100-101 traveler's diarrhea, 95
Oonychomycosis, 123-124 trichomoniasis, 115-116
Opportunistic infections, 51-54 tuberculosis, 141-142
Opportunistic Infections Guidelines, 54 vertebral osteomyelitis, 79-80
Oral contraceptives, 54 viral encephalitis, 37-38
Oral rehydration solution (ORS), 96 vulvovaginal candidiasis, 115-116
Organ transplants, 127-128, 131 Pediatrics
Oropharyngeal candidiasis, 54 acute otitis media, 5- 8
Oseltamivir, 3, 4, 10 pharyngitis, 13-15
Otalgia, 6 viral encephalitis, 39
OVIVA study, 81 Penciclovir, 109
Oxacillin, 24 Penicillin, 7
INDEX 151

allergyto,32,49,58,106 answers, 134-136


bacterial meningitis, 35 patient presentation, 133-134
necrotizing fasciitis, 73 references, 136
pharyngitis, 14 Pseudomonas aeruginosa, 23, 24, 32, 49, 58, 77, 89
resistance to, 43, 64 Purulent abscess, 63, 64
syphilis, 105 Pyelonephritis, 26, 29-32
vertebral osteomyelitis, 81 answers, 31-32
Penicillin/ beta-lactamase inhibitor combination, 49 patient presentation, 29
Peptostreptococcus, 68 questions, 30-31
Peramivir, 4 references, 32
Peripheral neuropathy, 144 signs and symptoms, 31
Peritonitis, 89 Pyrazinamide, 143
Perivalvular extension, 44 Pyuria, 31
Permethrin, 136
pH, ofurine, 26-27 Q
Pharyngitis, 13-15 Quality-of-care indicators (QCis), 45
answers, 14-15 Questions
patient presentation, 13 acute bronchitis, 10
questions, 13-14 acute otitis media, 5-6
references, 15 aspergillosis, 130-131
Phenazopyridine, 27 bacterial meningitis, 34
Pinworm, 138 bacterial vaginosis, 116
Piperacillin-tazobactam, 19, 58 chlamydia and gonorrhea, 112
Pneumocystis jirovecii pneumonia, 53 Clostridioides difficile infection, 92
Pneumocystis pneumonia, 54 community-acquired pneumonia, 18
Pneumonia cryptococcus, 126
community-acquired, 17-20 cystitis, 26
HIV and, 54 diabetic foot infection, 76
hospital-acquired and ventilator-associated, febrile neutropenia, 56-57
21-24 hepatitis C, 100
Polymicrobial infection, 77 herpes, 108
Polymorphonuclear cells (PMNs), 57 hospital-acquired and ventilator-associated pneumonia,
Polyuria, 26 22
Prednisone, 127 human immunodeficiency virus, 52-53
Pregnancy infective endocarditis, 42-43
antifungal therapy and, 123 influenza, 2
HIV and, 54 intra-abdominal infections, 88
influenza and, 4 necrotizing fasciitis, 72
malaria and, 135-136 nematodes, 137-138
Primary prophylaxis, 53, 54 pharyngitis, 13-14
Primary syphilis, 105 prosthetic joint infection, 84
Progestin, 54 protozoans, 134
Prophylaxis, 7, 89 pyelonephritis, 30-31
Prosthetic joint infection, 83-86 sepsis, 48
answers, 84-86 skin and soft tissue infection, 68
patient presentation, 83 skin and soft tissue infection I, 62- 63
questions, 84, 134 superficial fungal infections, 122
references, 86 syphilis, 104
Prosthetic valve (PVE) infective endocarditis, 44 traveler's diarrhea, 96
Protease inhibitor, 53 trichomoniasis, 116
Proton pump inhibitors (PPis), 92 tuberculosis, 142
Protozoans, 133-136 vertebral osteomyelitis, 80
152 INDEX

Questions (Contd.) Reye's syndrome, 15


viral encephalitis, 38 Ribavirin, 101
vulvovaginal candidiasis, 116 Rifampin,44,85, 143
Quick Sequential Organ Failure Assessment (qSOFA) Rifapentine, 143
criteria, 48 Ringw-orrn, 122, 123
Quinolones, 81 Risk classification, febrile neutropenia, 57-58
Ritonavir-boosted protease inhibitor regimens, 54
R
Rapid antigen detection test (RADT), 14 s
Rapid plasma reagin test (RPR), 104 S. maltophilia, 27
Rapid plasma region (RPR) card test, 112 S. pyogenes, 68
References Sainples,urine,31
acute bronchitis, 11 Screening
acute otitis media, 7-8 chlamydia, 113
aspergillosis, 131 for HIV, 106, 118
bacterial meningitis, 36 Secondary prophylaxis, 53, 54
bacterial vaginosis, 119 Secondary syphilis, 105
chlamydia and gonorrhea, 113-114 Seizures, 58
Clostridioides difficile infection, 94 Sepsis, 47-50
community-acquired pneumonia, 20 answers, 48-50
cryptococcus, 128 patient presentation, 47-48
cystitis, 28 questions, 48
diabetic foot infection, 78 references, 50
febrile neutropenia, 59 Septic shock, 48, 74
hepatitis C, 101 Sequential Organ Failure Assessment (SOFA), 48
herpes, 109 Serologic testing, 104
hospital-acquired and ventilator-associated pneumonia, Serology, 112
22-24 Serotonin syndrome, 85
human immunodeficiency virus, 54 Serum trough concentrations, 64
infective endocarditis, 45 Severity-of-illness index, 18
influenza, 4 Sexual partners, and STDs, 109, 113, 118
intra-abdominal infections, 89 Sexually transmitted diseases. see Bacterial vaginosis;
necrotizing fasciitis, 74 Chlainydia; Gonorrhea; Herpes; Syphilis;
nematodes, 139 Trichornoniasis; Vulvovaginal candidiasis
pharyngitis, 15 Sirnvastatin, 100
prosthetic joint infection, 86 Skin and soft tissue infection, 61-65, 67-69
protozoans, 136 answers, 63-65, 68-69
pyelonephritis, 32 patient presentation, 61-62, 67-68
sepsis, 50 questions, 62-63, 68
skin and soft tissue infection, 69 references, 65, 69
skin and soft tissue infection I, 65 Spirometrytesting, 10
superficial fungal infections, 124 Sputum production, 10, 22-23
syphilis, 106 Standardized definitions, 92
traveler's diarrhea, 97 Staphylococci, 44
trichornoniasis, 119 Staphylococcus aureus, 3, 23, 27
tuberculosis, 144 prosthetic joint infection, 84
vertebral osteornyelitis, 82 purulent abscess, 63, 64
viral encephalitis, 39 skin and soft tissue infection, 64
vulvovaginal candidiasis, 119 vertebral osteomyelitis, 81
Rehydration, 96 Stenotrophomonas maltophilia, 32
Renal dysfunction, 39, 64, 92 Stevens-Johnson syndrome, 69, 106
Resistance-associated substitutions (RASs), 101 Stool sainples, 93
Respiratory viruses, 10 Streptococci, 44
INDEX 153

Streptococcus,84 hepatitis C, 100


Streptococcus pneumoniae, 7, 19 herpes, 108
bacterial meningitis, 35, 36 syphilis, 104, 109
viral encephalitis, 39 traveler's diarrhea, 97
Streptococcus pyogenes, 14, 64 Traveler's diarrhea, 95-97
Strongyloidiasis, 138 answers, 96-97
Superficial fungal infections, 121-124 patient presentation, 95
answers, 122-124 prevention of, 97
patient presentation, 121 questions, 96
prevention of, 123 references, 97
questions, 122 transmission of, 97
references, 124 Treatment. see also Antibiotics; Antifungal therapy
risk factors for, 122 for abscesses, 63
Suprapubic pain, 26 bacterial vaginosis, 117-118
Surgery chlamydia and gonorrhea, 113
infective endocarditis, 44 of cystitis, 27
necrotizing fasciitis, 73 of herpes, 108
prosthetic joint infection, 84-85 of influenza, 3
Surviving Sepsis Campaign, 49 malaria, 135
susceptibility testing methods, 131 of necrotizing fasciitis, 73
Syphilis, 103-106 nematodes, 138
answers, 104-106 superficial fungal infections, 122-123
patient presentation, 103-104 trichomoniasis, 117-118
questions, 104 vulvovaginal candidiasis, 117, 118, 119
references, 106 Treponema pallidum, 104, 105
risk factors for, 104 Treponemal testing, 104-105, 106
Systemic azole antifungals, 123 Trichomonas vaginalis, 117, 118
Systemic inflammation, 68, 82 Trichomoniasis, 115-119
Systemic inflammatory response syndrome (SIRS), 48, 49, 63 answers, 117-119
patient presentation, 115-116
T questions, 116
Terbinafine, 123 references, 119
Tertiary syphilis, 105 Trimethoprim-sulfamethoxazole,27,64,69,89
Testing, for influenza, 2 Trough concentrations, 49
Test-of-cure, 113, 118 Tuberculosis, 106, 141-144
Tetracyclines, 81, 106 answers, 143-144
Throat culture, 14 patient presentation, 141-142
Thrombocytopenia, 135 questions, 142
Tigecycline, 32 references, 144
Tinea capitis, 122 risk factors for, 143
Tinea corporis (ringworm), 122, 123 Typhoid fever, 97
Tinea cruris, 122
Tinea pedis (athlete's foot), 122, 123 u
Tinea unguium (onychomycosis), 122, 123-124 Urethral discharge, 113
Tonsillectomy, 15 Urgency, urinary, 26
Tonsillopharyngeal erythema, 14 Urinary antigen tests (UAT), 23
Topical antifungal agents, 122-123 Urinary tract infection (UTI), 26, 27, 31-32, 117, 119
Toxic shock syndrome, 73 Urine samples, 31
Toxoplasma gondii, 53
Toxoplasmosis, 53 v
Transesophageal echocardiography (TEE), 43 Vaccines
Transmission acute otitis media, 7
Clostridioides difficile infection, 93- 94 hepatitis, 101
154 INDEX

Vaccines (Contd.) Vibrio spp., 72


for influenza, 4 Viral encephalitis, 37-39
influenza, 20 answers, 38-39
pneumonia, 20 vs. bacterial meningitis, 38
typhoid fever, 97 patient presentation, 37-38
Vaginal discharge, 112, 113, 117 questions, 38
Vaginal infections, 117. see also specific infections references, 39
Valacyclovir, 108-109 Viral pharyngitis, 14
Valvular dysfunction, 44 Viruses. see specific viruses
Vancomycin,19,35,49 Voriconazole, 131
Clostridioides difficile infection, 93 Vulvovaginal candidiasis, 115-119
diabetic foot infection, 78 answers, 117-119
febrile neutropenia, 58 patient presentation, 115-116
MRSA and, 63-64, 69 questions, 116
prosthetic joint infection, 85 references, 119
skin and soft tissue infection I, 64 treatment regimens, 118, 119
viral encephalitis, 39
Vegetation characteristics, 44 w
Venereal Disease Research Laboratory (VDRL) test, 104 West Nile virus, 39
Ventilators Whiff test, 112
resistance risk factors for VAP, 23 White blood cell (WBC) count, 57
ventilator-associated pneumonia, 21-24
ventilator-associated tracheobronchitis, 23 x
Vertebral osteomyelitis, 79-82 X-ray
answers, 80-82 chest, 19, 23, 143
patient presentation, 79-80 diabetic foot infection, 77
questions, 80
references, 82 z
Vertebral Osteomyelitis Guidelines, 81-82 Zanamivir, 3, 4

You might also like